Está en la página 1de 807
2500 SOLVED PROBLEMS IN FLUID |. os) ae Ics me eb 4N) v1 0 e) Revised First Edition @ Learn the best strategies for solving tough problems in step-by-step detail. © Prepare effectively for exams and save time in doing homework problems. ® Choose from the largest selection of solved problems yet published on this subject. © Use the index to quickly locate the types of _ problems you most need help with. ©@ Save this book for reference in other courses and even for your ae library. ee i RUG) Pa A SCHAUM’S SOLVED PROBLEMS SERIES 2500 SOLVED PROBLEMS IN FLUID MECHANICS AND HYDRAULICS I Jack B. Evett, Ph.D., Professor of Civil Engineering, and Cheng Liu, M.S., Associate Professor of Ciutl Engineering Technology, both at the University of North Carolina at Charlotte. Both authors have extensive teaching experience in the domain of fluid mechanics and hydraulics. They are coauthors of a textbook in fluid mechanics for the McGraw-Hill College Division. Project supervision by The Total Book Cover design by Wanda Siedlecka. Index by Hugh C. Maddocks, Ph.D. Library of Congress Cateloging-in-Publication Data Evett, Jack B. 2500 solved problems in fluid mechanics and hydraulics / by Jack B. Evett, Cheng Liu, p. cm. — (Schaum's solved problems series) ISBN 0-07-019783-0 1, Fluid mechanies—Problems, exercises, etc. 2. Hydraulies—Problems, exercises, etc. I. Liu, Cheng. I. Title. IIL. Title: Twenty-five hundred solved problems in fluid mechanics and hydraulics. 1V. Series, TAIS73.E84 1988 620.1°06076—de 19 88-1373, cP. 34567890 SHP/SHP 9321 ISBN 0-07-0197A4-5 (Formerly published under ISBN 0-07-019783-0) Copyright © 1989 McGraw-Hill, Inc. All rights reserved. Printed in the United States of America. Except as permitted under the United States Copyright Act of 1976, no part of this publication may be reproduced or distributed in any form or by any means, or stored in a data base or retrieval system, without the prior written permission of the publisher. CONTENTS To the Student List of Abbreviations List of Conversion Factors Chapter Chapter Chapter Chapter Chapter Chapter Chapter Chapter Chapter 1 2 9 ‘Chapter 10 Chapter 11 Chapter 12 Chapter 13 Chapter 14 Chapter 15, ‘Chapter 16 Chapter 17 Chapter 18 Chapter 19 (Chapter 20 Chapter 21 (Chapter 22 PROPERTIES OF FLUIDS FLUID STATICS FORCES ON SUBMERGED PLANE AREAS DAMS FORCES ON SUBMERGED CURVED AREAS BUOYANCY AND FLOTATION KINEMATICS OF FLUID MOTION FUNDAMENTALS OF FLUID FLOW FLOW IN CLOSED CONDUITS SERIES PIPELINE SYSTEMS PARALLEL PIPELINE SYSTEMS BRANCHING PIPELINE SYSTEMS PIPE NETWORKS FLOW IN OPEN CHANNELS FLOOD ROUTING FLOW OF COMPRESSIBLE FLUIDS FLOW MEASUREMENT DIMENSIONAL ANALYSIS AND SIMILITUDE. UNSTEADY FLOW PUMPS AND FANS TURBINES HYDRAULIC AND ENERGY GRADE LINES 1 aAaR Bee 3.8 i iv 0 CONTENTS Chapter 23. FORCES DEVELOPED BY FLUIDS IN MOTION Chapter 24 DYNAMIC DRAG AND LIFT Chapter 25. BASIC HYDRODYNAMICS Appendix Index 5 z 709 787 To the Student This book contains precisely 2500 completely solved problems in the areas of fluid mechanics and hydraulics. Virtually all types of problems ordinarily encountered in study and practice in these areas are covered. Not only you, but teachers, practitioners, and graduates reviewing for engineering licensing examinations should find these problems valuable. ‘To acquaint you with our “approach,” particular steps taken in presenting the problems and their solutions are itemized below. + First and most important of all, each problem and its solution are essentially independent and self-contained. That is to say, each contains all the data, equations, and computations necessary t0 find the answers. Thus, you should be able to pick a problem anywhere and follow its solution without having to review whatever precedes it. The exception to this is the occasional problem that specifically refers to, and carries over information from, a previous problem. + In the solutions, our objective has been to present any needed equation first and then clearly to evaluate each term in the equation in order to find the answer. The terms may be evaluated separately or within the equation itself. For example, when solving an equa- tion that has the parameter “area” as one of its terms, the area term (A) may be eva- luated separately and its value substituted into the equation [as in Prob. 14.209], or it may be evaluated within the equation itself [as in Prob. 14.94). + Virtually every number appearing in a solution is either “given” information (appearing as data in the statement of the problem or on an accompanying illustration), a previously computed value within the problem, a conversion factor (obtainable from the List of Con- version Factors), or a physical property (obtainable from a table or illustration in the Appendix). For example, in Prob. 1.77, the number 1.49, which does not appear elsewhere in the problem, is the dynamic viscosity (4) of glycerin; it was obtained from Table A-3 in the Appendix. + We have tried to include all but the most familiar items in the List of Abbreviations and Symbols. Hence, when an unknown sign is encountered in a problem or its solution, a scan of that list should prove helpful. Thus, the infrequently used symbol y is encountered in Prob. 25.6. According to the list, y represents the stream function, and you are quickly on your way to a solution. Every problem solution in this book has been checked, but, with 2500 in all, it is in- evitable that some mistakes will slip through. We would appreciate it if you would take the time to communicate any mistakes you find to us, so that they may be corrected in future printings. We wish to thank Bill Langley, of The University of North Carolina at Charlotte, who assisted us with some of the problem selection and prepar abs a (alpha) atm ‘atmos B (beta) bhp Btu or cB c eal ob. G G & g & cf eg. or CG G c. ° ep. @ ‘ G a D Abbreviations and Symbols acceleration or area area absolute angle between absolute velocity of fiuid in hydraulic machine and linear velocity of a point on & rotating body or coefficient of thermal expansion or dimensionless ratio of similitude atmosphere atmospheric angle between relative velocity in hydraulic machines and linear velocity of a point on a rotating body or coeficient of compressibility or ratio of obstruction diameter to duct diameter surface width or other width surface width or other width brake horsepower brake power British thermal unit speed of sound or wave speed (celerity) Celsius or discharge coefficient or speed of propagation, calorie center of buoyancy coefficient of contraction coefficient of discharge drag coefficient friction-drag coefficient {force coefficient ‘cubic foot per second center of gravity Pitot tube coefficient lift coefcient centimeter (10°? m) centipoise center of pressure specific heat at constant pressure specific heat at constant volume coefficient of velocity weir coefficient depth or diameter depth or diameter or drag force thickness of boundary layer thickness ofthe viscous sublayer change in (or difference between) critical depth effective diameter hydraulic diameter mean depth normal depth normal depth ‘modulus of elasticity or specific energy or velocity approach factor hydraulic efficiency elevation ump or turbine efficiency height or surface roughness ‘pump energy turbine,energy exponential frequency of oscillation (cycles per second) or friction factor vil vii 0 ABBREVIATIONS AND SYMBOLS F Fahrenheit or force R buoyant force fo drag force be horizontal force E lift force fps foot per second FS. factor of safety fe foot i uplift force on a dam K vertical force t acceleration due to gravity or gage height or gram G weight flow rate al gallon (gamma) specific (or unit) weight (Gamma) circulation GN siganewton (10° N) GPa sigapascal (10° Pa) pm gallons per minute h enthalpy per unit mass or height or depth or pressure head or hour fi average height or depth or head fi centhalpy per unit weight H energy head or total energy head hy ‘unit head loss hy vertical depth to center of gravity he vertical depth to center of pressure hy head loss due to friction Hg mercury HGL hydraulic grade line he total head loss bin head loss due to minor losses hp horsepower He, hertz (cycles per second) 1 inflow or moment of inertia 1D inside diameter in inch (infinity) sometimes used as a subscript to indicate upstream J joule kK bulk modulus of elasticity or Kelvin or minor loss coefficient k specific heat ratio eal Kilocalorie (10° cal) kg kilogram (10g) w joule (10°3) km meter (10° m) AN kKilonewton (10° N) kPa kilopascal (10° Pa) kw kilowatt (10° W) L length or lft force or liter A (lambda) model ratio or wave length pound be pound mass L equivalent length L linear dimension in model L linear dimension in prototype m mass or meter th mass flow rate M ‘mass flow rate or molecular weight or moment or torque MB distance from center of buoyancy to metacenter mbar millibar (10"" bar) me metacenter mgd million gallons per day MPa 2 (ohm) oo (omega) ? > Pa (phi) 0) T(pi) z Be pai ¥v (psi) aia psig He Be Pe q Q Qu Ow ABBREVIATIONS AND SYMBOLS 1) ix milititer (10° L) minute millimeter (10°? meter) ‘meganewton (10°N) rmegapascal (10° Pa) mile per hour ‘manometer reading absolute or dynamic viscosity megawatt (10° W) “Manning roughness coefficient or number of moles newton or rotational speed Brinkman number Froude number Mach number net positive suction head Reynolds number specific speed of pump or turbine kinematic viscosity Weber number outfiow outside diameter rotational rate angular velocity pressure or poise force (usually resulting from an applied pressure) or power pascal peripheral-velocity factor constant = 3.14159265 dimensionless parameter power ratio stagnation pressure pound per square inch stream function pound per square inch absolute pound per square inch gage pressure for condition at Nu= 1/VE ‘vapor pressure ‘wetted perimeter flow rate per unit width or heat per unit mass discharge or heat or volume flow rate heat transferred per unit weight of fluid volume flow rate per unit width of channel quart radius {g88 constant or Rankine or resultant force or hydraulic radius ‘manometer reading radian critical hydraulic radius hydraulic radius mass density inside radius outside radius revolutions per minute universal gas constant entropy of a substance or second or slope slope or storage critical slope specific gravity specific gravity of manometer fuid specific gravity of flowing fuid ABBREVIATIONS AND SYMBOLS (sigma) o E (sigma) s s 1 in (tau) Fo (tau) BESS S RARGSE SEE GEES pump cavitation parameter o stress or surface tension ‘cavitation index summation specific gravity of flowing fuid specific gravity of manometer fluid thickness or time surface width or temperature or torque o tension shear stress shear stress at the wall stagnation temperature velocity centerline velocity velocity velocity critical velocity velocity of volume average velocity centerline velocity volume of fluid displaced velocity in model velocity in prototype specie volume shear velocity tangential velocity terminal velocity width watt or weight or weight flow rate or work distance from center of gravity to center of pressure in x direction vorticity depth critical depth distance from center of gravity to center of pressure in y direction ‘normal depth ‘normal depth inclined distance from liquid surface to center of gravity inclined distance from liquid surface to center of pressure 0.00001667 m’/s= 1. L/min 0.002228 f'/s = 1 gal/min 0.0145 Ib/in? = 1 mbar 0.3048 m= 1 ft 2S4em= 3.281 ft Conversion Factors S80 ft-b/s Lh 778 fll = 1 Btw 1728in' = 1fe 200015 = 1 ton 3600s=1h 41843 = 1 keal 5280 ft= I mile 364005 =1 day 11000 000 N. 1 42 13 14 1s 16 ar CHAPTER 1 Properties of Fluids Note: For many problems in this chapter, values of various physical properties of fluids are obtained from ‘Tables A-1 through A-8 in the Appendix. A reservoir of glycerin (glyc) has a mass of 1200 kg and a volume of 0.952 m’. Find the glycerin’s weight (W), ‘mass density (p), specific weight (y), and specific gravity (s.g.). 1 F=W = ma =(1200)(9.81)=11770N or 11.77kN p= m|V = 1200/0.952 = 1261 kg/m* = WIV = 11.77/0,952 = 12.36 KN/m? 8.8: Yoyo! Yn aeare 12.36/9.81 = 1.26 ‘A body requires a force of 100N to accelerate it at arate of 0.20 m/s. Determine the mass of the body in kilograms and in slugs. 100 = (my(0.20) 500 kg = 500/14.59 = 34.3 slugs ‘A reservoir of carbon tetrachloride (CCl,) has a mass of $00 kg and a volume of 0.315 m*, Find the carbon tetrachloride’s weight, mass density, specific weight, and specific gravity. ' F=W=ma=(S00)(9.81)=4905N_ or 4.905KN p= m[V = 500/0/315 = 1587 kg/m? 5.57 N/m? 59 ‘The weight of a body is 1001b, Determine (a) its weight in newtons, (b) its mass in kilograms, and (¢) the rate of acceleration [in both feet per second per second (ft/s") and meters per second per second (m/s*)] if a net, force of 50 Ib is applied to the body. 1@ W = (100)(4.448) = 444.8 o FaWama 448=(m)Q81) m=45.34kg © m= 45.34/14,59 = 3,108 slugs Fama $0=3.108a a= 16.09 f/s* = (16.09)(0.3048) = 04 m/s? The specific gravity of ethyl alcohol is 0.79. Calculate its specific weight (in both pounds per eubie foot and kilonewtons per cubie meter) and mass density (in both slugs per cubie foot and kilograms per cubic meter) ' 7 =(0.79)(62.4)=49.31b/f° y= (0.79)(9.79) = 7.73 KN/m? p= (0.79)(1.98) = 1.53 slugs/f® p= (0.79)(1000) = 790 kg/m? {A quart of water weights about 2.08 1b. Compute its mass in slugs and in kilograms. ' FeWema —208=(m)(322) m=0,0646 slug m= (0.0646)(14.59) = 0.983 kg One cubic foot of glycerin has a mass of 2.44 slugs. Find its specific weight in both pounds per cubic foot and kilonewtons per cubic meter. BF =W =ma=(2.44)(32.2)= 7861b. Since the gycern’s volume is 1f?, y =78.61b/10 = (78.6)(4.448)/(0.3048)° = 12 350 N/m°, or 12.35 KN/m?. 2 0 CHAPTER 1 18 1.10 1 uz 13 144 1s 116 7 {A quart of SAE 30 ol at 68 F weighs about 1.851b. Calculate the oi’ specific weight, mass density, and specific gravity i V = 1/((4)(7-48)] = 0.03342 f° y= W/V = 1.85/0.03342 = 55.4 lb/ft” p= vl = 55.4/32.2=1.72 slugs 5.8: = Youl Ynz0 marc = 55.4/62.4 = 0.888 The volume of a rock is found tobe 0,00015 m. Ifthe rock’ speci gravity is 2.60, what i its weight? ! Youn = 2.60)(9.79)=25.5KN/m?——Wraa = (25.5)(0.00015)=0.00382 KN or 3.82 ‘A certain gasoline weighs 4.5 b/f. What are its mass density, specific volume, and specific gravity? Ui p= ig =46.5/32.2= 1.44 siugs/f? ——-V,= I/p-=1/1.44= 0.694 °/slug 44/1.94= 0.782 se. If the specific weight of a substance is 8.2 N/m", what is its mass density? ' p= vig =8200/9.81 = 836 kg/m? ‘An object at a certain location has a mass of 2.0 kg and weighs 19.0 N on a spring balance. What i the acceleration due to gravity at this location? ' F=Wema 19.0=200 a=9.50m/3! fan object has a mass of 2.0 slugs at sea level, what would its mass be at @ location where the acceleration due to gravity is 30.00 ft/s"? I Since the mass of an object does not change, its mass will be 2.0 slugs at that location ‘What would be the weight of a 3-kg mass on a planet where the acceleration due to gravity is 10.00 m/s*? ! F = W = ma (3)(10.00) = 30,00N Determine the weight ofa S-slug boulder at a place where the acceleration due to gravity is 31.7 A/S 1 F=W=ma=(5)(31.7)=1581b Tf 200 ft? of oil weighs 10 520 Ib, calculate its specific weight, density, and specific gravity. ! y= W/V =10520/200=52.61b/f p= y/g =52.6/32.2= 1.63 slugs/t 8.8. = You! Ynyo at arc = 52-6/62.4 = 0.843. Find the height ofthe free surface if 0.8 fof water is poured into a conical tank (Fig. 1-1) 20in high with a ‘base radius of 10in. How much additional water is required to fill the tank? 1 Voom = 27/3 ™ x(10)'20)/3=2004in? Vigo = 0.8 ft? = 1382 in? Additional water needed = 2095 ~ 1382 = 713in*. From Fig. 1-1, ,/10= h, /20, oF r, = hy/2.0; Vent top one = (h,/2.0}*h,/3= 713; h, = 13.96 in. Free surface will be 20 ~ 13.96, of 6.04 in above base of tank. Fig. 1 118 149 1.20 1a 12 123 12s 125 1.26 PROPERTIES OF FLUIDS 2 3 If the tank of Prob. 1.17 holds 30.5 kg of salad oil, what is the density ofthe oil? 1 Veoue = 2094 in* (from Prob. 1.17) = #78 (0.3048)° = 0.03431 m* p=m/V = 30.5/0.03431 = 889 ke/m* Under standard conditions a certain gas weighs 0.14 Ib/f. Calculate its density, specifie volume, and specific, gravity relative to air weighing 0.075 1b/ft. ! p= ylg =0.14/32.2 = 0.00435 slug/tt? ——_V, = 1/p=1/0.00435 = 230 f°/stug, 0.14/0.075 = 1.87 Ifthe specific volume of a gas is 360 f/slug, what is its specific weight? ' p=1V,=sto= 0.002778 slug/te? y= pg = (0.002778)(32.2) = 0.0895 Ib/te 'A vertical glass cylinder contains 900,00 ml. of water at 10°C; the height of the water column is 90.00.cm. The water and its container are heated to 80°C. Assuming no evaporation, what will be the height of the water ifthe coefficient of thermal expansion (a) for the glass is 3.6 x 10°**C" I Mass of water= pV = pisVio=PmxVeo _(1000)(900 x 10-*) = 971Vay Vio = 926.9 X 10°* m? = 926.9 em? Auo™ Vile = 900,00/90.00 = 10.000 em? Aw= rio 10.000: roo = Ful + (AT) a)] = (1.7841){1 + (80 — 10)(3.6 x 10°] = 1.7845 em. ‘Au = y= (1.7845)? = 10,004 cm? go = Veo Axp = 926.9/10.004 = 92.65 em ona = 1.7841 em Ifa vessel that contains 3.500 fof water at 50°F and atmospheric pressure is heated to 160°F, what willbe the Deroentage change in its volume? What weight of water must be removed to maintain the original volume? 1 Weight of water= WV = yiVio™= YuoVieo _(62-4)(3.500) = 61.0Vieo Vin = 3.5803 40° Change in volume = (3.5803 — 3.500)/3.000 = 0.027, or 2.7% (increase). Must remove (3.5803 — 3.500)(61.0), or 4.901b, ‘A vertical, cylindrical tank with a diameter of 12.00 m and a depth of 4.00 m is filled to the top with water at 20°C. Ifthe water is heated to 50°C, how much water will spill over? ' Viaus = (Viso)a0 #(12,00/2)°(4.00) = 452.4 m? Wrzo = (9-79)(452.4) = 4429 KN (Vio)s0 = 4429/9.69 = 457.1 m? Volume of water spilled = 457.1 ~452.4= 4.7 m* ‘A thick, closed, steel chamber is filled with water at 50°F and atmospheric pressure. Ifthe temperature of water and chamber is raised to 100 °F, find the new pressure of the water. The coefficient of thermal expansion of steel is 6.5 x 10° per ‘F. I The volume of water would attempt to increase as the cube of the linear dimension; heice, Veo = Vad + (100 — 50)(6.5 x 10°*)]* = 1.000975V.g;, weight of water = YV = Y:0Vin= YoVin, 62-4¥io= “7e{1.000975Vig), Yoo = 62.34 lb/ft". From Fig. A-3, Pye = 1300 psia (approximately). {A liquid compressed in a cylinder has a volume of 1000cm? at 1 MN/m? and a volume of 95 cm? at 2MN/m*. ‘What is its bulk modulus of elasticity (K)? 80 =) aon apa - k= Aviv (995 = 1000)/1000 ee Find the bulk modulus of elasticity of a liquid if a pressure of 150 psi applied to 10ft ofthe liquid causes 8 volume reduction of 0.02. ' _ 450 - 0)044) 2/10 10 800 000 Ib/ft? or 75.000 psi 4 0 CHAPTER 1 127 1.28 129 130 131 132 If K =2.2 GPa is the bulk modulus of elasticity for water, what pressure is required to reduce a volume by 0.6 pereent? ' a 22-27% p= 0.0132GPa or 13.2MPa Find the change in volume of 1.00000 fof water at 80°F when subjected to a pressure increase of 300 psi. ‘Water's bulk modulus of elasticity at this temperature is 325 000 psi ' AV = ~0,00092 4° 325000= ~ Ap aviv From the following test data, determine the bulk modulus of elasticity of water: at 500 psi the volume was 1.000 ft’, and at 3500 psi the volume was 0.990 8. 500-3500 e 1.000 — 0.990)/1.000 300.000 psi A tigid steel container is partially filled with a liquid at 15 atm. The volume of the liquid is 1.232001. Ata pressure of 30 atm, the volume of the liquid is 1.23100 L. Find the average bulk modulus of elasticity of the liquid over the given range of pressure if the temperature after compression is allowed to return to its initial value. What isthe coefficient of compressibility (8)? (0~15)(101.3) AVIV~ ~ (123100 =1.23200)/1.23200 872X10°kPa or 1.872GPa B= UK =1/1.872=0.534G1 A heavy tank contains oil (A) and water (B) subject to variable air pressure; the dimensions shown in Fig, 1-2 correspond to 1 atm. If air is slowly added from a pump to bring pressure p up to 1 MPa gage, what will be the total downward movement of the free surface of oil and air? Take average values of bulk moduli of elasticity of {he liquids 28 2050 MPa for oil and 2075 MPa for water, Assume the container does not change volume. Neglect hydrostatic pressures, 0 oo 1-0 3 : “""aviv = — a7 Teanxeaayyay AV ~20600 1-0 2075 = AViyo™ ~23850 mm? ~ AV go/7000 (300) 74] 44.540 mm? Water 300 min WHEL Fig. 12 A thin-walled spherical tank is filled with water at a pressure of 4666 psig; the tank's volume is then 805.407 in’. If the water is released from the tank, how many pounds will be collected at atmospheric pressure? 805.4069 in? PROPERTIES OF FLUIDS 9 5 when the pressure i 4666 psig, Use 305 000 psi as an average value of the bulk modulus of elasticity a 5 ' By 3500 ~ as a0 805 407 W = (62.4)(817.73/1728) =29.5 1b 17.73 in 1.33 Water in a hydraulic press, initially at 0 psia, is subjected to a pressure of 17 000 psia at 68°F. Determine the percentage decrease in specific volume if the average bulk modulus of elasticity is 365 000 psi. ae 700-20 av au Kev —““awi = 0.0465 or 4.65% decrease 134 Ata depth of 7km in the ocean, the pressure is 71.6 MPa. Assume a specific weight at the surface of 10.05 N/m" and an average bulk modulus of elasticity of 2M GPa for that pressure range. Find (a) the change in specific volume between the surface and 7 km; (b) the specific volume at Tkm; (e) the specific weight at 7km. 1@ (8). = py = gh. = 9.81/10 050 = 0,0009761 m'/kg 2 Om mat AV) =—0.0000259 m'/kg, o V.). + AV, = 0.0009761 ~ 0.0000299 = 0.000846 "kg © 17 8/¥s= 9.81/0.000846 = 10370 N/m? 135 Approximately what pressure must be applied to water at 60°F to reduce its volume 2.5 percent? ae, = P20 = ' kash mom= BP p= rs psi 1.36 A gas at 20°C and 0.21 MPa abs has a volume of 41 L and a gas constant (R) of 210m - N/(kg K). Determine the density and mass of the gas. BF p=piRT =0.21 x 10°/[210)(20+273)]=3.41 kg/m? m= pV =(3.41)(0.041) = 0.140 kg 1.37 What isthe specific weight of air at 70 psia and 70°F? I y=pIRT. From Table A-6, R= 53.3 f1/°R; y = (70)(144)/[(53.3)(70 + 460)] = 0.357 1b/f. Note: p/RT gives p (Prob. 1.36) or y (Prob. 1.37), depending on the value of R used. Corresponding values of Rin Table A-6 differ by a factor of g 1.38 Calculate the density of water vapor at 350 kPa abs and 20°C if its gas constant (R) is 0.462 kPa -m*/kg - K. ' p= pIRT = 350/{(0.462)(20 + 273)] = 2.59 kg/m? 1.39 Nitrogen gas (molecular weight 28) occupies a volume of 4.0 ft” at 2500 Ib/ft® abs and 750°R. What are its specific volume and specific weight? 1 R= R,/M = 49 709/28 = 1775 ft Ib/(slug -°R) where R,, the universal gas constant, = 49 709 ft: Ib/(slug -°R)] p= A/V, = pIRT = 2500/|(1775)(750)]__V, =$32.5 £°/slug Y= pe = (IIV.)(R) = (1/532.5)(32.2) = 0.0605 Ib/t* 1.40 One kilogram of hydrogen is confined in a volume of 200 L at ~45°C. What is the pressure if R is 4.115 kS/kg- K? ' p= pRT = (m/V)RT =(1/0.200)(4115)(—45 +273) =4.691 x 10" Pa or 4.691 MPabs LAL Whatis the specific weight of 1 Y= p/RT = 410/{(29.3)(30 + 273)] = 0.0529 kN/m* at a temperature of 30°C and a pressure of 470 kPa abs? 6 0 CHAPTER 1 1.42 Find the mass density of helium at a temperature of 39°F and a pressure of 26.9 psig, i atmospheric pressure is 14.9psia. i p= DIRT = (14.9 + 26.9)(144)/{(12.420)(39 + 460)) = 0.000971 Ib-s/ft* or 0.000971 slug/te 143 The temperature and pressure of nitrogen in a tank are 28°C and 600 kPa abs, respectively. Determine the specific weight ofthe nitrogen. ' Y= DIRT = 600/{(30.3)(28 + 273)] = 0.0658 KN/m? 1.4 The temperature and pressure of oxygen in a container are 60 °F and 20.0 psig, respectively. Determine the coxygen's mass density if atmospheric pressure is 14.7 psia. 1 P= pPIRT = (20.0 + 14.7)(144)/{(1552)(60 + 460)] = 0.00619 slug/te? 145 Calculate the specific weight and density of methane at 100 °F and 120 psia. ' {Y= pIRT = (120)(144)/(96.2)100 + 460)} = 0.321 1b /t? p= y/g = 0.321/32,2 = 0.00997 stug/ft” 1.46 At 90°F and 30.0 psi, the specif weight ofa certain gas was 0.0877 Ib/f?. Determine the gas constant and density ofthis gas ' Y=p/RT 0.0877 = (30.0)(144)/[(R)(90 + 460)] R= 89.6 41/°R. p= y/g =0.0877/32.2 = 0.00272 slug/tt? 1.47 Acylinder contains 12.5 ft of air at 120°F and 40 psia. The air is then compressed to 2.50 ft’. (a) Assuming. isothermal conditions, what are the pressure at the new volume and the bulk modulus of elasticity? (®) Assuming adiabatic conditions, what are the final pressure and temperature and the bulk modulus ofelatcty? 1@ PiVi=p:¥; (forisothermal conditions) (40)(12.5) = (p3)@2.50) P3= 200 psia dp ____ 40-200 VV" ~GZ3-2.5)/125 (©) p.Vi=p.V$ (for adiabatic conditions). From Table A-6, k = 1.40. (40)(12.5)" i= 381 psia; T/T, = (p3/p.)*-*, T3/(120 + 460) = C3), (4.40)(381) = 533 psi = 1104°R, oF 644°F; = (p9@.50)'*, kp 1.48 Airis kept at a pressure of 200 kPa and a temperature of 30°C in a 500-L container. What is the mass of the air? 1 p= p/RT = {(200)(1000)}/{(287)(30 + 273)] =2.300 kg/m? m= (2.300)(i88) = 1.15 kg 1.49 An ideal gas has its pressure doubled and its specific volume decreased by two-thirds. Ifthe initial temperature is 80°F, what is the final temperature? ' paiV,=piRT — p¥.=RT — p(Vv=RT, ——pAV,)2= RTs (ple MWe! V1 = (R/RVTIT) —2))= (80+ 460) Ty =360°R or ~100°F 1s 12 133 154 1.56 4s7 PROPERTIES OF FLUIDS 0 7 ‘The tank of a leaky air compressor orginally holds 90 L of air at 33°C and 225 kPa. During a compression proces, 4 grams of air is lost; the remaining air occupies 42 at S80 kPa, What isthe temperature ofthe remaining a Fox pul RT, = 225 x 107) /{(287)(38-+273)] =2.562 kg/m? m= (2.562)(0.050) = 0.2306 kg P2™=P2sRT, (0.2306 — 0.004)/0.042 = (550 x 10°)/(287T;) T,= 355K or 82°C In a piston-and-cylinder apparatus the initial volume of air is 90 L at a pressure of 130kPa and temperature of 26°C. If the pressure is doubled while the volume is decreased to 561, compute the final temperature and. density of the air. 1 p=plRT, Ps 130 x 10°)/[(287)(26 + 273)] = 1.515 kg/m’ ——_-m = (1.$15)(0.090) = 0.1364 kg IRT; 0.1364) = (2)(130% 107/287) =372K or 99°C 1p = 0.1364/(0.056) = 2.44 kg/m? For 2b mol of air with a molecular weight of 29, a temperature of 90°F, and a pressure of 2.5 atm, what is the volume? 1 PVInM=RT —((2.5)(14.7)(144)]{V/1)29))} = (53.3)000+ 460) V=321 te If nitrogen has a molecular weight of 28, what i its density according to the perfect gas law when p = 0.290 MPa and T =30°C? ' R= R,/M =8312/28= 297 J/(kg-K) [where Ry =83123/(kg- K)] P= PIRT = 290 000/{(297)(30 +273)] = 3.22 kg/m? fa gas occupies 1 m* at 1 atm pressure, what pressure is required to reduce the volume of the gas by 2 percent under isothermal conditions ifthe fuid is (a) air, (b) argon, and (e) hydrogen? I pV =nkT = constant for isothermal conditions. Therefore, if V drops to 0.98V,, p must rise to (1/0.98)p., or 1.020p,. This is true for any perfect gas (@) Calculate the density, specific weight, and specific volume of oxygen at 100°F and 15 psia,(b) What would be the temperature and pressure ofthis gas i t were compressed isentropically to 40 percent of is original volume? (e) Ifthe proces described in (b) had been isothermal, whet would the temperature and pressure have been? 1@ 2 = DIRT = (15)(148)/[(1552)(100 + 460)] = 0.00248 slug/t? = pg = (0.00248)(32.2)=0.07991b/f —V, = 1/p = 1/0.00288 = 403 £/shog ©) pulWot=palWo_ [25)(148)|403)'* = [(p_)04)](0.40(403)]* ps = 54.1 psia P2=PrRT, — (54.1)(144) = (0.00248/0.40)(1552)(7; + 460) T, = 350°F (6) Ifisothermal, = 7, = 100°F and pV = constant. [25)(4144)](403) = ((p.)(144)]{(0.40)(403)]__ p2= 37.5 psia Calculate the density, specific weight, and volume of chloride gas at 25 °C and pressure of 600.000 N/m? abs. ' RT = 600 000/{(118)(25 + 273)] = 17.1 kg/m* 17.19.81) = 168 N/m? V, =1/p = 1/17.1 =0.0585 m'/ikg y= 98 If methane gas has a specific gravity of 0.55 relative to air at 14.7 psia and 68°F, what ar its specific weight and specific volume at that same pressure and temperature? What isthe value of R for the gas? ! Yar DIRT = (14.7)(448)/[(53.3)(68 + 460)] = 0.07522 16/te° You ™ (0-55)(0.07522) = 0.0414 16/8 Up=aly Von 32.2/0.0414 = 778 f° /stug Since R varies inversely with density for fixed pressure and temperature, Ry, ~53.3/0.55 = 96.9 f/°R. 8 0 CHAPTER 1 138 139 1.0 161 18 1.63 164 1.65 1.6 A gas at 40°C under a pressure of 21.868 bar abs has a unit weight of 362.N/m?. What is the value of R for this gas? What gas might this be? 1 Y=pIRT 362 (21.868 x 10°)/[(R)(40+273)] R= 19.3m/K ‘This gas might be carbon dioxide, since its gas constant is 19.3 m/K (from Table A-6). If water vapor (R =85.7 fY°R) in the atmosphere has a partial pressure of 0.60 psia and the temperature is 80°F, what is its specific weight? ' Y= pIRT = (0-60)(144)/{(85.7)(80 + 460)] = 0.00187 Ib/f Refer to Prob. 1.59. Ifthe barometer reads 14.60 psia, calculate the partial pressure of the weight, andthe specific weight of the atmosphere (ar plus water vapor). 1 Por = 14.60~ 0.60 14.00psia y= p/RT Ya = (14.00)(14)/[(53.3)(80 + 460)] =0.0700Ib/f? Yam = Yar * Yruoom hon) = 0.00187 1b/F° (from Prob. 1.59) Yume = 0.0700 + 0.00187 = 0.0719 b/t? (a) Calculate the density, specific weight, and specific volume of oxygen at 20°C and 40 kPa abs. (b) If the ‘oxygen is enclosed in a rigid container, what will be the pressure ifthe temperature is reduced to ~100°C? 1@ p= p/RT = (40)(1000)/[(260)(20 + 273)] = 0.525 kg/m? 7 pg =(0.525)(9.81)=5.15N/m? —-V,=1/p = 1/0.525 = 1.90 m’/kg, (®) p=1/¥,= pIRT. Since ¥, and R are constants, V;/R = T/p = constant, (20 + 273)/40 = (—100+273)/ps, P2=23.6KPa. Helium at 149 kPa abs and 10°C is isentropically compressed to one-fourth of its original volume. What is its final pressure? 1 PVi=paVS — 149Vi%= (p.)(Vi/4)" pa = 1488 kPa abs (a) If 9 of an ideal ga at 75°F and 22 psa is compressed isothermally to 2, what isthe resulting pressure? (6) What would the pressure and temperature have been if the process had been isentropic? 1@ PiVi=pa¥e — (22(9)=(p)2) p2=99psia Oy} PiVimpaV$ — 22X9)'=(p:)(2)' pr = 155 psia TIT = (pap — T/(TS + 460) = (3 Te BADR or 380 (a) If 12m’ of nitrogen at 30°C and 125 kPa abs is permitted to expand isothermally to 30m’, what is the resulting pressure? (4) What would the pressure and temperature have been if the process had been isentropic? 1@ PiWi=p2¥s — (125)(12)=(p3)G0) —_p2= $0.0 kPa abs PiVi=paV$ —(125)(12)'*°=(p3)(30)'" py =34.7 kPa abs TIT.= (pap —T/(B0+273) = G4.7/125"""" T= 210K or ~63°C If the viscosity of water at 68°F is 0.01008 poise, compute its absolute viscosity (1) in pound-seconds per square foot. Ifthe specific gravity at 68°F is 0.998, compute its kinematic viscosity (v) in square feet per second. 1 The poise is measured in dyne-seconds per square centimeter. Since 1 1b = 444 800 dynes and I ft= 30.48em, 1 Ib s/ft? = 444 800 dyne - 5/(30.48 em)? = 478.8 poises 0.01008, _ we _pg_ (2.11 10-9(32.2) 3788 ve (0.998)(62.4) 2M x10Ib-s/f? v= .09 x 10° f/s ‘Convert 15.14 poises to kinematic viscosity in square feet per second ifthe liquid has a specific gravity of 0.964. U (from Prob. 1.65) b= 15.14/478.8= 0.03162 Ib s/f v= wg/y = (0.03162)(32.2)/[0.968)(62.4)] = 0.0169 #/s 1607 168 10 PROPERTIES OF FLUIDS J 9 ‘The fluid flowing in Fig 1-3 has an absolute viscosity (1) of 0.0010 Ib /ft and specific gravity of 0.913. Calculate the velocity gradient and intensity of shear stress at the boundary and at points lin, 2in, and 3in from the boundary, assuming (a) a straight-line velocity distribution and (6) a parabolic velocity distribution. ‘The parabola in the sketch has is vertex at A and origin at B. I (a) For the straight-line assumption, the relation between velocity v and distance y is v= ISy, du = 1Sdy. The velocity gradient = du/dy = 15. Since w= r/(du/dy), «= u (du/dy). For y =0 (i.e., atthe boundary), v=Oand du/dy = 155"; t= (0,0010)(15) = 0.015 Ib/ft. For y = Lin, 2in, and 3in, du/dy and tare also 15 3" and 0.015 lb/ft respectively. (b) For the parabolic assumption, the parabola passes through the points v = 0 when y = 0 and v = 45 when y =3. The equation of this parabola is v= 45 ~ 5(3~y), dv/d) 1 =0.0010 (dv/dy). For y Oin/s, dv /dy = 305", and r = 0.030 1b/f¢, For du/dy =20s", and r= 0.0201b/ft. For y =2in, v = 40in/s, du/dy = 10s", and v=45in/s, dv/dy =0s", and r= O1b/ft, 010 b/f. For y =3in, — Fig. 13 A cylinder of 0.40-ft radius rotates concentrically inside a fixed cylinder of 0.42-ft radius. Both cylinders are 1.00 ft long. Determine the viscosity of the liquid that fills the space between the cylinders if a torque of (0.650 Ib - tis required to maintain an angular velocity of 60pm. I The torque is transmitted through the field layers to the outer cylinder. Since the gap between the cylinders {s small, the calculations may be made without integration. The tangential velocity v, of the inner cylinder = rw, where 7 = 0.40 ft and @ = 2x rad/s. Hence, v, = (0.40)(2x) =2.51 ft/s. For the small space between cylinders, the velocity gradient may be assumed to be a straight line and the mean radius ean be used. Then, dufdy = (2.51 ~ 0)/(0.42 ~ 0.40) = 125.55". Since applied torque equals resisting torque, applied torque (#)(area)(arm), 0.650 = x[(1.00)(22(0.40 + 0.42)/2]{(0.40 + 0.82)/2], r= 0.615 I/F = u (du /dy), 0.615 = (W)(25.5), w= 0.00890 1b- s/f. Water is moving through a pipe. The velocity profile at some section is shown in Fig. 1-4 and i given mathematically as u = (B/4u)(d?/4—°), where v = velocity of water at any position r, =a constant, 4 = viscosity of water, d = pipe diameter, and r = radial distance from centerline. What is the shear stress at the ‘wall ofthe pipe due to the water? What is the shear stress at a position r = d/4? Ifthe given profile persists a distance L along the pipe, what drag is induced on the pipe by the water in the direction of flow over this distance? ' v= Blau@la—P)—— dusdr = (B/4p)(—2r) = ~26r/4p, =m (dv/dr) = w(-2Br/4u) = ~28r/4 At the wall, 7 =d/2. Hence, =26(d/2)__ Bd a 2BUd/4)__ Bd 4 4 Tena =" 8 Drag = (tyas)(atea) = (Feua)(tdL) = (Bd /4)(dL) = Ba mL [4 Velocity profile Fig. 14 10 0 CHAPTER 1 170 un in 3 A Large plate moves with speed vp over a stationary plate on a layer of oil (see Fig. 1-5). Ifthe velocity profile is ‘that of a parabola, with the oil at the plates having the same velocity as the plates, what is the shear stress on. the moving plate from the oil? Ifa linear profile is assumed, what is the shear stress on the upper plate? A Fora parabolic profile, v? = ay. When y =d, v= up. Hence, v= ad, a= vb/d. Therefore, w= (Udy) = (9/4) v= veVy/d — duldy =[(ve)(VVEYO)] r= (duldy) = alvod(V VA") For y=d, r= l(vo)(1/Va)(3)(d-")] = wvo/ (24). For a linear profile, dv/dy = veld, t= w(vold). » Assumed protie » Fig. 15 ‘A square block weighing 1.1 kN and 250 mm on an edge slides down an incline on a film of oil 6.0 um thick (see Fig. 1-6). Assuming a linear velocity profile in the oil, what is the terminal speed of the block? The viscosity of the oil is 7mPa-s. ' = 1 (dv/dy) = (7x 10" fv /(6.0x10-)] = 1167, Fy = tA = (1167v7)(0.250)? = 72.905, At the terminal condition, equilibrium occurs. Hence, 1100 in 20° = 72.9vp, vy = 5.16m/s. aks 19kN somm Co ‘ocosomm Fig. 1-6(2) =r Fig. 1646) ‘A piston of weight 21 Ib slides in lubricated pipe, as shown in Fig. 1-7. The clearance between piston and pipe is 0.001 in. Ifthe piston decelerates at 2.1 ft/s' when the speed is 21 ft/s, what is the viscosity of the oil? 1 =p (dv/dy) = wfv/(0.001/12)] = 12 00040 Fem tA = 12 000p0{(2)(8)()] = 7854u0 EF=ma 21 ~(7854)(u)(21) = (21/32.2)(-2.1) w= 1.36 10-*I- s/f? mA | Tiel Me sin | Fig. 1-7 ‘A piston is moving through a cylinder ata speed of 19 /s, a8 shown in Fig. 1-8. The film of ol separating the piston from the cyinder has a viscosity of 0.020 Ib s/f. What isthe force required to maintain this motion? I Assume a cylindrically symmetric, linear velocity profile for the flow of oil inthe film. To find the frictional resistance, compute the shear stress at the piston surface. pSeoclerg re|aay aie 5 =ea=ea(222)(2)] Pa) {__Cuner* Carbon tetrachloride ] Fig. 2-41 Calculate the pressure difference between A and B for the setup shown in Fig. 2-42. pa (62.4)(66.6/12) ~ [(13.6)(62.4)](40.3/12) + (62.4)(22.2/12) = [(13.6)(62.4)1(30.0/12) ~ (62.4)(10.0/12) = pe Pa-Px=A362Ib/fE or 31.7 Ib/in?® Calculate the pressure difference between A and B for the setup shown in Fig. 2-43. ' pa~(0.79)x ~[(0.8)(9-79) 0.70) + (9.79)(8 = 0.80) = Pp Pa Pa = 13.3 N/mm? Calculate the pressure difference between A and B for the setup shown in Fig. 2-44. ' Pat (2A) +4) ~[03.6)62.4)]4) + (2.4) 0-2) = Po Pa~Pa=270810/f or 18.8Ib/in* 42 9 CHAPTER 2 658 in Mereun ¥ Ge D6) Fig. 2-42 Oi 6g. = 08) LL xm +070 m~ 150m = (x 080) m | ale Fig. 2.43 FLUID STATICS J 43 Mercury 66= 139 ig, 244 262 Vessels A and B in Fig. 2-45 contain water under pressures of 40.0 psi and 20.0 psi, respectively. What is the deflection of the mercury in the differential gage? H (40,0)(144) + (62.4)(x + h) — [(03.6)(62.4)Jh + 62.4y = (20,0)(144). Since x + y = 16.00 — 10.00, oF 6.00 ft, head ldte : 63 For a gage pressure at A in Fig. 2-46 of ~1.58 psi find the specific gravity of gage liquid B. 1B (-1.58)(144) + [(1.60)(62.4)](20.50 ~ 9.00) ~ (0.6750)(11.25 ~ 9.00) + [(6.8.ug2)(62-4)](11.25 — 10.00) = 0 Sng = 1.00 44 264 2.465 2.66 a CHAPTER 2 In Fig. 2-47, liquid A weighs 53.5 lb/ft’ and liquid B weighs 78.8 1b/ff. Manometer liquid M is mercury. Ifthe pressure at B is 30 psi, find the pressure at A. ! a= (S3.5)(6.5+ 1.3) + [(1.6)(62.4)\(1.3) + (78.8)(6.5-+ 10.0) = (30)(144) ‘Pa = 2334 Ib/ft? or 16.2 Ib/in® tan Ta osk oor Fig. 2-47 ‘What would be the manometer reading in Fig. 2-47 if pp ~ pis 165 kPa? I. Converting to Ib/t, pp — p= 3446 Ib/f. The mercury level wil rise some amount, x, on the left side of the ‘manometer and will fall by that amount on the right side of the manometer (see Fig. 2.48). Hence, taking ight densities from Prob. 2.64, p, ~ (S3.5(6.5 + 1.3 +2) + [(13.6)(62.4)](1.3 + 2x) + (78.8)(6.5 + 10.0 ~ 2] “T6Ade + 1986 = py — p= 3446, x = 0.89 ft; manometer reading = 13+ (2)(0.89) = 3.08 ft Fig. 248 In Fig. 2-49, water is contained in A and rises in the tube to a level 85 in above A; glycerin is contained in B. ‘The inverted U-tube is filled with air at 23 psi and 70°F. Atmospheric pressure is 14.6 psia. Determine the FLUID STATICS 9 45 difference in pressure (psi) between A and B ify is 16 in. What is the absolute pressure in Bin inches of mercury and in feet of glycerin? ' Pa— (62.4)(8) = (23)(144) p= 3754.010/f Pa ~ ((1.26)(62.4)][(8S-16)/12] = (23)(144) py = 3764.1. b/ft? Pa— Pa = 3754.0 — 3764.1 = —10.11b/f? or -0.070 Ib/in? 3764.1/144 + 14.6)/{(13.6)(62.4)/(12)"] = 83.0 inFig (3764.1/144 + 14,6)/[(1.26)(62.4)/(12)"]=895.4in or 74.6ft of glycerin Fig. 249 267 Gas confined in a rigid container exerts a gage pressure of 150 kPa when its temperature is 7°C, What pressure would the gas exert at 67°C? Barometric pressure remains constant at 719 mmHg. ' Pam = {(13.6(9.79)|(0.719) = 95.7 KPa Pyyy=95.7 + 150= 245.7 kPa PMIT=paVsIT — (4S-1)(V)I2TS +7)=(psN(V)I(273 +67) [V (volume) is constant] P= 298.4 KPa (absolute) = 298.4 ~95.7 = 202.7 kPa (gage) 266 In Fig. 2-50, atmospheric pressure is 146 psia, the gage reading at A is 6.1 psi and the vapor pressure of the alcohol is 1.7 psi. Compute x and y I Working in terms of absolute pressure heads, (6.1 + 14.6)(144)}/(0.90)(62.4)] — x = (1.79(44)/(0.90)(62.4)], x = 48.72; [(6.1 + 14.6)(144)}/[(0.90)(62.4)] + (y + 4.2) — (4.2)(13.6/0.90) =0, y = 619K. Fig. 2-50 2.69 In Fig. 2:50, assume the following: atmospheric pressure = 858 mbar abs, vapor pressure of the alcoho! = 160 mbar abs, x = 2.90 m, y =2.10m. Compute the reading on the pressure gage (p.) and on the ‘manometer (2) 46 0 CHAPTER 2 4H Working in terms of absolute pressure heads, [(?)aye + 858](0.100)/[(0.90)(9.79)] 2.90 = {160X010 9040.7) (Pade = 42 mbar, [CA + 85810 10) (0 90)9.79)] + 2.104 2)~ (2)(13.6/0.90) = 0, z=! 2.70 A pipeline contains an incompressible gas (y = 0.051b/ft?) at rest; at point A the pressure is 4.69 in of water. What is the pressure, in inches of water, at point B, 492 ft higher than A? The change in pressure in the atmosphere must be considered; assume, however, that O76 byt is constant. (Pal Yam (Pal Yam + 4.69/12 ft of water @ (Pal Yan = (Pal Pon +¥/12 fof water @ Subtracting Eq. (2) from Eq. (1), (Pal Pe (Pa! Dam (Pal Yam ~ (Pal Yam + 4-69)12~ x [12 @ (Pal Yam (Pal Pam = 492K of at = (492)(0.076/62.4) = 0.599 ft of water (Pal Pan (Pal Pan = 492.1 of gas = (492)(0.05/62.4) = 0.394 fe of water Substituting these relationships into Eq, (3), 0.394 = 0.599 + 4,69/12—x/12, x =7.15in of water. 2.71 Determine the pressure difference between points A and B in Fig. 2-51. pa + {(0.88)(9.79)](0.21) — [(13.6)(9.79)](0.09) — [(0.82)(9.79)|(0.41 - 0.09) + (9.79)(0.41 0.15) ~ (0.0118)(0.10) = pp Pa=Po=10.2kPa 2.72 InFig. 2-52, if pp pa =97.4 kPa, calculate H. ' Pa ~(9.79)(H/100) ~ [(0.827)9.79)](i&) + [(13.6)(9.79)](34-+ H +17)/100] = py 1.234H + 66.53=py—p,=97.4 H=25.0em ted ol 3g = 0827 Mercury Fig. 2552 273 2 205 2.6 FLUID STATICS 0 47 For Fig. 253, if uid 1 is water and fluid 2 is mercury, and 24 = 0 and 2, Pa=Pam? 1 0+ (9.79)[0 ~ (1/100 ~ [(3.6)(9.79)]{22— (-1)100=0 2 Open. Pate =11.cm, what is level zz at which fava 2 Fig. 283 ‘The inclined manometer in Fig. 2-S4a contains Meriam red manometer il (6. = 0.827). Assume the reservoir is very large. What should the angle 0 be if each inch along the scale is to represent a change of 0.8 Ib/ft? in gage pressure p,? I From Fig. 2546, Ap = yAz, or 0.816 /8 = [0.827)(62.4 16/f€)] (i (sin 8) from which @ = 10.72°, w Zt wee ox eo hz Reservoir Fig. 2-54(a) ‘The system in Fig. 2-55 is at 20°C. Compute the absolute pressure at point A. ! a+ [(0.85)(62.4)]() ~ [(13.69(62.4)]() + (62.4)( (5) = (14.7)(144) p= 2691 Ib/ft* abs Fig. 254(6) Very small pressure differences p ~ pp can be measured accurately by the two-fiid differential manometer shown in Fig. 2-56. Density p, is only slightly larger than the upper fluid p,. Derive an expression for the proportionality between h and p~ pp if the reservoirs are very lage. pa + ough, ~ posh ~ prg(h ~h) = Pa. Pa~ Pa™ (D2 prgh. If (p2~ ps) is small, h will be large (sensitive). Mig. 2-56 48 0 CHAPTER 2 2.77 Water flows downward in a pipe at 35°, as shown in Fig. 257. The pressure drop p, ~ p2is partly due to gravity and partly due to friction. The mercury manometer reads a S-in height difference. What is the total pressure drop p,—p:? What is the pressure drop due to friction only between 1 and 2? Does the manometer reading correspond only to friction drop? ' Pit (62A)(6sin 35° + x/12+ 8) — [13.6)(62.4) 4) — (62.4)(2/12)~ pr Pi=pr= 1129 1b/ft? (total pressure drop) Pressure drop due to friction only = [(13.6)(62.4) ~ 62.4](%) = 327.6 1b/f Manometer reads only the friction los. Mer % Wig. 257 2.78 Determine the gage pressure at point A in Fig. 2-58. H pa~(9.79)(0.50) + (0.0118}(0.33) + [(13.6)(9.79)](0.17) — {(0.83)(9.79)](0.44) =0 p= —14.17 KPa Pe Adem Water Mercury Fig. 2.58 2.79 In Fig. 259, calculate level A of the il inthe right-hand tube. Both tubes are open to the atmosphere. 1 0+ (9.799(0.110 + 0.240) ~[(0.83)(9.79)](0.240-+ h)=0 = 0.1817 m= 181.7 mm = h ‘tomm ou, 59.2003 | 240mm Water 280 282 FLUID STATICS J 49 . 2-60a the inclined manometer measures the excess pressure at A over that at B. The reservoir diameter In £832, calibrate the scale in {is 25in and that ofthe inclined tube is in. For 9 =32° and gage Quid with s. si perf. Pa=WAh+Ay)+P» (see Fig. 2-606) p4—py= (Ah + Ay) From Fig. 2-606, (A,)(Ay) = (Ag)(R) or Ay = AgR/Aa, Ah = R sin 8, pa ~ po = ¥(R sin 0+ AgR/A,)= YR(sin 0+ AnlA4), An/An = [x(2)*/4\/Lx(2-5)/4] = hos Pa ~ Pa = [(0.832)(62.4)](R)(sin 32° + 9f)/144 = (.1947R. The scale factor i thus 0.1947 psi/t Fig. 2-60(6) Determine the weight W that can be equilibrated by the force acting on the piston of Fig. 2-61. ' Pimps RlAv= Bla, 1.25/{a(35)/4]=W/ln250)7/8] W=63.8KN 250mm giam 38mm aiam mr v sain > ES oi Rig. 261 Neglecting the container's weight in Fig. 2-62, find the force tending to lift the circular top CD. ' Peo ~ ((0.8)(62.4)(4)=0 peo = 199.7 Ib/te? F = pA = (199.7)[(2.5)*/4] = 980 Ib 1 as, 50 0 CHAPTER 2 283 2.84 2.85 2.86 287 2.88 2.90 Find the force of oll on the top surface CD of Fig. 2-62 if the liquid level in the open pipe is reduced by 1.3m. F co ~ [(0.8)(62.4)]{4 ~ (1.393.281) =0 peo = -13.241b/f?(i.., a downward pressure by CD) F =pA=(~13.24)[x(2.57/4] = ~65.0 16 ‘A drum 2.25 ft in diameter filled with water has a vertical pipe of 0.70-in diameter attached to the top. How ‘many pounds of water must be poured into the pipe to exert a force of 1500 Ib on the top of the drum? ' = FIA=1500/{(2.25)/4]=371.316/f2 = ply =377.3/62.4= 6.05 ft Wage = (6.05) -x(0.70/12)*/4](62.4) = 1.01 16 In Fig. 2-63, the liquid at A and B is water and the manometer liquid is oil with s.g. = 0.80, hy = 300 mm, ‘fiz 200mm, and hy = 600 mm. (a) Determine p~ Pp. (6) If pp = 50kPa and the barometer reading is, 730 mmHg, find the absolute pressure at A in meters of water. 1@ Pa (9.79)( 2) — [(0.80)(9-79)] fe) + 9.79)() = Pe Pa~Pa= —1.37 KPa o Pa ~ (9-79) ~ [(0-80)(9.79) (BR) + (9.79) (8) = 50 Pa=48.63KPa_ (gage) = 48.63/9.79 + 224(13.6) =14.90m water (absolute) Fig. 2-63 In Fig. 2-63, 5.g-1= 1.0, 5..2= 0.96, 5.g.5= 1.0, hy = hy = 269 mm, and hy = 1.2-m. Compute pa — pp in rillimeters of water. ' Pa ~ (1.0)(269) ~ (0.96)(269) + (1.0)(1200) =p» pa ~ Pe = ~673 mm of water In Fig. 2-63, 5.9-1= 1.0, 5.8..= 0.94, 5.8.5= 1.0, Ay = 300mm, hy= 1-1 m, and pa — po = ~360 mm of water. Find the gage difference (h,) pa (1.0)(300) ~ (0.94)(43) + (1.01100) = Py Pa Pe 3 800+ (0.94)(h3) hy = 468mm. ‘What isthe pressure difference, in pounds per square inch, of a 1000-ft water column? ! P= yh = (62.4)(1000)/144 = 433 psi Find the pressure at a point 9.5 m below the free surface in a fluid whose density varies with depth h (in m) according to p= (450 kg/m’) + (11 kg/m‘ Wp = ydh = pg dh =(g)(450+ 11h) dh, Integrating both sides: p = ()(880h + 1147/2) For h= = (.81)(450)(.5) + (11)(.5)2]= 46.807 kPa If atmospheric pressure is 29.72 inHg, what will be the height of water in a water barometer ifthe temperature of the water is (a) 50°F, (6) 100°F, and (e) 150°F? ' P= yh = ((13.6)(62.4)](29.72/12) = 2102 Ib/#t? or 14,60 1b/in? (a) At 50°F, 7 = 62.4 Ib/f and Pras = 25.7/144, oF 0.178 Ib/in*, hyo = (14.60 ~ 0.178)(144)/62.4 = 33.28 (8) AL 100°F, 7 = 62.01b/f and Pega = fl, oF 0.938 Ib/i’, hyo = (14.60 — 0.938)(144)/62.0 = 31.73 ft. (©) At 150°F, y= 61.2 lb/ft and prays = fl, oF 3.78 Ibn’, hyo ~ (14.60 ~ 3.78)(144)/61.2 = 25.46 ft. FLUID STATICS 0 51 291 A bicycle tire is inflated at sea level (where atmospheric pressure is 14.6 psia and the temperature is 69°F) to ‘65.0psi. Assuming the tire does not expand, what is the gage pressure within the tire on the top of Everest (altitude 300000 ft), where atmospheric pressure is 4.3 psia and the temperature is ~38°F? I Let subscript 1 indicate sea level and subscript 2 indicate altitude 30 000 ft. (Pan =14.6465.0=79.6 psia ps Vi/T = p.VlTs (79.6(V)/(460 + 69) = (p.)(V)/[460 + (—38)] (Vis constant) (Prdan= 63.5 psia —(Pa)jge = 63.5 ~ 4.3 = 59.2 psi 2.92 Find the difference in pressure between tanks A and B in Fig. 2-64 if d and d,= 230mm. 330 mm, d;= 160 mm, dy = 480mm, 1 Pa + (9.79)(0.330) ~ [(13.6)(9.79)(0.480 + 0.230sin 45") = pp pa~ py = 82.33 KPa 293A oulindrical tank contains water at a height of $5 mm, as shown in Fig. 2-65. Inside is a smaller open cylindrical tank containing cleaning fluid (6. = 0.8) at height h. If» = 13.40 kPa gage and pe= 13.42 kPa gage, what are sage pressure p, and height h of cleaning fluid? Assume thatthe cleaning fluid is prevented from moving to the top of the tank. ' Pa (9.790.055) = 13.42 pq = 12.88 kPa 12.88 + (9.79)(0.055 — fh) + [(0.8)(9.79)} = 13.40 4 = 0.00942 m= 9.42 mm Ri Orn Water 55mm +t | h mia 1 Kerosene Py Pe Fig. 2465 the tube, 234 An open tube is attached to a tank, as shown in Fig. 2-66. If the water rises to a height of 800 mu ‘what are the pressures p4 and p» ofthe air above the water? Neglect capillary effects inthe tube. i Pa — (9.79){(800 — 300 - 100)/1000}=0 p,=3.92kPa Pa~ (9.79){(800 ~ 300)/1000]=0 py =4.90KPa 52 0 CHAPTER 2 Precometer PT Pree! 2.95 For the setup shown in Fig. 2-67, what isthe pressure pif the specific gravity of the oil is 0.827 ' Pa + {(0.82)(9.79)](3) + (9.79)(4 ~ 3) ~ [(13.6)(9.79)](0.320)=0 py, =8.73 KPa 2.96 For the setup shown in Fig. 2-68, calculate the absolute 101.3 kPa, pressure at a. Assume standard atmospheric pressure, ! 101.3 + (9.79)(0.600 — 0.200) ~ [(13.6)(9.79)](0.140) + [(0.83)(9.79)](0.140 + 0.090) = px a= 88.44 KPa 2.97 A force of 460 is exerted on lever AB, as shown in Fig. 2-69. End B is connected to a piston which fits into a cylinder having a diameter of 6 mm. What force F, acts on the larger piston, ifthe volume between C and D is filed with water? I Let Fe = force exerted on smaller piston at C: Fe = (460)(%8%) = B43 N. Fe/Ac= Fo/Ap, (843)/{as8)"/4] = Fol {ex()/4], Fp = 15 830, oF 15.83 KN, 260mm —F, CHAPTER 3 Forces on Submerged Plane Areas 34 Ifa triangle of height d and base bis vertical and submerged in liquid with its vertex a te liquid surface (see Fig. 31), derive an expression forthe depth to its center of pressure. 3d : a f— b a Fig. 3-1 32 Ifa triangle of height d and base b is vertical and submerged in liquid with its vertex a distance a below the liquid surface (see Fig. 3-2), derive an expression for the depth to its center of pressure. when le +22) 4 b#18___(, e ' hema p= (03) + erauany” (3) * ae aa) $+ ad/3+ 4/9) +d° _ 64+ Bad +302 18(a-+ 24/3) a+ 24/3) a d — | bob mse 33 Ifa triangle of height d and base bis vertical and submerged in liquid with its base at the liquid surface (Gee Fig. 3.3), derive an expression for the depth to its center of pressure. 54 0 CHAPTER 3 Ld ns 3.4 A circular area of diameter dis vertical and submerged in a liquid. Its upper edge is coincident with the liquid surface (see Fig. 34). Derive an expression for the depth to its center of pressure. whe tent, m/e dd 5d ' hoo hat UA 3* Gad) 28 8 35 A vertical semicircular area of diameter d and radius is submerged and has its diameter ina liquid surface (see Fig. 3-5). Derive an expression forthe depth tits center of pressure 1x e)- ; tr {x18 ~8/08)K-) Ts) Gs) terete oo” ' hemhat jit he els 3.6 A dam 20m long retains 7 m of water, as shown in Fig. 3-6. Find the total resultant force acting on the dam and the location of the center of pressure. HF = yhA = (9.79){(0 + 7)/21{(20)(7/sin 607) = 5339 KN. The center of pressure is located at two-thirds the total water depth of 7 m, or 4.667 m below the water surface (ie., hay = 4.667 m in Fig. 3-6). FORCES ON SUBMERGED PLANE AREAS [7 55 37 A vertical, rectangular gate with water on one side is shown in Fig. 3-7. Determine the total resultant force acting on the gate and the location of the center of pressure. Fa phgA = (9.79)(3-+ 1.2/2)[2(1.2)] = 84.59 KN ah + = (3442) @a.2yn2 harhatgp= (0422) og -200 Fig. 3-7 38 Solve Prob. 3.7 by the integration method. ' Ff naa [om 925)= 09.39] +Z] =o ies yea [omersveay _09.58)19y +37? +9312 ws 59 39° A vertical, triangular gate with water on one side is shown in Fig 38. Determine the total resultant force acting ‘on the gate and the location of the center of pressure. 7 F = phegA = (62.4)(6+ 3/3){(2(3)/2] = 13101 oe 23)736 hag heat eG (6+3)+ Crs hg= = 3.633 m, 07 fe RISTO Fig. 38 86 0 CHAPTER 3 3.0 3a. aaa 33 Solve Prob. 3.9 by the integration method. ' F = phgA = [(0.82)9.79)][4-+ (1+ 1.2/2)(sin 40") (0.8)(1.2)] = 38.75 KN B= [ (624464 (2-293) ) = [(62.4)02—2y ~ 29/3) dy = (62.4129 —y?— 29/98 = 131018 fru [encrve-we [eon-o-yod F 1310 1310 (62.4) 72y = 29" = ¥' 168 - ait 7.07 ft ‘An inclined, rectangular gate with water on one side is shown in Fig. 3-9. Determine the total resultant force acting on the gate and the location of the center of pressure, 1! F = phegA = (62.4)[8 + 1(4 005 60°)]{(4)(5)] = 11 23010 ee 4), ana sereat = (Sart) * Gomer rT Fig. 39 Solve Prob. 3.11 by the integration method. rT Ff riaa~ [624y8+ycossorisy)= @ra[8y +2] = 11230 fora [eoaesyensrrsay [omers rye aa, T1230 11230 _GI2yI64y + 4y2 + yy oe =9.04 ft Note: /iy isthe vertical distance from the water surface to the center of pressure. The distance from the water surface t0 the center of pressure as measured along the inclination of the gate (z.,) would be 9.04/c0s 60°, or 18.08 ft. ‘An inclined, circular gate with water on one side is shown in Fig. 3-10. Determine the total resultant force acting on the gate and the location of the center of pressure. Ui F = thagA=(0-79)[1.5 + \(10sin 6°)][x(1.0)7/4] = 14.86 KN wreath ee DS (1.03/64 =2.260m a nar] mar cE? FORCES ON SUBMERGED PLANE AREAS J 57 Mig. 3:10 344A vertical triangular gate with water on one side is shown in Fig. 3-11. Determine the total resultant force ‘acting on the gate and the location of the center of pressure. 1 Fm phcgA = (9.79)[3 + H(1)NC-2\(1)/2] = 21.54 KN 0.290736 Brolaz@A 8" haga hat = (3+ OO Fig. 311 SAS Solve Prob. 3.14 by the integration method. I Fa yhaa, From Fig. 311, y/x = 1/12. Therefore, x=1.2y. (627990 + 1137 d7)= ['41.75)0y +7? 4p = (1.79) E42] =n. s4nN fra [omarvazay [01799 +67+y ra fem F 21.54 21.54 (= (11.75)199°/2 + 2y" + 148 ars 3.68m 58 0 CHAPTER 3 3.46 A tank containing water is shown in Fig. 3-12. Calculate the total resultant force acting on side ABCD of the ‘container and the location of the center of pressure. 5 Fr yha = (62.4){(0 + 6)/2I[(20)(6)] = 22.500 Ib ha = GX6)=4.00% (vertically below the water surface) Fig. 3:12 3.7 The gate in Fig. 3-13 is 4 ft wide, is hinged at point B, and rests against a smooth wall at A. Compute (a) the force on the gate due to seawater pressure, (6) the (horizontal) force P exerted by the wall at point A, and (©) the reaction at hinge B. 1@ F = tha = (6417 ~2){(4)(12)] = 30 106 6 oO yg Ths LOAD _ 9 5370 hed OT OOD) TMs=0 —— (PYT.2)~(30106)'12-6-0.537)=0 P= 22843 1b © De=0 B, + (30106)(32)—22843=0 — B, = 47791b DR=0 —_B,-G0106)%$)=0 By = 240851 Pat Wal Seawater y=64lb te we Fig. 3413(a) Fig. 31306) 3.18 Repeat Prob. 3.17, but instead let the hinge be at point A and let point B rest against a smooth bottom. F(a) From Prob. 3.17, F = 3010616. (6) From Prob. 3.17, y= ~0.837 46; E Ma =O; (8,)0.6) ~ (0 106)(6+0.537) = 0, B, = 205001b. © DA=0 0106) (%8)—4,=0 A, = 180641 TH=0 A, ~G0106)(48)+20500=0 4, = 3585 FORCES ON SUBMERGED PLANE AREAS J 59 ‘A tank of dye has a right-triangular panel near the bottom as shown in Fig. 3-14a. Calculate the resultant force ‘on the panel and locate its center of pressure. 1 P= thgA = pphegA = (S20)9.81)(6+ 88 + 168+ 4)] = 16.22 MN b — 2s)(hy/72 = (4-4 8)[(4 +8) — (24 + 8)|(8 + 1697/72 = -1152 m* mhysin @__ ~(-1152)(sin 30°) hed "GF 88+ 168+) Xe = +0.286m (The resultant force acts at 1.143 m down and 0.286 m to the right of the centroid.) \t bh? | Due er \i La I 4@-28) Pepe Pee Fig. 3-14(6) Gate AB in Fig. 3-15 is 1.0m long and 0.9 m wide. Calculate force F on the gate and the position X of its center of pressure. ' F = YhegA = [(0.81)(9.79)](3 + (1 + 1.0/2)(sin 50°)(0.9)(1.0)] = 29.61 kN her sin __=((0.99(1.0°/12Isin 50") hee BFC +1.0/2¥(sin SOH[O.I)C.0}} 015 m from the centroid 1.0/2 +0.015 =0.515 m from point A 60 9 CHAPTER 3 321 322 323 ‘A fishpond gate 6 ft wide and 9 ft high is hinged at the top and held closed by water pressure as shown in Fig 3-16. What horizontal force applied at the bottom of the gate is required to open it? 1 Fa yhegA = (CLAY8 + 4.5){(6(9)] = 42 12010 hy (oyna ine Gasooi” Oo" TMa=0 (PY) ~(42.120)(13.04~8)=0 P= 2358716 ag hag + Em (B44.5) +: Fig. 3:16 ‘A vat holding paint (6.g. = 0.80) is 8 m long and 4m deep and has a trapezoidal cross section 3 m wide at the bottom and 5m wide atthe top (see Fig. 3-17). Compute (a) the weight of the paint, (6) the force on the bottom of the vat, and () the force on the trapezoidal end panel 1@ W = WV =[(0.809(9.79)](8)(4)(5 + 3)/2] = 1002 KN ) Fa YhgA Fone ={(0:80)(9.79)\(4)(3)(8)] =752 KN (0) Fons Fane RF = [(0.80)(9.79)} (0+ 4)/21(4))] + (2)(0.80(9.79)] 1419/2] = 230 kN Fig. 3:17 Gate AB in Fig. 3-18 is 5 ft wide, hinged at point A, and restrained by a stop at point B. Compute the force on the stop and the components of the reaction at A if water depth h is 9 ft. ‘ F = phaA = (62.4)(9 — 3)[)(5)] = 7020 Ib Jog a8 _ SIE) ekg O= 3 DMa=0 — (BG)—(7020)(1.5+0.100)=0 B= 3744 1b DA=0 — 7020-374-A,=0 4, =32761b If gate weight is neglected, A, = 0. 0.100 FORCES ON SUBMERGED PLANE AREAS 1 61 ver Fig. :18(2) Fig. 3-18(5) In Fig. 38, stop B will break ifthe force on it reaches 9000 Ib. Find the critical water depth, = thyA= - 7 UEP /24sin HP) _ 1 Fo the = (624) heg(BVS)= 930 Yo FG eg VSN) mM, (9000)(3) ~ (936h)(1.5 + 0.750/h) =O LTB ft hg = 18.734 1.5= 20.23 In Fig. 3-18, hinge A will break if its horizontal reaction becomes equal to 80001b. Find the criti water depth. I From Prob. 3.24, F = 936, and y hg = 17.59 fhe = 1759 + 1.3 = 19.09. 0.750; E Mp = 0; (936h,.)(1.5 ~ 0.7507) ~ (8000)(3) = 0, Calculate the resultant force on triangular window ABC in Fig. 319 and locate its center of pressure. ' F = YhtagA = (10.08)[0.25 + (3)(0.60)}1(0.40)(0.60)/2} = 0.786 kN ug bh*[36 = (0.40)(0.60) /36 = 0.00240 mt ota sin 8 =(0.00240){sin 90°) Ye gA (0.25 GY(0.60)]{(0.40)(0.60)/2] 31mm (ce., below the centroid) ny = b(b ~25)(h)"/72 = 0.40f0.40 ~ (2)(0.40)](0.60)7/72 = ~0.000800 m* Hla sin =(--0.000800)(sin 90°) aA [025+ HO.60)]{(0.40)(0.60)72} a =+Homm tight of the centroid) 2c ‘y=10.08 N/m? F: 62 0 CHAPTER 3 327 328 Freshly poured concrete approximates a luid with s.g. = 2.40. Figure 3-20 shows a slab poured between wooden forms which are connected by four comer bolts A, B, C, and D. Neglecting end effects, compute the forces in the four bolts ' F = thaA = (2.40) 62.4](B)(0)12)]=97 44> sin 8 —[9)(2)"/12Ksi0 90) 20K A (eAIONaZy— ~~? TM=0 — OMAII-C704)6+2.0)=0 Rak =323481 LMc=0 (97 044)(6-2.00)—(2)(F,)(12)=0 Fy = Fy = 1617416 +1 Bin Fig. 3.20 Find the net hydrostatic force per unit width on rectangular panel AB in Fig. 3-21 and determine its line of action. ' Fo O.1)2414D1QNU)=78.32EN Fey, = (12.3641 + 912M] = 49.44 Fac Figo ~ Fae = 78.32 ~49.44-= 2858 RN son ee _al@eynaisinsoy Come ee1 +H) ~~ may = =LDANAGIN9) 5 66 (a+ DIQO1 =M,=0 ((78.32)(1 ~ 0.0833) — (49.44)(1 — 0.1667) = 28.88D D=1.059m (above point B, as shown in Fig. 3-21c) a + A we th ae cyan * ae TATOOS TTT 2 : Fig. 3:21(@), Fig. 3:21(6) Fig. 3:21(c) FORCES ON SUBMERGED PLANE AREAS 0 3.29 Acylindrical, wooden-stave barrel i 3.5 ft in diameter and 5 ft high, as shown in Fig. 3-72. It is held together by steel hoops atthe top and bottom, each with a cross section of 0.40 in’. Ifthe barre is filled with orange juice (6g. = 1.04), compute the tension stress in each hoop. ' P= yhgA = [(1.08)(62.4)\([.5)(5)] =2839 1b _rhesin OIG. 2N6in 90) 20 A F(a ST Mp0 —— 2839($— 0.833) —20Fege)(5) =O Faggr = ATID SMa 0 2Fnn)(5) ~2839(4 + 0.833) 0 Feng = 94610 Supe: =473/0.40 = 1182 psi Span = 946/0.40 = 2365 ps fe hey 2 Fy a ses 25 ome — 4t4.— : asee | TS Jerre wer ier B Fig. 3-22(a) 7 Fig. 3:22(6) 330 Gate AB in Fig. 3-230 is 16 t long and 8 ft wide. Neglecting the weight of the gate, compute the water level h for which the gate will start to fall. ' F = phegA = (62.4)(4/2){(8(h/sin 60°)] = 288.247 Iu sin @_ ~[8(h/sin 60°)°/12\(sin 60°) Yee = -0.1925h hha 2B sin 60] TMp=0 ——(11000)(16) - (288.247)[(h/sin 60°)/2-0.1925h]=0 = 117K 64 0 CHAPTER 3 3.31 Repeat Prob. 3.30, including the weight of the 2-in-thick steel (s.g. = 7.85) gate. (See Fig. 3-24.) Wye =[(7-85)(62.4)]1(16)(8)(4)] = 1045016. From Prob. 3.30, F = 288.247; Mp =0, (11.000)(16) ~ (288.245) [(h/sin 60°)/2 — 0.1925h] ~ 10 450(4# cos 60°) = 0, h = 10.7 ft. 000 te aa zoos co Fig. 3:24 3.32 A horizontal duct coming from a large dam is 2.5 m in diameter; itis closed by a circular door whose center or centroid i 45 m below the dam's water level. Compute the force on the door and locate its center of pressure. 1 P= phgA=(9.79)(45)[x(2.5)7/4)] = 2163kN = r'/4= x(38)"/4 = 1.917 mt sin _-(.917)in90°) Pe Tg ONS IAN Line of action of Fis 8.7 mm below the centroid ofthe door. '= ~0.0087 m 3.33 Gate AB in Fig. 325 is semicircular, hinged at B. What horizontal force Pis required at A for equilibrium? Farin) =(4)4)/Gx) = 1.698 F = yg = (9.79)(6 + 4— 1.658)[22(4)?/2)] = 2043 KN 8 _ -{(0.10976)(4)"](sin 90°) G+ 4—1.68)[x(4772)] (2043)(1.698-0.1347)—4P=0 P=798kN 0.1347 m = rera F Fig. 3-25(a) Fig. 3-25(6) Fig. a 334 Dam ABC in Fig. 3-26 is 38m wide and made of concrete weighing 22 kN/m°. Find the hydrostatic force on surface AB and its moment about C. Could this force tip the dam over? 3.38 FORCES ON SUBMERGED PLANE AREAS 0 65 HF = yhgA = (9.79Y$){(38)(80)] = 952371 EN. F acts at (2)(80), oF 53.33 m from A along surface AB (see Fig. 3266): For the given triangular shape, the akitude from C to AB intersects AB 5.2m from A (see Fig. 3-266). Hence, Mc = (952 371)(53.33 — 51.2) = 2028 550 KN. Since the moment of F about point C is ‘counterclockwise, there is no danger of tipping. = . a Water 20°C I oS, SZ 4a Zo) BLES: i 7 “ac k--4em Fig. 32640) BK sem tS Mig. 32616) Isosceles triangular gate AB in Fig. 3-27 is hinged at A. Compute the horizontal force P required at point B for equilibrium, neglecting the weight of the gate. FAB =3/sin 6 =3.464m P= hag =[(0.82)(9.79) 2+ 1.00){(1.2(3.464)/2] = 50.05 kN DMy=0 — 3P~($0.05).464/3-+0.1924) =0 P= 22.47 EN Fig. 3-270) Fig. 3-2706) 336 The tank in Fig. 3-28 is 40 em wide. Compute the hydrostatic forces on horizontal panels BC and AD. Neglect atmospheric pressure. ! P= th pac =[(0.84)(9.79)](0.35 + 0.40) + (9.79)(0.25) = 8.615 kPa F=pA —— Fyc=(8.615){(1.20)(0.40)] = 4.135 kN Pav = ((0.84)(9.79)|(0.40) = 3.289 kPa Fy = (3.289)[(0.55)(0.40)] = 0.724 KN 66 J CHAPTER 3 wf | ; ali alae “1 FHt0em jabem Solvent sgs086 water abe -+-———120cm—_+4° Fig. 3:28 337 Water in a tank is pressurized to 85 cmHg (Fig. 3-29). Determine the hydrostatic force per meter width on panel AB. Hn panel AB, pag ={(13.6)9.79 (0.85) + (.79)(6-+ 9) = 167.0 kPa, Fay = (167.0){(3)(1] = 501 kN. Fig. 3:29 338 Calculate the force and center of pressure on one side of the vertical triangular panel ABC in Fig. 3-30. i F= yh = (62.4)(1 + 6){(9)(6)/2] = 11794 Ib Tog = (6)(9)°/36 = 121.5 ft Jasin @ sin90)_ Jer RA "d+ H1@KOr ~ O°" ASO iggy gw ria, ~(-40 5650507) by Th en oe heA (1+ 6)f(9)(6)/2} ae ‘Thus, the center of pressure is 6 + 0.64, or 6.64 ft below point A and 2 +0.21, or 2.21 ft to the right of point B. 339 Fig. 331, gat AB is 4m wide andis connected bya rod and pulley toa massive sphere (6g. =2.40). What is the alles radius that wl Keep the gate closed? ’ Fa thgA = (.79)9-+ 31003) = 12344 (3)N2H5iN 7) 6 om ora@ay 7° ST Me=0 —— (Weyaee)(7+943)—(1234)3—1.5-0.071) =O Weynre™ 92.8 KN Woynace ™ Y(400°/3) 92.8 = [(2.40)(9.79) (4277/3) =0.98m 340 FORCES ON SUBMERGED PLANE AREAS J 67 ate : NN Fon 44° Fig. 3302) Fig. 33006) Spherical Fig. 331 ‘The triangular trough in Fig. 332i hinged at A and held together by cable BC at the top. If cable spacing is ‘1m into the paper, what is the cable tension? 7 F = phe = 0-1)(NB-TTD] = 213.3 N he sin 8 -[Q8.7177/124in 35%) re hgA HEA) IM=0 (TY + 5) — (213.3)(4.359 - 1.453)=0 T= 88.5KN = 1.453 m. Fig. 3:32(0) 68 0 CHAPTER 3 341 InFig. 333, gate Ais Aft wide and opens to lt fresh water out when the ocean tide is falling. The hinge at A is3ft above the freshwater surface. At what ocean depth will the gate open? Neglect the gate's weight. Bo Fa yhgA R= (2ANCRICN)]= 179 —_ = [C.025)62.4)}4/2914)0)] = 127.944 TMe=0 — (1279KIUZ+3-H/3)-(1797G+8)=0 ha LS 4 BFF ye ag [SSP 12 L Fr fo ait oe Fig. 333(0) Fig. 3:33(0) 3.42 Show that depth h in Prob. 3.41 independent of gate width b (perpendicular to the paper). I Areas, and hence pressure forces, are directly proportional to b. Thus b will cancel out of the equation ¥ My =0 that determines fh, 343 Compute the force on one side of parabolic panel ABC in Fig. 3-34 and the vertical distance down to the center of pressure. ! F = yhegA = (9.79)(1 + 6)[()(10)(6)] = 2741 kN, = A(AAY = (OH?) ~ [OHIO = GXOAOY ~ [XG IGA) = 274.3 m* fa sin 8 _ ~(274.3)(sin 90°) 20 hg T+ HNEIGOVE)) Hence, the center of pressure is 6 + 0,980, or 6.980 m below point A. 0.980 m, Parabola Fig. 3:34(0) Fig. 3:34(6) FORCES ON SUBMERGED PLANE AREAS J 69 344 Circular gate ABC in Fig. 3-35 is 4m in diameter and is hinged at B. Compute the force P just sufficient to keep the gate from opening when h is 8 m. 1 F = YhcgA = (9.79)(8){(4)*/4] = 984.2 kN d*/64 = x(4)'/64= 12.57 m* 257609) o5m A @le@maT Os DMa=0 (PY) (984.2)(0.125)=0 P= 61.SKN Yom Water h ave ie —_ 2m 2m cer F = | P Fig. 335(@) are Fig. 3:35(6) 345 For the conditions given in Prob. 3.44, derive an analytical expression for P as a function of h. i F=yhgA= phala(r)] Ing = mr /4 yup e888, =U P14 sin 907) _ =P? ena MNT TMe=0 Pr [hal aMFMOP MAN] = 0 P= yas (Note that force P is independent of depth h.) 346 Gate ABC in Fig. 3-36 is 2m square and hinged at B. How large must h be for the gate to open? The gate will open when resutant force Facts above point B—i.., when |yq1 <0.2m. (Note in Fig. 3:36 that yy is the distance between and the centroid of gate ABC.) _ Tha sin 8 _ ~[2(2)°/12\(sin 907) _ -1.333, ThA FLOOR) hes For |p| <0.2,1.333/(4h + 4) <0.2, h > 0.666%. (Note that this result is independent of Nuid weight.) rT sett am He te abn ee F Fig. 3-36(6) Yor Fig. 3:36(0) 70 0 CHAPTER 3 347 — Gate AB in Fig. 3-37 is 6 ft wide and weighs 2000 Ib when submerged. It is hinged at B and rests against a ‘smooth wall at A. Determine the water level h which will just cause the gate to open, Fo Fe thgA R=Mh+ DIINO] =37Hh-+ 14976 = 62.45 + H[(10)(6)] = 33.6961 clasin@ Gy _ fOr) _ ~67 Ihe te GOO" BHA _rlouoyn2y) : Ooe= “Se HG] ~~O7E DMs (B144h + 14976)[5 ~ 6.67/(h + 4)] ~ (33 696)(5 — 0.741) ~ (20003) "0h =5.32ft Fig. 33710) Fig. 33700) 3.48 The tank in Fig. 3.38 contains oil and water as shown. Find the resultant force on side ABC, which is 4 ft wide. ' F=YhgA Fan = [(0-80)(62.4)](F)1(10)(4)] = 9980 Ib Fp acts ata point (3)(10), or 6.67 t below point A. Water is acting on area BC, and any superimposed liquid ‘can be converted to an equivalent depth of water. Employ an imaginary water surface (IWS) for this calculation, locating IWS by changing 10 ft of oil to (0.80)(10), or 8 ft of water. Thus, Fac = (62.4)(8 + §){(6)(4)] = 16 470 Ib. a ahesin® _ [4NO"N246sin 90) peek NOC gece Fac acts at a point (248 + § +0.27), of 13.27 ft below A. 5: My =0; (9980 + 16 470)( lt.) — (9980)(6.67) — (16 470)(13.27)=0, iy = 10.78 ft from A. Thus, the total resultant force on side ABC is 9980 + 16470, or 26.450 Ib acting 10.78 ft below A. (ie., below the centroid of BC) 349 . a1 ase FORCES ON SUBMERGED PLANE AREAS 0 71 Gate AB in Fig 339s 4ft wide and hinged at A. Gage G reads ~2.17 psi, while oil (5.2. = 0.75) is in the right tank, What horizontal force must be applied at B for equilibrium of gate AB? ' Fa tha Fu {(0.75)(62.4) (){(6K8)} = 337010 Fa acts @)(6), oF 4.0ft from A. For the left sie, the negative pressure due to the air can be converted to its equivalent head in feet of water. h = p/y = (—2.17)(144)/62.4 = ~5.01 ft. This negative pressure head is. equivalent to having 5.01 ft less water above A. Hence, Fy.o = (62.4)(6.99 + $){(6)(4)] = 14 960 Ib. ta (6.99 + S{(6)(4)I Fiyo acts at (0.30-+ §), oF 3.30. below A. 5 Ma =O; (3370)(4.0) + OF ~ (14 960)(3.30) = 0, F'= 5980 1 (acting leftward) Jer = 0.308 AA vertical circular disk 1.1 m in diameter has its highest point 0.4m below the surface of a pond. Find the ‘magnitude of the hydrostatic force on one side and the depth to the center of pressure. ' F= phgA=(0.79)(0.4-+ 1.1/2) a)(L1)/4] = 8.84 KN Se Ln (21.1) 164 at ii” (04+) THOTT = 1.03m ‘The vertical plate shown in Fig. 3-40 is submerged in vinegar (s.g. = 0.80). Find the magnitude of the hydrostatic foree on one side and the depth to the center of pressure. : nee (0.80)(9.79)](2 + 3){(3)(7)1 = 905 KN hat Saas = ant rg a (0.80)69.79)][2-+ 3+ 4/2124] = 439 KN (ayn ue: een 2434401 Say” om = 905 +439= SHEEN 1344.5 = (005)(6.24) + (439)(7.19) hag = 655m surtace 4m Fig. 3-40 ‘The irrigation head gate shown in Fig. 3-41a is a plate which slides over the opening to a culvert. The coefficient of friction between the gate and its sliding ways is 0.5. Find the force required to slide open this 1000-1b gate ifit is set (a) vertically and (6) on a 2:1 slope (n= 2 in Fig, 3-41a), as is common, 72 0 CHAPTER 3 354 HW (@) F = phegA = (62.4)[14 + (f8)/21[(1)(€8)] = 25 740 Ib. Let T = force parallel to gate required to open it. YE =O; T — 1000 —(0.5)(25 740) =O, T = 13870 b. (6) See Fig. 3415. F= (62.4){14 + 8(1/V3)/2112)()] = 23 $84 Ib. Let N = total force normal to gate; N= 23 584 + (1000)(2/V5) = 24478 Ib. EF, = 0; T — (1000)(1/V5) — (0.5)(24 478) = 0, T = 12.686 1b. “ Fig. 341(6) {A 65-i-square floodgate, weighing 2200 Ib, is hinged 44.5in above the center, as shown in Fig. 3-42, and the {ace is inclined 5° to the vertical. Find the depth to which water will ise behind the gate before it will open. ' Closing moment of gate about hinge = (2200) (*(sin 52] = 71 Ib f F = tha A= (62.4) (h/2M (8)(5)lc08 5" = 169.64 TMi 0 (169.642) (65 + 129/12 - Choos St/3]-T11=0 = 0.8264 "2 Fig. 3-42 Gate MN in Fig. 343 rotates about an axis through N, If the with ofthe gate is 5 ft, what torque applied tothe shaft through IN is required to hold the gate closed? Fo Pa thgA — R=OA6-+G+4)/211G+4)(5)]= 207481 = (62.4)(4)[(5)(4)] = 24961 (5)(3-+ 4)*/12}(sin 90°) _ wea OMG GrHMTG+ HE] Oe Facts at (3)(4), or 1.333 ft from N. E My = 0; (20 748)(3 + 4/2 ~ 0.430] —(2496)(1.333) — torque = 0, torguey = 60 369 1b ft achasin 3.56 FORCES ON SUBMERGED PLANE AREAS 0 73 Find the minimam depth of z for which the gate in Fig. 344 will open, ifthe gate i (a) square and (6 isosceles triangular, with base = height. 1@ F=yhgA Fugo = (62.4\(z ~ 33)(3)] = (561.6)(z ~ 1.5) ‘asin @ 3)(3)"/12\(sin 90°) _ -0.750 fA OMMO~"G_ HEY F-15 Moment due to water = [(561.6)(z — 1.5)][3 + 0.750/(z — 1.5)] = (561.6)(1.5z — 1.500) Free pA=[S)[(3X9) = 480%, Fr aca or 15 blow ge, Moment eto gas (680)(.5)= Yo= ‘7201 - ft. Equating moments gives (561-6)(1.52 — 1.500) = 9720, z = 12.54 ft. ) Fao = (62.4)fz — (@)M@)13)/21 = (280.8)(2 — 2.000) [3)(3)"/36)(6in 907) __ 0.500 Ohio GYAIGYB)/A] z= 2.000 ‘Moment due to water = (280.8)(2 ~ 2.000)][3 + 0.500/(z — 2.000)] = 280.82 — 421.2 Fone ((5)(144)]{)(3)/2] = 3240 Ib. Fy, acts at 3, or 1.000 ft below hinge. Moment due to gas = (4240)(1.000) = 3240 Ib - ft. Equating moments gives (280.82 ~ 421.2)= 3240, z = 13.06 ft et frsven Fig. 344 ‘The triangular gate CDE in Fig. 3-45 is hinged along CD and is opened by a normal force P applied at E. It holds a liquid of specific gravity 0.82 above it and is open to the atmosphere on its lower side. Neglecting the ‘weight of the gate, find (a) the magnitude of force exerted on the gate, by direct integration; (6) the location of the center of pressure; and (¢) the force P needed to open the gate. H (@ F=f yhda=f yly sin ed). When y linearly with y. Hence, =y~8. When y = 14, x with y- Hence, x=20~y. x =0, and when y = 8+ %, or 14, x =6, with x varying ‘and when y =8 +12, or 20, x =0, with x varying linearly F -[ [(0.82)(62.4)](y sin 30)[(y — 8) dy] + [v .82)(62.4)}(y sin 30°)[(20 ~ y) dy] ~102ny62.(6nan{[2-07]"+ [197-2 ]}-a0n sengin® Sie y= 0,290 ah sin 8 __—{2(6)(F)*/12\(sin 30°) Me pg [E+ BY sin IP 12)(G)/2} (he resue emeris048 flo te ened eased inte pln of he). 0.43 tt © TMeo=0 OP = (128948) P= 429816 74 0 CHAPTER 3 3.57 Determine the force acting on one side of vertical surface OACO in Fig. 3-46 and the location of the center of pressure, if y= 8.4 N/m". The curved edge isan arc of the parabola y = 27/8. 1 Fafaa~[enoneed)= f (8A) 2VH) dy = [ss dy = [19.01y"™],= 19.01 kN Joraa [enoyera a 19.01 3.58 Find the force exerted by water on one side of the vertical annular disk shown in Fig. 3-47. Also locate the center of pressure. a F = hagA = (9.79)3) (a) (1) ~ (7) (88) = 59.05 KN og = ()(1)*/4 — (9) f88)*/4 = 0.6836 m* gt 3 zm her hat ia? aye aay ot? 39 361 FORCES ON SUBMERGED PLANE AREAS 0 75 Determine y in Fig. 348s thatthe lashboards will tumble only when the water reaches ther top. 1 The fashboards will tumble when y is atthe center of pressure. Hence, y =, or 1.333 m. 7 Fig. 3.88 Determine the pivot location y of the square gate in Fig, 3-49 so that it will rotate open when the liquid surface isas shown. The gate will open when the pivot location is at the center of pressure 1 (wey hamhatpg~O-D+G! Bee 21-23-2167 0.853m Fig. 349 ‘The gate in Fig. 3-50a (shown in raised position) weighs 350 Ib for each foot normal to the paper. Its center of sravity is 1.5 ft from the left face and 2.0 ft above the lower face. For what water level below the hinge at 0 does the gate just begin to swing up (rotate counterclockwise)? I Refer to Fig. 3-50b and consider 1 ft of length. F = yh = (62.4){(ho/2)]{(ho)(1)] = 31.2h3; E Mo=0; (2)G50) = (5 = ho/3)(31.2K8) = 0, ho = 2.30 ft. Fig. 350(0) Fig. 3-50(6) 76 0 CHAPTER 3 3.62 For the gate described in Prob. 3.61 and Fig. 3-50a, find h for the gate just to come up to the vertical position shown in Fig. 3-50a I See Fig. 351, F.= yh = (62.4)(H)|(SY(1)] = 312h, Fe = (62.4)(4/21(4)()] = 31.28%; E Mo = 0; (1.5)650) + (1/3)(12H") ~ (25)(3124) =0, h= 0.686. Fig. 351 3.63 For the gate described in Prob. 3.61 and Fig. 3-50a, find h and the force against the stop when this force is a ‘maximum forthe gate. I See Fig. 351. F, = yhA = (62.4)(H)|(5)(1)] =3124, (1.5)(350) + (4/3)(31.2h°) — 2.5) 12K) + 5) Fy) oa Pa - Seta 156-6244 =0 hh = 5.008 Fay = (156)(5.00) ~ (2.08)(5.00)' ~ 105 = 41516 62.4){h/2)|(H)(1)] = 31.24"; E Mo =0; , Fay = 156h ~ 2.08h° — 105. 3.64 Compute the air pressure required to keep the gate of Fig. 3-52 closed. The gate isa circular plate of diameter 0.8m and weight 2.0 KN. ' Faya f, (2)(9.79)]L1.7 + (3)(0.8)(sin 45°)If-x(0.8)"/4] = 19.52 kN gf AT (t al0.8)'/4 wamee weet Leas 2)09] ira crONTROHTA 2 ZL Mine =O (19.52)(2.818 ~ 1.7/e08 45°) + 2.0[(4)(0.8)(cos 45°) ~ [2(0.8)*/4](pax{(4)(0-8)] = 0 Par™ 42.99 kPa an CHAPTER 4 Dams In Fig. 41, calculate the width of concrete dam that is necessary to prevent the dam from sliding. The specific weight of the concrete is 1501b/ft’, and the coefficient of friction between the base of the dam and the ‘foundation is 0.42. Use 1.5 as the factor of safety (F.S.) against sliding. Will it also be safe against overturning? I Working with a 1-f “slice” (.e., dimension perpendicular to the paper) of the dam, Waa = (20)(w)(1)(150) = 3000w, F = yhA, Fi =" sliding force "7020 (3000)(8.36)|(8.36/2) coon) {otal righting moment ‘overturning moment FS ring ‘Therefore, it should be safe against overturning. Figure 42 is the cross section of an earthwork (s.g. =2.5) dam. Assuming that hydrostatic uplift varies linearly from one-half the hydrostatic head at the upstream edge of the dam to zero at the downstream edge, find the ‘maximum and minimum pressure intensity in the base of the dam. ! Fa yhA — Fy= (624 (0+ 99 /21(G7(0)] = 293 561 10 For equilibrium, R, = 293 561 Ib. W, = [2.5)(62.4]{(1)(10}(90 + 30)]= 187 200 Ws = [(2.5)(62.4)]{(1)(60)(90)/2] = 421 20010 Fy =((62.4)(48.5+0)/2}(60-+ 10)41)] = 1059241 _R, = 187200-+ 421 200 — 105924 = 502 47610 ST Mo= 0 (293:561)32.33) + (187 200)(5) + (421 200)(30) — (108 924)[(60 + 10)/3] — 502-4762 = 0 = 40.98 ft Eccentricity = 40.98 ~ (60 + 10)/2 = 5.98 ft ‘Since the eocentricity is less than one-sixth the base of the dam, the resultant acts within the middle third of the 502.476, {(502.476)(5.98)](60 + 10)/2 B+ring* —aeoriyr — *0=778*3679 Pane ® 7178+ 3679 = 108571b/f? Pain = 7178 — 3679 = 3499 1b/ft? 78 0 CHAPTER 4 43 Fig. 42 For linear stress variation over the base of the dam of Fig. 43a, find where the resultant crosses the base and compute the maximum and minimum pressure intensity at the base. Neglect hydrostatic uplift. I Figure 43b shows the forces acting on the dam. F,= y{(19 + 6)/21[(19 + 6)(1)] = 312y, i= W@G)(1)] = 18y, Fi= yi(1)(19)(3)/2] = 28.5y, Fe= [(2.5)(YI419 + 6)(1)] = 2507, Fs = [(2.5\(y)MAI19)(3)/2] = 71.257, (2-5\(y)I{A)A9)(11)/2} = 261; R, = 18y + 28.5y + 290y + 71.25y + 26ly = 628.75y. E My = 0; (€28.75y)(x) ~ B12y)I(9 + 6)/3] ~ (187)(-5) — 28.5y)(1) — (2507) + 2) ~ (71.257) ~ 1) ~ Qby}(4 +3 +4) x = 10.87 m. Eocentrcity = 10.87 ~ (11 + 4+ 3)/2= 1.87 ft. Since the eccentricity is ess than one-sixth the base fof the dam, the resultant acts within the middle third of the base. E Mx, Mey _ (628.75)9.79) , [(628.75)(9.79)(1.87](1-+4+3)/2 4 9 At tT U+44 (G1 + 443)712 eee Poa =342+213=555KP2 aug = 342-213 = 129 kPa 4s DAMS 0 79 For the conditions given in Prob. 4.3 with the addition that hydrostatic uplift varies linearly from 19m at A to zero at the toe of the dam, would the resultant still act within the middle third of the base? F Fp= 119 + 0/2443 Uy] =1Tly R= 18y-+28.5y + 2507 + 71.25y + 261y — ITY = 457.757 LMy=0 (457.75 y)(x)~ G12y)[(19 + 69/3] — (1By)(1.5) ~ 8.57) (1) — (2507) +2) ~ (71.28y)(3 - 1) - Q617)(44+3-+ 4) + ATL) +3 +11)/3]=0 x =12.68m Eccentricity = 12.68 — (11 + 4 + 3)/2 = 3.68 ft Since the eccentricity is greater than one-sixth the base of the dam, the resultant acts outside the middle third of the base, ‘A-concrete dam retaining water is shown in Fig 44a. If the specific weight of the concrete is 150 b/f®, find the factor of safety against sliding, the factor of safety against overturning, and the pressure intensity on the base. ‘Assume the foundation sol is impermeable and that the coefcient of friction between dam and foundation soil isos. I. The forces acting on the dam are shown in Fig. 4.4b, F = yh, F, = (62.4)[(0-+ 42)/2I{(42)(1)] = 55 0401, From Fig, 446, CD/42= 8, CD = 8.40 ft; F, = (62.4)(8.40)(42)/2](1) = 11 01016. righting moment ‘moment arm about toe, B component | weight of component (kips) from toe, B (ft) (ip) 1 (10x 50)(0.1) = 37.50 204+ = 23.33 875, 2 (10 x 50)(0.15)(1) = 75.00 10+ = 15.00 12s 3 (10 x 50)0.15)(1) = 37.50 @a0 = 6.67 250 5 01 30~ (8-40) = 27.20 299 Y V= 161.01 hips EM, =2549 kip ft (0.45)(161.01 Morrsnine = (55.04)($) = 771 kip-ft 1.32 FSi = Syerturning moment ~ 771 $5.04 kips and R, = 5, V = 161.01 kips; hence, R = 55.04" + 161.01 = 170.16 kips. My _T.M,~My_2549. REV 6LOL 1.08 ft Eccentricity = —11.04= 3.96 t Since the eccentricity is less than one-sixth the base of the dam, the resultant acts within the middle third of the base. F Mx, Moy _ 161.01 , [(161.01)0.96))45) ag Ze (30)(1) (@GoyP/12 374425 =9.62kips/f —py=5.37—4.25-—=1.12kips/tt $3724.25 Pi ‘The complete pressure distribution on the base of the dam is given in Fig. 4-4c. 46 CHAPTER 4 Foationtioa! Fig. 44(0) jp, = 16101 se | +n 4 Mig. 446) ‘A concrete dam retaining water is shown in Fig. 4Sa. Ifthe specific weight ofthe concrete is 3.5 KN/m’, find the factor of safety against siding, the factor of safety against overturning, and the pressure intensity on the base. Assume there isa hydrostatic uplift that varies uniformly from full hydrostatic head atthe heel of the dam to zero atthe toe and that the coeficient of friction between dam and foundation soil is 0.45. 1 The foes acting on te dam are shown in Fig. 5b, F= yh, B= (9-9)0+ W/2TG4N | = 859.44, = (9.79)[(3)(14 — 3)(1)] = 323.1 KN. Hydrostatic uplift varies from (14)(9. .79), or 137.1 KN/m_ at the heel to nar the toe, as shown in Fig. 4-56. F, = (137.1/2)(15)(1) = 1028 KN. It acts at (})(15), or 5.0m from point A, a shown in Fig 45. DAMS 0 a1 ‘righting moment ‘momeat arm about toe, B ‘component ‘weight of component (KN) trom toe, B (mm) aN: 1 (as—3— 9122351) =1128 | G)GS-3-4)= 5.333 6016 2 (402 +3)(23.5)(0) = 1410 (15—3—$)= 10.000 14100 3 (15)@)@3.5)(1) = 1058 B= 7.500 1935 B = 33 (15-4) =13.500 4360 EV =3919kN Marans = (959.4) $) + (1028)(10) = 14.760 KN _stiding resistance _(0.45)(3919 ~ 1028) _ FS ae St 1.36 20 a __total righting moment _ 32411 ‘overturning moment ~ 14760 = 959.44N and R, = 5; V — Fy = 3919 — 1028 = 2891 KN; hence, R = VO50.4+ 2897 = 3046 KN. EM _¥M,~Mo_32411 ~ 14760_ eer Ln eS Fa = 6.105m Eccentricity = ¥ —6.105 =1.395 m ‘Since the eccentricity is less than one-sixth the base of the dam, the resultant acts within the middle third of the ‘ase. Fg Ma, Mey 2691 (891)0.395)108) , pn eM ON IE 40m 190.7 21075 Pa = 192.7 + 107.5 = 300.2 KN/m? ‘Pa = 192.7 ~ 107.5 = 85.2 KN/m* ‘The complete pressure distribution on the base of the dam is given in Fig. 4c. Mig. 4-5(a) 82 2 CHAPTER 4 Fig. 45(6) | a | . cea | I 3002KN jm _—_ +1395 m R= 291AN Fig. 45(0) 4.7 A concrete dam retaining water is shown in Fig. 4-6a. Ifthe specific weight of the concrete is 23.5 kN/m*, find the factor of safety against sliding, the factor of safety against overturning, and the maximum and minimum pressure intensity on the base. Assume there is no hydrostatic uplift and that the coefficient of friction between ‘dam and foundation soi is 0.48, I The forces acting on the dam are shown in Fig. 46. F = yA, Fy = (9.79)(0-+ 6)/2](6)(1)] = 176.28N. righting moment ‘moment arm from toe, sbont toe, component | weight of component (KN) A (a) AGN: m) 1 DOMES.) = 164.5 QQ) =1.333 219 2 ‘@)G3.5) = 329.0 24 $= 3.000 987 EV =493.54N DM.= 1206KN-m DAMS 0 83 Merearig = (176.2)(§) = 352.4 KN FS scan, dling resistance _(0.48)(493.5) _ 44 ‘Seta siding force 1762 otal righting moment _ 1206 FS oumaniet” overturning moment 352.47 >? Ry = Fy = 176-2KN and R, = 5, V = 493.5 EN; hence, R= VITEP TGA = SUN. Eccentricity = $- 1.730=0.270m EM, EM. My_ 1206 ~ 382.4 FR EY ages 730m Since the eccentricity is es than one-sixth the base of the dam, the resultant acts within the middle third ofthe base, Mes 5 iB 493.5, [(493. (0.2701) 4 9 173.44 50.0 oo* owe Pain = 123.4 ~50,0= 73.4 KN/n? ke i Pra 123.4 + 50.0= 173.4 N/m? Powe = 123.4 Fig. 4-6(@) Fig. 46(6) 84 0 CHAPTER 4 48 For the dam shown in Fig. 4-7, what is the minimum width b for the base of a dam 100 ft high if hydrostatic ‘uplift is assumed to vary uniformly from full hydrostatic head at the heel to zero at the toe, and also assuming ‘an ice thrust P, of 12 480 Ib per linear foot of dam at the top? For this study, make the resultant of the reacting {forces cut the base at the downstream edge of the middle third of the base (e., at O in Fig, 4-7) and take the weight of the masonry as 2.50y. HF yha Fy =(62.4)(100-+ 0)/21(100)1)] = 312.0001 Fy = [(100)(62.4)/2]{(1)(b)] = 31200 = [@50)(62.4)}(20)(100)(1)]= 312.0001 w= (2.50)(62.4)]{(b — 20)(100)(1)/2} = 78006 ~ 156.000 TMo=0 (312,000)(42) + (31206)(b/3) — (312.000){(4)(b) ~ #] ~ (78006 — 156 000){(3)(6 — 20) ~ 6/3] + (12.480)(100) = 0 36*+ 1006 -24400=0 b= 75.0 Fig. 47 1 [] CHAPTER 5 Forces on Submerged Curved Areas ‘The submerged, curved surface AB in Fig. 5-1a is one-quarter ofa circle of radius 4 ft. The tank’s length (Gistance perpendicular to the plane of the figure) is 6 ft. Find the horizontal and vertical components of the {otal resultant force acting on the curved surface and their locations. I The horizontal component ofthe total resultant force acting on the curved surface is equal to the total resultant force, Fy, acting on the vertical projection of curved surface AB (i.c., BF in Fig 5-1b). This projection is a rectangle 6 ft long and 4 ft high. For the portion of Fi resulting from horizontal pressure of BHEF in Fig. 5-1b, pi = (8)(62.4) = 499 lb/ft, A = (6)(4) = 24 ft, F, = (499)(24) = 11 980 Ib. For the portion of Fy resulting from horizontal pressure of HGE in Fig $-1b, p. = (62.4)(0 + 4)/2] = 125 Ib/t, F; = (125)(24) = 3000 Ib; Fu= K+ Fy= 11 980 + 3000 = 14 980 Ib. The vertical component of the total resultant force acting on the curved surface is equal to the weight of the volume of water vertically above curved sutface AB. This volume consists of a rectangular area (AFCD in Fig. 5-1c) 4ft by 8{t and a quarter-circular area (ABF in Fig. 5-1c) of radius 4ft, both areas being 6 ft long. This volume (V) is V = [(4)(8) + ()(4)"/4](6) = 267.4, Fy = weight of water in V = (267.4)(62.4) = 16 650 I. The location of the horizontal component (F,)is along a (horizontal) line through the center of pressure for the vertical projection (ie., the center of gravity of EFBG in Fig. 5-16). This can be determined by equating the sum of the moments of F, and F, about point C to the moment of Fy about the same point. (11 980)(8 + $) + (3000)[8 + (3)(4)] = 14 980h., f= 10.13 ft. (This is the depth from the water surface to the location ofthe horizontal component. Stated another way, the horizontal component acts at a distance of 12 - 10.13, or 1.87 ft above point B in Fig, 5-16.) The location ofthe vertical component (F,) is Fig. 5-1(@) iy st Fig. 5-106) Fig. $100) 86 J CHAPTER 5 52 53 along a (vertical) line through the center of gravity of the liquid volume vertically above surface AB (i.e, the center of gravity of ABCD in Fig, -1c). This can be determined by referring to Fig. 5-1e and equating the sum of the moments ofthe rectangular area (AFCD in Fig, 5-le) and of the quarter-circuar area (ABF in Fig 5-1c) about a vertical line through point B to the moment of the total area about the same line. (x)[(8)(4) + (YC) 41 = (BNC) + (62) /4I14))/GH)], x =1.91 fe. (This is the distance from point B to the line of ‘action of the vertical component.) Solve Prob. 5.1 for the same given conditions except that water is on the other side of curved surface AB, as shown in Fig. 5-2. I If necessary, refer to the solution of Prob. 5.1 for a more detailed explanation of the general procedure for solving this type of problem. p = Pan = (y){ (Ht, + h)/2] = (62.4){(8-+ 12)/2] = 626 Ib/tt2, A = (6)(4) = 248, Fir = pA = (624)(24) = 14980 Ib. The vertical component (F,) is equal to the weight of the imaginary volume of water vertically above surface AB. Hence, Ry = {(4)(8) + (2)(4)'/4](6)(62.4) = 16 690 Ib. The location of the horizontal component is 10.13 ft below the water surface (same as i Prob. 5.1 except that Fy acts toward the left). The location ofthe vertical component is 1.91 ft from point B (same as in Prob. 5.1 except that Fy acts upward). Fig. 5-2 ‘The submerged sector gate AB shown in Fig. 5-3a is one-sixth ofa circle of radius 6 m. The length of the gate is 10m. Determine the amount and location of the horizontal and vertical components ofthe total resultant force acting on the gate. I If necessary, refer to the solution of Prob. 5.1 for a more detailed explanation ofthe general procedure for solving this type of problem. Refer to Fig. 536. Fy= yhA = (9.79)[(0 + 5.196)/2]{(10)(5.196)} = 1322 KN, ‘Areasnc = @fCAacap + af€8pp0 ~ aF€8 ano = (5.196)(3) + (3.000)(5.196)/2 ~ (7)(6)/6= 4.532 m, Fy = (areasac)(length of gate)(y) = (4.532)(10)(9.79) = 444 KN. The location of the horizontal component (F,) is along a (horizontal) line 5.196/3, or 1.732m above the bottom of the gate (A). The location of the vertical component (F,) is along a (vertical) line through the center of gravity of section ABC. Taking area moments shout AC, 4552 = (5196) + (2) 005 196)13 + 3.000) — (6/616 — fon (O2ICNO)=), Fig. 5-310), FORCES ON SUBMERGED CURVED AREAS J 87 6 sin 60° = 5.196 m 6 cos 60° = 3.000 a Fig. 5-3(6) 54 The curved surface AB shown in Fig. 5-4ais a quarter of a circle of radius 5 ft. Determine, for an &-ft length perpendicular to the paper, the amount and location of the horizontal and vertical components of the total resultant force acting on surface AB. I ifnecessary, refer to the solution of Prob. 5.1 for a more detailed explanation of the general procedure for solving this type of problem. Refer to Fig. 4b, Fy = yh = (62.4)[(0+ 5)/2]{(5)(8)] = 6240 Ib, ateaany = ated gorp ~ aFedanc = (5)(5) — ()(5)*/4= 5.365 ft, Fy = (areaano)(length)(y) = (5.365)(8)(62.4) = 2678 Ib. Fyis located at §, or 1.67 ft above C. Fy is located atx from line AD. 5.365x = [(5)(5)](2) ~ [GEV VAIS = SGA], x = 1.126. . 7 = | [—* —s Fig. S-4(a) S$ Determine the value and location of the horizontal and vertical components of the force due to water acting on curved surface AB in Fig, 5-5, per foot of its length. I Afnecessary, refer to the solution of Prob. 5.1 for a more detailed explanation of the general procedure for solving tis type of problem. E, = yh = (62.4)[(0-+ 6)/21(6)(1)]= 1123, F, = (area)(length)(y) = [()(6)°/4)(4)(62.4) = 1764 1b. Fy is located at (3)(6), or 4.00 t below C. F, is located at the center of gravity of area ABC, or distance x from line CB. x = 4r/(3x) = (4)(6)/(3x) =2.55 ft. $6 The 6-ft-diameter cylinder in Fig. 5-6 weighs 5000 Ib and is 5 ft long. Determine the reactions at A and B, neglecting friction. Fig. 55 4 The reaction at is due to the horizontal component of the liquid fore acting onthe cylinder (Fy). y= kA = [(0.800)(62.4)](0 + 6)/2](3 + 3)(5)] = 493 Ib. Fy acts to the right; hence, the reaction at A is +493 Ib to the left. The reaction at B isthe algebraic sum of the weight ofthe cinder and the net vertical component ofthe force due tothe liquid. (F)up™= (areaeconoe)length)(7), (F)aem™= (ate@ece(lensth)(), Foden Fidan = (Fraren = (ate conn length) (7) = (4) /21.5|(0.800)(62.4)} = 3529 1b (upward). The reaction at Bis S000 ~3529, or 1471 Ib upward FORCES ON SUBMERGED CURVED AREAS J 89 S17 Referring to Fig. 5-7, determine the horizontal and vertical forces due to the water acting on the cylinder per foot ofits length. (Bidcon = 62.4 {(4 + (4+ 4.24 + 0,88))/2}((2.12 + 3)(1)] = 2006 1b (Fiddan = (62.4){((4 + 4.24) + (4+ 4.24 + 0.88)]/2}((0.88)(1)] = 477 Ib (Ferdea = (Fe)coa — Fudan = 2096 — 477 = 1619 ib (right) (Bua = Fionn ~ (Fr)oc = weight of volume aren — weight of Volume necro = weight of volume asraco = weight of (rectangleame+ trianglecrs + semicirclecnas) = 62.4(4)(4.24) + (4.24)(4.24)/2-+ ()(3)*/2]() = 2501 Tb (upward) Fig. 5-7 In Fig. 5-8, an 8-t-diameter cylinder plugs a rectangular hole in a tank that is 3 ft long. With what force is the cylinder pressed against the bottom of the tank due to the 9-ft depth of water? 8 Fide = (Fy)eve ~ (Frdea (ne = 2.404 +4)(1) ~ (a V7/210) — GAL THO.54) + (BMV) - 2VG.46)/21) ~~ 62.41(7)0.54) + (4)(=)(4)? ~ (2(3.46)/2]() = 4000 b (downward) In Fig. 5.9, the 8t-diameter cylinder weighs 00 Ib and rests on the bottom ofa tank that is 3 ft long. Water and oil are poured into the left- and right-hand portions of the tank to depths of 2ft and 4, respectively. Find the magnitudes of the horizontal and vertical components of the force that will keep the cylinder touching the tank at B. H Edon™ (Fudan ~ Faden = ((0.750)(62.4)]{(0 + 4)/2]1(4)(3)] ~ (62.4)[(0 + 2)/2]{(2)(3)] = 749 Ib (left) (Fe)ne = (Fe)an + (Foden = [(0:750)(62.4)]1 (4/413) + (62.4){ N\A? — 2)(VI2)/218) ‘= 2684 Ib (upward) ‘The components to hold the cylinder in place are 749 Ib to the right and 2684 ~ $00, or 2184 Ib down. 90 2 CHAPTER 5 Fig. 5.9 $10 The half-conical buttress ABE shown in Fig. 5-10 s used to support a hal-cylindrical tower ABCD. Calculate the horizontal and vertical components ofthe force due to water acting on the buttress. 1 Fig thegA = (CLAYS + MMO + 2)/2} = 3744 (right) = weight of (imaginary) volume of water above curved surface = (€2.49(4)(6)2)12)13 + (()E)'@)] = 1960 b (wp) Fig. 5-10 5.11 A dam has a parabolic shape 2 = 2(x/x?, as shown in Fig. S-11a. The fluid is water and atmospheric pressure may be neglected. Ifx= 10 ft and 2 = 24 ft, compute forces F, and F, on the dam and the position «.p. where they act. The width ofthe dam is 50 ft I Fig= yhA = 62.4{(24 + 0)/2](24)(50)] = 898 600 Ib. The location of Fis along a (horizontal line *, or 8,00 above the bottom ofthe damn Fy = (areaaay)(width of dam)(y). (See Fig. $-11b.) Aredaas = 2024/3 {(2)(10)(24)/3 = 160 fF = (160)(50) (62.4) = 499°200 Ib. The location of Fis along a (vertical) line through the center of gravity of From Fig. 5-11b, x = 3xo/8 = (3)(10)/8 = 3.75 ft, z = 32/5 = (3)(24)/5= 14.4 ft, ness V@SD NF +698 600 = 1028 O00 Ib, As seen in Fg 5-11, Fenn ats down and to the ight a ah angle of arctan (499 200/868 600), or 29.1°. Femi Passes through the point (x, 2) = (3.75 ft, 8ft). If we move down alone the 29.1°line until we strike the dam, we find an equivalent center of pressure on the dam at x= 5.43 ft and z = 7.07. This definition of ep. is rather artifical, but this isan unavoidable complication of dealing with a curved surface. Ne ) Fig. S-11(a) Fig. S110) sa $13 FORCES ON SUBMERGED CURVED AREAS 0 91 ° saya Fig. S-11(0) ‘The canal shown in cross section in Fig. 5-12a runs 40 m into the paper. Determine the horizontal and vertical components ofthe hydrostatic force against the quarter-circle wall and the point c.p. where the resultant strikes the wall, I Fy= yh =9.79{(18 + 0)/2}{(18)(40)} = 63 439 KN, The location of Fy is along a (horizontal) line ¥, oF 6.0m above the bottom of the wall. Fy = 9.79{(40)(:r)(18)'/4] = 99 650 KN. The location of Fy is along a (vertical) line through the center of gravity of areaoan x= 4r/(3x) = (4)(18)/(3x) = 7.64 m, Fenauee= “V63 439" + 99 650" = 118 130 KN. As seen in Fig. 5-125, Fenn: acts down and to the right at an angle of arctan (9650/63 439), oF 57.5. Fan Passes through the point (x, 2)= (7.64 m, 6.00 m). If we move down along the 57.5" line until we strike the wall, we find an equivalent center of pressure at x = 8.33m and 2 = 2.82 m. 18,190 KN Feesettaat = 99.650 KN 63.438 Fig. $:12(0) Fig. 51206) Gate AB in Fig. 5-13ais a quarter circle 8 ft wide into the paper. Find the force F just sufficient to prevent rotation about hinge B. Neglect the weight of the gate. I y= yA =62.4{(7 + 0/2]{(1)8)] = 12230 kb (left). The location of Fy is along a (horizontal) line 3, or 2:33 ft above point B. (See Fig. 5-136.) Fy = F— R= 62.4[(8)(7\)] ~ 62.4(8)(x)(1)/4] = 24461 ~ 19211 = ‘250 Ib (up). The location of Fy can be determined by taking moments about point B in Fig. 5-136. 5250x = (24 461)(3) ~ (19 211)[7 ~(4)(7)/m), x = 1.564 ft. The forces acting onthe gate are shown in Fig. 513e, DM =O; 9F ~ (2.333)(12 230) — (1.568)(5250) = 0, F = $249 b (down). 92 0 CHAPTER 5 i Fig. 51306) | "Soary + | 4 azs0m zasstt Aa 52501 Mig. $-13(€) 5.14 Repeat Prob. 5.13 ifthe gate is steel weighing 30001, I. The weight of the gate act atthe center of gravity ofthe gate shown in Fig. 5:14. 2r/2= Q\(I)/n = 4.456 ft; Z Mp =0. From Prob. 5.14, 7F — (2.333)(12 230) — (1.568)(5250) + {G000)(7 ~ 4.456) = 0, F=4159Ib. 4000 Ib *} : an : fo-8 #4} ng sue 5.15 Compute the horizontal and vertical components of the hydrostatic force on the quarter-circle face of the tank shown in Fig. 5-154. ' Fig = Yigg = 9.79{4 + HL 7)] = 308 KN =F B= O.7M HAN) - O.7M Hay 14] = 289 EN (See Fig. 5-156.) FORCES ON SUBMERGED CURVED AREAS 0 93 Vo tm Fig. 5-15(6) $16 Compute the horizontal and vertical components of the hydrostatic force on the hemispherical boulder shown in Fig. 5-162. A From symmetry, Fy, = 0, Fy =F Fi (see Fig. 5-16b). F, = 62.4{()(3)°(12)} — (62.4)[(4)4(2)G)'] 176431, SFP radius Se Ft radius Fig. 5-16(6) 5.17 The bottled cider (s.g. = 0.96) in Fig. 5-17 is under pressure, as shown by the manometer reading. Compute the net force on the 2-in-radius concavity in the bottom of the bottle. A From symmetry, Fu = 0, pas + {(0-96)(62.4)]() — [(13.6)(62.4)](4) = Puen = 0, Pan = 339 Ib/ft? (gage); F.=PasAronon + weight of liquid below AA = 3391()(i)*/4] + [(0.96)(62.4)]()(=)(8)/4] — [(0.96)(62.4)11G)G) AY = 32.11. 94 0 CHAPTER 5 sas 519 Mercury Fig. 5-17 Halfcylinder ABC in Fig. 5-18 is 9ft wide into the paper. Calculate the net moment of the pressure fore on the body about point C. From symmetry the horizontal forces balance and produce no net moment about point C. (See Fig 5-186.) By = BB = Fawrntsny ase = [(0.85)(62.4)](9)4)()'/2}= 15 184 Wb, x =4r] Bx) = (4YE)/GH) = 1.910, ‘Me (15 184) 910) = 29 O01 Ib fe (clockwise). Fig. 5-18(0) Fig. 5-18(6) ‘Compute the hydrostatic force andits line of action on semicylindrical indentation ABC in Fig. 5-19a per meter ‘of width into the paper. ' Fig phegA = [(0.88)(9.79)]2-+2 + ¥)1@-5)(1)] = 113.1 N ype asi _ LVR. 7) ThA 2424 IAI O] ‘As demonstrated in Prob. 5.18, Fy = Fyn cr maty anc and it acts at 4r/(3) from point C. Fy [(0.88)(9.79)]{4)()@2Y/2]= 21.14 KN, = 4r/Gx) = ()CP)/(Bx) = 0.531 m. The forces acting on the indentation are shown in Fig. 5-196. Be RE ete iN asa Fig. 5-196, Fann passes through point O and acts up and to the right at an angle of arctan (21.14/113.1), or 10.59". 0.099:m FORCES ON SUBMERGED CURVED AREAS 0 95 Fig. 5-196) 520 Find the force on the conical plug in Fig. 5-20. Neglect the weight of the plug. ' Fy = PAs. + weight of water above cone = [(4.5)(144)]{()(1)*/4] + 62.4){(4)(x)(1)*/4] ~ (62.4)|(3)(1.207)()(1)°/4] = 685 Ib pea.Sibjin? gage wane | ale — 7 sh Fig. 520 ‘S21 The hemispherical dome in Fig. 5-21 is filled with water and is attached tothe floor by two diametrically ‘opposed bolts. What force in ether bot is required to hold the dome down, ifthe dome weighs 25 KN? ' weight of imaginary) water above the container = 9.29§(S + 1.5)(a)(1.5)'] ~ 9.795)(-(0.04)/4] ~ 9.794(8)(3)().5)"] = 380.5 N (up) net upward force on dome = 380.5 ~25 = 355.5kN force per bolt = 355.5/2 = 177.7 KN 96 0 CHAPTER 5 saa 523 smu ‘A 3-m-diameter water tank consists of two half-cylinders, each weighing 3.5 kN/m, bolted together as shown in Fig. 5-22a. If support of the end caps is neglected, determine the force induced in each bolt. I Sce Fig. 5-22b. Assuming the bottom half is properly supported, only the top half affects the bolt force. Pim (9.79)(L5 + 1) = 24.48 N/m DF, = prAy — BR — Wis ~ Wasa mr 0, 24,48{(3)(48)] ~ 2Rroe — 9.79 (4) e)(1-5)/2] — 3.5/4=0, Fg 4.42 KN. Bott spacing em Fig. 5:22(0) Fig. 5:22(6) ‘The cylinder in Fig. $-232 extends 5 tito the paper. Compute the horizontal and vertical components of the pressure fore on the cylinder. I See Fig 5.235. Note that the net horizontal force is based on the projected vertical area with depth AB. Fy= phgA ~62.4(6-+ 2.828)/2}(4+2.828)(5)] = 7273 ib; Fy = equivalent weight of fuid in regions 1, 2, 3, and 4 = (62.4)(5)(x)()/2+ (2. 828)(8) + (2.828)(2.828)/2 + (2)(8)78] = 14579 Ib. 4s" Water Fig. $.23(0) Fig. 5-23(6) ‘A 3.ft-diameter log (6.8. = 0.82) divides two shallow ponds as shown in Fig. 5-24a. Compute the net vertical and horizontal reactions at point C, ifthe log is 12ft long. FORCES ON SUBMERGED CURVED AREAS 0 97 I F-= yhA. Figure 5.248 shows the forces acting on the log. (Fu); = 62.4[(0 + 3)/2][(1.5 + 1.5)(12)] = 3370 > (Frys = 62.4{(0 + 1.5)/2]f(1.5)(12)] = 842 Ib (Fy) = 62.4[(12)(22)(1.5)"/2] = 2646 Ib (Fy) = 62.4(12)(8)(1.5)"/4] = 1323 Ib DR=0 — 337-82-C.=0 C,=2528Ib (lett) Deo 2646 + 1323 — [(0.82)(62.4)]{(12)(")(1.5)]}+C,=0 — C,=371 1b (up) . Fe Fig. 5-24(a) Fig. 5-240) fe ee a cee ee ese cen ears eget es ae ee Batata wats pan Gutce te seat cong at tems I See Fig. 5-256. The wall reaction at B is purely horizontal. Then the log weight must exactly balance the ‘vertical hydrostatic force, which equals the equivalent weight of water in the shaded area. Wing = Fy = (0.79) (8G) 0.5)? + 0.5)(0.5)] = 65.71 KN, Yong = 65.71/(8)(a)(0.5)] = 10.46 KN/m, 5g 107. Fig. 5:25(a) F20.5m Fig. 5:25(6) $26 The tank in Fig. 5-26a is 3 m wide into the paper. Neglecting atmospheric pressure, compute the hydrostatic horizontal, vertical, and resultant force on quarter-circle panel BC. 1 y= ph = (9.79)(4-+ DL()3)] = 954.5KN, Fy = weight of water above panel BC = (9.79){(3)(5)(4)] + (0.79){(3)(2)(5)/4] = 1164 EN, Fata = V954.5+ TIF = 1505 KN. As seen in Fig. 5260, Fran SSCS through point O and acts down and to the right at an angle of arctan (1164/954.5), or 50.6". Mig. 5:26(a) e Fig. 5-266) 98 7 CHAPTER 5: 5.27 Gate AB in Fig, 5-Z7ais a quarter circle 7 ft wide, hinged at B and resting against a smooth wall at A. Compute the reaction forces at A and B. ha sin 8 _—[OP/2sin 907) __ hel GID] ~ 077" ‘Thus, fy ats at $ - 0.375, or 2.625 ft above point B. Ry = weight of seawater above gate AB = (64 TCI) 6)] ~ (64) 11) 6)/A] = 29 568 ~ 12.67 = 16 901 Ib. The location of F, canbe determined by taking moments about point in Fig. -27b, (29 568)() ~ (12 667) (4)(6)/(3a)] = 16 901s, x =3.340ft. The forces acting on the gate are shown in Fig. 527e. SMy=0 — (21504)(2.625) + (16901)(6-3.90)~6A,=0 4, = 16901 5 TA=0 — 21504-B,-16901=0 2, =46031b DR=0 — B-16901=0 B= 169011 WE Fg hg = (64N(11 — DIG)] = 21 50410 Seawater yeouih it ~ Fig. 5-27(a) efr rd F, = 29,568 Ib = F,= 2667 ib = nel — ae 3 A A ett Bet Fig. 52706) F,= 16A01 a Y Fig. 52710) FORCES ON SUBMERGED CURVED AREAS 0 99 (Curved wall ABC in Fig. 5-28a is a quarter circle 9ft wide into the paper. Compute the horizontal and vertical ‘hydrostatic forces on the wall and the line of action of the resultant force. I See Fig. 5-28b. Fy = yhgA = (62.4)(3.536)|(7.072)(9)] = 14 044 Ib, Fy = weight of (imaginary) water in ‘rosshatched area in Fig. 5-280 = (62.4)(9)[()(5)"/4 — (2)(5 sin 45°)(5 cos 45°)/2] = 4007 ID; Frenne = VAO0F + 14 0447 = 14.604 1b. Fenane Passes through point O and acts at an angle of arctan (4%, or 15.9", as shown in Fig. 5-28c. 3.836 ft 7.072. ¢ F, asset Fy Fig. 5-28(6) << \ Fresaltent = 14,604 Ib Fig. 5-28(c) Pressurized water fills the tank in Fig. 5-29a. Compute the net hydrostatic force on conical surface ABC. I From symmetry, Fy =0. The gage pressure of 100 kPa corresponds to a fictitious water level at 100/9.79, or 10.215 m above the gage or 10.215 ~ 7, or 3.215 m above AC (see Fig. 5-29b). F, = weight of fictitious water above cone ABC = 9.79{(3.215)(22)(3)/4 + (4)(6)()(3)"/4] = 361 KN (up). — Fletitions water level 100KPa gage Fig. 5.292) 100 0 CHAPTER 5 5.30 Gate AB in Fig. 5-30c is 7 m wide into the paper. Compute the force F required to prevent rotation about the hinge at B. Neglect atmospheric pressure. H Fg= YA =9.79(9.6-+ 0)/24(9.6)(7)] =3158EN. Fy ats at %, oF 3.200 m above B (see Fig. 5-206). Fy = weight of water above the gate = 9.79{(3)(6\(9.6)(7)] = 2632 KN. Fy acts at, or 2.250 m right of B (see Fig. 5-306). L My = 0; (3.200)(3158) + (2.250)(2632) — 9.61 ), F = 1670 KN, 7 F Fy = 26a2kN sem Fy = 3158 KN 9200m ae = — Shag Fig. 5:20(0) Fig. 5-30(a) ‘The cylindrical tank in Fig 531 has a hemispherical end cap ABC. Compute the total horizontal and vertical forces exerted on ABC by the oil and water. ' Fa hgA (Fads ={0.9)0.71G +82) Q2)12]=221 kN (let) Fohs= {I(0.99(9.79)13 +2) + (.79)(3))[(a)2Y/2=338 KN (let) Fv =221-+338= S59KN- (left) Fy = weight of fud within hemisphere = (0:9}(9.79)I(4)(3\()@)] + (9-79)[4)(3Y-aX2)"] = 156KN (down) sat Fig. 5:31 532 Aclindrical barrier holds water, as shown in Fig. 5-32. The contact between cylinder and wal is smooth. Consider a 1-m length of cylinder and determine its weight and the force exerted against the wall ! Fodco = 0-79) A)(2)/2+ (22) + (2(2)]=139.8KN (up) Fav = 0.79) 1QQ) — (AEP A]=84KN (down) DH=0 — 39.8— Wane 8.4=0 Weta = 131AKN, Fy™ YhagA —— (Fidane = (9-79)2){(2+ 2\()] = 78.3 KN (ight) Foc = (9.79)2+ IQKD]=S8.TEN (left) Fain nt =78.3 ~ 58.7= 19.6N (right) FORCES ON SUBMERGED CURVED AREAS 0 101 533 The revolving gate in Fig. 5-33 is a quarter-cylinder with pivot through O. What force Fis required to open it? (Treat the gate as weightless.) I Ateach point of ABC the line of action ofthe pressure force passes through O; hence the pressure has no moment about Q. It follows that any F, no matter how small, suffices to produce a net opening moment. Fig. 5:33 524 Find the vertical component of force on the parabolic gate of Fig. 5-34a and its line of action. ' eeeigoinaginry tid above ge [ye (ee 53) = (0.0098) ["" @— V5x) de = (9.00))[25 wer = 14.40 kN wt | 1-year fg (ue ig 530) any [” 2 - VSe x de 2.0 [ ax V5e) de Tao 10 = (9.00)(3)fx* (V5x*)/3]39/14.40 = 0.240 m y= 4.00 kN/m* Gate amr ‘mde Fig. 534(a) Fig. 5346) $35 Determine the moment M needed to hold the gate of Fig. 5a shut. Neglect its weight. 1 Fy = yA = 9.004(0-+2)/2][@2)(3)] = 5A0KN (lft). Fy acts at 3, oF 0.667 m above point 0. Fy = 14.40 kN (up) and x.5 =0.240 m (from Prob. 5.34 and Fig. 5-340). 5 My = 0; M ~ (14.40)(0.240) ~ (54.0)(0.667) M=30.5KN-m. 102 0 CHAPTER 5 536 5.38 Find the force on the body (part of a parabolic cylinder) of Fig. 5-35. The length normal to the paper is L=4.5m, and yis 9.20kN/m’. ' Fu= yhA = (9.20\3){(1)(4.5)] = 20.70 kN 39.03 kN ve (x = wight tiguid above 04 [riya = [0.204.9(2) d= 0.2015) 24], Fecctne = NIG F BOTP = 44,18 KN Fig. 5:35 ‘The curved plate in Fig. 5-36 isan octant of a sphere. Find the resultant force, including its line of action, acting ‘on the outer surface, ifthe radius ofthe sphere is 600 mm and its center is 2 m below the water surface. W Seo Fig. 5:36. f= yhA = y[H! — &r/(32)|(x0*/4), = F, = Fy =9.7912 ~ (4(0.6)/(3x)]{(2(0.6)14] = 4.831 kN (both F, and F, act toward 0); F, = Fy = weight of water above curved surface = y|(H)(2x)(r)"/4— (8)(8)(0)?/8] = 9-79{(2)(2)(0.6)°/4 — (4)(-2)(0.6)°/8] = 4.429 KN. Feegines 21S on a line through 0 making a 45° angle with the x and z axes because of symmetry; Freunao = V4.429" + 4.831" + 4.831" = 8.142 KN. It acts at an angle 6 = arcoos (4.429/8,142) = 57.0". Fig. 5:36 Find the horizontal and vertical components of the force per unit width exerted by fluids on the horizontal cylinder in Fig. 5-37 if the id tothe left ofthe cylinder is (a) a gas confined in a closed tank at a pressure of 35.0kN/m* and (b) water with a free surface at an elevation coincident withthe uppermost part of the cylinder. ‘Assume in both instances that atmospheric pressure occurs to the right of the eylinder. F(a) The “net vertical projection” (see Fig. 5-374) ofthe portion ofthe cylinder surface under consideration js 4 (2— 200s 30), or 3.752. Fy =pA ~35.0[(1)G.732)] = 130.6 KN (Fight). Note that the vertical force of the gas on surface abis equal and opposite to that on surface be. Hence, the “net horizontal projection” with regard tothe gas is ae (se Fig. 5-380), which is 2sin 30", o 1.000m. Fy = 35.0((1(1.000)] = 38.0 EN (up). ” y= WA = (2.9)(3.732/2)()3.732)] = 68.2KN (right) Ky =-weight of crosshatched volume of water (Fig. $-37b) = (.19)(INBBM NAY 4 + (4)(.000)(3.732 —8) + (1)8)]= 99.8 KN (up) FORCES ON SUBMERGED CURVED AREAS J 103 Net vertical ejection Fig. $3710) Fig. 5-37(6) A vertical-thrust bearing consists of an 8-in-radius bronze hemisphere mating into a steel hemispherical shell. At ‘what pressure must grease be supplied to the bearing so that an unbroken film is present when the vertical ‘thrust on the bearing is 600 000 1b? ! Projected area = 27? = (7)(8)*=201.1in® p= F/A = 600 000/201.1 = 2984 Ib/in? Find horizontal and vertical forces per foot of width on the Tainter gate shown in Fig. 5-38. A Fy = yhA = (62.4){(0 + 25)/2][(25)(1)] = 19 500 Ib. Fy, acts at (3)(25), or 16.67 ft below the water surface. R, = weight of imaginary water in ACBA = (62.4)(1){(1)(25)"/5 ~ 2)(25 eos 36°)(25 sin 36°)/2] = 5959 Ib. Fy acts through the centroid of segment ABCA. Fig. $38 ‘The tank indicated in cross section in Fig. 5-39 is 6m long normal to the paper. Curved panel MN is one-quarter of an elipse with semiaxes b and d. If b= 5m, d=7m, anda = 1.0m, calculate the horizontal and vertical components of force and the resultant force on the pancl. ' Fig= YA =9.79(1.0+ 3)[(6)(7)] = 1850 KN seer ee OMe water Soe eAg aang (10 OU egeuaene eee h, Ry = weight of water above surface MN = (9.79)(6){(=)(S)(7)/4 + (1.0(5)] = 1908 kN b/(3t) = (4)(5)/(3) = 2.122 m to the right of N Fagan = VIS08T + 1850" = 2658 KN Frenam 8s through the intersection of Fy and Fy at an angle of arctan (1908/1850), or 45.9". a mak Arete garter of ipso = = thee Avra = Fig. 5.9 104 0 CHAPTER 5 $42 Solve Prob. 5.41 ifa = 1.0ft, b= 5ft, d =7ft, the tank is 6 ft long, and MIN represents a parabola with vertex an, u For = YhegA = (62.4)(1.0+ 3)[(6)(7)] = 11 794 I = hg = (1.04) + ON _ Iam hat 5g (0+ D+ a Gree S07 below water sre F,= weight of water above surface MN = (62.4)(6)[@)(7)(5) + (1.0)5)] = 10 60816 y= (10) = (ES) = 1.88410 the right of N Fanta = VIOGOE + 1179 = 15 863 1b Faun 28 through the intersection of Fi and Fat an angle of arctan (10 608/11 794), or 42.0. ‘543 In the cross section shown in Fig. 5-40, BCis a quarter-cicle. Ifthe tank contains water to a depth of 6 ft, determine the magnitude and location of the horizontal and vertical components on wall ABC per 1 ft width. i Fry = YA = (62.4){(0 + 6)/2]{(1)(6)] = 1123 1b hy = G)(6) = 4.00 y= weight of water above surface BC = (62.4)(1)[(6)(5)] ~ (62-4)(1)[()(5)*/4] = 1872 — 1225 = 647 Ib ‘The location of F, can be determined by taking moments about point B. (1872)($) ~ (1225)[(4)(5)/Ga)) AT ap Xap = 3.22 A. Fig. $40 5.44 Rework Prob. 5.43 where the tank is closed and contains gas at a pressure of 10 psi ' Fir pAy = ((20)(144)}{(1)(6)] = 8640 Ib (10)(444)]{(1)(5)] = 7200 tb 5.45 A spherical steel tank of 22 m diameter contains gas under a pressure of 300 kPa. The tank consists of two hhalf-spheres joined together with a weld. What wil be the tensile force across the weld? Ifthe stee! is 25.0 mm thick, what isthe tensile stress in the steel? __ force/length _ 114 040/(22) thickness -25.0/1000 een ' F = pA = 300f()(22)"/4] = 114 040 KN 5.46 Determine the force required to hold the cone shown in Fig. 5-41a in position. I Figure 5-416 shows the vertical projection above the opening. Pax = 0.6 ~ [(0.83)(62.4)|(5.1)/144 = 23 psi, Far = ((1.23)(144)]{(2")(0.804)"] = 360 tb, Feyanser = (62.4)(0.83)[()(0.804)"(5.1 + 3)] = 852 1b, = (62.4)(0.83){(3)(22)(0.804)"/3] = 105 Ib; 5, = 0, 360 ~ 852+ 105+ F =0, F = 387 1b. Foe HF inter .? — |< osot ft Fig. $4110) (radians) Fig: 54106) Fe ee ee ee ea eae eee SaT 548 FORCES ON SUBMERGED CURVED AREAS J 105 ‘The cross section of the gate in Fig. 5-42 is given by 10x = 3y*; its dimension normal to the plane of the paper is. ‘7m. The gate is pivoted about O. Find the horizontal and vertical forces and the clockwise moment acting on the gate ifthe water depth is 1.8m. ' Fy = yhA =9.79{(0 + 1.8)/2]{(7)(1.8)] = 111.0 kN y= weight of water above the mie= [29,06 4y)= (0.7911) if “nay =0. aoyen| 22") *=40.0KN Mo= crr.ayes)+ [9.79 (5) cy) = 656+ 0.7990) 03 gy 156 02040) 222] ares Fig. 5-42 Find the wall thickness of steel pipe needed to resist the static pressure in a 36-in-diameter steel pipe carrying water under a head of 750 ft of water. Use an allowable working stress for steel pipe of 16.000 psi. ' p= yh =(62.4)(750) = 46 800 lb/ft? or 325 b/in* T = pd/2= (325)(36)/2 = 5850 Ib/in of pipe length 5850/16 000 = 0.366 in ‘A vertical cylindrical tank is 6 ft in diameter and 10 ft high. Its sides are held in position by means of two steel ‘hoops, one at the top and one at the bottom. The tank is filled with water up to 9ft high. Determine the tensile stress in each hoop. WF See Fig. 5-43. hhoop = (7582)(4) = 2275 Ib stress in bottom hoo} 7582 Ib; stress in top yh =62.4{(0+9)/2](9)(6)] = 15 163 Ib, T = F/2 = 15 163/3 (7582)(10 — 3)/10] = 30716. 10 ft 1 ft 10 ft fe Fig. 5-43(@) Fig. 5-43(6) 106 9 CHAPTER 5 5.0 S51 582 583 554 ‘A 48.in-diameter steel pipe, } in thick, caries ol of sg. = 0.822 under a head of 400 f¢ of oil. Compute the (@) stress in the steel and (6) thickness of stel required to carry a pressure of 250 psi with an allowable stress of 18000 psi @ ow ‘A wooden storage vat, 20ft in outside diameter, i filed with 24 t of brine, s.g. = 1.06. The wood staves are bound by flat stee! bands, 2in wide by 4 in thick, whose allowable stress is 16 000 psi. What isthe spacing of the ‘bands near the bottom of the vat, neglecting any initial stress? Refer to Fig. 5-44 I. Force P represents the sum of ll the horizontal components of smal forces dP acting on length y of the vat, and forces T represent the total tension carried in a band loaded by the same length y. DHH0 ATP HO T= Accu nani (€2)(2)](16 000) = 8000 Ib = yhA = [(1.06)(62.4)](24)(20y)=31 749y _(2)(8000)~ 31 749y =0 y =0.5044t or 6.05in ee Fig. 5-44 A.4.0-in-ID steel pipe has a }-in wall thickness. For an allowable tensile stress of 10 000 psi, what is the nau poet ' on 0c 242) pe sna A thin-walled hollow sphere 3.5 m in diameter holds gas at 1700 kPa. For an allowable stress of $0000 kPa, determine the minimum wall thickness. Considering half a sphere of diameter d (3.5 m) and thickness , (xd‘)(0) = (p)(d?/4), [6)8.5)(0](50 000) = 1700[()(3.5)/4], ¢ = 0.02975 m, oF 29.75 mm. A cylindrical container 8 ft high and 3 ft in diameter is reinforced with two hoops a foot from each end. When it is filled with water, what is the tension in each hoop due to the water? I See Fig. 5.45, F = yA = 62.4[(0+ 8)/2}(8)(3)] = 5990 b. F acts at (3(8), oF 5.33 ft from the top of the container. De=0 21,+2%,-s90=0 0 mM, @T)(1.667 ~ @T.XA4.333) =0 h=200T, @ Solve simultaneous equations (1) and (2). 27; + (2)(2.60) ~ $990 = 0, T; = 8321b, T; = (2.60)(832) = 2163 b. FORCES ON SUBMERGED CURVED AREAS J 107 Fe sasstt asst FE fe iT, Fig. 5-45 $55 A.30-mm-diameter steel (y = 77.0 KN/m’) ball covers a 15-mm-diameter hole in the bottom of a chamber in ‘Which gas pressure is 27 000 kPa. What force is required to push the ball up into the chamber? 1 F = pA + weight of ball = 27 000{(2)( H)?/4] + [(4)(=)(a85)'1(77.0) = 4.780 kN et ao CHAPTER 6 Buoyancy and Flotation 61 62 63 64 65 108 ‘A stone weighs 105 lb in air. When submerged in water, it weighs 67.01b. Find the volume and specific gravity of the stone. ' Buoyant force (F) = weight of water displaced by stone (W) = 105 ~ 67.0= 38.016 WawW=esv 380=624¥ V=0.0098" ‘Weight of stone in air ‘weight of equal volume of 58. 2.16 {A piece of iregularly shaped metal weighs 300.0.N in air, When the metal is completely submerged in water, it weighs 232.5N. Find the volume ofthe metal ' R=W 3000-2325 = (9.791000) V = 0.00689 m* ‘A cube of timber 1.25 ft on each side floats in water as shown in Fig. 6-1. The specific gravity of the timber is 0,60. Find the submerged depth of the cube. ' B= W — 62.4[(1.25)(1.25)(D)] = {(0.60)(62.4)}{(.25)(1.25)(1.25)] D = 0.7508 | 125" —+! Teg ff! Determine the magnitude and direction of the force necessary to hold a concrete cube, 0.300 m on each side, in ‘equilibrium and completely submerged (a) in mercury (Hg) and (6) in water. Use §.8 connie = 2-40. F (@ Since 5.815 = 13.6 and 5.8 mene = 2.40, itis evident thatthe concrete will float in mercury. Therefore, a force F acting downward will be required to hold the conerete in equilibrium and completely submerged in mercury. The forces acting on the concrete are shown in Fig. 6-22, where Fis the force required to hold the concrete cube in equilibrium and completely submerged, W is the weight of the concrete cube in air, and Fis the buoyant force. 5 F, =0, F+W ~ Fi, =0, F + {(2.40)(9.79)}{(0.300)(0.300)(0.300)] = [(13.6)(9.79)}{(0.300)(0.300)(0.300)] = 0, F = 2.96 KN (downward). (6) Since 5.8-camrae = 2-40, it will sink in ‘water. Therefore, a force F acting upward will be required to hold the conerete in equilibrium and completely submerged in water. The forces acting on the concrete in this case are shown in Fig. 62b. E = 0, ,[(240)(9.79)]{(0.300)(0.300)(0.300)} ~ F — 9.794(0.300)(0.300\(0.300)] = 0, F = 0.370KN Fig. 6-1 A concrete cube 10.0 in on each side isto be held in equilibrium under water by attaching a lightweight foam. ‘buoy to it, as shown in Fig. 6-3. (In theory, the attached foam buoy and concrete cube, when placed under water, will nether rise nor sink.) If the specific weight of concrete and foam are 150 b/ft and 5.01b/ft, respectively, what minimum volume of foam is required? BUOYANCY AND FLOTATION J 109 — F< 0mm | _ L TL Ue ; Concrete A me cas ee 1 Tne forces acting in his problem are shown in Fig. 63, where Wand W, are the respective weights of the foam and the concrete, and fi and Fi are the respective buoyant forces on the foam and the concrete. F, = 0, Wy — Fig + We — Fie = 0, 5-0Vioum ~ 62-4Vionm + 150{(39)(12)(13)] ~ 62-4{(72)(12)(12)] = 0, Vom = 0.883 f°, Concrete “ih ne ‘A barge is loaded with 150 tons of coal. The weight of the empty barge in air is 35 tons. If the barge is 18 ft ‘wide, 52 long, and 9 ft high, what is its draft (Le., its depth below the water surface)? ' B=W — 62.44(18)(52)(D)] = (150 +35)(2000) D = 6.334 Concrete cube 1000 in on each side Determine the submerged depth of a cube of stee! 0.30 m on each side floating in mercury. The specitic gravities of steel and mercury are 7.8 and 13.6, respectively. ' A=W [(13.6(9.79)]{(0.3)(0.3)(D)] = [(7.8)09.79)](0.3)(0.3)0.3)]_ D = 0.172m. A.cube of wood (s.g. = 0.60) has Sin sides. Compute the magnitude and direction of the force F required to hold the wood completely submerged in water. I Since 58-0 = 0.60, itis evident that the wood will float in water. Therefore, a force F acting downward will be required to old the wood in equilibrium and completely submerged. The forces acting on the wood are essentially the same as those shown acting on the conerete cube in Fig. 62a: 5 F,=0, F+W —K,=0, F + [(0.60)(62.4)}(8)(8)()] ~ 62.4{(&)(2)(B)] = 0, F = 10.51 (downward). 110 0 CHAPTER 6 69 6.10 oat 612 633 6a4 A hollow cube 1.0m on each side weighs 2.4 KN. The cube is tied to a solid concrete block weighing 10.0 KN. il these two objects tied together float or sink in water? The specific gravity of the conerete is 2.40. I Let W = weight of hollow cube plus solid concrete block, (F,), = buoyant force on hollow cube, and (&);= buoyant force on solid concrete block. W = 2.4 + 10.0= 12.4 KN, (F,), =9.79{(1)(1)(1)] = 9.79 EN, Vea = 10/{(2-40)(9.79)] = 0.4256 m?, (a= (9.79)(0.4256) = 4.17 KN, (Fy + (5)2 = 9-79 + 4.17 = 13.96 N. Since [W = 12.4] <[(F),+ (B= 13.96], the two objects ted together wll loat in water. ‘A conerete cube 0.5 m on each side isto be held in equilibrium under water by attaching a light foam buoy tot. ‘What minimum volume of foam is required? The specific weights of concrete and foam are 23.58 KN/m’ and 0.79 kN/m:, respectively I. Let W, = weight of foam in air, (F,), = buoyant force on foam, W. = weight of concrete in ar, and (B)_= buoyant force on concrete. 3 =0, W; ~ (B+ We~ (Be 0, 0.79Venun ~9.79V ian + 23,58{(0.5)(0.5)(0.5)} ~ 9.79{(0.5)(0.5)(0.5)] = 0, Visua = 0.192 m°, ‘A prismatic object 8 in thick by 8in wide by 16in long is weighed in water at a depth of 20in and found to ‘weigh 11.01b. What is its weight in air and its specific gravity? The forces acting on the object are shown in Fig. 64. 2.5 =0, T +B, ~ W'=0, Fy = weight of displaced water = 62.4{(8(8)(16)/1728] = 37.01b, 11.0+ 37.0 W = 0, W = 48,016, sg. = 48,0/37.0=1.30. r=n0 f, Fig. 64 ‘A hydrometer weighs 0.00485 Ib and has a stem at the upper end which is cylindrical and 0.1100 in in diameter. How much deeper will it float in oil of s.g. 0.780 than in alcohol of s.g. 0.821? H Wozonses = Wane iaus: For position 1 in Fig. 6-5 in the aleohol, 0.00485 = [(0.821)(62.4)|(V), ¥; = 0,0000947 fe (in aleobol). For position 2 in Fig. 6-5 in the ol, 0.0048 = [(0.780)(62.4)][0.0000947 + (4)(\(0.1100/12)"/4], f= 0.0750 f, oF 0.900in. sgront) —sgcomm Fig. 6S {A piece of wood of s.g. 0.651 is 3 in square and 5 ft long. How many pounds of lead weighing 7001b/f? must be fastened at one end of the stick so tha it will oat upright with 1 ft out of water? ' Wace sote = Wepeet wnat ((0.651)(62.4)]1(5)(4)(i)] + 700V = 62.4[(5 — 1)G3)G) + V) V=0.00456 0? Wags = (0.00456)(700) = 3.191b ‘What fraction of the volume of a solid piece of metal of s.g. 7.25 floats above the surface of a container of, ‘mercury? I Let V =volume of the metal and V’ = volume of mercury displaced. F, = W, [(13.6)(62.4)|(V")= [(7.25)(62.4)|(V), V'/V =0.533. Fraction of volume above mercury = 1 ~ 0.533 = 0.467. 616 on 69 BUOYANCY AND FLOTATION J 111 ‘A rectangular open box 25 ft by 10 ft in plan and 12 ft deep weighs 40 tons and is launched in fresh water. (@) How deep will it sink? (B) Ifthe water is 12 ft deep, what weight of stone placed in the box will cause it to rest on the bottom? ! R=w @ 62.4{(25)(10)(D)] = 40)(2000) = 5.13 ® {62.4f(25)(10)(12)] = (40 + Waone)(2000) Wane = 53.6 tn, {A block of wood floats in water with 2.0 in projecting above the water surface. When placed in glycerin of sg, 1.35, the block projects 3.0 in above the liquid surface. Determine the specific gravity of the wood. 4H LetA =areaof block and h = height of block. Waa =[(6-8.)(62.4)|(AR/12), Wares wnes=62.4[(A)(H—2)/12, Wass grata = (1.39)(62.4)]{(A)(h ~ 3)/12. Since the weight of each displaced liquid equals the weight of the Block Wasnt ener = Wanner: 62-4{(A)(H ~2)/12]=[(1.35)(62.4)]{(A)(H —3)/12), h = 5.86%n, ‘Also, Wht = Wavnnseoer’ [-8 62-0)M(A)(S.86/12)] = 62.4{(A)(5.86~2)/12}, x, = 0.659. ‘To what depth will an 8t-diameter log 15 ft long and of s.g. 0.425 sink in fresh water? I The logis sketched in Fig. 66 with center O of the log above the water surface because the specific gravity is less than 0.5. (Had the specific gravity been equal to 0.5, the log would be haf submerged.) F, = W, F, = weight ‘Of displaced liquid = 62.4{(15)[(26/360)( 2.4”) ~ (2)(4)(4 sin 8)(4 cos 6)]} = 261.48 — (14 976)(sin @)(cos 8), W = ((0.425)(62.4)][(15)(x4*) = 19 996. 261.40 — (14976)(sin 6)(cos 8) = 19 996 ‘This equation can be solved by successive trials. Try 0= 85": (261.4)(85) ~ (14976)(sin 85°)(cos 85") = 20919 (#19996) ‘Try 8= 83"; (261.4)(83) — (14 976)(sin 83°)(cos 83") = 19 885 (#19996) Try O= 83.2%: (261.4)(83.2) ~ (14 976)(sin 83.2°)(c0s $3.2") = 19988 (#19996) ‘Try @= 83.22": (261.4)(83.22) — (14 976)(sin 83.22°)(c08 83.22") = 1998 (close enovgh) Depth of flotation = DC = OC - OD = 4.00 ~ 4.00 cos 83.22° = 3.53 ft (a) Neglecting the thickness ofthe tank walls in Fig. 6-7a, ifthe tank floats in the position shown what is its ‘weight? (b) Ifthe tank is held so thatthe top is 10 ft below the water surface, as shown in Fig. 6-7b, what is the {force on the inside top of the tank? Use an atmospheric pressure equivalent to a 34.0-ft head of water. 1@ Waane = Wepaesiqais = 62.4{(1)(14"/4)] = 784 10 (6) The space occupied by the air will be les at the new depth shown in Fig. 6-7b. Assuming that the temperature of the air is constant, then for postions a and b, pVs = PoVo. (62.4(34.0+ 1)]{(4)(-r4?/4)] = ([62.4)34.0+ 10+ y)II(v)(4°/4)], y? + 44.09 — 140 =0, y = 2.98. The pressure at D is 10+ 2.98, or 12.98 ‘of water (gage), which is essentially the same «s the pressure on the inside top of the cylinder. Hence, the force on the inside top ofthe cylinder i given by F = yh = (62.4)(12.98)(=r4?/4) = 10 180 Ib, A ship, with vertical sides near the waterline, weighs 4000 tons and draws 22 ft in salt water (y = 64.01b/f)(see Fig. 68). Discharge of 200 tons of water ballast decreases the draft to 21 ft. What would be the draft d of the ship in fresh water? 112 0 CHAPTER 6 eet art 2 nea | i tt \ pal ¥ ae oT Fig. 670) Fig. 6-706) Because the shape of the underwater section of the ship isnot known, its best to solve this problem on the basis of volumes displaced. A 1-ft decrease in draft was caused by a reduction in weight of 200 tons, or 7¥z= 64.0{(1)(A)] = (200)(2000) where V, represents the volume between drafts 22 ft and 21 ft, and [(1)(4)] represents the water-lin area times 1 ft, or the same volume V,. From the equation above, Vz (210)(2000)/64.0 = 6250 f (this is per fo0t depth), F; = weight of displaced liquid = yV, Vi= F,/?. In Fig. 68, the vertically crosshatched volume isthe difference in displaced fresh water and salt water. This difference in volume can be expressed as W /eua 1,0 ~ W/ Years» OF (4000 — 200)(2000)/62.4 ~ (4000 — 200)(2000)/64.0. Since V, = 6250 f/ft depth, the vertically crosshatched volume can also be expressed as 6250y. Hence, {6250y = (4000 ~ 200)(2000)/62.4 — (4000 ~ 2002000)/64.0, y = 0.49 ft; d = 21 +0.49 = 21.49 ft 6 aia ap et Fig. 68 A barrel containing water weighs 283.5 Ib. What will be the reading on the scales if a 2in by 2in piece of wood is held vertically inthe water to a depth of 2.047 I For every acting force, there must be an equal and opposite reacting force. The buoyant force exerted by the ‘water upward against the bottom of the piece of wood is opposed by the 2in by 2 in area of wood acting downward on the water with equal magnitude. This force will measure the increase in scale reading. F,=€2.4{(2)(3)()] = 3.5lb, new scale reading = 283.5 + 3.5 = 287.0Ib. Find the weight of the floating can in Fig. 69. ' K=W 9IAGE\AGRYI=W W=0.00344KN or 3.44N BUOYANCY AND FLOTATION 7 113 “The weight of a certain crown in air was found to be 14.0'N and its weight in water, 12.71N. Was it gold (5-193)? ' F=140-127=1.3N —— Vanianano = Vewen =1:3/[(9.79)(1000)] = 0,0001328 m? “Yexoma = 14.0/0.0001328 = 105 422'N/m* or 105.4KN/m? —5.8-ceen = 105.4/9.79 = 10.77 ‘Thus the crown was not pure gold. Repeat Prob. 6.22 assuming the crown isan alloy of gold (6. reasured weights, compute the fraction of silver inthe crown. I From Prob, 6:22, 8 own 10.77, Lt a= fraction of silver in crown. (a)(10.5)+ (1 ~ a)(19.3) = 10.77, 10504193~ 19.30 1071, «= 0.969. 3) and silver (s.g. = 10.5). For the same A plastic sphere is immersed in sea water (y = 64.0 b/ft°) and moored at the bottom. The sphere radius is 15in. ‘The mooring line has a tension of 160 Ib. What is the specific weight of the sphere? The mooring line tension (T) and sphere weight (W) act downward on the sphere, while the buoyant force (K) acts upward. EF, = 0; F, — T — W = 0, 64.0{(3)()(18)"] ~ 160 — (YreneredS))(H)] =, Yoptere = MAA If the total weight of the hydrometer in Fig. 6-10 is 0.035 Ib and the stem diameter is 0.35 in, compute the elevation h for a uid of specific gravity 1.4 I Let AV = submerged volume between s 8. = 1.4, Yo= submerged total volume when sg. = 1.0, 1/= specific weight of pure water, and W = weight of hydrometer. W = 7¥o=(s-8.(7)(¥e~ AV) = (8 ))(V) ~ GB YMAV). Since (7)(Ve)= Wand AV = ha = h(d?/4), W = (8. )() — GE AMC ne/4)}, 0.035 = (1.4)(0.035) ~ (1.4)(62-4|(6)(2)(0.35/12)/4),h = 0.240 fe, oF 2.88in. Fig. 6-10 For the hydrometer of Fig. 6-10, derive a formula for float position h as a function of s.g., W, d, and the specific ‘weight y of pure water. Are the scale markings linear or nonlinear asa function of s..? I From Prob. 6.25, W = (6.8.)(W)—( oan (5-g.)0W) = (W)(s.g.=1) “Ge. ser Canad) When plotted in Fig. 6-11 (in arbitrary units), itis slightly nonlinear. ‘A hydrometer weighs of 0.17N and has a stem diameter of 11 mn. What is the distance between scale markings forsg.= 1.0 and sg. = 1.1? Between 1.1 and 1.2? 8 Leth,= distance between markings for sg. 1.0 and .g.=1.1 and hy = distance between scale markings forsg.= land sg.= 12 From Prob, 6.26, __ sg.) xN0.1=)—_s rggm “Gennady “TST GOONTOoTVTA|~ OS™ oF 166mm 0.1701.2-1) L297 cOOO)I Ca) 0-01 74] hy=W5~16.6=13.9mm hyth= =0.0305m of 30.5mm 114 0 CHAPTER 6 630 +2 +l le wo -t 2 7”, oF 049 0 hl eae Fig. G11 ‘A square pole (s.g.= 0.68), 80 mm by 80 mm by 6m long, is suspended by a wire so that 4m is submerged in ‘water and 2m is above the surface. What i the tension inthe wire? I Tension (T) and buoyant force (F,) act upward on the pole, while pole weight (W) acts downward. © F, =0; T+ -W=0, T +9-79{(0.080)(0.080)(4)] ~ [(0.68)(9.79)}{(0.080)(0.080)(6)] =0, T = 0.00501 kN, oF 5.01. ‘The spar in Fig. 6-12 is wood (5.g.= 0.62), 2in by 2in by 10ft, and floats in sea water (s.g. = 1.025). How many pounds of stel(5.g.= 7.85) should be attached tothe bottom to make a buoy that floats with exactly h ‘ofthe spar exposed? ' Voyar = (&)(E)(10) = 0.2778 f° Veena Vat = Waa (7-85)(62.4)] =0.002041 Wag = Wage + Waa {(0.025)(62.4)}(0.2361 + 0.002041.) = (0.62)(62.4)](0.2778) + Waa o1tb Fig. 6-12 ‘A tight circular cone is $0 mm in radius and 170 mm high and weighs 1.5 N in air. How much force is required to push this cone vertex-downward into ethanol so that its base is exactly at the surface? How much additional force will push the base 6.5 mm below the surface? I Downward force (F) and cone weight (W) act downward on the cone, while buoyant force (f,) acts upward. EB =0; F— F — W =, {(0.79)(9.79)(1000)][()(0.050)"(0.170)/3] ~ F~ 1.5 =0, F = 1.94N. Once the cone is fully submerged, F is constant at 1.94 ‘A.2in by 2.in by 104 spar has 710 of steel weight atached (Fig. 6-12); the buoy has lodged against a rock 7 ft ‘deep, 28 depicted in Fig. 6-13. Compute the angle @ at which the buoy wil lean, assunking the rock exerts no ‘moment on the buoy. H From Prob. 6.28, Vi = 0.2778 f°. Woons = ((0.62)(62.4)](0.2778) = 10.75 tb and F, = 62.4{(2)(2)(L)] = 1LTSBL. Ways acts downward ata distance of Ssin 8 tothe right of A, and Facts upward ata distance of (L/2)(6in 6) tothe right of A; while the steel force passes through point A. Hence, Mx =0, 10.75(5sin 8)— (.733L)](L/2)(sn 6)}=0, L=7.876 ft cos 8 = 7/L=7/7.876= 0.88878, 0 = 27.3". ‘The submerged brick in Fig. 6-14 is balanced by a 2.54-kg mass on the beam scale. What is the specific weight of the brick, if it displaces 2.197 liters of water? BUOYANCY AND FLOTATION 2 115 (sg2062) Seawater sin mig. 613 1 F=mg = 2.580981) =24.92N. Upward force (F) and buoyant force (F.) act upward on the brick, while its weight (W) acts downward. J. F=0; F, + F — W =, (9.79)(1000)](2.197 x 10") + 24.92—W=0, W= 254Kg Mig. 614 ‘The balloon in Fig. 6-15 is filed with helium pressurized to 111 kPa. Compute the tension in the mooring line. y= PIRT; Yau {(101)(1000)}/{(29.3)(273 + 20)] = 11.76 N/m, Yq = [(111)(1000)}[(212.0)(273 + 20)] = 1.787 N/m*. Weight of helium (WW) and tension in mooring line (T) act downward on the balloon, while buoyant force (F,) acts upward. IF, = 0; fy ~ W ~ T =O, 11.76{(3)(2)(3)'} ~ 1.7871G)(n)G) - T = 0, T = 3807N. Airat 101KPa ‘and 20°C, Vi Fig. 6-15 ‘A 1.1-ft-diameter hollow sphere is made of steel (s.g. = 7.85) with 0.015-ft wall thickness. Flow deep will the sphere sink in water (ic, find h in Fig. 6-16)? How much weight must be added inside to make the sphere neutrally buoyant? ' B= W =wcight of displaced water= 7(x/3)(h)°Gr—h)] = 62.4{(/3)()(3)(4)~ h)]} = 107.8? ~ 65.358? W= (aes)(Vons) Yam (7.85624) = 489.816 /80 S)(a)E) — (Ca) {1 — 2)(0.01500)}/2)° = 0.05548 f° W = (489.8)(0.08848)=27.171 107.842 ~65.35H°=27.17 Von = 116 0 CHAPTER 6 ‘Two roots of this equation are complex. The other, obtained by tril and error, ish = 0.643 ft. For neutral buoyancy, the total weight ofthe sphere plus added weight must equal the weight of water displaced by the entire sphere. Hence, 27.17 + Wass = 62-4(3)(2)(4)", Wasa = 16.32 Ib. deta tt t 20.015 ft ‘Submerged volurm Ere Ene3r-h) ag gas 6.35 When a 6-Ib weight is placed on the end of a floating S-in by S-in by 10-ft wooden beam, the beam tilts at 1.6° with the weight atthe surface, as shown in Fig. 6-17. What is the specific weight of the wood? 1 tan 6° h/10 k= O.2793F — Venwa = ()(8)(10) = 1.736 f° Fy = W = 62.4[1.736 ~ 1}(0.2798)(8)(10)} = 72.026 W= (Yeou)( 1-736) +6 72.02 (YonusX1-736)+6 Yous = 38.01b/f° 18 ew vor Mig. 617 636 A wooden beam (s.g. = 0.64) is 140 mm by 140 mm by 5m and is hinged at 4, as shown in Fig. 6-18. At what angle @ will the beam float in water? I The forces acting onthe beam are shown in Fig. 6-18. Wan =[(0.68)(9.79)}(0-140)(0.140)(5)] = 0.6140 &N and F, = 9.7{(0.140(0,140)(L)] = 0.1919L. 5. My = 0; (0.1919L)[(5~ L/2)(cos 8) ~ (0.6140)1)(c0s 6))=0, —0.0960L? + 0.9595L — 1.535 = 0, L = 2.000 m; sin 8 = 1/(5 — 2.000) = 0.33333, 8 = 19.5°, Fig. 618 637 A barge weighs 45 tons empty and is 18ft wide, 45 ft long, and 9 ft high. What will be its draft when loaded with 125tons of gravel and floating in sea water (s.g. = 1.025)? ' R=W_ [(1.025)(62.4)}[18)(45)(h)] = (4S + 125)(2000) = 6.568 638 A block of steel (s.g. = 7.85) will “float” at a mercury-water interface as in Fig, 6-19. What will be the ratio of distances a and b for this condition? I Let w = width of block and Z = length of block. FW, (wo ML) + (13.6)(Ynyo)(6LW) = (O85) ynpNla +b)(Lw), a-+ 13.6b = 7.850 + 7.856, a/b = 0.839, 60 oat a 6a BUOYANCY AND FLOTATION J 117 yy Wie) « = GiekZa Mercury Fig. 619 ‘A balloon weighing 3.2 Ib i 5.5 ft in diameter. Upon release iti filled with hydrogen at 17 psia and 65°F. At ‘what altitude in the standard atmosphere will this balloon be neutrally buoyant? ' R=W — y=pIRT y= (17)(044)/{(765.5)(460 + 65)] = 0.006001 Ib/t Cra AER] = 3.2+ (0.006091) NEF] 04282 1b/ft° From Table A-7, altitude = approximately 18 600 ft ‘A rectangular barge 18 ft wide by 46 ft long by 9 ft deep floats empty with a draft of 4ft in a canal lock 28 ft wide by 56ft long and water depth 7 ft when the empty barge is present. If 170 000 Ib of steel is loaded onto the barge, what are the new draft ofthe barge (t) and water depth inthe lock (H)? 1 The weight ofthe barge (W,) is equal tothe buoyant fore when the draft is 4. Wy =62.4{(18)(46)(4)] = 206 68 Ibs Fs = W, 62.4[(18)46)(4)] = 206 669 + 170000, f= 7.290. Volume of water in lock = (7)28)(S6) ~ (4)(18)(46) = 7664 f°. After steel is added, (H)(28)(56)— (7.290)(18)(46) = 7664, H = 8.74 ft ‘A 4in-diameter solid cylinder of height 3.75 in weighing 0.85 Ib is immersed in liquid (y = 52.0 b/ft) contained ina tall, upright metal cylinder having a diameter of Sin. Before immersion the liquid was 3.0 in deep. At what level will the solid cylinder float? See Fig. 6-20. I Let x = distance solid cylinder falls below original liquid surface, y = distance liquid rises above original liquid surface, and x +y = depth of submergence. Vi, = Vp, x{()(4)*/4] = y[()(5)/4] ~ yl(ee)(4)/4], x = 0.5625y. F = W, 52.0{(2e)(i4)*/4][(x + y)/12] = 0.85, x + y = 2.248, 0.5625y + y = 2.248, y = 1.44in, x = ((.5625)(1.44) =0.81 in. The bottom ofthe solid eylinder willbe 3.0— 0.81, oF 2.19 in above the bottom ofthe hollow cylinder. ; ! Treen Re Re ee ee ee ee eee gern relative to ocean water? What portion of ts volume would be above the surface if ce were floating in pure water? Youano = 64.0 lb/ft. ' (Veatars)tmanged = (1 ~ 4) Viesere = 0.857 Vater, BW Yaa ri0) Viste ates = Yates) Vines) 5 Bama = Yee Yoon 0 = (Varennes Vite = 00.857V cess! Verten = 0.857 (relative to ocean water) exter ™ (0.857)(64.0) = 54.851b/A° 5 Bateng = 54.85/62. Therefore, 1— 0.879 = 0.121, or 12.1 percent of its volume would be above the water surface in pure water. .879 (relative to pure water) A hydrometer consists of an 11-mm-diameter cylinder of length 220 mm attached to a 26-mm-diameter weighted sphere. The cylinder has a mass of 1.5 g, and the mass of the sphere is 13.0 g. At what level will this device float 118 0 CHAPTER 6 in liquids having specific gravities of 0.8, 1.0, and 1.2? Is the scale spacing on the cylindrical stem uniform? Why ‘or why not? IF Lety = submerged length of cylinder in millimeters. Vijere= ($)(™)(4) = 9203 mm”, Vusmerstrtaser = (y)(a)(B)? = 95.03y, F, = W = (6.2.){(9-79)(1000)/1000"](9203 + 95.03y) = (0.09010)(s.g.) + (0.0009303)(5.2.)(9), W = mg = (1.5 + 13.0)(9.81) = 142.2 g- m/s?, or 0.1422N, (0.09010)(6.g.) + 1422, y = [0.1422 — (0.09010)(s.g.)}/(0.0009303)(s.¢.)} (0.0008303)68.8.)(9) = Fors.g.=08 y= [0.1422 (0.05010)(0.8)/{(0.0008303)(0.8)] = 94.2: mm Fors.g.=1.0 1422 — (0.09010)(1.0)}/[(0.0009303)(1.0)] Fors.g.=1.2 1422 —(0.09010)(1.2)1/(0-0009303)(1.2) ‘Seale spacing is not uniform because buoyant force is not directly proportional to submergence. 644 A typewriter weighs 6 b in water and 8 Ib in oil of specific gravity 0.86. Find its specific weight. (0.86)(62.4)|(V) = W ~ 8 Subtracting the second equation from the fist gives 6—(-8), V = 0.22910"; (62.4)(0.229) = W -6, W=20.31b. y = 20.3/0,229= s8.61b/te, 645 A balloon weighs 270 1b and has a volume of 14 900 f°, Its filled with helium, which weighs 0.0112 Ib/f? at the ‘temperature and pressure ofthe air, which weighs 0.0807 lb/ft’. What load will the balloon support? A=W (0.0807)(14 900) = 270 + (0.0112)(14900) +10ad Load = 76616 646 A small cylindrical drum 32 cm in diameter and 52 cm high, weighing 27.0N, contains perfume (s.g. = 0.83) to ‘depth of 22 em. (a) When placed in water, what will be the depth y to the bottom of the drum? (6) How much perfume can the drum hold and still oat? 1@ B= W_—_ 9.791(y)()(G)/4] =27.0/1000 + [(0.83)(9.79) (0.22) (0.32)/4] y=0.217m or 21.7em (8) 9.79{(0.52)(2)(0.32)*/4] = 27.0/1000 + [(0.83)(9.79)]{(H)(a)(0.32)°/4] k= 0.585m or 58.5em Since h = 58.5 cm is greater than the height of the drum (52cm), the drum will float when full. Therefore, Vaux = (0-52)(1)(0.32)/4 = 0.0418 m?, or 41.8L. 647 Ablock (7 = 1241b/fe) 1 ft square and 9 in deep floats on a stratified liquid composed of a 7-in layer of water ‘above a layer of mercury. (a) Determine the position of the bottom of the block. () Ifa downward vertical force of 260 Ibis applied tothe center of mass of this block, what isthe new position ‘of the bottom of the block? BG), =W. Let x= depth into mercury below water-mercury interface. [(13.6)(62.)}(1)(1(2)} + 62.4{(1)(1)()] = (124)[(1)(1)(&)], * = 0.0667 ft, or 0.800 in. (b) In this case the top of the block will be below the water surfce, ence, [13.6462 ){)AN=)} + 62-4 ()N 2) = ONIN B) + 260, = 0.389, 648 Twospheres, each 1.3m in diameter, weigh SKN and 13 kN, respectively. They are connected wit a short rope and placed in water. What i the tension (7) inthe rope and what portion ofthe lighter sphere protrudes from the water? ' I For the lower (heavier) sphere, the buoyant force and Tact upward and its weight acts downward. Hence, ER =0, K=9.79[()(x)(1.3/2))] = 11.26 KN, 11.26 + T 13 =0, T= 1.74KN. For the upper (lighter) sphere, ‘the buoyant force acts upward and its weight and T act downward. Hence, F, — 5— 1.74=0, F, =6.74kN. Portion above water = (11.26 ~6,74)/11,26~=0.401, or 40.1 percent of volume. 649 A board weighing 2.2 Ib/ft and of cross-sectional area 8 in* dips into oil as shown in Fig. 6-21. If the hinge is frictionless, find 8. The forces acting on the board are shown in Fig. 6-21. W = (2.2)(11) = 24.2 Ib; F, = (53)[(sh)(2)] =2.944x, E Mange = 0, (24.2)[(4)(sin 6)] ~ (2.944x)[(11 — x/2)(sin 8)] = 0, 1.4727 ~ 32.8x + 133.1 = 0; x, = 16.53 ft and ost 62 63 BUOYANCY AND FLOTATION 0 119 x= 5.AT ft. Using x = 5.47 ft (since x = 16.53 fis impossible for this situation), cos @ = 5/(11 ~ 5.47) = 0.90416, O=253. Fig. 6-21 ‘A.cube 2.2ft on an edge has its lower half of s.g. = 1.6 and upper half of s.g. = 0.7. It rests in a two-layer fiuid, with lower s.g. = 1.4 and upper s.g.= 0.8. Determine the height h of the top of the cube above the interface (Gee Fig. 622). ' R=w [(1.4)(62.4)]{(2.2)2.2)(2.2 — A] + 10.8)(62.4 112.292.2908) (4.6)(62.4)]{2.2(2.2)72)] + [(0.7(62.4)112.202.22)) Tp 22tt bh | —_— y intertace een sget4 k— 22tt—>| mes Determine the volume and density of an object that weighs 4N in water and $N in an alcohol of s.g. 0.80. 1 B= W__[(9.79)(1000)](Vorjen) = Wonjea 4 [(0-80)(9.79(1000)] Von) = Water 5 Subtracting the second equation from the frst gives 9790Vijai ~ 7832Vonjea [(9.79)(1000)}(0.0005107) = Woejar = 4, Wetnr = 9.000 N; ¥ = 9.000/0. 00051 17623 1 yea M6kelm 1 * 5 Typ 0200557 mes With how many pounds of concrete (y= 25 kN/m") must a beam of volume of 0.2 m? and s.g. = 0.67 be coated ‘to insure that it sinks in water? B&W 9.79)(0.2) + 9.79Vanre = {(0.67)9.79)]0.2) + 25Veonte Vemma 0.04248 m? Woonawe = (0.04248)(25) = L.062KN or 1062N or 1062/4.448= 23915 “The gate of Fig. 623 weighs 160Ib/ft normal to the page. Its in equilibrium as shown. Neglecting the weight of the arm and brace supporting the counterweight, find W (weight in air). The weight is made of concrete, sg. =2.50. 1 Fur ha ~ (62-4YG NOVO] = 112310 Mage w= 89810 (1123)(8) ~ (WG sin 30°) = 0 ‘This is the submerged weight. B= W— 62.4Vecmcrte = [(2-50)(62.4)](Veoncee) = 898 Veonerne = 9-594 £0? Weems = {(2-50)(62.4)](9.594) = 1497 Ib 120 0 CHAPTER 6 Fig. 623 ‘A wooden pole (s.g. = 0.55), 550mm in diameter, has a concrete cylinder (6. = 2.50), $50 mm long and of the same diameter, attached to one end. Determine the minimum length of pole forthe system to float vertcaly in static equilibrium. The system will oat at minimum length of wooden cylinder as shown in Fig, 624. K = W, (ANCE + 0.550) = (0.559 1(LA) + (@.50)(7)](A)(0.550), L = 1.833 m, LZ } L | sss0ss 4 sso] —sqn850 inn ane eee econ A hydrometer weighs 0.040N and has a stem 7 mm in diameter. Compute the distance between specifi gravity ‘markings 1.0 and 1.1. I From Prob. 6.26, ne Gained) ‘What is the weight ofthe loaded barge in Fig. 6-257 The barge is 7m in width. 1 RaW 9,79{(1{(14)(2.4) + 2)(2.4)(2.4)/2)} = WW =2359KN sD Fig. 625 In Fig. 626, a wedge of wood having specific gravity 0.66 supports a 160-Ib mermaid (not shown). The wedge is 3 fe in width, What is depth a? I The 160-1 force and the weight ofthe wood (W) act downward on the wedge, while the buoyant force (F) acts upward. EF, = 0, F ~ 160 W = 0, 62.4{(2)(3)(¢)(d tan 30°)/2] ~ 160 {(0.66)(62.4)]((2)3)@1G)/tan 30°Y/2) = 0, d= 2.44 ‘The tank in Fig. 627s filed brimfull with water. Ifa cube 700 mm on an edge and weighing S30N is lowered slowly into the water until it floats, how much water flows over the edge of the tank? Neglect sloshing, etc. 1 =W. Let h= the depth to which the cube will sink in the water. [(9.79)(1000)]{(0.700)(0.700)(h)] = 530, ‘h=0.120.m, Vaunes [(0:700)(0.700)(0.120)] = 0.0588 m?. This is the amount of water that will overflow. 69 “a 6a BUOYANCY AND FLOTATION 0 121 Som Fig. 627 ‘A stone cube 280 mm on a side and weighing 425.N is lowered into a tank containing a layer of water over a layer of mereury. Determine the position of the block when it has reached equilibrium. Youu = 425/(0.280)" = 19.360 N/m. Since the cube is heavier than water but lighter than mercury, it will sink beneath the water surface and come to rest at the water-mercury interface, as shown in Fig. 6-28. F, = W, 9.794 (0.280)(0.280)(0.280 — x)] + [(13.6)(9.79)}{(0.280)(0.280)(x)] = 0.425, x = 0.0217 m, or 21.7 mm. Thus, the bottom of the cube will come to rest 21.7 mm below the water-mercury interface. ed oe Aniceberg (y = 9 kN/m*) floats in ocean water (7 = 10 kN/m’) with 3000 m of the iceberg protruding above the free surface. What is the volume ofthe iceberg below the free surface? ' = W > 10000V ine = 9000 Vie + 3000) Vining = 27 000m? ‘A rectangular tank of internal width 7m, partitioned as shown in Fig. 6-29, contains oil and water. (a) If the oil's specific gravity is 0.84, fin its depth h. (6) If a 900-N block of wood is floated in the oil, what is the rise in free surface of the water in contact with air? H (0) Pan ((0.84)9.79)|() + (9.79)(3) ~ 9:79) 4) = Pas h = 1.190 m. (6) Let h'= the new valu of h with the 900. block in flotation, Since the volume of oil does not change, (1.190)(0.5)(7)= (h’)(0.3\(7) = ‘00/{(0.84)(9.79)(1000)], b= 1.221 m. Ifthe oil-water interface drops by a distance 6, the free surface of water with air will rise by 5/2. Pssm + [(0.84)(9.79)|(1.221) + 9.79(3 ~ 5) — 9.794 + 5/2) = Pstm, 5 = 0.01709 m, (0F 17.09 mm. The free surface ofthe water wil ie by 17.09/2, or 8.54mm. 122 0 CHAPTER 6 6.2 663 ‘A balloon is filled with 3000 m’ of hydrogen having specific weight 1.1 N/m’. What lift does the balloon exert (@) atthe earth’s surface, ifthe balloon weighs 1400 N and the temperature is 15°C; (b) at an elevation of, 10 km, assuming that the volume has increased 6 percent? I. From Table A-8, yuy~ 12.01 N/a at elevation 0 and 4.04 N/m’ at elevation 10000. ZF, =0, B.— Woatom ~ Wa 0. @ (22.01)(3000) ~ 1400 ~ (1.1)(3000)—lift= 0 Lift=31330N- or 31.33kN oy 4,04{(1.06)(3000)] — 1400 - (1.1)(3000) —lift=0 Lift=8147N or 8.15KN ‘A wooden rod weighing 4 Ibis hinged at one end (Fig. 6-30). The rod is 9ft long and uniform in eross section, and the support is 4ft below the free surface of a freshwater pond. At what angle a will it come to rest when allowed to drop from a vertical position? The cross section of the stick is 1.4in* in area. I The forces acting on the beam are shown in Fig. 630. F,=62.4[(9—0)(1.4/148)] =5.460-0.6067e SS M,=0 (4.5 c0s a) ~ (5.460 ~ 0.6067)[(9~ e)/2|(cos a) =0 ~0.303e" +5.46e ~ 6.57 =0 sin @=4/(9—e)=4/(9-1.297) =051928 = 31.3" Fig. 6:30 {A block of wood having a volume of 0.034 m* and weighing 300N is suspended in water as shown in Fig. 6-31. ‘A wooden rod of length 3.4m and cross section of 2000 mm is attached to the weight and also to the wall. If the rod weighs J6N, what will angle 0 be for equilibrium? Fue ={(0.79)(1000)]0.034) =333N (Fou = [(9.79)(1000)]{(AC)(2000/107)] = 19.584CN DM=0 333(3.4 cos 6) + (19.58AC){(AC/2) + (38)/sin 6](cos 6) ~ 300(3.4 2s 8) ~ (16)3.4/2}(c0s 6) = 0 AC =3.4~(B)/sin 333(8.4.008 6) + 19.58(3.4 — (Al8)/sin 6] % {(34— (B)/sin 0)/2+ (2B)/sin 8} (c0s 8) ~ 300(3.4 cos 8) — (16)(3.4/2)(c0s 6) =0 4.341 = [3.4 - (@By/sin OIL1.700 + (28)/(2sin )] 4.341 = 5.780 ~0,048/sin* 0 sin? 0=0.033357 sind =0.18264 = 10.5° /we 2 Fig. 631 BUOYANCY AND FLOTATION J 123 ‘A barge with a flat bottom and square ends has a draft of 6.0 ft when fully loaded and floating in an upright position, as shown in Fig. 6-322. The center of gravity (CG) of the barge when fully loaded is on the axis of symmetry and 1.0ft above the water surface. Is the barge stable? If itis stable, what is the righting moment ‘when the angle of heel is 12°? B_MB=1/V, = ((42)(25) /12\/[(25)(42)(6)] = 8.68 ft. Therefore, the metacenter (mc) is located 8.68 ft above the center of buoyancy (CB), as shown in Fig. 6-32b. Hence, it (the metacenter) is located 8.68 - 3— 1, or 4.68 ft above the barge’s center of gravity and the barge is stable. The end view of the barge when the angle of hee! is 12°is shown in Fig. 6-32c. Righting moment = (F,)(x), F, = 62.4{(25)(42)(6)] = 393 120 Ib, x= (sin 12°)(distance from me to CG) = (sin 12")(4.68) = 0.973 f, righting moment = (393 120)(0.973): 382.500 Ib ft (@) Top view 1 ar 4 t 250 (©) End view Fig. 6320) Water 124 0 CHAPTER 6 water Fig. 63210) 6.66 Would the wooden cylinder (s.g. = 0.61) in Fig. 6-33a be stable if placed vertically in oil as shown in the figure? I The first step is to determine the submerged depth of the cylinder when placed in the oil. K; = W, {(0.85)(9.79)][(D)(z2(0.666)*/4)] = (0.61)(9.79)][1-300)( +e) (0.666)"/4)], D = 0.9333 m. The center of buoyancy is located at a distance of 0.933/2, or 0.466 m from the bottom of the cylinder (see Fig. 6-336). ‘MB = 1/V,= ((x)(0.666)'/64)/[(0.933)(z)(0.666)'/4] = 0,030 m. The metacenter is located 0.030 m above the ‘enter of buoyancy, as shown in Fig. 6336. This places the metacenter 1.300/2 — 0.466 — 0.030, or 0.154m, below the center of gravity. Therefore, the cylinder is not stable. BUOYANCY AND FLOTATION 0 125, 666m iumeter Oil sg = 085) Figure 6-34a shows the cross section of a boat. The hull of the boat is solid. Show if the boat is the boat is stable, compute the righting moment when the angle of heel is 10°? FMB =1/V,=((20(10)/12/[(10)(5)(20)] = 1.67 ft. Therefore, the metacenter is located 1.67 ~0.5, or 1.17ft above the center of gravity, as shown in Fig. 634, and the barge is stable. The end view of the barge when the angle of heel is 10° is shown in Fig. 634c. Righting moment = (F5)(x), F, = 62.4{(10)(5)(20)] = 62.400 Ib, x= (sin 10)(1.17) = 0.203 ft, righting moment = (62 400)(0.203) = 12.670 Ib ft. (©) Top view |~——0n. Mig. 6342) 126 0 CHAPTER 6 Fig. 634(6) k ; Fig. 6-34(c) 6.68 A solid wood cylinder has a diameter of 2.0 ft and a height of 4.0 ft. The specific gravity of the wood is 0.60. If the cylinder is placed vertically in oil (S.g. = 0.85), would it be stable? HK =W, [00.85)(62.4)]l(Dx)(2)/4] = [(0.60)(62.4)]{(4)(x)(2)*/4], D = 2.82 ft. The center of buoyancy is located at a distance of 2.82/2, or 1.41 ft from the bottom of the cylinder (see Fig. 6-35). MB = 1/V, = [C=)(2)*/641/12-82)(22)(2)'/4] = 0.09 f. The metacenter is located 2 ~ 1.41 — 0.09, oF 0.50 ft below the center of sravity, as shown in Fig. 6-35. Therefore, the cylinder is not stable. siinieemenacniaaiel 6 on BUOYANCY AND FLOTATION 0 127 4 ft 041 (sg. = 0.85) Fig. 635 ‘A wood cone floats in water in the position shown in Fig. 6-36a. The specific gravity of the wood is 0.60. Would it be stable? I The center of gravity is located #, or 2.50 in from the base of the cone or 7.50 in from the tip, as shown in Fig. 6:365. Woe = [(0.60)(62.4)][(10}(1)(7)'/12]/1728 =2.779 Ib. Let x = submerged depth. z= 0.700x Vs = (x)(2)(D,)°/12 = (x)(-)(0.700x)"/12 = 0.12832? R=W @2A(0.1283x")= 2.779 x=0.703ft or 8.44in Dx = (0.700)(8.44) (0.1283)(0.703)° = 0.0446 for 77.1in? MB = 11V, = ((a)(5.91) 168/77. = 0.7810 “The metacenteris located 0.78 in above the center of buoyancy. Hence, the metacenter is located 7.50 ~ 6.33 ~ 0.98, of 0:39 in below the cone’s center of gravity, and the cone isnot stable. 91 in A block of wood 6 ft by 8 ft floats on ol of specific gravity 0.751. A clockwise couple holds the block in the position shown in Fig. 6-37. Determine the (a) buoyant force acting on the block and its position, (6) magnitude of the couple acting on the block, and (c) location of the metacenter for the tilted position. H @ B=W =((0.751)(62.4)][10)(4 + 4(4.618)/2} = 86561. F, acts upward through the center of gravity O” of the displaced oil. The center of gravity ies 5.333 ft from A and 1.540 from D, as shown in Fig. 637 AC= AR+RC=AR + LO’ = (5.333)(c0s 30?) + (1.540)(sin 30?) = 5.388 ft. Hence, the buoyant force of {8650 Ib acts upward through the center of gravity ofthe displaced oil, which is 5.388 ft to the right of A. () One method of obtaining the magnitude of the righting couple (which must equal the magnitude ofthe ‘external couple for equilibrium) isto find the eccentricity e. This dimension is the distance between the two parallel, equal forces W and F, which form the righting couple. e = FC: (6.333)(cos 30°) + GR sin 30° = 4.619 + (0.691)(sin 30?) = 4.964 ft, (€656)(0.424) = 3670 t-Ib. (€) Metacentrc distance MG = MR ~ GR = RC/sin 30° ~ GR = 0.770/sin 30° - 0.691 128 0 CHAPTER 6 d= Fig. 63640) on on on BUOYANCY AND FLOTATION 0 129 Ail lengths in feet, Fig. 637 A rectangular scow 7 ft by 18 ft by 32 ft long; its load of garbage has its center of gravity 2 ft above the watering, a3 shown in Fig. 638. I the cow stable for this configuration? HMB =1/V, = ((32)(18)/12)/(5)(18)(32)] = 5.40 ft. The metacenter is located 5.40 ft above the center of ‘buoyancy, which is 2.5 ft above the bottom of the scow. Hence, the metacenter is located at 5.40 ~ 4.5, or 0.90 ft above the center of gravity, and the scow is stable. =. ees In Fig. 6-39, a scow 20 ft wide and 60 ft long has a gross weight of 225 tons. Its center of gravity is 1.0ft above the water surface. Find the metacentric height and restoring couple when Ay = 1.0ft. I 5=W, 62.4((60)(20)(4)] = 225)(2000), h = 6.01. To locate CB’, the center of Buoyancy in the tipped position, take moments about AB and BC. (6.01)(20)(x) = (6.01 ~ 1.0)(20)(10) + [(1.0 + 1.0)(20)/21@), x = 9.45 ft; (6.01)(20)(y) = (6.01 ~ 1.0)(20){(6.01 - 1.0)/2] + [(1.0 + 1.0)(20)/2]{(6.01 — 1.0) + (1.0 + 1.0)/3},, 1 =303 By similar triangles AEO and CB'PM, by CBP 1.0 _10-9.45 b/2) MP 20/2 CG is 6.01 + 1.0, 0 7.01 from the bottom. Hen 01 - 3.03 = 3.98, MCG = MP - TGP 5.50~ 3.98 = 1.52.. The scow i stable, since MCG is positive. Righting moment {(225)(2000)}(1.52)[1/ VP? +. 1.0 = 68 060 ft - 1b. MP =5.508t ‘Fig. 6-39 ‘What are the proportions of radius to height (ra/h) of a right-circular cylinder of specific gravity s.g. so that it will loat in water with end faces horizontal in stable equilibrium? 130 0 CHAPTER 6 6m 6.75 F see Fig, 640. A Wo Hhyara) = [9 (harrs) a= (58. )(H) hy=(g)()2 —- MG=MB-GB MB = 1/V, = (2eri/4)(hy2erd) = r3/(4hy) = PILCAY(6-8.(0)) 2 (SB Mh)I2=(M\A—-Sg.)/2 MG = rA{(4)(6.2.)(H)]— HY -8.8,)/2 2 Hizey begs cael oes geal Fig. 640 ‘The plane gate in Fig. 6-41a weighs 2.1 kN/m normal to the paper, and its center of gravity is 2m from the hinge at O. (a) Find h as a function of @ for equilibrium of the gate. (b) Is the gate in stable equilibrium for any values of 67 Consider a unit width of gate, as shown in Fig. 6-410, @ FayhA —— R=[(9.79)(1000)K(4/2)(h/sin 0) = 4895K"/sin 6S Mo=0 (4895h?/sin 6){(h/sin 6)/3]~2100{($)(cos 6)]=0 h*=2.574sin? @cos @ fh =1.370(sin® 80s 6)" (8) From part (a) E Mo = (1632h)/sin? 9 — 4200.cos 8, dd 64h? sin? 6 c0s 0 + 4200 sin 8. Substituting h = 1.370(6n? 8 c0s 8)" [from part (a)}, dM /d@ = ~(3264)(1.370)"(cos? 8/sin 8) + 4200 sin @ = ~(8393)(cos* 6/sin 8) + 4200 sin 8. For stability, dM/d0 <0, in which case 4200 sin 6 < 8393(cos? 8/sin 8), (0.500 sin 0 0, (6.7/2 ~ (0.5)(6.g.) + 0.08333 > 0. This condition is true (ie., the cube is stable) for s.g. > 0.789 and s.g. <0.211. CHAPTER 7 Kinematics of Fluid Motion m1 12 13 14 1s 132 ‘A nozzle with base diameter 75-mm and a 35-mm-diameter tip discharges 12 L/s of uid, Derive an expression for fluid velocity along the nozzle’s axis. Measure distance x along the axis from the plane of the larger diameter. Tit 0.040%/L, sives vin m/s. length of nozzle and D = diameter of nozzle at any point. D = is ~ (ss — i) (&/L) =0.075 ~ Q/A = 0.012/[x(0.075 ~ 0.040x/L)"/4] = 1.528/(0.70 ~ 0.40x/L)*. Note: x and L in millimeters ‘What angle a of jet is required to reach the roof of the building in Fig, 7-1 with minimum jet velocity vo at the nozzle? What is the value of uo? A dyldt = ~g, dy/dt~ ~gt +6). Att=0, dy/dt = vo sin a. Therefore, ¢; = vosin a, and dy/dt vosin a, y= ~gt/2+ tuesin a+ cs. Att=0, y= 0. Therefore c:=0, andy = ~g'/2 + vosin a, 15 Li(vp00s 2) H = ~g[L/ (vp 00s @)F/2 + [L/(vpcos @)|(vp sin a) o Let F = gL?/(2v}). Then, from Eq. (1), F = (cos «)(L sin a — H cos a) = L cos a sin a ~ H cos" a. Find ‘maximum F for minimum vp. dF der = L(cos? « ~ sin? a) + 2H sin @cos @= 0 2H/L= ~(cos' a ~sin® a)/(sin e 608 @) @esyr2 ‘Substituting into Eq. (1), 28 = ~(9.807)[24/(vo cos 69.7°)['/2 + [24/ (vo C0 69.7°)|(ve sin 69.7"), vo = 25.2 m/s. 2e0t 2a 2eot2a a =69.7° Fig. 7-1 Given the velocity field, V(x, y, 2, 1) = (Sxy? + Di-+ 22 +8)j + 18k m/s, with x, y, z in meters and ¢ in seconds. Calculate V(9, ~2, 1,4). What is the magnitude of this velocity? ' V=((S)ON-2P + 4+ [OIC + 8]j + 18k = 1844+ 10) + 18k m/s IV = Vige + 10+ TF = 185 m/s Boldface letters are used herein and hereafter to denote vectors. Ne ‘The velocity components in a flow of fluid are specified as v, = 4xt + y*z + 14 m/s, v, = 2xy?+ P+ y m/s, and u,=3-4 2ty m/s, where x, y, and 2 are given in meters and fin seconds. What is the velocity veetor at (2,4,3) mat time ¢= 45? What is the magnitude of this vector at this point and time? ' V=[(4)(2)(4) + 7G) + 14H + 12 2)(4)? + 4 + 4]j +13 + (YA) (A) Ihe = 945 + Bj + 35K m/s IV] = VOEH BEF 3S? = 131 m/s Given the velocity feld V = (Se)I+ (I5y + 11) + (197)km/s, determine the path ofa particle which i at (4,6,2)m at time #= 3s. 16 ” KINEMATICS OF FLUID MOTION J 133 1 wo sy +11 @ v,=dz/dt= 197 @ From (1), dx/x = Sde, Inx = S-+ cy. Ate=3, x= 4, Hence, In4 = (5)(3) +¢4, ¢ = —13.6. Ins =5t—13.6 o From (2), dy/(1Sy + 11) = dt, In (15y +11) = 154-4 c,, Att=3, y =6, Hence, In [(15)(6) + 11] = (15)(3) +e, =~ 40.4, in(sy +10 ‘ From (3), dz = 19" dt, 2 = 19P'/3+ cs. Att=3, 2 =2. Hence, 2197/3 ~ 1609 6 ‘Add Eqs. (4) and (5) to get Ing + In (I5y +11) = 200-540 o Solve for tin Eq. (6): = [(z +169)(3)I!. Substitute this value of t into Eq. (7): Inx +In (15y + 11) = 204(z + 169)(4)]"” ~ 54.0, In ()(ASy + 11)] = 10.81(z + 169)" ~ 54.0. ‘An incompressible ideal fluid flows at 0.5 fs through a circular pipe into a conically converging nozzle, as shown in Fig. 7-2. Determine the average velocity of flow at sections A and B. I Asa frst step, an approximate flow net is sketched to provide a general picture of the low. Since this isan axially symmetric flow, the net is not a true two-dimensional flow net. At section A, the streamlines are parallel, hence, the area at right angles to the velocity vectors isa circle. Thus, v= Q/A.4 = 0.5/{(=r)(B)'/4] = 1.43 fs. ‘At section B, however, the area at right angles to the streamlines is not clearly defined; it is a curved, dish-shaped section. As a rough approximation, it might be assumed to be the portion of the surface of a sphere of radius 2.0 in that is intersected by a circle of diameter 2.82in. Up = Q/Ap = O/(2arh) = 0.5/{2)()(4)(0.59/12)] = 9.71 fs. Fig. 72 ‘Water flows at 6 gal/min through a small circular hole in the bottom of a large tank. Assuming the water in the ‘tank approaches the hole radially, find the velocity in the tank at 2, 4, and 8 in from the hole. I The area through which flow occurs is a hemispherical surface, with A = 2x0. Q = 6/{(7.48)(60)] = 0.01337 ft'/s, v = Q/A. At 2in from the hole, v = 0.01337/{(2)(x)(4)'] = 0.0766 ft/s. At 4in from the hole, v= 0.01337/{(2)()(i4)'] = 0.0192 ft/s. At Bin from the hole, v = 0.01337/{(2)(+")(4)'1 = 0.00479 ft/s. 194 0 CHAPTER 7 78 Given the eulerian velocity-vector field V(x, y,z, 1) = 3 +22] + 1%, find the acceleration ofa particle, av_ av, (av, Vv, av 2 ' + (user h ew) wast vex wap ou, aw wv av, vee Skea ase Taagk Vass ave Ge 7 HF YH) + GEH) + (x2)(Rerk) + (O*YGH) = B+ Bez + eey?M + (y? + DayztIe If Vis valid everywhere as given, this acceleration applies to all positions and times within the flow field 7.9 Flow through a converging nozzle can be approximated by a one-dimensional velocity distribution u = u(x). For the nozzle shown in Fig. 7-3, assume the velocity varies linearly from u = vp at the entrance to u = 3p at the exit: u(x) = ug(1 + 2x/L); 3u/@x =2v4/L. (a) Compute the acceleration du/dé as a general function of x, and (6) evaluate du/dr at the entrance and exit if vg = 10 ft/s and L = 1 ft. Bin eee Ment BevBtenBen wen(ied) Bate a0 [oo FNC) 040-0) (6) Atthe eftrance, where x= 0, du/dt = [(2)(10)'()]{1 + 2)(0)/() ddu/dt = ((2)(40)°)IA + (2)(1)/)] = 600 fs Aya= oe VY 7 =o F733 7.10 A ttwo-dimensional velocity field is given by u = 2y*, v=3x, w=0. At (x, y, 2)=(1, 2,0), compute the {@) velocity, (b) local acceleration, and (e) convective acceleration. 1@ V=HQDO'1 +O = 44+ 35 © ® +o en-08 079094 0014) +00)=129 ou, 20, ou ust ot Se = 0+ YI) + (HO) + a= (12)(1)Q)i-+ (2)4 = 21+ 245 6? TAL For the velocity field described in Prob. 7.10, at (1,2,0) compute the (a) acceleration component parallel to the velocity veetor and () component normal to the velocity vector. I From Prob. 7.10, V = 4i +3) and a= 241 + 24j at (1,2,0). (@) Tangential accelerati B=VIVIRH+H a an, = (28 +.24))- (f1+ 9) = 19.24 14.4 = 33.6 units parallel to V m2 13 14 KINEMATICS OF FLUID MOTION 0 135 (b) From Fig. 7-4, the angle 0 between V and a is determined by cos 0 = a,/lal = 33.6/(24 + 24°)" = 0.98995, 8 =8.13°, a, =[al sin O = (24? + 24°)!%(sin 8.13°) = 4.80 units normal to V. On Mig. 74 Is the flow with velocity field V = 2tri ~ yj + 3xzk steady or unsteady? Is it two- or three-dimensional? At the point (x,y, 2) 0), compute the (a) total acceleration vector and (b) unit vector normal to the acceleration. 4 Flow is unsteady because time & appears expliily. low is three-dimensional because u,v, w #0 De + (2te)(24) + (~Fy)(0) + (Bxz)(0) =2e + 4 Dry + (2e2)(0) + (—Fy)(—F) + Bxz)(0) = —2y7 + Hhy 0+ (2ex)(3z) + (—F°y)(0) + (3x2)(3e) = 6urz + 9x7 a fe Sa ae @ . At point (2, ~2,0), du/dt = (2)(2) + (4(2)Q) = 4-48, du/di = —(2)(0(—2) + (F)(-2) = 41 —2¢4, dw/de = (O20) + CAPO) = 0. Hence, a= (4+ 8F)i+ (4t 21. (6) The unit vector normal to-a must satisfy a-m=0=n,(4+ 87) +7,(4~ 21°) + n,(0) plus ni +n +n2= 1. A special case solution isn =k. For steady flow through a conical nozzle, the axial velocity is approximately u = U,(1 ~ x/L)-*, where Uy is the entrance velocity and L is the distance to the geometrical vertex of the cone. Compute (a) a general expression for the axial acceleration du/dr and (6) its values atthe entrance and at x = 2m, if Uy=4m/s and L= 3m. WG Boaden enone luli) [-g) col p]oveen (2) A) (8) At entrance (x = 0): = S13] = 10.7 mys? ai Aty=2m: Sle = 3) *[2)4)"/3] = 2592 m/s ‘A two-dimensional velocity field is given by V = (x? — 2y? + 2x)i— (xy + y)j. Atx =2 and y =2, compute the (@) accelerations a, and a,, (B) velocity component in the direction @ = 32", and (e) directions of maximum acceleration and maximum velocity. du_ | Ou, du, au, ou : oe ' Sit mg POS twas = OF Oy" + 2)Rx +2) + (Bay —V)(—4y) +0 136 0 CHAPTER 7 as 116 a7 Bag wR 4 Rt Wm O+ (= 292 + 20-39) + (Bay —I—SE =) +0 a ae an ay ai @ 0, = [2 — Q2P+ DAMA) +2] + [(-3@)2) — 2M-42)] = 12 4, = [2 ~ (2)2)* + 2YQIIM—3V2 + (3) 2VR) — 2(-3)Q) ~ 1] = 98 o Ve = Vite VP — P+ QM [G)QV2) +2 = 01 145 hog = 0.8481-+0.530} tax = (Oi 14))(0.8484 + 0.530) = ~7.42 (0) Direction of a: a = arctan (98/112) = arctan 0.87500 = 41.2°. Direction of V (direction of —}): B = —90". ‘The velocity field in the neighborhood of a stagnation point is given by u = Usr/L, v = —Usy/L, w =0. (@) Show that the acceleration vector is purely radial. (b) If L = 3, what is the magnitude of Uy if the total acceleration at (x, y)=(L, L) is 29 ft/s"? ' Pe Ua) (4) geo aaa \(2)+o+0- Ts + EEN oon ita = (Ui/L'\(x1 + yi) = (UBIL2Y(0). (Hence, purely radial.) @ ‘a(L, L) = (U3 L2) \LA + Lj] = Usy3/L. If L=3 ft and [al = 29 ft/s", 29 = U3V2/3, Up=7.84 ft/s. ) 1m? at a uniform speed of 4 m/s. Express the u and v at t= 0. See Fig. 75. ‘A particle moves around the circular path 23+ y components as functions of time, assuming 0 1 u=u,cos @~uysin@=—4sind m/s v=u,sinO +v,.cosO=+4c0s@ m/s But vp = rd, 4= 30, 0 =; hence u= —4sin fr and v= +4 00s uz 4 . On ee? 4 Fig. 7-5 ‘A perfect fluid flows from the bottom of a large tank through a small hole at the rate Q = 0.9L/s. Ifthe fuid ‘flows radially toward the hole with the same volume flow across every section, compute the convective acceleration at points 100 mm and 200 mm from the hole. I Consider the radial velocity (v,): v, = -Q/A, = -Q/(2nr*) (A, is the area of a hemisphere). Pa At r=0.100.m, a, = ~(0.9 10-%)/[(2)(2)*(0.100)'] = ~0.0041 m/s?, or ~4.1 mm/s", Atr =0.200m, a, ~0,0081/32 = ~0,000128 m/s", or ~0.128 mm/s? 738 "9 10 12 KINEMATICS OF FLUID MOTION 2 137 Given the velocity field V(x, y, z, t) = 10x44 — 20yxj + 100rk, determine the velocity and acceleration of a particle at position x = 1m, y=2m,z=5m, and=0.1s. 1 “V = (10)(1)*1— (20)(2)(1)§ + (100)(0.1)k = 104 — 40} + 10K m/s v1, Vv, Ov ms, y 2, = Se grt ag tse) ‘a= 100k + {(10x7)(20e4 — 20H) + (—20yx)(—20eH) + (1002)(0)] = 200% + (~200r%y + 400)" + 100k At position x=1m, y =2m, z= 5m, and ¢=0.1s, a= (200)(1)" + [(-200)(1)*2) + (400)(2)(1)"1) + 100k = ‘2001 + 400 + 100K m/s". ee tenet econ ternenters IF Since the flow at section A is uniform and also steady, a, = 0. fe pre reraerentdn oe Yay a ay For point B on the axis of the pipe at section B, 0; hence, ‘The effective area through which the low is occurring in the converging section of the nozzle may be expressed approximately as A = 2zthr, where h = r(1— cos 45°) = 0293+ and r is the distance from point C. Thus A= (2n)(0.2937*) = 1.847, and the velocity inthe converging nozzle (assuming the streamlines flow radially toward C) may be expressed approximately as v = Q/A =0.50/(1.84r). At section B, r = 2in = 0.167 ft; hence, v= 0,50/[(1.84)(0.167)"] = 9.744 fps. BV__aV__f(-20.50))__@x0.50) Ne aan %. -X- -(e} Tan wrery~MET PIA — ay —uST= (.748)(116.7)= 1137 ts ‘A two-dimensional flow field is given by u = 2y, v =x. Sketch the flow field. Derive a general expression for the Yelocity and acceleration (x and y are in units of length L; u and v are in units of L/T). Find the acceleration in the flow field at point A (x =3.5, y =1.2) I The flow field is sketched in Fig. 7-6a. Velocity components u and v are plotted to scale, and streamlines are sketched tangentially to the resultant velocity vectors. This gives a general picture of the flow field. ou, at veaeerearesy® acuertaa eager aaudevaryarenO=2 a4 tay @)=2e=7OLIT (ay), = 29 = 24LIT? Lene + (an)5]' = (07.0)? + 2.4) = 7.4/7? ‘To get a rough check on the acceleration imagine a velocity vector at point A. This vector would have a magnitude approximately midway between that ofthe adjoining vectors, or V4~4L/T. The radius of curvature of the sketched streamline at A is roughly 3L. Thus (a), ~4"/3 ~5.3L/T°, The tangential aceleration of the particle at A may be approximated by noting that the velocity along the streamline increases from about 3.2L/T, where it erosses the x axis, to about 8L/T at B, The distance along the streamline between these two points is roughly 4L. Hence a very approximate value of the tangential acceleration at A is a= (+a) oaavtnd (822) -anusr 7 ‘Vector diagrams of these roughly computed normal and tangential acceleration components are plotted (Fig. 7-66) for comparison with the true acceleration as given by the analytic expressions (Fig. 7-6¢). ‘The velocity along a streamline coincident with the x axis is u= 9.4 x"°. What is the convective acceleration at x= 3.2? Specify units in terms of L and T. Assuming the fluid is incompressible, is the flow converging or diverging? bu ' au We 138 0 CHAPTER 7 12 723 Flow field hone tt Fig. 7-6(@) AW a=70ur® Fig. 7-6(6) “D5 Fig. 7-60) since aun oe 29(S) aa ee Atx=3.2, a, =(3)(3.2)-% + (G2)! = L.61L/7*. For incompressible flow, the flow is converging. A large hemispherical vat has a small taphole centered on its lowest point. Ideal liquid drains through the hole according to Q = 11 ~0.5t, where Q isin cubic feet per second and tis in seconds. Find the total acceleration at ‘point 3ft from the center of the hole at ¢= 165. Assume that liquid approaches the center of the hole radially I v= Q/A. The area through which flow occurs is a hemispherical surface, so v = (11 — 0.51)/(2x7"). sa] s 2a’ Aer a Stand = 168, ou (4 — OSNLOM AN AIAD (U0 - O.S\1)1/M@3)"1 — 0.5/1ANHIEN = 0.0107 f/s* ‘Under what conditions does the velocity field V= (a,x + buy +eiz)I+ (asx + bay + ,2)) + (asx + yy +652)h, where a;, d;, etc. = constant, represent an incompressible flow which conserves mass? ' Bu, 2, Ow a a a 0 Flext biy tee) +Z (axe t bay + e.2) +5 (ox + bsy tea2) =0 14 125 126 ro 128 KINEMATICS OF FLUID MOTION 0 139 ora; +b, +¢,=0. Atleast two of the constants a,, b,, and ¢; must have opposite signs. Continuity imposes no restrictions whatever on the constamts by, ¢;, dz, C2, ds, and bs, which do not contribute to a mass increase or decrease of a differential element. ‘An incompressible velocity field is given by w = a(x?—y ‘What must the form ofthe velocity component v be? ‘This is integrated partially with respect to ‘that satisfies the incompressible continuity equation. The function of integration fis entirely arbitrary since it vanishes when v is differentiated with respect to y. ‘An incompressible flow field has u =.x2* and w =.xe"” (dimensional factors omitted). What form does continuity imply for te velocity component 0? 1 au , 90, ow By + f(x, 2) ‘A two-dimensional incompressible velocity field has u = K(1~ e~), forx = L and 0=L ==, What isthe most general form of u(x, y) for which continuity is satisfied and v = vo at y = 0? What are the proper dimensions for the constants K and a? HF Dimensions of constants: (K} = {L/T}, {a} = (1/L}. au , au , aw. av Rratarte ots ton0 Ifu= eat y =0 forall x, then v= vp everywhere. v=f(x) only ele ‘Which of the following velocity fields satisfies conservation of mass for incompressible plane flow? (@) u=-x,u=y () w=3y, v= 3x (© und v= sy @ um3ut, v=3yt © vax ty, veay tet CD) unary’, v= day" Ignore dimensional inconsistencies F tnorder to satisfy continuity, oH By ou av i «@ = and $1 therefore, i does satis continuity. au au rn ® Fao and 2-0 therefore, it does satis comity au 3u — © Has ama B therefore, it does satisfy continutiy au au : @ Saar and nse thereto, it doesnot satisfy continuity. © Seay and Fmx therefor, it doesnot sais contin. o Ba tay? and Baty? therefore it does satiny continuity. If the radial velocity for incompressible flow is given by v, = b cos @/r", b = constant, what is the most general tom fan) at sie ony ra) Pe 450 ' 2 +t Zonet cos 8 140 0 CHAPTER 7 7.29 —_Atwo-dimensional velocity field is given by pes = eevee Pty where Kis constant. Does ths fed satisfy incompresible continuity? Transform these velocities into polar components v and vs. What might the flow represent? ou , vw aKy __2yKe_ ' 8, Ow gq _2eKy +0=0 ax ay az Sry ery ‘Therefore, continuity is satistied. x* + y=’, cos @=x/r, sin @ = y/r. “EOE --# sind +veawo=—[-(3ika) IG) (ep) ) re Hence, in polar coordinates vr, 8) and v9(r, 6), ¥, =0, ve = K/r. (This represents a potential vortex.) cos 8 +u sin 730 For incompressible polar coordinate flow, what is the most general form of a purely circulatory motion, vp =(r, 8, 1) and v, = 0, which satisfies continuity? 1a 18 1a eanrtZen+Bao0 Limos+tZiro-0 LF w)-0 w=s0 731 What is the most general form of a purely radial polar coordinate incompressible flow pattern, v, = v,(r, 8, #) and vp = 0 that satisfies continuity? 2u, a 22@)+(Joro-0 tm-0 u=(@ 13,18 Bot Geor 7.32 Anincompresible steady ow pattern is given by u = x*+ 32" and w = y*—3yz, Whats the most general form of the third component, u(x, y, 2), that satisfies con we ay=0 Rasy ae valy'ar'y thle, 2) 733 wy a Enforce no-stip condition: u(x, 0) = U(0— 0) + f(x) =0, f(x) =0. y ° Fig. 7-7 734 Consider the flat-plate boundary-layer flow in Fig. 7-8. From the no-slip condition v =0 all along the wall y = 0, and u = U= constant outside the layer. Ifthe layer thickness 6 increases with x as shown, prove with 135 136 139 138 KINEMATICS OF FLUID MOTION 0 141 incompressible two-dimensional continuity that (a) the component u(x, y) is everywhere positive within the layer; (6) v increases parabolcally with y very near the wall and (c)v reaches a positive maximum at y = 8 (a) 1f 3 increases with x, the streamlines inthe shear layer must everywhere move upward to satisty continuity. Therefore, 6u/@x <0 everywhere inside the shea layer. Since continuity requires au, 3 Rt? everywhere, it follows that 3u/3y >0 everywhere in the shear layer. (6) Near the wall, u = y f(x), 9u/ay = ~du/ax = — f"(x); therefore, SFG) (parabolic) (©) Aty = (2), 3u/dx =O; therefore 3u/ay = 0, and v = maximum. Layer hee 80) = Fig. 7-8 The axial velocity field for fully developed laminar flow in a pipe is U, = au(1 ~ r°/R?) and there is no switl, ‘¥)~0. Determine the radial velocity field v,(r, ) from the incompressible relation if mais constant and v, = 0 at r= R. (rdenotes radial distance from the pipe's center; R denotes the pipe’s radius.) au, o (8.2) ape esto 1a a o FHM + sl) +n 72 (u)+0+0—0 Sim=0 v, iftv,(R)=0 forall 6, z, v,=0. ‘An incompressible low field has the cylindrical components vg = Cr, v, = K(R?~ ), v,=0, where C and K are constants and r = R, z = L. Does this flow satisfy continuity? What might it represent physically? 12 12 : rie tee (4 Zee (mort Zen +2icee—ry= 0+0+0=0 (satisfies continuity) ‘This flow represents pressure-driven, laminar, steady flow in a rotating tube (fully developed). ‘An incompressible flow in polar coordinates is given by v, = K c0s 8(1 ~b/r?), Up = ~K sin (1+ b/r*). Does this field satisfy continuity? For consistency, what should the dimensions of the constants K and b be? 12 [rom o(i-2)] +12 [-xsno(1+2)]+0=0 rer +736 0=0 (satisfies continuity) 1a 13 ou, Tat 5g) +5. Siam o(1+2)—2oso(1+ Dimensions of constants: (K} = (L/T}, (6) = {22 ‘The x component of velocity is u =x? + 2* + 6, and the y component is v= y* +24, Find the simplest z component of velocity that satisfies continuity. 142 0 CHAPTER 7 739 Istthe continuity equation for steady, incompressible flow satisfied if the following velocity components are involved? ae . 7 . we2tmay tet vextm dey ty? we dey ye ty (x -y)+(-4e +2y) + (-y) =O (Gatisties continuity) 740 For steady, incompressible flow, are the following values of u and v possible? @uady ty v=Gyt3r u=2'+y', v= — dy au , 2v, aw ' HS (@) 4y+6x+0#0 (Flows not possible.) () 4e—42+0=0 — (Flowis possible) 7.41 Determine whether the velocity field V = 3+i + x2j + yk is incompressible, irrotational, both, or neither. I The divergence of this velocity fel is vereLoneSeereZoneo ‘Therefore, this velocity fields incompressible. The cr of this velocity felis tok & & Fl -ey-niem Yaz 9? ‘This isnot zero; hence, the lw field is rotational, not rotational. vxv= 742 If.a velocity potential exists for the velocity field u = a(x*~y*), ‘2axy, w =0, find it and plot it. Since w =0, the curl of V has only one (2) component, and we must show that itis zero. i =F Ba S (tay) 5 (ae? ~ay")=—2ay +2ay =O checks ‘The flow is indeed rotational. A potential exis. To find (x,y), set « e@ Integrate (1) @ Ditlerentiate (3) and compare with (2) $o= 200 +'0) = 2009 « ‘Therefore f’=0, or f = constant. The velocity potential is ¢ = ax°/3 ~ axy? + C. Letting C=0, we can plot the 6 lines as shown in Fig. 7-9. 743 Given the velocity field V= 13x"yl + 18(y2 + x)j + 15k, find the angular velocity vector of a fluid particle at @.3.4)m. a¥,_ aM _y 1(3¥, . ' =7(F- FB) =10- 1) = 0, =5 (34-4) -10-0)=0 13% _3¥ 7 7 o, 3 (Ge F) = 308-138) = — 6.5 99h +O) + (96.527) At point (2,3, 4) m, «= (-9)(3)i + [9 ~ (6.5)(2)"k = -271— 17k rad/s. 14 148 146 a 8 KINEMATICS OF FLUID MOTION 2 143 Mig. 79 Given the velocity field V. yi — (Ax ~ 42)j + 122% m/s, compute the angular velocity field w(x, y, 2) 1 =} (%-%)-10-0~0 ont Bateseye-aet) ax-aeg—essmentl Show that any velocity field V expressible as the gradient of a scalar must be an irrotational field. Af Show curl (grad 9) = 0 out (3812444281) <0 SO20hEO-2OhlES-sOko Since 3°66 /8y 3: = 8°4/82 By, etc., we see that we have proven our point provided the partial derivatives of @ are continuous. Is the following flow field irrotational or not? V = 12x'yi-+ 3x'j + 10k ft/s. For the velocity vector V = 31+ x2] + 17° evaluate the volume flow and the average velocity through the square surface whose vertices are at (0, 1,0), (0, 1,2), (2,1,2), and (2, 1,0). See Fig. 7-10. The surface $ is shown in Fig. 7-10 and is such that m= and dA = dr dz everywhere. The velocity field is 311+ x2] + 9°. The normal component to Sis V-n=V-J = v, the y component, which equals xz. The limits on the integral for Q are 0 to 2 for both dx and dz. The volume flow is thus 0=[yar=[[redede =40units ‘The area of the surface is (2)(2) = 4 units. Then the average velocity is Ver = Q/A = 4.0/4.0= 1.0 unit. At low velocities, the flow through along circular tube has a paraboloid velocity distribution 1U= tga(1 = 7/R?), where Ris the tube radius and tine is the maximum velocity, which occurs atthe tube centerline, (a) Find a general expression for volume flow and average velocity through the tube; (6) compute the volume flow if R = 3.cm and tas = m/s; and (c) compute the mass flow if = 1000 kg/m’. 144 0 CHAPTER 7 19 750 731 As de de (2.1.0) O13 4 Fig. 7-10 F(a) The area Sis the cross section of the tube, and m=, The normal component V- m= V+i=u. Since u varies only with r, the element dA can be taken to be the annular strip dA = 2r dr. The volume flow becomes = [dam ["nea(1-Fanrar Carrying out the integration over r, we obtain Q = Higa stR*, The average velocity is ty = Q/A = WoecAR?/s0R? = lg. The average velocity is half the maximum, which is an accepted result for low-speed, ‘or laminar, flow through along tube. (b) For the given numerical values Q = (8)2(0.03)* = 0.0113 m'fs (© For the given density, assumed constant, r= pQ = (1000)(0.0113) = 11.3 kg/s. For low-speed (laminat) flow through a circular pipe, as shown in Fig. 7-11, the velocity distribution takes the form w= (B/u)(r3— 7), where 1 is the fuid viscosity. Determine (a) the maximum velocity in terms of B, 1, and rp and (b) the mass flow rate in terms of B, 4, and r FB (@) thay, occurs when du/dr = 0. du/dr = ~2Br/p = 0, 7 =0, toe = Bre) a. ® (Pear dr) = 240: sie 8 a 72) If the fuid in Fig. 7-11 is water at 20°C and 1 atm, what is the centerline velocity Up if the tube radius is 20mm and the mass flow through the tube is 1.3 kg/s? # From Prob. 7.49, rit = (p/2)ina( 773), 1.3= FY Uns [()(0.020), ts = Uo = 2.07 m/s. (Actually, tis is unrealistic. At this sz, Nq > 2000, so the flow is probably turbulent.) ‘A velocity field in arbitrary units is given by V = 3x"t~ ay] — 6xzk. Find the volume flow Q passing through the square with comers (x, y, z)= (1, 0,0), (1, 1,0), (1,1, 1), and (1, 0, 1). See Fig. 7-12. H Q=Sf (usa dy dz. Since m=i, V 32°, O = J2 3G) dy dz =3 units. FO KINEMATICS OF FLUID MOTION 0 145 182 For an incompressible fluid, express the volume flow Q across the upper surface in Fig. 7-13 in terms of the inlet ‘velocity Us and the height 6 of the fluid region shown in the figure? ; 2 [uss -['usn(2)ou=vapnt-aa(2)] =upo—ue(2)i.+11= uss—200(2) = UpbS(1 = 2/) = 0.363Uyb5 y=0 Plate (width 6 into paper) Mig. 7-13 183 The velocity profile in water flow down a spillway i given approximately by w= (U,)(y/h)"” where y =0 denotes the bottom and the depth ish (see Fig. 7-14). If Uy = 1.4m/s, h=3m, and the width is 17 m, how long will t take 10° m° of water to pass this section ofthe spillway? ' o- [uda~['w(2)"ay= oooh] = ivadh = GL. 4NI7V)=02.5m/5 1=V/Q = 10'/62.5= 1603s or 26.7 min Mig. 7-14 148 0 CHAPTER 7 154 185 136 181 Find the total derivative, dp/dt, at x = 1 and y =3 of the density field p = 3x° — 4y? corresponding to the velocity field V = (x? y?+x)i— Gay + y)j. O88 By Py y Pg 5 2 y242)02)— ff ' Basu Bay Paw Laos ty +2V90)~ Cry + (+0 Atz= Landy =3, dpldr= (2-3 + 1()(1I-(@)B) + 31(-8G)} = 279 units. A frictionless, incompressible (p = pa) steady flow fields given by V = 3xyi~ 2y%j in arbitrary units, Neglecting ‘gravity, caleulate the pressure gradient and evaluate this gradient at (3, 1,0). av, av, al BS ol udet w+ WZ) = prey + (-27%GHt— hy) +0)=—Vp Yp= pdt 4°) AtG.1,0), Vp = pul(6)(3)(1)4 + AVI] = pull + 49). ‘A temperature field T = Szy* is associated with a velocity field given by u = 2y?, v= rate of change dT /dt at the point (x, y) = (3, 4). 0. Compute the 1 SF + WS = OVW) + 009) +0 10) 30c%y At (3,4), ar = (10) + (30)(3)°(6) = 3640 units a “ake the velocity eld u= a(x? »2), v= —2ary, w =0 and determine under what conditions i is solution to the Navier-Stokes momentum equation. Assuming that these conditions are met, determine the resulting, pressure distribution when zis “up” (g.=0, » dy ' ps, - 2 * Make a direct substitution of u,v, w. 9(0)- 2 + wea ~20) =2a°pe" +37) o) (0) B+ w= 200609 +7?) @ p(-8)- 2+ u)=0 @ ‘The viscous terms vanish identically (although 1 is not zero). Equation (3) can be integrated partially to obtain P= — paz + fix, ¥) @ , the pressure is hydrostatic inthe z direction, which follows anyway from the fact thatthe flow is two- sional (w = 0). Now the question is: Do Eqs. (1) and (2) show that the given velocity field is a solution? ‘One way to find out is to form the mixed derivative &p/(ax 2y) from (1) and (2) separately and then compare them. Differentiate Eq, (1) with respect to y pope bray” ‘Sa pxy ) Now differentiate Eq, (2) with respect tox FP. 8 pagan = — 4a"; Seay atl y= ~Aeony © ‘Since these are identical, the given velocity field is an exact solution to the Navier-Stokes equation. 18 19 18 KINEMATICS OF FLUID MOTION 0 147 To find the pressure distribution, substitute Eq. (4) into Eqs. (1) and (2), which will enable us to find fix, y) Ben -aet pee? 429°) o Faeroe +9) ® Integrate Eq. (7) partially with respect to x fea -Weplat +287) +60) ” Differentiate this wth respect to y and compare with Eq. (8) F_ _atroe'y 4 Fa troy +430) (a0 Comparing (8) and (10), we se they are equivalent if Si) =—2a'py? or ar KO) =4etpy" + wo where C is a constant. Combine Eqs. (4), (9), and (11) to give the complete expression for pressure distribution PCa, y, 2) = pgs — hatp(a'+ y+ 2r7V7) +6 (2) ‘This is the desired solution. Do you recognize it? Not unless you go back to the beginning and square the velocity components: w+ ut Ww Vm ae yt 2'y?) aay ‘Comparing with Eq. (12), we can rewrite the pressure distribution as P+ pV? + pgz=C us ‘The sprinkler shown in Fig, 7-15 on p. 148 discharges water upward and outward from the horizontal plane so that it makes an angle of 6° with the «axis when the sprinkler arm is at rest. It has a constant cross-sectional flow area of Aa and discharges q cfs starting with o = 0 and r= 0. The resisting torque due to bearings and seals isthe constant T, and the moment of inertia of the rotating empty sprinkler head is, Determine the equation for w as a function of time. I Te control volume isthe cylindrical area enclosing the rotating sprinkler head. The inflow is along the axis, so that it has no moment of momentum; hence, the torque ~T, due to friction is equal tothe time rate of change of moment of momentum of sprinkler head and uid within the sprinkler head pus the net efx of moment of momentum from the contro volume. Let ¥; =q/2Ao 24 [por dr + 192 [2222 cos 0-07] ‘The total derivative can be used. Simplifying gives £2 (+ ipAard = pardV 008 8 ~ on) -T, For rotation to start, pqraV, cos @ must be greater than Tj. The equation is easily integrated to find w as a function oft. The final value of wis obtained by setting du/dt = 0 in the equation. ‘Aturbine discharging 10 m?/sis to beso designed that a torque of 1000 N- mis to be exerted on an impeller {uring at 200 rpm that takes all the moment of momentum out ofthe fd. At the outer periphery of the impeller, r= 11m. What must the tangential component of velocity be at this location? 1 T= pQl(rv,)e—(r)s] 10000 = (1000)(10){(1)()m~0] (07) = 1.00m/s “The sprinkler of Fig. 7-16 discharges 0.01 fs through each nozzle. Neglecting friction, find its speed of rotation. “The area of each nozzle opening is 0.001 f. 1 Tne fd entering the sprinkler has no moment of momentum, and no torque is exerted on the system externally; hence the moment of momentum of uid leaving must be zero. Let «be the speed of rotation; then 148 0 CHAPTER 7 708 Fig. 7-15 the moment of momentum leaving is pQsrt1a + pQ:r2¥a in which v, and va are absolute velocities. Then van ba wr, = Q,/0.001 ~ wr, = 10 w and vq = v2 — or = 10 4o. For the moment of momentum to be 00("0 + Fata) =0 oF (1)(10~ @) + (3)(10 — fo) = O and « = 11.54 rad/s, oF 110.2 rpm. — {Cee me a "The velocity profile for laminar flow between two plates, as in Fig. 7-17, is = Man(h=9) ra vew=0 If the wall temperature is T, at both walls, use the incompressible-flow energy equation to solve for the temperature distribution T(y) between the walls for steady flow. ' K a (Hey = 2hy?+ OOP + Gi] Since dT /dy = 0 at y=h/2, C 2. (a [ey 2" + HOY WT =T,aty=Oandy=h, then C= T,. (Bee) ayy 4% 18 18 KINEMATICS OF FLUID MOTION 0 149 Consider a viscous, steady flow through a pipe (Fig. 7-182). The velocity profile forms a paraboloid about the pipe centerline, given as C(?— D4) m/s o ‘where Cis a constant. (a) What is the flow of mass through the left end of the control surface shown dashed? (6) What isthe flow of kinetic energy through the left end of the control surface? Assume that the velocity profile does not change along the pipe. I In Fig. 7-185, we have shown a cross section ofthe pipe. For an infinitesimal strip, we can say noting that V and dA are collinear but of opposite sense: pV - dA = p[C(r* ~ D?/4)|2ar dr. For the whole cross section, we have Jfeveta=o [ce 2 )aardr=anpc[e-SE]" = PP agi 2 We now tum to the flow of kinetic energy through the left end of the control surface. The kin ‘energy for an clement of fuid is} dmV?. This corresponds to an infinitesimal amount of an extensive property N. To get 7, the corresponding intensive property, we divide by dm to get a=W @ We accordingly wish to compute ff npV-dA.= ff (AV?)(pV-dA). Employing Eq. (1) for V, and noting again that V and dA are collinear but of opposite sense, wee Jfrov- an~ [3c (r-2)| folc(e-F)emrar]} ween ["( es @ In Prob. 7.62, assume a one-dimensional model with the same mass flow. Compute the kinetic energy flow through a section of the pipe for this model. That is, compute kinetic energy flow with an average constant velocity. What is the ratio ofthe actual kinetic energy to the kinetic energy flow for the one-dimensional model flow? I We first compute the constant velocity at a section for the one-dimensional model. Hence, using Eq. (2) of Prob. 7.62, (vy 22) = 20 Y= Z mis @ The kinetic energy flow forthe one-dimensional model is then ve = 0 (€D?)'(xb?) __pC’Dtx Jf Fev-an~ -3(S) EP) Pape No ® ‘We now define the kinetic-energy correction factor «asthe ratio ofthe actual flow of kinetic energy through a 150 0 CHAPTER 7 164 165 16 167 ‘cross section to the flow of kinetic energy for one-dimensional model for the same mass flow. That is. KE flow for section ® ‘KE flow for 1-D model For the case at hand, we have from Eq. (2) of this problem and Eq. (4) of Prob. 7.62 2 @ “The factor a exceeds unity, so there is an underestimation of kinetic energy flow for a one-dimensional model. “The velocity field in a diffuser is u = Ue~*”*, and the density field is p = pye*, Find the rate of change of density tx = L e205 P,P 5 y Pog (tye m0y( PE) 4 94 9m —Poleg me ' Basu Cro Baw a04 Gem(Pe—) +040— —AF AtreL, de_ _ pate, aT it = 9.0498 90s HoT Gas is flowing in a long 4-in-diameter pipe from A to B. At section A the flow is 0.30 Ib/s, while at the same instant at section B the flow is 0.33 Ib/s. The distance between A and B is 700 ft. Find the mean value of the time rate of change of the specific weight of the gas between sections A and B at that instant. HA reAiva (22) (wotume of section). Since G = yAv. a30~035= (32) tamynycsy/a @).. 000491. /A/s An incompressible flow field is given by V =x" z'j ~ 3xzk with V in meters per second and (x,y, z) in ‘meters. Ifthe fuid viscosity is 0.04 Pa s, evaluate the entire viscous stress tensor at the point y,2)= G20. ue Fo OOS At (ey, 2) = G,2, 1) for = 0.04 kg/(m - 0 0.2 =| 0 0 -008) Pa 0.12 -0.08 -0.72 Given the velocity distribution ua Ke “Ky a where kis constant, compute and plot the streamlines of fow, including directions, and give some possible interpretations ofthe pattern 1 Since time does not appear explicitly in Eqs. (1), the motion i steady, so that streamlines, path lines, and strealins wll eoinide, Since w =O everywhere, the motion is two-dimensional, inthe 2y plane. The streamlines can be computed by substnating the expression fr u and v into fz y yee) fz Integrating, we obtain nx @ 18 KINEMATICS OF FLUID MOTION J 151 This is the general expression forthe streamlines, which are hyperbolas. The complete patter is plotted in Fig, 7-19 by assigning various values to the constant C. The arrowheads can be determined only by returning to Eqs. (1) to ascertain the velocity component directions, assuming K is positive. For example, inthe upper Fight quadrant (x>0, y >0), wis positive and v is negative; hence the flow moves down and to the right, establishing the arrowheads as shown. Note thatthe streamline pattern is entirely independent ofthe constant K. It could represent the impingement of two opposing streams, or the upper half could simulate the fow of a single downward stream against a flat wall. Taken in isolation, the upper right quadrant is similar to the flow in a 90" corner. Finally note the peculiarity that the two streamlines (C= 0) have opposite directions and intersect each other. This is possible only ata point where w= ), which occurs atthe origin in tis case. Such a point of zero velocity is called a stagnation point Fig. 7-19 A velocity field is given by w= V cos 8, v= V sin 8, and w =0, where V and @ are constants. Find an ‘expression for the streamlines ofthis flow. ’ dx_dy_de_dr dc) de uv w VV Veos6 Vsind 0 (Note: dz/0 indicates that the streamlines do not vary with z.) éy_Vsin 7 et yereTme = -yextand+C ‘Hence, the streamlines are straight and inclined at angle @, as illustrated in Fig. 7-20. 152 0 CHAPTER 7 1.6 Atwo-dimensional steady velocity field is given by u = 3x*~2y?, v = —Gxy. Derive the streamline pattern and sketch a few streamlines in the upper half-plane. dy W-3 ey lay dea (32-2 dy df oxyde +Gx*—2y)dy fle, y)=3x'y — 29°/3 = const Hence, the streamlines represent inviscid flow in three corners, as illustrated in Fig. 7-21. 7.10 A two-dimensional unsteady velocity field is given by u =x(1 +3), v =y. Determine the one-parameter (1) {family of streamlines through the point (xo, yo). ' de _dy dz_dr _dx_dy v ay ow xa+ay oy Integrate, holding t constant. yee yaceun Hy =yoat x =o, Yo CHU™. = e=on(Z) Fig. 7-22 om cy KINEMATICS OF FLUID MOTION J 153 Investigate the stream function in polar coordinates y= Usin O(r — Rr) @ “where U and R are constants, a velocity and a length, respectively. Plot the streamlines. What does the flow represent? Isit a realistic solution to the basic equations? The streamlines are lines of constant y, which has units of square meters per second. Note that y/UR is dimensionless. Rewrite Eq. (1) in dimensionless form /UR=sin nq —1/n) nn =rIR @ (Of particular interes isthe special line y =0, From Eq, (1) or (2) this occurs when (a) 8 = 0 or 180° and (@)r= R. Case (a) is the x aris and case (b) isa circle of radius R, both of which are plotted in Fig. 723. ‘For any other nonzero value of w it s easiest to pick a value of rand solve for 8: vIUR 7IR-RiF In general, there will be two solutions for 0 because of the symmetry about the y axis. For example take v/UR= 41.0: sin ® Guesr/R | 30 | 25 | 20 | 18 | 17 | LOB ‘Compute 6 | Ea | 28 | ae | se | of | 0 158° 13s 138° 26 ne ‘This line is plotted in Fig. 7-23 and passes over the circle r = R. You have to watch it, though, because there is a second curve for y/UR = +1.0 for small r (mass flow in unit time)y = 5; (mass flow in unit time) ux (PMG*HE?/A125) = (ONL ® BY 4110) + (pYICVA.5/12)7/4](12) + (H)LCRDRY*/AN(ah/aty dh/dt=0.1910 fe/s (8) Consider only air in the control volume. It must be conserved. Hence, (pu){(x\RY/4](0) = (@ud{(#)2)74)(0.1484), v= 21.4 8/5. 4 fe ra. 63 ‘The piston of a hypodermic apparatus (Fig. 8-4) is being withdrawn at 0.30in/s; air leaks in around the piston at the rate 0.0012 in’/s. What is the average speed of blood flow in the needle? I Choose as a control volume the region between the piston and the tip of the needle. M=pAv=pQ (mass low in unit time), = 5 (mas fow in unit time) a (Prsoos)[()(0.02/12)7/4](v) + (Ppto0a){0.0012/1728) = (Prrooa){()(0.2/12)7/4](0.30/12) v= 2.18 ft/s Fins FUNDAMENTALS OF FLUID FLOW J 159 Air at 30°C and 110 kPa flows at 16N/s through a rectangular doct that measures 160 mm by 320 mm. Compute the average velocity and volume fux. ' We=yAv y= p/RT=(110)(1000)/{(29.3)(30-+273)] = 12.39 N/m? 16 = 12.39{(0.160)(0.320) Kv) v = 25.2m/s Q = Av = [(0.160)(0.320)](25.2) = 1.29 m/s il (6.g. = 0.86) lows through a 30-in-diameter pipeline at 8000 gpm. Compute the (a) volume flux, (b) average velocity, and (c) mass ux. 1@ © = 8000/(7.48)(60)] = 17.8 °/s o Qa Av I78=[ANBP/) v= 3.63 4/5 © ‘M= pA = ((0.86)(1.94)}{(2)(8)°/4]@.63) = 29.7 sugs/s In the rectilinear chamber of Fig. 85, section 1 has a diameter of 4in and the flow in is 2 cfs. Section 2 has a diameter of 3in and the flow out is 36 fps average velocity. Compute the average velocity and volume flux at section 3if Dy = Lin. Is the low at 3in or out? ' 2:=2:+, (assuming Q, is out) 2= [BY /AIGS) + Qs Q,=0.233cfs (out): v= Q/A =0.233/f(x)(14)?/4] = 42.7 fps we Ue Q| ae 1 FO mes 812 The water tank in Fig. 8-6 is being filled through section 1 at v, = 5 m/s and through section 3 at aa O12 m’/s. If water level his constant, determine exit velocity vs Q+Qs=O2 —— [(A)(0.040)*/4](5) + 0.012= 9, 2,=0.01828m'/s uv, = Qx/A, = 0.01828/{(1)(0.060)"/4) = 6.47 m/s ® Jo,:0012m1s D, 240mm |. Fig. 86 Ifthe water level varies in Prob. 8.12 and v,=8 m/s, find rate of change dh/dt. Assume d= 1.0m, dad 7) _ dh - 1 D4 Qs Dt T [Mate AS) + 0.012 = (Ca) GH) 418) +FDO] dh[de= -5.52mm/s (ic. falling) 160 0 CHAPTER 8 84 as 87 818 819 For the general case of the flow depicted in Fig. 8-6, derive an expression for dh/dt in terms of tank size and volume flows 01, Qs, and Q at the three ports. ' Qroveorrdiet — MO1= 0,409 ‘Water at 20°C flows steadily through the nozzle in Fig. 8-7 at 60 kg/s. The diameters are D, = 220 mm and D,= 80mm. Compute the average velocities at sections 1 and 2. ' = MIp = =0.0601 m?/s = Q/A, = 0.0601/{()(0.220)'/4] =1.58m/s__ v= Q/A, = 0.0601/{(n)(0.080)'/4] = 12.0 m/s Ta a Fig. 8-7 ‘The inseminator in Fig. -8 contains fui of s.g.= 1.04. If the plunger is pushed in steadily at 1.0n/s, what is crit velocity Vz? Assume no leakage past the plunger. i NAM = YAY {(1.04)(62.4)][(-")(0.80/12)7/4](1.0/12) = [(1.04)(62.4)][(")(0.04/12)"/4](V;) 33.3 ft/s (ote thatthe answer is independent ofthe fi’ specific gravity.) 2, s080In Cy Net Wa as Repeat Prob. 8.16 assuming there is leakage back past the plunger equal to 1/4 of the volume flux out of the needle. Compute V; and the average leakage velocity relative to the needle walls if the plunger diameter is 0.796iin, ' = Qs = AV, = [(4)(0.750/12)'/4](1.0/12) = 0.000256 f/s 2. (2)(0.000256) = 0.000192 f/s a= QslAs = 0.000192/{()(0.040/12)'/4] = 22.0 /s Qu = ()(0.000256) = 0.000064 f'/s Via = Qran/ Areas = 0.000064 {(3)(0.80/12)"/4 — (2)(0.796/12)?/4] = 1.84 ft/s ‘A 100-mm-diameter plunger (1) is being pushed at 60 mm/s into a tank filled with a uid of s.g. = 0.68. If the fluid is incompressible, how many pounds per second is being forced out at section 2, D, = 20 mm? ' Ay = Avs [(")(0.100)7/4](0.060) = [()(0.020)°/4](v,) v= 1.500m/s W = yAv = [(0.68)(9.79)][()(0.020)'/4](1.500) = 0.003137 KN/s or 3.137N/s = 3.137/4.448 = 0.705 Ib/s ‘A gasoline pump fils a 80-L tank in 1 min 15s, exit velocity? HQ =V/e= (8)/(60 +15) = 0.001067 m/s v= Q/A =0,001067/{()(0.04)?/4] = 0.85 m/s the pump exit diameter is 4m, what is the average pump-flow FUNDAMENTALS OF FLUID FLOW J 161 420 The tank in Fig. 89s admitting water at 100N/s and ejecting gasoline (s.g. = 0.69) at 52N/s. If all three Auids are incompressible, how much sir is passing through the vent? In which direction? ' 2:40, (assuming airflow is out) 2, = Wil Yugo = 100/[(9.79)(1000)] = 0.01021 m?/s 22 = Wil Yom = 52/(0.69)(9.79)(1000)] = 0.007658 m/s 0.01021 = 0.007698+, Qs=0.002512m"/s (out) ‘Yor = P/RT = (1)(101.3)/{(29.3)(20 + 273)] = 0.01180 KN/m* ‘Ws = (YasQs) = (0.01180)(0.002512) = 0.00002964KN/s or 0.0296 N/s re Le Water GU Fig. 89 821 Air at 72°F and 16 psia enters a chamber at section 1 at velocity 210 fps and leaves section 2 at 1208 °F and 202 psia. What is the exit velocity if D, = 8 in and D, =3 in? Assume the flow is steady. ' PrAwi= 2A, — p=p/RT —_ p= (16)(144)/[(1716)(460 + 72)] = 0.002524 slug/f ps = (202)(14)/[(1716)(460 + 1208)] = 0.01016 stug/t 0,002524{(n)(8)°/41210) = 0.01016{()(2)*/4Kv2) v2 = 371 fps 82. Kerosene (6.g. = 0.88) enters the cylindrical arrangement of Fig. 8-10 at 1, at 0.08N/s. The ‘80-mm-diameter plates are 2 mm apart. Assuming steady flow, compute the inlet average velocity vs, outlet average velocity v, assuming radial flow, and outlet volume flux. ' Wi = YowAsvs 0.08 = {(0.88)(9.79)(1000)]{(#)(0.004)"/4}(u,)_ vy = 0.739 m/s. Q,= Avy, = {(-7)(0.004)"/4(0.739) = 0.00000929 m/s z= Q, =0.00000929 m/s or 9.29 mL/s Vs = Qa/Az= 0.00000929/{(2)(0.080)(0.003)] = 0.0123 m/s or 12.3mm/s [#9 20mm ——+ CELE CT Fig. 8:10 $23 In Fig. 8:11, pipes 1 and 2 are of diameter 3 cm; D,= 4m. Alcohol (s.g. = 0.80) enters section 1 at 6 m/s while water enters section 2 at 10m/s. Assuming ideal mixing of incompressible fluids, compute the exit velocity and density of the mixture at section 3. The temperature is 20°C. ' Q.+Q2= 2, [(x)(0.03)'/416) + )(0.037'/4]10) = 2, Qy=0.01131 m/s 1s" QA = 0.01131/[(4)(0.047/4]=9.00m/s My + = My Panis + PrsoAs¥™ Pmt [(0.803(998)]{()(0.03)/4)(6) + 998{()(0.08)7/4](10)= PaunwnI(#10.04)°/4](9.00) Pare 923 kg/m? 162 0 CHAPTER 8 eal mixing Vee Fig. 8-11 824 Inthe wind tunnel of Fig. 812, the test-section wall is porous; fui is sucked out to provide a thin viscous ‘boundary layer. The wall contains 800 holes of 7-mm diameter per square meter of area, The suction velocity ‘out each hole is ¥, = 10 m/s, and the test section entrance velocity is V; = 46m/s. Assuming incompressible flow of air at 20°C and 1 atm, compute (a) Ve, (b) the total wall suction volume flow, (e) Vz, and (d) Vp. 1@ AcVo=AsVi — {()(2.6)°/4](V6) = [(4)0.9)°/4](45) Vom S.A m/s o Quon Nici Ore Nets = 8004(7)(0.9)(4)] = 9048 Qo = ArieVint = [()(rd)?/4](10) = 0.0003848 m"/s Qrcson * (9048)(0,0003848) =3.48 m/s © Q1= 2+ Qnucsion —_[(*(0.9)"/4](46) = Q2+3.48 — Q2= 25.78 m"/s ¥2= Qa/Az™ 25.78/{()(0.9)"/4] = 40.5 m/s. @® AN, = AN, ((")(2-4)/4)(¥,) = [(")(0.99°/41(40.5) Vy = 5.70 m/s. ae ao my ye My. 612 8.25 A rocket motor is operating steadily as shown in Fig, 8-13. The exhaust products may be considered an ideal gas ‘of molecular weight 26. Calculate v,. ' M.= M+ M, 7+0.1=08slug/s=p:Asv, R= 49709/26= 19121b-fe/(slug-*R) Pa= pIRT = (16)(144)/{(1912)(1105 + 460)] = 0.000770 stug/tt? 0.8 = 0.000770((16.0/12F/4](v:) _ v2= 5291 ft/s —s ran D.=60in foriers Mig. 8:13, FUNDAMENTALS OF FLUID FLOW 0 163 426 For the solid-propellant rocket in Fig. 8-14, compute the rate of mass loss of the propellant, assuming the exit ‘ges has a molecular weight of 30. ' Ma Moat 4 (Mgmt) — 0 Moat £ (Mpa) (yep) = Mou PAs R=SP = 277N-mi(kg-K) pe. =p./RT, = (105)(1000)/{(277)(800)] = 0.4738 kg/m* Cyaan) = (04738 O24 300) = 16.4 eas Combustion 1300. 950 kPa Fig. 8:14 827 The water-jet pump in Fig. 8-15 injects water at U; = 80 ft/s through a 4-in pipe which is surrounded by a secondary flow of water at Us= 8 ft/s. The two flows become fully mixed downstream, where Us is approximately constant. Ifthe flow is steady and incompressible, compute U;. HQ + D2= Os [Cx )C4/41060) + (EF - V4) = (EP NUD) = 16.0 ft/s seme Fly 128 The fow in the inlet between parallel plates in Fig. 8-16 s uniform at Us= 50 mm/s, while downstream the flow develops into the parabolic laminar profile w= az(z9~2), where asa constant. If z= 20 mm, COMPULE tw. 1H Let b = width of plates (into paper). Mig. 8-15 Qn Qm robo [uaa satis [ecu spoee=os[22-2] = a= 6Uszs w= ax(%9~2)= 6UNzE 02) aac OCOUTS at 2 = Z/2 = 0.020/2= 0.010 m: tins = [(6}{0.050)/(0.020)"I0.01(0.020 — 0.010) = 0.0750m/s or BOmm/s. Fig. 8-16 164 0 CHAPTER 8 29 830 831 832 8.33 834 Assuming the container in Fig. 8-17ais large and losses are negligible, derive an expression for the distance X ‘where the free jet leaving horizontally will strike the floor, asa function of h and HY. Sketch the three trajectories for h/H =0.25, 0.50, and 0-75. B= Vig(H=h) eel2 t= VOhg X= vot = V2g(H=h) Vib = 2VK(H A) For h/H =0.25, ot h=0.25H, X = 2V(@25A\H (25H = 0.866H. For h/H = 0,50, ot h= 0.50H, X= 2V(0SOFT OST) =H. For hdl ©0.95, or b= 0.75, X= 2VLTSE = O73) - trajectories are sketched in Fig. 8178. onsi i . 0.30! i Free it o.tsu ay in | : L a ig 8170) ; OB He Fig. 8170) In Fig. 818 what should the water level k be for the free jet just to clear the wall? ! v9= Vigh Fall distance = gt*/2= 0.40 = 0.8944/VE Horizontal distance = nyt = (V2ghK0.9944/Vg)=0.50 = 0.156m=15.6em A Li oem 800m ata wa Fig. 8:18. ‘When 500 gpm flows through a 12-in pipe which later reduces to a 6-in pipe, calculate the average velocities in the two pipes. ' Q = S00/{(7.48)(60)]=1.114 8/8 Y= O/An= LIM A)EV A= 1428/8 ye= O/Ag= 1.114/[(a (8/4) =5.67 ls I the velocity in a 12-in pipe is 1.65 ft/s, what is the velocity in a 3-in-diameter jet issuing from a nozzle attached to the pipe? ' Ais = Avs (()(B/410.65)= [()(AP/4I(02) v2 = 26.4 fe/s Air flows in a 6-in pipe at a pressure of 30.0 psig and a temperature of 100 F. If barometric pressure is 14.7 psa and velocity is 10.5 ft/s, how many pounds of ar per second are flowing? t ‘y= p/RT = (30.0+ 14.7)(144)/[(53.3)(100 + 460)] = 0.2157 Ib/f W = yAv = 0.2157{(3)((8)°/4](10-5) = 0.445 tb/s, Carbon dioxide passes point A in a 3-in pipe at a velocity of 15.0 ft/s. The pressure at A is 30 psig and the temperature is 70°F. At point B downstream, the pressure is 20 psig and the temperature is 90 F. For a barometric pressure reading of 14.7 psia, calculate the velocity at B and compare the flows at A and B. ' y=pIRT —_yq= (30+ 14.7)(144)/[(35.1)(70 + 460)] = 0.3460 Ib/te? 20-+ 14.7)(144)/[(35.1)(90 + 460)] =0.25881b/A? —¥4AaUa=YeAnve at FUNDAMENTALS OF FLUID FLOW J 165 Since A4 = Ap, (0.3460)(15.0) = (0.2588)(v,), vp = 20.1 ft/s. The number of pounds per second flowing is constant, but the flow in cubic feet per second will differ because the specific weight is not constant. Qa = Aga = [(H)(B)/A](1S.0) = 0.736 P/5; Op = Anda = [(*)(12)'/4](20.1) = 0.987 ft/s. What minimum diameter of pipe is necessary to carry 0.500 b/s of air with # maximum velocity of 18.5 ft/s? ‘The air is at 80°F and under an absolute pressure of 34.0 psi. fo W=yAv —— yur=p/RT = (34.0)(144)/{(53.3)(80-+ 460)] = 0.170 1b/f° 0.500 = (0.170){()(d)"/4\(18.5) d= 0.450ft or 5.40in {In the laminar flow of a uid in a circular pipe, the velocity profile is exactly a true parabola. The rate of discharge is then represented by the volume of a paraboloid. Prove that for this case the ratio of the mean ‘velocity to the maximum velocity is 0.5. I See Fig. 8:19. For a paraboloid, u = tga ~ (r/n)I 0- us [aali-Q)]oeaomfs- fl nom dln) Veen = QA = Wa 73/2) {(72) = tge2- THUS Vinee ner = 05. Fig. 819 A gas (y =0.05 Ib/f¢) flows atthe rate of 1.6 b/s past section A through a long rectangular duct of uniform cross section 2ft by 2ft. At section B, the gas weighs 0.060 lb/ft. Find the average velocities of flow at sections A and B. FB WayAv 1.6=(0.05)12)2)1(1.) va =8.00F/5 1.6 = (0.060)1(2)(2)Kus) vn =6.67 ft/s ‘The velocity ofa liquid (s.g. = 1.26) in a 6-in pipeline is 1.6 ft/s. Caleulate the flow in: (a) fP/s, (6) slug/s. '@ = Av = [(x)(8)'/4](1.6) = 0.314 f/s ©) -M = pAv = [(1.26)(1.94)][(#)()'/4]1.6) = 0.768 slug/s ‘Orygen flows in a 3-in by 3-in duct at a pressure of 42 psi and a temperature of 105 °F. If atmospheric pressure is 13.4 psia and the velocity of flow is 18 fps, calculate the weight-flow rate. ' 7 = pIRT = (42 + 13.4)(144)/[(48.2)(460 + 105)] = 0.2929 1b/f W = yu = (0.2929){(#)(2)1(18) = 0.330 Ib/s Air at 42°C and at 3 bar absolute pressure flows in 2 200-mm-diameter conduit at a mean velocity of 12:m/s Find the mass flow rate. Hp = DIRT =3 x 1071287273 + 42)] =3.318kg/m?—-M = pAv=3.318{(-)(0.200)"/4](12) = 1.25 kg/s ‘A 120-mm-diameter pipe enlarges to a 180-mm-diameter pipe. At section 1 of the smaller pipe, the density of a ‘38 in steady flow is 200 kg/m” and the velocity is 20 m/s; at section 2 of the larger pipe the velocity is 14 m/s. Find the density of the gas at section 2. ' PA, = prAsvs — 200[()(0.120)7/4}(20) = (pa){()(0.180)7/41(14) pz = 127 kg/m™ 166 845, 8.46 847 849 0 CHAPTER 8 ‘The Peconic River is spanned by Noyack Bridge and Smith’s Bridge. At noon on July 4, the measured flows under the two bridges were Qy = S0m’/s and Q, = 40 m°/s. Neglecting losses, compute the instantaneous rat ‘of water storage between the two bridges. ' Qy-Qs=aS/dt —50-40=dS/adt dS/de=10m'/s ‘A worker in a children’s playground is cleaning a lide with a hose. She observes that a horizontal stream dlected ino the low end climbs toa point 12 above the nozzle. What isthe nozle velocity ofthe steam? - v= Vigh = V@GEAD) = 27.8 tls At section 1 of a pipe system carrying water the velocity is 3.0fps and the diameter is 2.0 ft. At section 2 the diameter is 3.0 ft. Find the discharge and velocity at section 2. Fe Q= = Av=[)ROVING.O)= 9-42 v= QalAr=9.42/1(2)(3.0)/4] = 33 fps In two-dimensional flow around a circular cylinder (Fig. 8-20), the discharge between streamlines is 34.56in/s per foot of depth. At a great distance the streamlines are 0.25 in. apart, and at a point near the cylinder they ae 0.12in, apart. Calculate the magnitudes of the velocity at these two points. I v= (Q/d)/w. At great distance, v = (34.56/12)/0.25 = 11.52in/s. Near the cylinder, v = (34.56/12)/0.12= 24.0in/s. — SS res A pipeline carries oil (6.g. = 0.86) at v =2m/s through a 20-cm-ID pipe. At another section the diameter is Sem. Find the velocity at this section and the mass flow rate. He Q=A,v,=[()(0.20)'/8]2) = 0.06283 m'/s vs = Q/Az = 0.06283/[()(0.08)*/4] = 12.5 m/s -M = pAv={(0.86)(1000)]{()(0.20)/4](2) = 54.0 ke/s Hydrogen is flowing in a 3.0-in-diameter pipe at the steady rate of 0.03 Ibm/s. Calculate the average velocity lover a section where the pressure is 30 psia and the temperature is 80°F. ! M=pav RT = (30)(144)/{(765.5)(460 + 80)] = 0.01045 tbm/ ft? 0.03 = (0.01045)[(2)3.0/12)/4\(v) v= 58.5 ft/s Ifa jet is inclined upward 30° from the horizontal, what must be its velocity to reach over a 10-ft wall ata horizontal distance of 60 ft, neglecting friction? F (v,)o= 19005 30° = 0.866000, (v.)o= Vo sin 30° = 0,50000e. From Newton’s laws, x = (0.86600.)(#) = 60, ‘2 =0.5000v9t ~ 32.2/2= 10. From the first equation, f = 69.28/o. Substituting this into the second equation, {(0.5000)(1.)(69.28/v4) — (32.2)(69.28/vq)*/2= 10, v9 = 56.0 fps. Water flows at 10 m”/s in a 150-cm-diameter pipe; the head los ina 1000-m length ofthis rate of energy loss du to pipe fiction. 1 Rate of energy loss = yQH = (0.79)(10)(20) = 1958 kW is 20m. Find the Oil with specific gravity 0.750 is flowing through a 6-in pipe under a pressure of 15.0 psi Ifthe total energy relative to datum plane 8.00 ft below the center of the pipe is 58.6 ft Ib/lb, determine the flow rate of the oil. 1 H=ztv/2g+ply — 58.6= 8.004 v'/{(2)(32.2)] + (15)(144)/[(0.750)(62.4)] V=16.92At/s OQ = Av [()((8)/4)(16.92) = 3.32 f/s FUNDAMENTALS OF FLUID FLOW 0 167 In Fig. 8-21, water flows from A, where the diameter is 12in, to B, where the diameter is 24in, atthe rate of 13.2cfs. The pressure head at A is 22.1 ft. Considering no loss of energy from A to B, find the pressure head atB. ' aly +UR2¢+ 24 Daly + Ual2g + 2p U4 Q/Ay=13.2/(()(B)"/4] = 16.81 ft/s An = 13.2/{(a) (3/4) = 4.202 ft/s al y+ 4.202? [(2)(32.2)] + (25.0~ 10.0) ppl = 11.2 ft of water 22.14 16.81°(2)G2.2)]+0= ‘A pipe carrying oll with specifi gravity 0.877 changes in size from in at section E to 18in at section R. Section E is 121 lower than R, and the pressures are 13.2 psi and 8.75 psi, respectively. If the discharge is 5.17efs, determine th lost head and the direction of flow. I = 2403/29 + ply. Use the lower section (E) as the datum plane. vg = Q/Ag = 5.17/[(=)(8)?/4] = 26.33 ft/s, Up = O/ Ag = 5.17/{()(18)"/4] = 2.926 ft/s; He = 0 + 26.33°/{(2)(32.2)] + (13.2)(144)/{(0.877)(62.4)] = 45.50 ft, Hy = 12+ 2.9267 /[(2)G2.2)] + (8.75)(144)/[(0.877(62.4)] =35.16 Since the energy at E exceeds that at R, flow occurs from E to R. The lost head is 45.50 ~ 35.16, or 10.34 ft, EtoR. A horizontal air duct is reduced in cross-sectional area from 0.75 f to 0.20 f°. Assuming no losses, what pressure change will occur when 1.50Ib/s of air flows? Use y = 0.200 1b/f forthe pressure and temperature conditions involved. 1 Q=1,50/0.200= 7.5008 /s — paly + 04/28+24= pol + vbl28 +29 aly + (1.500/0.75)/{(2)(32.2)] + 0= paly + (7.500/0.20)?/{(2)(32.2)] +0 Pal ~ aly = 20.28 of air p4~ Pa = (20.28)(0.200)/144 = 0.0282 psi ‘A turbine is rated at 600 hp when the flow of water through itis 21.5 fs. Assuming an efficiency of 87 percent, ‘what head is acting on the turbine? ! Rated horsepower = (extracted horsepower)(effciency) = (7 QH/550)fficiency) (600 = ((62.4)(21.5\(H)/S50]0.87) HH = 283 ft ‘A standpipe 20 ft in diameter and 40 ft high is filled with water. Calculate the potential energy of the water ifthe ‘elevation datum is taken 10 ft below the base of the standpipe. ' PE = Wz = [(62.4)(40)(1)(20)*/4](10 + 40/2) 35% 107 ft Ib How much work could be obtained from the water in Prob. 8.55 if run through a 50-percentffcient turbine that discharged into a reservoir 30 ft below the base ofthe standpipe? ' ‘Work = PE = nWe = (0.50)[(62.4)(40)(2)(20)°/4](30 + ) = 1.96 x 107 f- Ib Determine the kinetic-energy flux of 0.01 m/s of oil (s.g. = 0.80) discharging through a 40-mm-diameter nozzle ! v= Q/A = 0.01/{(7)(0.040)"/4] =7.96 m/s ww /2-= pOQv"/2 = [(0.80)(1000)](0.01)(7.96)"/2 = 253 W [Neglecting air resistance, determine the height a vertical jet of water will rise if projected with velocity 58 ft/s ' PESKE = Wz =mv*/2=(W/32.2\(58)'/2 s2.2t 168 0 CHAPTER 8 89 8.60 861 82 8.64 8.65 Ifthe water jet of Prob. 8.58 is directed upward 45° with the horizontal and air resistance is neglected, how high will it rise? BE AL45%, vy = vy = (580.7071) = 41.01 fs; We = mv?/2 = (W/32.2)(41.01)"/2, 2 = 26.1. ‘Show that the work aliquid can do by virtue ofits pressure is { p dV, in which V isthe volume of liquid displaced. I Work =f Fas. Since F = pA, work = J pA ds, Since A ds = dV, work =f paV. A fiuid is flowing in a 6-in-diameter pipe at a pressure of 4.00 Ib/in® with a velocity of 8.00 ft/s. As shown in Fig. 8-22, the elevation of the center ofthe pipe above a given datum is 10.0 ft. Find the total energy bead above the given datum ifthe Suid is (a) water, (6) oll with a specific gravity of 0.82, and (¢) gas with a specific weight of 0.042 1b/fe 1 Haz+v'gtply @ H = 10.0 + 8.007/[(2)@2.2)] + (4.00)(144)/62.4= 20.22 o H = 10.0 + 8.007/{(2)(32.2)1 + (4.00)(144)/[(0.82)(62.4) = 22.25 © H = 10.0 + 8.007/{(2)(32.2)] + (4.00)(144)/(0.042) = 13 725 ft _L SE ae fe Datum, ‘Fig. 8:22 ‘A.100-mm-diameter suction pipe leading to a pump, as shown in Fig. 8-23, carries a discharge of 0.0300 m/s of oil (s-g. = 0.85). If the pressure at point A in the suction pipe is a vacuum of 180 mmHg, find the total energy head at point A with respect to a datum at the pump. B= Q/4 = 0.0300/1(7)18)"/4] =3.820.m/s_—_ p= yh = [(13.6)(9.79)|(— HER) = ~23.97 KN aa? H=24+v"/2g + ply = 1.200 + 3.8207/[(2)(9.807)] + (—23.97)/[(0.85)(9.79)] = -3.337 m Figure 8:24 shows a pump drawing water from a reservoir and discharging it into the air at point B. The ‘pressure at point A in the suction pipe is « vacuum of 10 in mercury, and the discharge is 3.00 f/s. Determine the total head at point A and at point B with respect to a datum atthe base of the reservoir. i H=z+v'/2g+ply U4 = Q/Aa=3.00/((4)(BY/4] = 5.50 ft/s. Hy =25 + 5.50P/[(2)(32.2)] + [(13.6)(62.4)](—19)/62.4 = 14.14 Up = Q/An =3.00/[(n)(A)'/4]=8.59 ft/s Hy = (25+ 15 + 40) + 8.59°/((2)(32.2)] +0 = 81.15 ft If the total available head of a stream flowing at arate of 300 f0/sis 25.0 ft, what is the theoretical horsepower available? 1 P = QyH = (300)(62.4)(25.0) = 468 000 ft - Ib/s = 468 000/550 = 851 hp ‘A.150-mm-diameter jet of water is discharging from a nozzle into the air at a velocity of 36.0 m/s. Find the Power in the jet with respect to a darum at the jet i Q = Av = [(-")(5%)/4](36.0) = 0.6262 m"/s H=2z+v"/2g + ply =0 + 36.0/{(2)(9.807)] +0 = 66.08: P= QyHt = (0.6362)(9.79)(66.08)=412EN- m/s or 412kW FUNDAMENTALS OF FLUID FLOW 0 169 Suction pipe Datum Fig. 8:23 WOindiameter ——Bindiameter suction pipe discharge pipe 466 Oil (s.g. = 0.84) is flowing in a pipe under the conditions shown in Fig. 8-25. If the total head loss (f.) from point 1 to point 2is 3.0, find the pressure at point 2. : pily + Wilde + 2= ply + vH28 + 24h, Y= Q/A\=2.08/{(n)(8)/4]=10.59ft/s v= Q/A,=2.08/{(n)(8)"/4] = 4.71 fs (65)(144)/{(0.84)(62.4)] + 10.59°/[(2}(32.2)] + 10.70 = poly + 4.717/{(2)(32.2)] + 4.00 + 3.00 paly= 183.678 p=[(0.84)(62.4)](183.67) = 9627 Ib/f€ or 66.9 bin? 170 8.69 0 CHAPTER 8 Fig. 8:25 {a2 Ein-diameter horizontal pipe is attached to a reservoir, as shown in Fig. 8-26. If the total head loss between {he Natr surface inthe reservoir and the waterjet atthe end ofthe pipe is 6.0 ft, what ae the velosty ood flow Tate of the water being discharged from the pipe? ' pily + vile + z= pily tv tath, 040+ 15=04v9/[(2)(32.2)]+0+6.0 v= 2A1 fs = Ayu, =[(a)(4)"/4] 24.1) = 841 fs Water Sa diameter Pig. 8:26 2 3tmm-diameter siphon is drawing oil (sg. = 0.82) from an oll reservoir, as shown in Fig. 827. Ifthe head {oss from point 1 to point 2is 1.50 m and from point 2 to point 3is 2.40 m, find the discharge of cil era ehe siphon and the oil pressure at point 2, Fpl suite t= pily +g tath, — 04045.00=04 v31(2)(.807}+043.90 4S m/s Q = Avy = [()(s)/4](4.645) = 0.00912 m"/s uly + vil2g + 21 paly + Ui2g + 24h, 0+ 0+ 5.00— ps/y + 4.6454/[(2(9.807)] +7.00 + 1.50 paly=~4.60m py [(0.82)(9.79)](—4.60) = —36.9 N/m? or —36.9kPa {ruse £28 shows a siphon discharging ol (sg. = 0.84) from a reservoir into open ai. If the velocity of fow in ‘he Pie isu, the head loss from point 1 to point 2 is 2.0v/2g, andthe head loss from point 2 to pomt ig FUNDAMENTALS OF FLUID FLOW J 171 — so-mm-diameter siphon p30 en 3.0v?/2g, determine the volume flow rate inthe siphon pipe and the absolute pressure at point 2. Assume an atmosphere pressure of 14.70 psia. W pily+vilag + z=psly + vi2g + z+h, — 0+0+10=0+ v3/[(2)(32.2)] + 0 + 5{v3/{2)(32.2)}} v3= 10.36 ft/s Q=Agus= [()()'/4](10.36) = 0.509 ft/s ply + vi/2g +21 = poly + V3/28 + z+ he 040+ 10= pa/y + 10.367/[(2)(32.2)] + 15 + 2(10.36/12)32.2))) pal = -10.0t of oil P= ((0.84)(62.4)|(—10.0) = -S241b/f? or -3.641b/in? §—_ p= 14.70 - 3.64 = 11.06 Ib/in? abs Hndiameter pipe ° fo { f Wg. #28 870 Once it has been started by sulficient suction, the siphon in Fig. 8-29 will run continuously as long as reservoir ‘uid is available. Using Bernoulli's equation with no losses, show (a) that the exit velocity v; depends only ‘upon gravity and the distance H and (6) that the lowest (vacuum) pressure occurs at point 3 and depends on the distance L+H. W pily + vil0g + 24=paly + V82g + 20 hi, 040+ 2,= 04 v9/2g + 2240, v= Vig — 4) = VBA. For any point B in the tube, pp/'y + v3/2g + 5 = pa/y + v3/2g + 2+ h,. Since Vp = V2 and P; = Paes Pe ™ Pum (Ze — %). The lowest pressure occurs at the highest zp, Of Pain Ps = Pem — Y(L +H). Fig. 8-29 172 0 CHAPTER 8 8.71 The siphon of Fig. 830s filled with water and discharging at 150L/s. Find the losses from point 1 to point 3 in terms of velocity head v'/2g. Find the pressure at point 2 if two-thirds ofthe losses occur between points 1 and 2. ' pily + vi/2g+21=psly + 02g + 2th, — O+041.5=04vi/2¢ +04 (K\(vH/25) y= Q/Ag= (iits)/{()(GY /4] = 4.775 m/s 1.5 = 4.775*/[(2)(9.807)] + K (4.77511 2)(9.807)) K=0.2904 hy, = (0.2904){4.775/[2)(9.807)]} =0.338.m Pily + vile +2, paly + UY2g+ 2h, — 0+0+0=paly +4.775%/[2)(9.807)] + 310.338) poly = ~3.388m of water pz (—3.388)(9.79) = ~33.2 N/m? ‘Contre volume Boundary 1.2. Fig. 8:30 8.72 For the water shooting out of the pipe and nozzle under the conditions shown in Fig. 8-31, find the height above the nozzle to which the waterjet will “shoot” (i.c., distance h in Fig. 8-31). Assume negligible head loss. ' Pal + Val28 + 24 = Progl 7 + Ving!28 + Ziop + he. 55.0/9.79+ v3/2+0=0+0+(1.00+h)+0 = 4.5184 03/28 Pal + Ua)26 + Za Proce! Y + Vive 28 + Zamnte +, 55.0/9.79+ V3/2g +0 0+ Uirete!26 +1.100+0 Anta =Agoatetanate — [()( fin)? 4]04 = [() (185) /4 ramets Yrote = 4.0004 55.0/9.79 + v3/[(249.807)] + 0= 0+ (4.00v,)*/{(2)(9.807)] + 1.100 +0 w=2431m/s k= 4518+2.4317/(2)0.807)] = 4.82m 8.73 Water flows from section 1 to section 2 in the pipe shown in Fig. 8-32. Determine the velocity of flow and the fluid pressure at section 2. Assume the total head loss from section 1 to section 2is 3.0m. ' Da Awi=Aavs [4 )(HB)*/42.0) = [()GHR)/4](02) v2 = 8.00m/s. pil + Uil2g +2 =paly + 022g + 2h, 300/9.79 + 2.0°/{(2)(9.807)] + 2= pa/9.79 + 8.007/[(2)(9.807)] +0+3.00 p= 260 KPa 8.74 A nozzle is attached toa pipe as shown in Fig. 8.33. The inside diameter ofthe pipe is 100 mm, while the water jet exiting from the nozzle has a diameter of 50 mm. If the pressure at section 1is 500 kPa, determine the water jets velocity. Assume head loss in the jet i negligible. i Qa Aw =Aav2 — [(")R)"/41(04) = [)ahia)*/41@2) v= 0.2500. Dil + Vil2g +2 = pal + 3/28 + 22 he 500/9.79 + (0.250v2)°/[(2)(9-807)] + 0= 0+ v3/[(2)(9.807)] +0+0 — v.= 7 m/s FUNDAMENTALS OF FLUID FLOW 0 173 Nozzle D= 100 mm Point A D=20mm P= 5S0KPa Fig. 8:31 ee © Searer Fig. 8:32 174 0 CHAPTER 8 8.75 Oil Bows from a tank through S00 of 6in-diameter pipe and then discharges into the air, as shown in Fig. 834, Ifthe head loss from point 1 to point 2 is 1.95 tof oil, determine the pressure needed at point 1 to cause 0.60 ft of ol to flow. ' v= Q/A =0.60/[(n YF IA =3.06t/s ply + vi/2G +2=paly + VHB +t AL uly + 0+ 80 = 0+ 3.06%/[(2)(32.2)]+ 1004 1.95 ply =22.10 fof oil P= [(0-84)(62.4)(22.10)/144 = 8.04 bint Spe. Blew, 1008 ‘ie pressure =? ® —s08n Gin-diameter pipe le 0% Fig. 834 8.16 Water is to be delivered from a reservoir through a pipe to a lower level and discharged into the air, as shown in Fig. 8-35. If head loss inthe entire system is 11.58m, determine the vertical distance between the point of water discharge and the water surface in the reservoir. i ¥2= Q/Az = 0.00631/{()(i86)"/4] = 3.214 m/s pily + vil2g + 2; = paly + v3/2g + 2+ hi, 0404 24 =0+3.214/12)0.80]+0+11.58 = 12.11m ‘S0-mm-diameter pipe (Q= 000631 ms Mig. 8:35 8.77 Determine the velocity and pressure at section 2 and section 3 if water flows steadily through the pipe system shown in Fig. 8-36. Assume a head loss of 6.0 ft from section 1 to section 2 and of 15.0 ft from section 2 to section 3. ! A= Avs — {()(8)7416.0)= [4 )(8)/4](02) vs 20.0fe/s pil + vil2g + 2 paly + U2/2g+ 24+ hy (25)(144)/62.4-+ 5.07(2)(32.2)] + 20 = (p3)(144)/62.4 + 20,07/[(2)(32.2)] + 15+ 6.0 ‘P2= 22.0 Ib/in? Au, = Ags [IAVVAIE.0) = (NBII) v= 8998/8 Duly + vile + 2, psly + vi/2g + 25+ hy (25)(144)/62.4 + 5.0°/[(2)(32.2)] + 20 = (p;)(144)/62.4 + 8.89°/{(2)(32.2)] + 10 + (15.0-+ 6.0) ‘ps= 19.9 b/int® FUNDAMENTALS OF FLUID FLOW 0 175 4.78 Compute the ideal flow rate through the pipe system shown in Fig. 8-37. TF pily + viltg +2," ply + via +2+h, — prly + vi/{(2)(9.807)] + 0.6sin 30° = p,/y +0+0+0 vi/[(2)(9.807)] = poly ~ pri ~ 0.300 From the manometer reading, p, ~ 9.79(1.2sin 60°) 10.17 N/mm; v3/[(2)(9.807)] = 10.17/9.79 — 0.300, v; = 3.807 m/s; Q =. (()((88)*/41@.807) Fig. 637 $79 A large tank with a well-rounded, small opening as an outlet is shown in Fig. 8-38. What is the velocity of a jet issuing from the tank? ' ply + vil2g+a=pily+vl2gt+nth, O+0+h=O+vi/2¢+0+0 v,=VIgh Fig. 838 480 Neglecting friction, find the velocity and volumetric discharge at the exit 2in Fig. 839. Fo pilytviltg+2=ply tug t nth, 0+0+(3.040.6+1.5)=0+ vi/{(2(9.807)] +0+0 v= 10.0m/s OQ = Av =[(-)(si)*/4}(10.0) = 0.177 m/s 176 0 CHAPTER 8 15m 4 3 m BY Fig. 8:39 881 One end of a U-tube is oriented directly into the flow (Fig. 8-40) so that the velocity of the stream is zero at this point. The pressure at a point in the flow that has been stopped in this way is called the stagnation pressure. The other end of the U-tube measures the undisturbed pressure at that section in the flow. Neglecting friction, ‘determine the volume flow of water in the pipe. W pily + vil2g + 2 = poly + v2 + 22+ he, pily +0+0= aly + vi/2g + 0+ 0, v3/2g = p,ly ~ pal. From the manometer reading, p, + (62.4)(42) — [(13.6)(62.4)|@) = ps, ps — Pa= 163.81b/ft; v3/[(2)G2.2)] = 163.8/62.4, v, = 13.00 ft/s; Q = Av = [()(A)'/4](13.00) = 4.54 7s. Fig. 8-40 882 A cylindrical tank contains ai, oil, and water, as shown in Fig 8-41; the aris under gage pressure p = 4Ib/in’ Find the ext velocity at 2, neglecting any friction and the kinetic energy of the fluid above elevation A. The jet cof water leaving has a diameter of If Bo pil + vig tam paly tulle tit h, —pr= (A(04A) + (0.8)(62.4)1@) = 675.81b/te 675.9/62.4+0+8=0+ vI[QG22]+0+0 v= 348f/s air on specific gravity 08 Water amy Fig. 8-41. 8.83 A large tank contains compressed air, gasoline at specific gravity 0.68, light oil at specific gravity 0.80, and water, as shown in Fig, 8-42. The pressure p ofthe air is 120 kPa gage. If we neglect friction, what is the mass flow of oil from a 20-mm-diameter jet? ' pil + vil2g + 2,=paly + 2g + 22h, py 120+ [(0.68)(9.79)]@) = 133.3 kN/m? 133,3/{(0.80)(9.79)] + 0 +0 = 0+ v3/{(2)(9.807)]+4+0 v= 15.98 m/s -M = pAv = {(0.80)(1000)()(i)"/4](15.98) = 4.02 kgs FUNDAMENTALS OF FLUID FLOW 0 177 Oil specitie gravity = 0.80 Gasoline specific gravity = 0.68 Water Fig. 8-42 ‘A flow nozale is a device inserted into a pipe as shown in Fig. 8-43. If A2 is the exit area of the flow nozzle, show that for incompressible flow we get for OQ, where C,is the coefficient of discharge, which takes into account frictional effects and is determined experimentally [ ee ree 28 But pe ~ Pa =P2~p and v= (v;)(AV/A,); hence, +H onPtsZeov0 vim ad ze(PA=PA) = Vizcarag VHF") On Av Cot ara eS”) = OV maa VS”) ‘Maltiplying by 43/4? in the numerator and denominator of the radical gives fara) oe ol retiay Pe Fig. 8-43 In Prob. 8.84, express @ in terms of h, the height of the mercury column (Fig. 8-43), and the diameters of the pipe and flow nozzle. I From Prob. 8.84, o- ol ta HE) e=P2)) 178 0 CHAPTER 8 a7 From the manometer, p; ~ P= (Yne— Ynso)(h). on ol ag VO] ‘A hump of height 6 is placed on the channel bed in a rectangular channel of uniform width over its entire width (Gee Fig. 8-44). The free surface has a dip d as shown. If we neglect friction, we can consider that we have ‘one-dimensional low. Compute the flow q for the channel per unit width. This system is called a venturi flume. ' ily + 02g += paly +vi/2g+ nth, O+vi/2g +h =0+vi/2g +(h—d)+0 =i 2gd Awi= Aw, — [(1Mh)|(vs) = [Gh —d - 6) Kv) vs oo'(z ap- 7 7 2 Zig, 844 In the fountain of Fig. 845, water flows steadily up the vertical pipe, enters the annular region between the circular plates, and emerges as a free sheet. Find the volume flow of water through the pipe, ifthe pressure at A is 70 kPa gage and friction is negligible. Paly +URl2g +24 poly + vEl28 + 20 +h. 70)9.79 + v3(2\9.807)]+0= 0+ vEs{(2)(9.807)] + 1.5+0 Aavan Arve —((#)(0.200)/4](u,) =[(0.013)(}(0.3+0.3))(v2) — v4= 0.78006 709.79 + (0.7800e)*/(2)(9.807)] + 0= 0+ vE/[@).807]+2.0+0 vp = 16.06m/s Q= Agus = [(0.015)(x)(0.5 + 0.5)} 06.06) = 0.757 m/s osm 18mm -—-] oo we 200mm Ww BB FUNDAMENTALS OF FLUID FLOW J 179 888 Ifthe velocity at point A in Fig. 8-46 is 18 m/s, wh: isthe pressure at point B if we neglect fiction? t Poly + Vnl2g +25 =Paly + Val2B + Za the o/9.79-+ va/{(2V(9.807)] +0 =0 + 18/[(2)(9.807)] + (05 + 15) +0 Po= 049915 +3135 poly +ub/2¢+2¢=pal¥ + UIE + tah 0+ v2/[(2)(9.807)] + 0 = 0 + 18°/[(2)(9.807)] +1540 ve =24.86 m/s Aavy = Acte Ua BRYAN v9) = (C2) (dn) /4124.86) vg =3.496 m/s (—-0.4991)(3.496)? + 313.5 = 319.6 kKN/m? deflect water from a stationary tank of water. To what height h does a sled traveling at 100 km/h deflect water? 1 In an inertial fame fixed tothe sled apply Bernoulli's equation between the scoop (point 1) and the highest point inthe trajectory (point 2); py/y + vi/2g + 21 pal + v8/2g + 2+ hy, 0+ v¥/[Q)9.807)] + 0=0+ (v, cos 20° /{(2)(9.807)] + (h — 0.150) + 0. From the data, v; = (100)(1000)/3600 = 27.78 m/s, 27:78112N(9.807)] = (27.78 cos 20)/(2(9.807)] + (h 0.150), h = 4.90 m. Scoop from sed O Fig. 847 } 890 A venturi meter isa device which is inserted into a pipe line to measure incompressible flow rates. As shown in t Fig. 8-48, it consists of a convergent section which reduces the diameter to between one-half and one-fourth the i pipe diameter. This is followed by a divergent section. The pressure difference between the position just before ‘the ventusi and at the throat of the venturi is measured by a differential manometer as shown. Show that on cl tira HO) 180 0 CHAPTER 8 where Cy is the coeficient of discharge, which takes into account frictional effects and is determined experimentally. BBB en aPetentn PetonBeteore ut-ubeze BoP! ag By 2g 2g Oy 2g (5) Awan, ve (ua() - win 2g( BPS = [By -tJoa=2e oranncaniray TAF Fe “btw eP)| Pa Fig. 8-48 891 Annecked-down, or venturi, section ofa pipe flow develops a low pressure which can be used to aspirate fluid ‘upward from a reservoir, as shown in Fig. 8-49. Using Bernoulli's equation with no losses, derive an expression for the exit velocity us that i just sufficient to cause the reservoir fluid to rise in the tube up to section 1. ' pily + vile +2 =paly UH +th, Ply + Vi/2g +0=Pawly + vag +040 Awi=Aws — (ndi/4{v,) = (adi/4)(v,)— v= (UN Pil + [(U2N dal dF 128 = Pal + VR Pam ~ Pr = (y/28\vDl (dal ds)*— 1} For fuid to rise in the tube, Pam ~ ps > yh; hence, (y/2g)(v3)[(dald,)* — 1} = yh, * Naat 892 Neglecting losses, find the discharge through the venturi meter of Fig. 8-50. Huby + vil2g + 2,= poly + v3/2g + 2,+ h,. From the manometer, ps/y ~ (K+ 0.200) + (2,— 22+ K)= pals Ast = Avs, [(4)(0.300)/4](v,) = [()(0.150)/4](v,), v, = 0.2500; pay + (0.2500,)2\9.807)] + 2,= pal ~ k +0.250) + (2, ~ 2, + k)] + v3/[Q)(9.807)] + 2,40, vy =2.287 m/s; O= Av, = {(2)(0.150)'/4]@2.287) = 0.0408 ms. 250m $150 mm aia 200 mm sim — 394 89s FUNDAMENTALS OF FLUID FLOW 0 181 With losses of 0.2u/2g between sections 1 and 2 of Fig. 8-50, calculate the flow in gallons per minute. From Prob. 8.92, vs = 0,250v, = (0.250)(2.287) = 0.5718 m/s; pu/y + (0.250V.)*12)(9.807)] + z= Ipaly ~ & + 0.250) + (2, ~ 22+) + v3/(2)0.807)] + z+ 0. For Prob. 8.93, add a term 0.2v4/2 to the Previous equation, giving ps/ + (0.250u,)"/(2)(9.800)] + 24 = [ps/y ~ (& + 0.250) + (2, ~ 22 + B+ V¥/[2V9.807)] + 2,+ (0.2)(0.5718"/[(2)9.807)}, v2= 2.272 m/s; Q = Ave = [()(0.150)'/4] 2.272) = (0.0401 n°/s = (0.0401 /(0.3048)](7.48)(60) = 636 gpm. ‘The device shown in Fig. 8-51 is used to determine the velocity of liquid at point 1. It isa tube with its lower end directed upstream and its other leg vertical and open to the atmosphere. The impact of liquid against ‘opening 2 forces liquid to rise in the vertical leg to the height 2 above the free surface. Determine the velocity at 1. E pily + vig + n=pily tvi2gt nth, — kt+viI2+O=04(k+42) +040 v= V2gAE Fig. 851 In Fig. 8-52 the losses in the exit pipe equal Kv*/2g, where K = 5.0. The tank reservoir is large. Compute the flow rate in cubic feet per minute. ' pil + Ug +2 =paly + vig + thy (15)(144)/{(0.86)(62.4)] +0 + 6 = (14.79(144)/[(0.86)(62.4)] + v¥/{(2)(32.2)] + 0 + (5.0)(v3/12)(32.2)]} VR BSS Q= Av=[(x)()/4]8.54) = 0.0466 8/5 or 2.80fF'/min 14.7 ein? abs t Fig. 852 ‘The manometer fluid in Fig. 8-53 is mercury, Neglecting losses, calulate the flow rate in the tube ifthe flowing (a) water, (6) air. Use 60°F as the fluid temperature. W pil + vildg +4) pal + vU2g +24 he, pily + vil2g +0 paly +0+0+0, pa~ p= (y)(0i/28). From the manometer, ps + (62.4)(y/12) + ((13.6)(62.4)}(h) — (62.4)(i) — (62.4)(9/12) = pay Pa Ps = 65.52 Ib/; (r)(uil2g) = 65.52. Ce) (2A){UFN2VG22Y} =65.52 y= BWA/s = Aw. =[(=MA)/4B.223) = 0.718 10/5 (©) (0.0763){vi/1(2NB2.2)}} = 65.52 vy = 2B5.24t/s = Ayvy = [)(4)?/4] 235.2) = 20.5 07s Fig. 853 182 J CHAPTER 8 897 8.98 39 In Fig. 8-54 the uid is air (y = 12.N/m’), and the manometer fuid has s, compute the flow rate in L/s B ply + viltg + = pal + vil2g + 22+ he, pil +0+0=psly + vi/2g +040, p,— Pr (YMvi/2g). From the manometer, p; + (12x 10°3)(y + 0.080) ~ [0.827)(9.79)|(0.080) ~ (12 x 10") = ps, ps ~ P= 0.827. Assuming no losses, (0.6467 KN /e?s ()(u3/2g) = 0.6467, (12 x 10°)(v3/[(2)(9.807))) = 0.6467, v,=32.51 m/s; Q= A.v,= {(£)0.050)'/4]22.51) = 0.0638 m/s = 63.8L/s. By 400mm Dy 50mm 1 f 7 7 ¥ as Jeomm FE Fig. 8-54 ‘The flow from two reservoirs mixes together and flows through a common pipe. The elevations and pipe diameters are indicated in Fig 8-55. Both reservoirs contain the same liquid and are open to the atmosphere. ‘The common pipe empties tothe atmosphere. Neglecting any frictional effects, ind the flow rate through the common pipe. ' pily + vi/2g + z= paly +vi/2¢4 24h, Pani +0+hi =paly + v3/2g +0+0 Pan + Yh = pat YUH o poly + vi/2g + 25=paly +VU2G+20+h, — Pamly +0+h2=paly + vi/2g+0+0 Pant Yh = pat 2g 2 pily + Vi/2g +26 poly +val2¢ + 24+, poly + ¥3/2g +0= Pualy + v/2g + (—hs) +0 Since v5 = vs, Ps= Pam Yhy @ Assume ps = p2= Pa. Substituting this common value of pressure back into Eqs. (1) and (2) and solving for the velocity in each branch, we get vz = V2g(Vn + hs), va= V2g(hs + hs) Q = Au = (adi/4)V2¢(h + hs) + (xd3/4)V2gh, + hy) = (x/4)|d (hy + hs) + dzv2g(hz + hs))- A steady jet of water comes from a hydrant and hits the ground some distance away, as shown in Fig. 8-56. If the water outlet is 1 m above the ground and the hydrant water pressure is 862 kPa, what distance from the hydrant does the jet hit the ground? Atmospheric pressure is 101 kPa. Te magnitude of v, can be obtained by noting that at the hydrant outlet the low is entirely inthe x direction, v= V,. Applying the Bernouli equation between the interior of the hydrant and the outlet gives s+ pay. + ipVi= p+ pay,+ ipV3. The pressure inthe hydrant ps given, and the outlet is open to the atmosphere, p:= Pam The elevation of points 1 and 2s the same, y=)». We assume thatthe outlet area is ‘small enough compared with the hydrant cross-sectional area for the hydrant to be essentially a reservoir, Vice V3. Neglecting ¥,, we get v, = V(2/P)(Ps— Pan). Since vis constant, it can be brought outside the integral for giving '=v, 7. ‘To find the ime T required for a fluid particle to hit the ground, we apply the Bernoulli equation between point and some arbitrary point on the jet having elevation y and velocity V: p+ pgh = Pam + pay + 3pV" Now V?=u!+ v3. When we use the previously determined value of v, and note that v, = dy/d, the Bernouli equation becomes (dy dt) = 2g(h —y). We take the square root (the negative rat isthe appropriate one since ‘dy/di must be negative): dy/dt = —V2g(h —y). Then we separate variables and integrate between the limits of Yoh t=Oandy=0, [a--a[« Integrating and solving for T gives T = VZh/g. The y component of the Suid motion is that of a body freely falling under the infuence of gravity. Finally, we substitute numerical values to get v, = Vral(662 ~ 101)(1000)] = 39.0 (m - N/kg)"*, oF 39.0 mis, VQNE-/9.807 = 0.4525; L = ur = (39.0)(0.452) = 17.6. Tr FUNDAMENTALS OF FLUID FLOW 0 183 3 Pames Reservoir 2 fy Hl we | | | cohort £190 Water flows between the two reservoirs in Fig. 857 at the rate of 16 L/s. What is the head loss in the pipe? If atmospheric pressure is 100 kPa and the vapor pressure is 8 kPa, for what constriction diameter d will cavitation ‘cccur? Assume no additional losses due to changes in the constriction Wily + vil2g +21 pal + Vi/2g +24 hy, 0404 20=04045 +h, f= 15m; Yaron = O/A grat ™ 0.016/(xd?/4). Assume a central constriction, with 'f, or 7.5-m head loss on each side. Apply Bernoulli's ‘equation between point 1 and the constriction, with P, = Pym = 100 kPa and p, = 8 kPa at the constriction. Fig. 8:56 184 0 CHAPTER 8 100/9.79 +0 +20 = 8/9.79 + via/{(2)9.807)] +0+7.5, Yu = 20.7 m/s; 20.7 = 0.016/(axd"/4), d= 0.0314 m, oF 2.98. Sem Pig. 857 8.101 The horizontal wye fitting in Fig. 858 splits Q, into two equal flow rates. At section 1, Qy = 4ft'/s and ‘Pi = 20 psig. Neglecting losses, compute pressures p2 and ps. ' pily + vi/2g + 21= ply + 032+ 24h, vi= Qi/A=4/{(a)(8)"/4]=20.37 fs vam Qala SMC) 14] = 22.92 8s (209(144)/62.4 + 20.37/(2)(32.2)] + O= (p:)(144)/62.4 + 22.92/[2)(32.2)] +040 P2= 19.3 psig Pil ¥ + Vi/2g + 2,= pal + V3/2g + 25+h, — Vs= QslAy= A/{(a MAY /4] = 40.74 fe/s (20)(144)/62.4 + 20.377/[(2)(32.2)] + O= (p,)(144)/62.4 + 40.747/[2)32.2)] +040 py = 11.6psig 4, 24in ? 1 \Ga.sin ng. 8.102 A cylindrical tank of diameter do contains liquid to an intial eight hy. At time r= 0 a small stopper of diameter is removed from the bottom. Using Bernoulli's equation with no losses, drive a diferental equation forthe free-surface height h during draining and an expression forthe time fa to drain the entire tank. 4 Lesing point 1 be the guid srtace and point athe ext, ply 2 +21 = poly +i +s 0+ vi/2g +h =0+ vif2g +0+0, vi Avni = Arua, (ad¥/4)(0,) = (nd"/4)(02), va™ (dol) (0)s vi= [(deld)*(o.) — 28h, rghi( 1). Or v, = VKA where K = 2g/[(do/d)* — 1}, But also, v= —dh/dt, dh/de = Ri th ‘ _. yp VR? Sy [ve QR = VR QR W)=—VEr = (ns -) Or, b= (ho? ~ [g/(2Mas/a* — IY. 8.103 Inthe water flow over the spillway in Fig. 59, the velocity is uniform at sections 1 and 2 and the pressure ‘approximately hydrostatic. Neglecting losses, compute v, and v;. Assume unit width. i ily + vi/2g + 21=prly + v2 t+ mth, OF vi/2g+6=0+ v3/2+1+0 Ama Awr [OMKD)=(OMK) v= 6v, ViL2)(9.807)] + 6 = (6v,)°2)(9.80)] +1 v= 1.67 m/s ‘vs= (6)(1.67) = 10.02 m/s FUNDAMENTALS OF FLUID FLOW J 185 af Fig. 8-59 {$8106 For the water channel flow down the sloping ramp of Fig. 8-60, h, = 1m, H'=3m, and v, = 4m/s. The flow is ‘uniform at 1 and 2. Neglecting losses, find the downstream depth h, and show that three solutions are possible, of which only two are realistic. Neglect friction. FW pily tutte +zy=pily vig tn th, — 0+4/1(2)(9.807)] + 3+ 1) = 0+ vYI29.807)] +h, +0 Awi=Aw2 [4 =[A)0)e2) v= 4/hy #N(2)9-807)] + 4= (4/hYMAVO.8OT)| + hy 3 -4.816K3 + 0.8157 =0 ‘There are three mathematical solutions to this equation: h,=4.78m (subcritical) 432m (supercritical) ‘h, = —0.396m (impossible) Fig. 8-60 8105 For water flow up the sloping channel in Fig. 8-61, hy =0.5 ft, v, = 15 f/s, and H =2{t. Neglect losses and. assume uniform flow at 1 and 2. Find the downstream depth , and show that three solutions are possible, of which only two are realistic. © pily + vide +2 =paly + 022g + mth, — 0+ 157/{(2)(32.2)] + 0.5 = 0+ v3/[(2)32.2)] + (2+h,) +0 Avi = Ave [(0-5)(1)IAS)=[(.))N@2) v2 = 7-5/ha 15*/{(2)(32.2)] + 0.5 = (7.5/h2)M[(2)G2.2)] + (2+ ha) 3 - 1.994H} + 0.8734 = 0 There are three mathematical solutions to this equation: hy=1.69ft (subcritical) fh, =0.887ft (supercritical) A, =—0.582ft (impossible) 186 0 CHAPTER 8 8.107 8.108 8.109 8.10 106 A constant 12-Ib force is applied to the piston in Fig. 8-62, and flow losses are negligible. Compute the water-jet exit velocity vs, Waly + 0302 + 21= Pam!Y + Ui/2g + 22+ hi Considering the force acting on the piston, 12 (= Pam{CBY AD, Py ~ Pam = 34.38 10/f; Avy = Av, (CYB) /4](01) = [GAY /4](02), v1 = 0.2500; (D1~ Pam) / + (0.2500) M(2}G22)] + 0 vf[2)32.2)] + 0+ 0, 34.38/62.4 + (0.2500,)*/12\32.2)] + 0= vY[Q)G22)] +040, v»= 6.15 ft/s. 428in dgzain Fer — war —4 Pde} Pas Hig. 8462 ‘The horizontal lawn sprinkler of Fig. 8-63 is fed water through the center at 1.2L/s. If collar friction is negligible, what is the steady rotation rate for (a) 0= 0° and (6) 8= 30°? ‘Choose an inertial (nonrotating) frame with origin atthe center of the sprinkler; let (p, ) be polar coordinates relative to this frame. ‘An emergent waterjet has velocity components v= vysin@ vy = ¥9c08 8— ro For zero reactive torque—the criterion for the steady state—v, =0, or a = (0.08 8)/r. @ = (10.39)(c050°)/0.20= 51.95 rad/s or 496xpm o @ = (10.39)(c05 30°)/0.20 = 44.99 rad/s or 430 1pm. ‘Water flows at 6 ft/s through a pipe 500 ft long with diameter Lin. The inlet pressure p; = 200 psig, and the exit ‘section is 100 ft higher than the inlet. What is the exit pressure p if the friction head loss is 350 ft? ! pily + vil2g + 2=psly + vi/2¢ + 2+ he (200)(144)/62.4 + vi/2g +0= (p.)(144)/62.4+ v3/2g +100+350 vi/2g=vi/2¢ ps 5.00 psig ‘A.30-in-diameter pipeline carries oil (6.g. = 0.86) at 600 000 barrels per day. The friction head loss is 10 ft per 1000 tof pipe. Compute the pressure drop per mile. ' pily + vile +2. pily + vile + 25+ he, iI {(0.86)(62.4)] + v}/2g +0 = pz/{(0.86)(62.4)] + v3/2g + 0+ (10/1000)(5280) vi/2g = vi/2g Ps — P2= 2833 Ib/f® or 19.7 Ib/in* ‘The long pipe in Fig. 8-64 is filled with water. When valve and water flows at 10'/s, p, ~ p= 25 psi. What isthe fri ‘condition? 1 pilt + vig +2=pily +vil2gt nth, — vi/2g=vi2g is closed, p,— p: = 12 psi. When the valve is open head loss between 1 and 2 for the flowing FUNDAMENTALS OF FLUID FLOW 0 187 ‘Valve closed: (Pa~ pi(h44)/62.4 = <= (12)(144)/62.4 =27.09 8 Valve open: @ )(I4A)/62.44 (21-2) =A, (25)(144)/62.4427.09—h, = 5.4K Constant- diameter vie Fig. 8.64 ALLL Find the manometer reading in the lossless system of Fig. 8-65. 1 Pill + vile +2," paly + vig + 2th, Amr =Av2 (CRY AIR) =(NAY AW.) v2 = 18.0ft/5 pul +2/[(2\32.2)] +0= 0+ 18.0°/((2)(32.2)]+8+0 — pyly = 12.978 For the manometer, 12.97 +2.5 ~13.6h =0, t= 1.14 ft _L. |lese | Mercury Fig. 8-65 A112 In Fig. 8-66 00 p. 188 the pipe exit losses are (1.5)u/2g, where vis the exit velocity. What isthe exit weight fox of water? B pul + vifag + 21=paly + vite + 2th, pr= 2O)(144) + [(0.68)(62.4)](4) = 3050 1b/° 3050/62.4 + 0+5=0-+ v3/{(2)(32.2)] +0+ (1.5)(v9/[2)G2.2))) v= 37.25¢e/s W = yAv = (62.4){()(4)/4]G7.25) = 50.7 Ib/s 8113 In Fig. 8-67 the fluid is water, and the pressure gage reads p, = 180 kPa gage. Ifthe mass flux is 15 kg/s, what is the head loss between 1 and 2? ' ily + vi/2g + 2,= ply + vig + athe M=pAv 15 = 1000{()(0.08)7/4](v,) vy =2.984m/s 15 = 1000{(7)(0.05)/4](v;) _ v,=7.639 m/s 180/9.79 + 2,984%/[(2)(9.807)] + 0=0 + 7.6397/[(2)(9.807)] + 12+h, — h, =3.86m 188 0 CHAPTER 8 8a 81s ir 20 fin? gage tom Fig. 867 il at specific gravity 0.761 flows from tank A to tank E, as shown in Fig. 8-68, Lost head items may be assumed to be as follows: A to B = 0.60u3,/2g; B to C= 9.0vb,/2g; C to D = 0.40v2/2g; D to E = 9.0v4/2g. Find the flow rate and the pressure at C. ' aly + ads + 24=pely + vEl28 +26 +f 0+ 0+ 40.0=0+0+0+ (0.60v}, +9.0v%, + 0.4003 + 9.0v2)/((2)(32.2)} 9.6002, +9.40v3= 2576 Ay = Avs [UVB n) = [VHD V6 = 4.00042 | 9.60vis + (9.40)(4.0002)"=2576 vn = 4.012 M/s Q = Av = [(0)(B)/4](4.012) = 3.15 £75 Pal + Val2g'+ 24% Pel + U2/28 + 20+ hy, 040+ 40,0 = (p)(144)/{(0.761)(62.4)] + 4.0127/12)(32.2)] + (40.0 + 2) + [(0.60)(4.012)" + (9.0)(4.012)"1/[(2)(32.2)] Pe -1.S3 Ib/in? Fig. 8-68 (© What is the pressure on the nose of a torpedo moving in saltwater at 100 ft/s ata depth of 30.01%? (6) Ifthe pressure at point Con the side ofthe torpedo at the same elevation asthe nose is 10.0 psig, what i the relative telocity at that point? 1H (@)Inttis case, greater clarity inthe application of the Bernouli equation may be attained by considering the relative motion ofa stroam of water past the stationary torpedo. The velocity of the nose of the torpedo will a6 au a8 a FUNDAMENTALS OF FLUID FLOW 0 189 then be zero. Assume no lost head in the streamtube from a point A in the undisturbed water just ahead of the torpedo toa point B onthe nose ofthe torpedo: pal + ua/2g + 24" aly + v2/26 + Ze 4 hue S007 TO yQNGZDI + Om poly + 0-040, poly = 185.38, py = (4.2)(185.3)/144 = 82.6 si. This pressure is called the stagnation pressure and may be expressed as 7, = Po pv4/2. ) pal + Ua28 + 24 Del + UE + 20+ In 30.0 + 1007/{(2)(32.2)] + 0 = (10.0)(144)/64.2 + v2/[Q)G2.2)] +040 vem 1024 ft/s ‘Asphere i placed in an air stream which sat atmospheric pressure and is moving at 100.0f/s. Using he sar etsr af constant at 0.00238 slag), calculate the stagnation pressure and the pressure on the surface of the sptere at point B, 75* from the stagnation point, i the velocity there is 220.0 ft/s. 1. From Prob. 8.115, p,= po pus? = (147)(144) + (0.00238)(100.0)72 = 2129/0 14.8 bin. ply + dap nce poly + UAI2g + Zn h, 2129 0.00238)(32.2)]+ 0+ 0 pal + 220.0'/(2)32.2)] + 0+ 9, papa 2 009 of ait, po = (0.00258) (32,2) 27 029) = 2071 I/F 14.4 Thin ‘A arge closed tank i fled with ammonia (NH,) under a pressure of 5.30 psig and at 65°F. The ammonia, Gucherges imo the atmosphere through a small pening in the side of the tank. Neglecting friction loses, aaaeane tne velocity ofthe ammonia leaving the tank assuming constant density. The gas constant for ammonia is 89.510/'R. I Apply Bernouli's equation between the tank (1) and the atmosphere (2). pily + uil2g + 247 paly 4 Tae et has y =pIRT, Yhon,= (530+ 14.7944) / 9.5) (450 + 65)] = 0.06129 I/F, (.30)(144)/0.06129 + 0+ 0=04 v3{(2)G22)] +040, ¥,= 896 H/s. Water at 90°F isto be lifted from a sump ata velocity of 6.50 f/s through the suction pipe of « pump. Calculate the tneoretical maximum height of the pump setting under the following conditions: Pam = 14.25 psi, . =0.70 psa, and hy in the suction pipe = 3 velocity heads, ‘The minimum pressure atthe entrance to the pump cannot be less than the vapor pressure ofthe liquid ‘Apply Bernoulli's equation between the water surface ouside the suction pipe and the entrance to the Pow: Jeet alae + = paly + vig + 2+ he, (14.25) 144)/62.1 +04 0= (0.70)(44)/62.1 + 6.0°/(2)G2.2)) © Pult (6 30 102)G2-)), = 28.8. (Under these conditions, serious damage duc to cavitation will probably ‘ccur.) For the Venturi meter shown in Fig. 8-69, the deflection of mercury in the differential gage is 14.3 in. Determine the flow of water through the meter if no energy is lost between A and B. ' aly + 042g + 24= Pal + Val2g + 20+ hi. paly + viMM(2\32.2)] + 0= poly + v5/[@)G22)] +30.0/12+0 — paly Pal = 0.01553(v} ~ v4) + 2-500 Anva= Ande (eN(BY IAI) = (EVI 2) V4 0.2500 From the manometer, paly + 2-+ 14,3/12 ~ (13.6)(14.3/12) ~ 2 ~30.0/12 = pol, Pal ~ Pal Try O.O1553lob — (0.2805) + 2.500, vy = 3212 fs; Q = Av = [(x}(B)/A](32.12) = 6.31 fs 7.524, | | | | 190 J CHAPTER 8 8.120 az 8.123 8.128 For the meter in Fig. 8-69, consider air at 80°F with the pressure at A = 37.5 psig. Consider a deflection of the gage of 14.3in of water. Assuming that the specific weight of the air does not change between A and B and that the energy loss is negligible, determine the amount of air flowing in pounds per second. ! pal + Uil28 + 24% poly + v2 + 20th, Pal + V4M(2)G2.2)1+ 0 poly + vb/[2)32.2)] +30.0/12+0 aly ~ poly =0.01553(05 ~ v3) + 2.500 Aava=Anve —(Ca)CBYIA|(04) = (VAY) v4 =0.25009 Y=PIRT Yor = (315+ 14.79(144)/(53.3)(460 + 80)] = 0.2612 I6/f° From the manometer, pa/y +z + 14.3/12 ~ (62.4/0.2612)(14.3/12) ~ 2 — 30.0/12=ps/y, pal ~ Pal y= 286.0 ft of air; 286.0 = 0.01553[v} — (0.250u,)"] + 2.500, vp = 139.5 ft/s; W = yAv = 0.2612{(2)(4)*/4](139.5) = 7.15 Ib/s. Given a frictionless flow of water at 125.6 ft/s in a long, horizontal, conical pipe, of diameter 2ft at one end and 6 t at the other. The pressure head at the smaller end is 18 ft of water. Find the velocities at the two ends and the pressure head at the larger end. ' v= QUA, = 125.6/[()(2)'/4]= 39.98 ft/s vg = Q/A,= 125.6/{(2)(6)7/4] = 4.44 fe/s pul + vil2g + 2=paly + vi/2g + 2+ he 18 + 39.987/{(2)(32.2)] +0= paly + 4.48°/[2)(32.2)]+0+0 — py/y =42.5 ft of water Water flows through a long, horizontal, conical diffuser atthe rate of 4.0m’/s. The diameter of the diffuser ‘varies from 1.0m to 2.0m; the pressure at the smaller end is 8.0 kPa. Find the pressure at the downstream end of the diffuser, assuming frictionless flow and no separation from the walls. ' Dil + Vil2e +2 = paly + Vi 28 + the O/[(#)(1.0)/4] =5.093 m/s v= Q/A,= 4.0/](2)(2.0)/4] = 1.273 m/s £8,0/9.79 + 5.093/[(2)(9.807)] + 0= p3/9.79 + 1.2737/1(2)(9.807)] + 0:0" p2=20.13kPa ‘A vertical pipe 3 ft in diameter and 30 ft long has a pressure head at the upper end of 22 ft of water. When water flows through it with mean velocity 1S fps, the friction loss is 6 ft. Find the pressure head at the lower end of the pipe when the flow is (a) downward and (6) upward. 1@ pily + ilg + 2=paly + vil2g + 2+ he, 22+ 15*/[(2)(32.2)] + 30 = pay + 15°/(2)32.2)]+0+6 — paly = 46.0 o pal + vi/2g-+ 24 ply + vi/2g + 24+ he pal + 15°/(2)G2.2)] + 0= 22 + 154/[2)G2.2)]+ 3046 pal = S80 ‘A vertical conical pipe has diameter 1.5 ft atthe top and 3.0ff at the bottom, and is 60 ft long. The friction loss is 10 ft for flow in either direction when the velocity at the top is 30 fps and the pressure head there is 6.5 ft of water. Find the pressure head at the bottom when the flow is (a) downward and (b) upward. ' = Avy = [()(1.5)7/4](30) = 53.01 00/5 v= Q/A,=53.01/{()3.0)'/4] = 7.50./s @ pil + Vil2g +2 =paly + Vi/2g + +h, 6.5 + 30°/[(2)(32.2)] + 60 = poly + 7.50°/1(2(32.2)] +0410 pa/y = 69.68 o paly + vi/2g + 2=pily + vi/2g + 2+ hy paly +75 [2N32.2)] + 0=6.5 + 30/1(2)(32.2)]+ 60+ 10 psy =89.6 8 The inclined pipe in Fig. 8-70 is of uniform diameter. The pressure at A is 20 psi and at B, 30 psi. In which direction isthe flow, and what isthe frietion loss ofthe fui, ifthe liquid has specific weight (a) 30 lb/ft? and @) 100 /t02 W pair + v4 I2g +24=paly + vol2g + 25 + h,. Assume flow is from A to B. FUNDAMENTALS OF FLUID FLOW J 191 (a) (20)(144)/30 + v3 /2g + 25 = (30)(144)/30 + v9/2g + 0+ hy, v5/2g = vb /2g, hy = -23.0ft. Since hy is negative, flow is actualy from B (0 A (b) (20)(144)/100 + w2/2g + 25 = (30)(144)/100 + v5/2g + 0+ he, vi positive, low is from A to B, as assumed. = vb/2g, hi = 10.6 tt. Since h, is Fig. 8-70 $8126 In Fig. 8-70, ifthe difference in elevation between A and B is 10m and the pressures at A and B are 150 kPa and 250 kPa, respectively, find the direction of flow and the head loss. The liquid has specific gravity 0.85. IW paly + vil24 + 24=paly + vbl2g + 20+ he. Assume flow is from A to B. 150/{(0.85)(9.79)] + v3/28 + 10 = 250/{(0.85)(9.79)] + vp/2g +0-+ h,, v4/2g = v3/2g, h,, = -2.02m. Since h,, is negative, flow is actually from B to A. 8.27 An irrigation line carries water from a lake down into an arid valley floor 810 ft below the surface of the lake. ‘The water is discharged through a nozzle with a jet velocity of 220 fps; the diameter of the jet is 4 in, Find the power of the jet and the power lost in friction. i aly + V3/2g + 24=paly + val2g + zy +h, — 0+0+810=0-+2207/[(2)(32.2)] +04 hy, hy = 58.45 ft Q = Av =[(x)(4)/4](220) = 19.2 f'/s Pha = Qyv3/2g = (19.2)(62.4){220"/{(2)(32.2)]} = 900.000 ft - Ib/s = 900 000/550 = 1636 hp Pax = Qh, = (19.2)(62.4)(58.45) = 70 000 ft b/s = 70 000/550 = 127 hp 8.428 Water is flowing in a channel, as shown in Fig, 8-71. Neglecting all losses, determine the two possible depths of flow y, and ys 7 1 aly +UR2g + 24= Daly + URI2g +29 + hi, Q=Aava=[(4(10)](16.1) = 644 ft/s Up = Q/Ag = 644/(10y) = 64.4/y 0+ 16.17/(2)G2.2) + B+ 4) =0+ (AMP NAQGLDI+y +0 GAd/y*+y-16.02=0 yP-16.02y?+644=0 y,=216f y,=15.8ft Fig. 8-71 8.129 Neglecting all losses, in Fig. 8-71 the channel narrows in the drop to 6 ft wide at section B. For uniform flow across section B, determine the two possible depths of flow. 1 paly + al2g + 24 = paly + vbl2¢ + 20+ he, = Agua =[(MUOII6.1) = 6460/5 v9 = O/Ay = 644/(6y) = 107.31 0+ 16.17/[(2)32.2)] + (8 + 4) =0 + (107.3/y) M322] +y +0 1788/y? + y ~16.02=0 Y= 16.0277+1788=0 yy =3.8 eH IS3 A 192 0 CHAPTER 8 8.130 8.131 8:32 8.133 8.14 8.138 [the losses from section A to section B of Fig. 8-71 are 1.9, determine the two possible depths at section B. ' pal + UR/2g + 24= Daly +528 + 20+ he = Agu, = [(4)(10)](16.1) = 644 ft/s vy = Q/Ay = 644/(10y) = 64.4/y_ 04+ 16.17/12)32.2)] + (B+4)=0+ (4.4/9 FMQG22|+y +19 — OAAly?+y~1412= y= 14127 +64.4=0 y= 2.346t 13.8 ft High-velocity water flows up an inclined plane, as shown in Fig. 8-72. Neglecting all losses, calculate the two possible depths of flow at section B. ' aly + 04/26 + 24 Poly + Ub/28 + 20 + hy = Ava = (48) 2)19.806) = 9.806 m'/s vp = Q/Ay=9.806/(2y) = 4.93/y 0+ 9.8067/[(2)(9.807)] + i =0-+ (4.903/y)/[2)0.807)] + (25+y) +0 1.226/y?-+y-2.903 =0 YP-2.905y7+1.226—0 y= 0.75m y= 274m 06 500 ~— cont mws | 4 2 meen In Fig. 8-72, the channel changes in width from 2m at section A to 3 m at section B, For losses of 0.3 m between sections A and B, find the two possible depths at section B. a aly + ¥4/28 + 24> Poly + Ua/2¢ + Zh, — = Anda = [(48)(2)](9.806) = 9.806 m/s Up = Q/Ag=9.806/(3y) =3.269/y 0 + 9,8067/[(2)(9.807)] + #885 = 0 + (3.269/y)*/[(2)(9.807)] + (2.5 +y)+0.3 O.S448/y?+y—2.603=0 y?—2.603y"+0.5448=0 y= 0.510m — y, = 2.52m For loses of 0.05H through the nozzle of Fig. 8-73, what i the gage diference R in terms of H? ' L2H +1.2y + 1.28 -3.0R ~1.2y =(0.95)(1.2(H) R= 0.0333 so 3 Fig. 873, ‘Neglecting losses, calculate H in terms of R for Fig. 8-73. W128 + 1.2y + 1.28 ~3,0R ~1.2y =1.2H, Therefore, R =O for ll H. ‘At point A ina pipeline carrying water, the diameter is 1 m, the pressure 100 kPa, and the velocity m/s. At point B, 2m higher than A, the diameter is 0.5 m and the pressure is 20kPa. Determine the head loss and the direction of flow. 8.136 a7 a8 81398 FUNDAMENTALS OF FLUID FLOW 0 193 I Assume the direction of flow is from A to B. pa/y + v3/2g +24 Daly + 3/24 + 9+ hy, Q= Aii= [Ga )(4y/4)(1) = 0.7854 m/s, vs = Q/Az = 0.7854 /{(\(0.5)/4] = 4,00 m/s, 100/9.79 + 1°/[(2)(9.807)] + 0= 20/9.79 + 4.007/{(2)(9.807)] + 2+ hz, he = 5.40 m. Since h, is positive, flow is from A to B as assumed. ‘Water is flowing in an open channel at a depth of 2m and velocity of 3 m/s, as shown in Fig. 8-74. It then flows down a contracting chute into another channel where the depth is 1 m and the velocity is 10 m/s. Assuming frictionless flow, determine the difference in elevation ofthe channel floors. I The velocities are assumed to be uniform over the cross sections, and the pressures hydrostatic. Daly + U%/2g +24 poly + val24 + 29 +h, 0+3*/[QN9.807)] + (y +2) = 0+ 10/[(2)9.807)] +140, y= Sob. : aan Fig. 8-74 For losses of 0.1 m, find the velocity at A in Fig, 8-75. The barometer reading is 750 mmHg, ! Poly + Val2g +25 pal + Val28 + 24+ he, 7519.79 + 0+3= [(13.6)(9.79)}(0.750)/9.79 + v3/[(2)(9-807)] + 040.1 v4=2.66m/s For flow of 375 gpm in Fig. 8-76, determine H for losses of Su7/2g. a 2 _ G75/7.48)160 pag teeth PA" Gaya 0404 H=0+4255%/[2)G2.2] +04 5(6.25872)G22)) H= LOK ' = 4.255 fils Fig. 8-76 For 1500-gpm flow and H = 30 ft in Fig 8-76, calculate the losses through the system in velocity heads, Ku?/2g. (1500/7.48)/60 Oey 040+ 30=0+ 17.027/[(2)32.2)] +04 KITOZMG2M —-K=5.67 Gi = 17.02 ft/s 5.61 velocity heads) 194 0 CHAPTER 8 8.140 The losses in Fig. 8-76 for H = 20 ft are 8(v*/2g). What is the discharge? B paly-+vil2e + 29m paly +Ul2e + 24th, 0+0+20— 0+ v4/l2(32.2)] +0+ 8(¥% M2322). ‘U4 = 11.96 ft/s = Avg = [(4)(5)'/4](11.96) = 2.35 Ys, 8.141 In Fig. &-77, the losses up to section A are Sui/2g and the nozzle losses are 0.05u3/2g. Determine the discharge and pressure at A, if H=8m. t Paly + Vbl2g + 29 =Pely + UEI2 + 20 +h 0+ 04 8= 0+ vi/[(2)(9.807)] + 0+ S(vi/[(2)9.807)}) + 0.05 (v3/[(219.807)}) 0.05353u3+0.258907-8=0 A= Ars —_[()(5B)*/41(0.) = [Ca VG) /4](v2) v= 0.1110 0.05353u3 + (0.2549)(0.1111v,)7—8=0 — v,=11.88m/s vy = (0.1111)(11.88)=1.320m/s Q = Avs = [C1 G8) 411.88) = 0.0233 m/s poly + 3/29 + 29 =paly + UR28 + 24th 040+ 8=p4/9.79 + 1.3207/{(2\9.807)] + 0+ 5{1.320°/1(2)(9.807)]} pa = 73.1 KN/m? Léicisomm's ¢ om’ ee comm 77 8.142 For pressure at A of 25 kPa in Fig. 8-77 with the losses given in Prob. 8.141, determine the discharge and the head H. Paly + vbl2g +29 paly + 04/28 + 24+ he 040+ H =25/9.79 + vii{(29.807)] + 0 + 5(v3/[(2)(9.807)]) H=0.30594 +2.554 @ Pal + Vil28 + 29 poly + Vel28 + 20+ hi 0+ 0+ H=0+ v¥{(2\9.807)] + 0+ 5{vF/[(2)(9.807)]} + 0.05(v3/[2)(9.807))) 0.053533 +0.254903—H=0 Ayu = Asus [(x)(48B)"/41(vs) = [()i8e)/41(02)—_ v= 9.00004 (0.05353)(9,0000,)* + 0.25450 @ Solving Eqs. (1) and (2) simultaneously, (0.05353)(9.000v,)° + 0.2549v} = 0.3059u3 + 2.554, v, =0.7720m/s; Q = Ayu, = [()(488)*/41(0.7720) = 0.0136 m*/s, H = (0.3059)(0.7720)* + 2.554-= 2.736 m. 8.143 The system shown in Fig. 8-78 involves 6:in.-.d. pipe. The exit nozzle diameter is 3 in. What is the velocity v, of flow leaving the nozzle? Neglect losses. ' ily + 012g + 21=paly + V4/28 + 24+ he (14.79(144)/62.4 + 0-4 40 = 4000/62.4 + v2/1(2)(32.2)] +040 W=BAMYs — Aava=Anvo (CEYIRS.14) = [VAI v, = 100.5685 4000 lb/ft? abs —T wae 2 water 40ft 10 ste a BD ve TOTTI Fig. 6-78 FUNDAMENTALS OF FLUID FLOW J 195 AIM In Fig. 879, H = 6m and h =5.75m, Calculate the head loss. t v2= Vigh = VQ)OBOIVGS.75) = 10.62m/s ply + vi/2g +2 =paly + V/2g + thy 0+0+6=0+ 10.627/[(2)(9.807)]+0+h, hy =0.250m ‘845 In Fig. 6-80, 0.1 m’/s of water flows from section 1 to section 2 with losses of 0.4(v, ~ v.)?/2g. If p,= 100 kPa, find ps. ' pil + vil2¢ + 2.= poly + UU28 + 2,4 h. 1, = Q/Ay = (0.1)/{(#)(0.300)'/4] = 1.415 m/s v2 = O/A, = (0.1)/{(*)(0.450)"/4] = 0.629 m/s 0.4(1.415 ~ 0.629)? ‘oa P2=100.8kPa 100/9.79 + 1.415°/[(2)(9.807)] + 0 = p,/9.79 + 0.629/[(2)(9.807)] +0 + saondan.; 20° 650 mmdiam ® @® Fig. 8-80 4146 Neglecting losses, determine the discharge in Fig. 8-81 F pulysvigtn=pily + vig tnt, [(0.86)(62.4]3)/62.4+ 04 404 v3/[2N32.2)] +040 v= 20.5948 Q= Av= [(x)(4)/4]20.59) = 0.45 1/5 Fig. 861 8107 A pipeline leads from one reservoir to another which has its water surface 10m lower. For a discharge of 1.0 m’/s, determine the losses in meters and in kilowatts. ' Pir + vig +z =pily + vY2%g +2 +h, —0F0+10=040404h, k= 10m Losses = Qyh, = (1.0)(9.79)(10) = 97.9 kW 196 0 CHAPTER 8 8.448 In the siphon of Fig. 8-82, 4y= 1m, hy =3m, d,=3m, and d,= 5m, and the losses to section 2 are 2.62/26, with 10 percent of the losses occurring before section 1. Find the discharge and the pressure at section 1. 1 Poly + val2g + 2n=pily + v2 +2, +h, 040+ (1 +3) =0+ v3/[(2)(9.807)] + 0+ 2.6u3/[(2)(9.807)] v= 4.668 m/s Q = Ayu, = [()(5)7/4](4.668) = 91.7 m/s vy = Q/Ay = 91.7/[(2)(3)7/4] = 12.97 m/s Poly + vb/2g + t0= pily + vil2g + +h, 0+ 0+ (1 +3) =p4/9.79-+ 12.97/12)(9.807)] +3 + 0.10((2.6)(4.668)°/12)(9.807)}} y= T10KPa Fig. 8-82 8.149 Find the pressure at A of Prob. 8.148 if it is a stagnation point (velocity zero). De poly + vi/2g + 20 =paly + V5/28 + 24+h, — OF0+0=p,/9.79+044+0 p,m —39.2 kPa 8.150 In the friction-free siphon shown in Fig. 8-83, what are the pressures of the water in the tube at B and at A? 1 Poly + vp/2g +20 = Ply + vbl2g + 20+ hi 0+0+43.0=04 vi/[(2).807)] +040 vc=7.67m/s=uy™ 04 Poly + vipl2g + Zo ™ pal y + V5/28 + 25+ he 0+0+3.0=pp/9.79 + 7.67 /{(2)(9.807)] + (3.0+1.5)+0 py = —44.0kPa Poly + vpl2g + 20 = Pal + val2g + Za + he 0+043.0=p4/9.79-+ 7.67 /[(2)9.807)]+3.0+0 p= —29.4kPa Fig. 8.83 8.151 Ifthe vapor pressure of water is 0.1799 m of water, how high (h) above the free surface can point B be in Prob. 8.150 before the siphon action breaks down? Assume atmospheric pressure is 101 KPa. F poly + vbl2g + 2 =poly + 3/28 + 2p + hz. Using absolute pressures and considering that vp = 0 at ‘maximum h when the siphon action breaks down, 101/9.79 + 0+ 3.0=0.1799 +04 (3.0+h) +0, h=10.14m. a 4 9 36 CHAPTER 9 Flow in Closed Conduits Water at 10°C flows in a 150-mm-diameter pipe at a velocity of 5.5 m/s. Is this flow laminar or turbulent? Ng dv]y = (0.150)S.5)/(1.30 x 10-4) = 634 615. Since 634.615 > 4000, the flow is turbulent. SAE10 oil at 68°F flows ina 9in-diameter pipe. Find the maximum velocity for which the flow willbe laminar. N= pdv/. For laminas flow, assume Ng =2000. 2000 = (1.68)({3)(u)/(1.70 x 10"), v= 2.704t/s. ‘The accepted transition Reynolds number for flow past a smooth sphere is 250000. At what velocity will this ‘occur for airflow at 20°C past a 10-em-diameter sphere? 1 New duly 250000~ (0.10)(0)/(1.51 x10") v= 37.8mi/s, Repeat Prob. 9.3if the fd is (a) water at 20° and (b) hydrogen at 20°C (v= 1.08 x 10°*m*/s). 1e@ Nq=dv/y 250000 (0.10\v)/(1.02x 10") v= 2.55m/s ® Nem duly 250000 (0.10)(v)/(.08x 10) v= 270m/s ‘A bin-diameter water pipe is 60 ft long and delivers water at S gpm at 20°C. What fraction of this pipe is taken ‘up by the entrance region? HQ =(5)(0.002228) = 0.01114 ft/s V = Q/A = 0.01114/{(#)(0.5/12)'/4] = 8.170 ft/s. Na=dv/v From Table A-2, v= 1.02 x 10°* m’/s at 20°C, which equals 1.10 x 10 f/s; hence, Ne = (0.5/12)(8.170)/(1.10 x 10°*) = 30.947, Since 30947 > 4000, the flow is turbulent and for entrance length, LJd = 4.4NS*= (4.4)30947)"*= 25, The actual pipe has L/d = 60/{(4)/12] = 1440; hence, dd Leg 8 Tyg 7 Wis 0.017 oF 1.7 percent ‘An oil with p = 900 kg/m’ and v = 0,0002:m*/s flows upward through an inclined pipe as shown in Fig. 9-1 ‘Assuming steady laminar flow, (a) verify thatthe flow is up and find the (b) head loss between section 1 and section 2, (€) flow rate, (d) velocity, and (e) Reynolds number. fs the flow really laminar? Te HGL=2+p/og — HGL,=0+ 350 000/[(900)(9.807)] = 39.65 m HL, = (10)(sin 4%) + 250 000/[(900)(9.807)] = 34.75 m Since HGL, >HGL, the flow is upward. ® hy = WGL,— HGL, = 39.65 - 34.75 = 4.90m © ‘p= (900)(0.0002) = 0.180 kg/m s) Q sagt ia ern 99) — 6.00764 m" @ v= Q/A =0.00764/{(7)(afo)?/4] = 2.70 m/s © Ng = du/v = (afy)(2.70)/0.0002 = 810 ‘This value of Ng is well within the laminar range; hence, the flow is most likely laminar. Is ‘py = 350,000 Pa, 2) =0 Fig. 4 197 198 0 CHAPTER 9 97 98 99 oat 92 933 For flow of SAEI0 oil through a 100-mm-diameter pipe, for what flow rate in cubic meters per hour would we ‘expect transition to turbulence at (a) 20°C [u = 0.104 Pas}, and (6) 100°C [1 = 0.0056 Pa 5}? Assume transition to turbulence occurs at Nx = 2300. Ne = pdv/t. (a) 2300 = (869)(0.100)(v)/0.104 2.153 m/s Q = Av = [()(0.100)*/4](2.753)= 0.0216 m?/s or 77.76 m'fh (6) 7300 = (869)(0.100)(v)/0.0056 v= 0.1482 m/s = Av = {(2)(0.100)?/4](0.1482) = 0.00116 m"/s or 4.18m'/h A fluid at 20°C flows at 0.8 L/s through an 100-mm-diameter pipe. Determine whether the flow is laminar or turbulent if the fluid is (a) hydrogen (y = 1.08 x 10~* m?/s), (6) air, (e) gasoline (v = 4.06 x 10? m?/s), (@) water, (e) mercury (v= 1.15 x 10 m*/s), or (A) glycerin. ' Ne=duly v= Q/A=(0.8 x 10)/{()(0.100)"/4]= 0.1019 m/s @ Na (0.100¥0.1019)/(1.08 x 10-*) = 94 (laminar) Oy ‘Nq=(0.100)(0.1019)/(1.51 x 109) =675 (laminar) © ‘Nq=(0.100)(0.1019)/(4.06 x 10-7) =25099 (turbulent) @ ‘Na= (0.100)(0.1019)/(1.02 x 10-*) = 9990 (turbulent) © ‘Na= (0.100)(0.1019)/(1.15 x 10°") = 88609 (turbulent) wo ‘Nq= (0-100)(0.1019)/(1.18 x 10) = (aminar) Oil (6g. =0.9, v = 0.0003 m"/s) enters a S0-mm-diameter tube. Estimate the entrance length ifthe flow rate is 1Lbs. FT Ng=dvly = Q/A = 0.001/{(7)(0.050)7/4] = 0.5093 m/s Ng = (0.050)(0.5093)/0.0003= 85 (laminar) L,Jd=0.06Ny —_L, = (0.050)(0.06)(85) = 0.255 m ‘What isthe Reynolds number fora flow of ail (.g. = 0.8, ¢ = 0.00200 Ib - s/f) in a 6-in-diameter pipe a alow rate of 10/t/s. Is the flow laminar or turbulent? ' v= QA = 10/10=)(8)/4 Na = pdu/u = [(0.8)(1.94)]()(50.9)/0.00200 50.9 ft/s 9749 (turbulent) Gasoline at a temperature of 20°C flows at the rate of 2.L/s through a pipe of inside diameter 60 mm. Find the Reynolds number. 1 v= Q/A= (2x 107)/[()(0.060)?/4] = 0.707 m/s [Ng = pv /y = (7199(0.060)(0.707)/(2.92 x 10-4) = 104 452 ‘The Reynolds number for fuid in a pipe of 10 in diameter is 2000. What will be the Reynolds number in a ‘in-diameter pipe forming an extension of the 10-in pipe? Take the flow as incompressible. I Np= duly. Since y is constant, [Na/(dv)], = [Ne/(dv]»» Aiv.= Ave, [()(8¥/41(0:) = [EP /A1(09, 14, = 0.3600, 2000/{(1)(0.36002)] = (Naa ()(ua)], (Ne) 3333. ‘Water is flowing through capillary tubes A and B into tube C, as shown in Fig. 9.2. If Q,=3mL/s in tube A, what is the largest Oy allowable in tube B for laminar flow in tube C? The water is at a temperature of 40°C. With the calculated Qp, what kind of flow exists in tubes A and B? 4 or mina ow, assume N= 2300, Ne = dv/v. ln tube C, 2900» (0006)(ve(6 56x 10", 0.2515 m/s; Qe = Ace ™ [()(0.006)"/4](0.2515) = 7.11 x 10°* m’/s, oF 7.11 mL/s, O, In tube A, v4 = Qala = (3 x 10"*/{(22)(0.005)*/4] = 0.1528 m/s, 11 mL/s. ‘Ng = (0.005)(0.1528)/(6.56 x 10° 1165 (laminar) FLOW IN CLOSED CONDUITS 7 199 In tube B, vp = Qn/Ay = (4.11 x 10-*/{(22)(0.004)'/4] = 0.3271 m/s, Na = (0.004)(0.3271)/(6.56 x 10"")= 1995 (turbulent) d= 5mm Samm Fig. 92 9.14 Incompressible steady flow of water occurs in a tube of constant cross section, as shown in Fig. 9-3. What is the head loss between sections A and B? ' pal + Ua/2¢ + 24 ol + Ub/2g + Zn he (00)(144)/62.4 + vi/2g + 0 (30)(144)/62.4+ vp/2g + 100+, Wtg=vbl2g hy =38S 8 Fig. 9-3 9.45 Water flows through a pipe at 5L/s, as shown in Fig. 9-4. If gage pressures of 12.5 kPa, 11.5 kPa, and 10.3 kPa are measured for p,, p2and ps, respectively, what are the head losses between 1 and 2 and 1 and 3? Wl pily + vil2g +2." pal + 2g + 2+ (hi)i-a — 12.5/9.79 + vil2g + 10=11.5/9.79 + vi/2g + 10+ (hii Vig = vig (hy-2=0.102m — py/y + vi/2g +2 =psly + VIE +254 (hea Y= QIA,= (5 x 10)/{(-)(0.050)"/4] = 2.546 m/s v= Q/Ay™ (5 x 10-7) /{(-)(0.030)"/4] = 7.074 m/s 12,5/9.79-+ 2.546'/{(2(9.807)] + 10 = 10.3/9.79 + 7.0747/[(2)(9.807)]+0+ (hi) (hes = 8.0m. Fig. 4 9.46 A large oil reservoir has a pipe of 3in diameter and 7000-ft length connected to it, as shown in Fig. 9-5. ‘Assuming laminar flow through the pipe, compute the amount of oil issuing out of the pipe as a free jet. 200 0 CHAPTER 9 9a7 9.8 ‘Compute the velocity and Reynolds number to see if the flow is laminar. ¥aq=1 x 10 ft'/s. Neglect entrance losses to the pipe. FW pay + vig + n= psy + vi/2gt rth, — prl624+vi/2g+0=0+v3/2g+0+h, vi/2g =vi/2g 2/624 = hy o 08d, Oo peal 128QuL _ 128{(nd*/4)(uyJule_32v,vL _32)(v:)(1 x 10-9000) _ =; Ta te @ pily + vil2g +2) =paly + v2 + th, — 0+0+10=p,/62.4+ vi/[2)G2.2)]+0+0 pr/62.4= 10- v¥[2)G32.2)] o ‘Equating hy from Eqs. (1) and (2), P/62.4= 11.130, o Equating p,/62.4 from Eqs. (3) and (4), 11.13v, = 10 ~ vi/[(2)(32.2)], v3 + 716.80. ~ 644 = 0, v= 0.8973 fs; Q = Av = [()(3)/4)(0.8973) = 0.0840 f/5; Nx = dv/v = ()(0.8973)/(1 x 104) = 2243 (barely laminar). Fig. 95 If 140.L/s of water flows through the system shown in Fig. 9-6, calelate the total head loss between 2 and 3. ' aly + Ui2g + 2.=psly + il2e + 24+ he, a= Q/A2= (140 x 10™)/[(s")(0.300)*/4] = 1.981 m/s v5 = Q/Ay= (140 x 10-*)/[()(0.150)"/4] = 7.922 m/s pr/9.79 + 1.981?/[(2)(9.807)] + 0= 0+ 7.922"/{2(9.807)} + 15+, he =pa/9.79— 18.00 ily + vi/2g + %5=paly + v/2g tz th, — 0+0+30=pa/9.79+ 1.9817/[(2)(9.807)] +040 p/9.79=29.80m hy =29.80~18.00= Determine the maximum velocity for laminar flow for (a) medium fuel oil at 60°F (v= 4.75 x 10™*f/s) flowing through a 6-in pipe and (6) water at 60°F flowing in the 6-in pipe. FF For laminar flow, assume Nq = 2000. Nq = dv/v. @ 2000 = (8)(U)/(4.75 x 10° v= 0190/8 O) 2000 = (£)(0(1.21 x10) v= 0.0486 fs 99 ro) 904 9.6 99 FLOW IN CLOSED CONDUITS J 201 Determine the type of flow occurring ina 12-in pipe when (a) water at 60°F flows ata velocity of 3.50 ft/s and (@) heavy fuel oil at 60°F (v = 221 x 10-*f'/s) flows atthe same velocity. I Np=duly @ ‘Na = (8)(3-50)/(1.21 x 10°*) = 289256 (turbulent) @ Ne = (4)(3.50)/(221 x 10°) = 1584 (laminar) For laminar flow conditions, what size pipe will deliver 90 gpm of medium fuel oil at 40°F (y= 6.55 x 10" f/5)? 1 Q= (90)(0.002228) = 0.2005 f?/s. For laminar flow, assume Nx <2000. Ne = dv/v, v= Q/A = (0.2005) (x d/4) = 0.2553d-*, 2000 = (€)(0.2853d *)/(6.85 x10"), d= 1.95.8. ‘What is the Reynolds number of low of 0.4 m’/s of ol (s.g. = 0.86, = 0.025 Pas) through a 450-mm-diameter pipe? ! = Q/A=0.4/[(#)(0.450)'/4] =2.515 m/s Na=p du/u = [(0.86)(1000)](.450)2.515)/0.025 = 38932 ‘Anoil wth sg. =0.85 and v = 1.8 10-* m/s flows in a 10-cm-diameter pipe at 0.50 L/s. Is the flow laminar or ‘turbulent? ! 1A = (0.50)(1000)/{()()*/4] = 0.06366 m/s Na = du/v = (38\(0.06366)/(1.8 x 10°) =354 (laminar) Fluid with kinematic viscosity 0.00015 f/s flows through a pipe of diameter 9in. What isthe maximum velocity for laminar fow? I For laminar flow, assume Nq = 2000. Ny = dv/v, 2000 = (#)(v)/0.00015, v = 0.400 ft/s. ‘An oil with v = 0.005 ft/s flows through a 6-in-diameter pipe at 10 ft/sec. Is the flow laminar or turbulent? ' Nq = du/v = (4)(10)/0.005 = 1000 (laminar) Hydrogen at atmospheric pressure and 50°F has a kinematic viscosity of 0.0011 ft/s. Determine the maximum mass flow rate for laminar flow in a 3-in-diameter pipe. y = 0.00540 b/ft’. I For laminar flow, assume Ny =2000. Nq = dv/y, 2000 = (j3)(v)/0.0011, v = 8.80 ft/s; W = yAv = (0.00540)|(2)(2)*/4](8.80) = 0.001037 Ib/s. Air at 1500 kPa abs and 100°C flows in a 20-mm-diameter tube. What is the maximum laminar flow rate? A For laminar flow, assume Nx = 2000. Nx = pdv/u, p = p/RT = (1.5 x 10°)/{(287)273 + 100)] = 14.01 kg/m’, 2000 w= (14.01)(0.021)(v)/(2.17 10°), 1 =0.1589 m/s; Q = Au = [(x)(0.020)"/4](0.1549) = 0.000487 m’/s, oF 0.087 Lis. ‘What is the hydraulic radius of a rectangular air duct 8 in by 14 in? ' Ry= Alpe =[(GIV(8+8+ 14414) =2.55in or 2.554 ‘What is the percentage difference between the hydraulic radii of 30-cm-diameter circular and 30-cm square ducts? ' R=Alp, (Ruane = ((4)(30)/41/((2)(30)] = 7.50.6m (Re )agace = (30)(30)/(30 + 30+ 30+ 30) = 7.50.6, Since they are equal, the percentage difference is zero. Note that the hydraulic radius of a circular section is, ‘one-fourth its diameter. ‘Two pipes, one circular and one square, have the same cross-sectional area. Which has the larger hydraulic radius, and by what percentage? 202 930 931 932 9.33 934 925 0 CHAPTER 9 I Letd= diameter of the circular pipe and a = the side of the square one. Since they have the same cross-sectional area, xd°/4= a", a = Vi/2; (Rs)anaaw = d/4 = 0.2500d, (Ry aque = A/D = a°/4a = a/4, Since 4 = Vad!2, (Ry aye ™ (Wid /2)/4=0.2216d, hence, the circular pipe has the larger hydraulic radius by (0.2500 ~ 0,2216)/0:2216 = 0.128, or 12.8 percent. Steam of weight density 0.26 1b/f flows at 100 fps through a circular pipe. What isthe shearing stress atthe wal, ifthe friction factor i 0.0152 ! to (F/4)(y)(v/2g) = (0.015/4)(0.26)(100°/[(2)(32.2)}} = 0.151 to/te™ Giycerin at 68°F flows 120 through a 6-in-diameter new wrought iron pipe ata velocity of 10.0.t/s. Determine the head loss due to friction i hy =(f(LId)(v"/2g) Nu = pdu/w = (2.44)((4)(10.0)/(3.11 x 10°) = 392 Since Nn <2000, the Now it laminar and f = 64/N = = 0.1633, hy = 0.169120 (8)](10.0°7[@2)32.2)) = 19. ‘SAEI0 oil flows through a cast iron pipe at a velocity of 1.0 m/s. The pipe is 45.0m long and has a diameter of 150mm, Find the head loss due to friction. 1 hy = (AYLId\ v2) Na= pdv/u = (869)(H)(1-0)/0.0814 = 1601 Since Ne <2000, the flow is laminar and f = 64/), (0.0400(45.0/( (8) (1-0°/[(2)2.807)} = 0.612m. A 60-mm-diameter pipe (Fig. 9-7) contains glycerin at 20°C flowing at 8.5 m’/h. Verify that the flow is laminar. For the pressure measurements shown, isthe flow ascending or descending? What is the head loss for these pressures? ! v= Q/A = (8.5/3600)/[(1)(0.060)"/4] = 0.835 m/s ‘Ne = pdu/e = (1258)(0.060)(0.835)/1.49 = 63 (laminar) HGL=2+p/pg — HGL,=0-+ (2.0)(101 400)/{(1258)(9.807)] = 16.44 m HG, = 12 + (3.8)(101 400)/{(1258)(9.807)] = 43.23 m. Hence, the flow is from B to A (i.e, descending). Head loss = 43.23 — 16.44= 26.79 m. 38am Fig. 9-7 For the data of Prob. 9.33, compute the theoretical head loss if the pipe length is 30m between A and B. ‘Compare with the head loss corresponding to the measured pressures. BS _ (#)(1258)(9.807)(0.060)"(h,) 3600 (128)(.49)(30) ‘which is only 10m greater than the value found in Prob. 9.33. hy =26.89 m, ‘Two horizontal infinite plates keep a distance h apart as the upper plate moves at speed V, as in Fig. 9-8. There is a uid of constant viscosity and constant pressure between the plates. If V = S m/s and h = 20mm, compute the shear stress at the plates, given that the fuid is SAE 30 oil at 20°C. 938 939 340 FLOW IN CLOSED CONDUITS 1 203 Ng phV /u = (888)(0.020)(5)/0.440 = 202. Since the flow is laminar, au 05 where u = (V/K)(y), € = (u)(V/k) = 0.440{5/(2.0/100)] = 110 Pa, —? ye Ee os Find the head loss per unit length when a fui of s.g. 0.86 and kinematic viscosity 0.008 ft/s lows in a ‘bin-diameter pipe ata rate of 5 gpm. HQ =(5)(0.002228) = 0.01114 fe'/s (PML Ia)(07/28) A = 0.01114/{(22)(8)?/4] = 0.2268 ft/s Ng = du/v = (3)(0.2269)/0.008 = 7.09 ‘Since Nq<2000, the flow is laminar and f = 64/Ng = 64/7.09 = 9.03, a, = 9.03{1/(3)]{0.2269°/[(2)(32.2)}) = 0.0289 ft per foot of length. ‘Tests made on a certain 12-in-diameter pipe showed that, when V = 10 fps, f = 0.015. The fluid used was water at 60°F, Find the unit shear at the wall and at radii of 0, 0.2, 0.3, 0.5, and 0.75 times the pipe radius, ! To= (F/4)(YV*/2g) = (0.015/4)(62.4) {10° (2)(32.2)]} = 0.3634 Ib te? ‘The stress distribution is linear; hence, tr, ‘x Ib/t? 0 0 02 0.0727 03 0.1090 05 0.1817 07s 0.276 foil with a kinematic viscosity of 0.005 f?/s weighs 54Ib/ft, what wil be the flow rate and head loss in @ '3600-t length of 4-in-diameter pipe when the Reynolds number is 800? BH Ng=du/y — 800=(i)(v)/0.005 v= 12.00ft/s OQ = Av =[()(i4)"/4](12.00) = 1.047 ft/s = 64IN = 64/800 = 0.0800 fy = (F)(LId){v/2g) = 0.0800,3600/()}(12.007/[(2)(32.2)}) = 1932 How much power is lost per kilometer of length when a viscous fluid (1 = 0.20 Pas) lows in a 200-mm-diameter pipeline at 1.00 L/s? The fluid has a density of 840 kg/m” 1 v= QIA = (1.00 x 10)/[()(0.200)"/4] = 0.03183 m/s [Ny= pdv/ = (840)(0.200)(0.03183)/0.20= 26.74 y/L= (F)(1/d)(v?/28) Since Nx < 2000, the flow is laminar and f = 64/Ne = 64/26.74 = 2.393, hy/L= 2.393{1/(0-200)] (0.08183"/[(2)(9.807)]) = 0.0006180 m, P/L = Qyh;/L = Qnghy/L = (1.00 x 10"2)(840)(9.807)(0.0006180) = 0.00509 W/m = 5.09 W/km, Calculate the discharge of the sytem in Fig. 9-9, neglecting all losses except through the pipe. 002089 s/t yahd __(Ss)sQy/2p ‘32uL~ (32)(0.0002089)(16) Ne = pdv/ yu = (y/e)(d)(v)/u = (55/32.2)[(2)/12](4.017)/0.0002089 = 684 (laminar) Q = Av = ((x)[(4)/127/4} (4.017) = 0.00137 ft/s Assume laminar flow and use the conversion 1.0 centipoise 4.017 fe/s 204 0 CHAPTER 9 SSESTFest contipoise mg. 9.9 941 InFig. 9:10, H=25m, L= 40m, 0 = 37, d= 8mm, y= 1OKN/m’, and 4 =0.08 Pas, Find the head loss per unit length of pipe and the discharge in iters per minute. F Assuming laminar flow, yAhd? _ (105258 x 107° ‘Sauk (32)(0.08)(40) ‘yig\(d)(w)/u = [10*/9.807\(8 x 10-)(0.1563)/0.08 = 16 (laminar) Q = Av = [(a)(8 x 107)/4](0.1563) = 7.857 x 10-* m?/s = 0.471 L/min Ah/L = =0.625 m/m 0.1563 m/s 942 For the data of Prob. 9.41, find H if the velocity is 0.1 m/s. By proportion, H = (0.1/0.1563)(25) = 16.0 m. 9.43 Water flows at 0.20.m'/s through a 300-mm-diameter, 120-m-long pipe, under a pressure difference of 280 mmHg. Find the fiction factor. Hh, =(fy(L/a)(v"/24). From the pressure gradient, hy = (13.6/1)(0.280) = 3.808 m; v= Q/A = 0.20/{()(0.300)*/4} = 2.829 m/s, 3.808 = (f)[120/(0.300)]2.829"/[2(9.807)]}, f = 0.0233. 9.44 Use the Blasius equation for determination of friction factor to find the horsepower per mile required to pump 3.0 ft/s of liquid (v = 3.3 x 10~*ft'/s, y = 601b/f’) through an 18-in pipeline. i hy = (PMLIAv7/2g) f= 0.316/NE* v= Q/A = 3.0/{()(1)?/4] = 1.698 ft/s Ng =du/y = (18)(1.698)/(3.3x10-)= 78 f =0.316/718"= 0.03371 ‘hy = 0.03371[5280/(48)]{1.698°/[(2)(32.2)]} = 5.312 ft P= Qyh, = (3.0)(60)(5.312) =956.2ft-Ib/s per mile 956.2/550-= 1.74 per mile 945 Determine the head loss per kilometer required to maintain a velocity of 3 m/s in a 20-mm-diameter pipe, if va4x 10? m/s. ' hy = (NLIdv/2g)—Nq = dv = (0.020)3)/(4 x 10) = 1500 (laminar) {f= 641Nq = 64/1500 = 0.04267, = 0.04267{1000/(0.020)}{3"/[(2)(9.807)]} = 979.0 m per km 30 FLOW IN CLOSED CONDUITS 0 205 Fluid flows through a 10-mm-diameter tube at a Reynolds number of 1800. The head loss is 30 m in a 120-m length of tubing. Calculate the discharge in liters per minute. Fh, = )(L/d)(v'I2g). Since Nq <2000, flow is laminar and f = 64/Ny = 64/1800 = 0.03556, = 0.03556{120/(0.010)](v"/[2)(2.807))}, v= 1.74 m/s; O = Av = [(%)(0.010)"/4](1.178) = 92.21 x10" m/s = 5.53 L/min. Oil of absolute viscosity 0.00210 Ib s/f and specific gravity 0.850 flows through 10 000 ft of 12-in-diameter cast iron pipe at the rate of 1.57 fs. What i the lost headin the pipe? ' fy = LIA v*?2g) v= Q/A=1.57/{(H)GRY IA] = 1.999 ft/s [Na = pdu/ yu = [(0.850)(1.94)}(\(1.999)/0.00210= 1570 (laminar) = 64/Np = so = 0.04076 fh, = 0.04076{10 000/(1)]{1.999°/[(2)(32.2)}) = 25.3 ft ‘When first installed between two reservoirs, a 4-in-diameter metal pipe of length 6000 ft conveyed 0.20 ef of water. (a) If after 15 years a chemical deposit had reduced the effective diameter of the pipe to 3.0in, what then ‘would be the flow rate? Assume f remains constant. Assume no change in reservoir levels. (b) What would be the flow rate if in addition to the diamater change, had doubled in value? H (a) (f(Lalav3/2g) = (f.)(Lald:)(v3/24). Since f, L, and g are constant and v = Q/A = Q/(xd°/4), Qildi = O3/d;, 0.20714? = Q3/3.0°, z= 0.0974 cfs. (8) (GY Qi/di) = (FN O5/d). Since fr= 2f,, Qildi = (2)( O35), 0.20714 = (2)(03/3.0%), 2, = 0.0689 efs. A liquid with y = 58 1b/ft’ flows by gravity through a 1-ft tank and a 1-ft capillary tube ata rate of 0.15f07/h, as shown in Fig, 9-11, Sections 1 and 2 are at atmospheric pressure. Neglecting entrance effects, compte the viscosity of the liquid in slugs per foot-second. Te pur + vite + 2,= pil + vie + 25h, vs= QUA, (0.15/3600)/(22\(0.008)/4] =3.316 0/5 OF 0+ (+1) =043316/12)822]+0+h, hy = 1.8298 ‘Assuming laminar flow, 2W)E.316) (58)(0.004)" [Na = pdu/y = (r/g)(d)(v)/u = (58/32.2)(0.004)(3.316)/(1.600 x 10~ 1.829 = = 1.600 10" stugi/(ft -s) 493 (laminar) a LY Q= 015 0m Fig. 9-11 In Prob. 9.49, suppose the flow rate is unknown but the liquid viscosity is 2.1 x 10™ slug/(ft-). What willbe the flow rate in cubic fet per hour’ Is the flow still laminar? 1 pil + vil2g + 2,=paly + vi2g + 244 h, OFO+(141)=0+ VALI] +0+h, hy =2-0.015530" 206 0 CHAPTER 9 Assuming laminar flow, hy =32uLv/yd? 2 —-0.01553v? = (32)(2.1 x 10-*)(1)(0)/[(58)(0.004)] v°+46.63-1288=0 v=26164t/s Q = Av = |(-)(0.004)/4](2.616) = 0.00003287 ft/s = 0.118 fa Na = pdu/u = (vig (a)(v)/u = (58/32.2)(0.004)(2.616)/(2.1 x 10)=897 (laminar) 951 Inthe syringe of Fig, 9-12 the drug has p= 900 kg/m’ and y= 0.002 Pa.-s. What steady force F is required to produce a flow of 0.4 mL/s through the needle? Neglect head loss in the larger cylinder. i Pal ps + Va/28 +24 = Pal pg + Vnl2g + Zn + hy, Un = Q/Ay = 0.4 x 10°*/|(x)(0.25 x 10-"/4] = 8.149 m/s ‘Na = pdv/ = (900)(0.25 x 10"°)(8.149)/0.002= 917 (laminar) yp = Rubv _ (32) 0.002)(0.0209(8.149) ‘pad? (000)(9.807)(0.25 x 10") Pal{(900)(9-807)} + 0 + 0 pp/{[(900)(9.807)] + (8.149)*/[(2)(9.807)] +0 + 18.91 Pa~ Pa 196788 N/m? F=(Pa~Pa\(Arion) = =1891m 196 788{(-)(0.010)'/4] = 15.5 — Le ee 9.52 Paint issues from the tank in Fig. 9-13 at Q = 45 f€/h. Find the kinematic viscosity. Is the flow laminar? Fraley +5128 + 24= palo + Ual2g + 0th, Uy = QIAn= (shn)/{()(0.5/12)/4] = 9.167 tls 0+0+9=049.167°/[(2)(32.2)]+0+h, hy = 7.6958 ‘Assuming laminar flow, (128)(y)(6)(s5) @e@2r2,0.5/12 (0.5/12)(9.167)/0.0002444= 1563 (laminar) 7.695 = ‘y= 0.002444 ft/s Laon afin Fig. 9:13, 9.53 In Prob. 9.52, what will the flow rate be if the paint properties are p = 1.78 slugs/ft? and ys = 0.00217 Ib s/f” ' Pale + v3l2g-+ z4= alee + val2g + 29th, 0+049=0+07/((2)32.2)] +04 hy FLOW IN CLOSED CONDUITS 0 207 Assuming laminar fow, O4049m0FWMMOMR2I+0+41 vE426.60-5I26=0 v=2139HV6 Q = Av =[(2)(0.5/12)'/4)(2.133) = 0.002908 ft/s or 10.47 7h [Nj = pdv/u = (1.78)(0.5/12)(2.133)/0.00217 = 73 (laminar) 954 ‘The smaller tank in Fig. 9-14 is 50 m in diameter. Ifthe fluid is ethanol at 20°C, find the flow rate. 1 Daly + Vi/28 + 24™ Poly + vbl2g +25 +h, — O+04(0.4+0.6)=040404h, (128)(1.20 x 10-Y(0.8 + 0.40) y= 1.000m= 00 = CRO aUNODID) 1.000 = eet bo (a )(788)(9.807)(0.002)" Q=2107%10-m/s or 7.59L/h Mig. 9:14 955 For the system in Fig. 9-14, if the luid has density of 920 kg/m* and the flow rate is unknown, for what value of viscosity will the capillary Reynolds number exactly equal the critical value 2300? ee ee ! y= 1.000 = 2EEP— Grom Prob. 954) 1,000 = G2UJO8+0.4(u) —_0.0009398 (20y9.807)0.002)" |e Nq=pdu/ 2300 = (920)(0.002)(0.0009398/12)/n jx = 0.000867 Pas For the pressure measurements shown in Fig. 9-15, determine (a) whether the flow is up or down, and (b) the flow rate. Use p = 917 kg/m? and 4 = 0.290 Pa -s. HF HGL=2+p/oe @ HL, = 15 + (200)(1000)/1(917)(9.807)] = 37.24 m HGL, = 0 + (600)(1000)/[(917)(9.807)] = 66.72 m Since HGL,>HGLs, the flow is from A to B (i. up). (®) Assume flow is laminar. T2BWLD _ 66 79 3724029.48m 7 aa __(128)(0.290)25.00)(0) hy RR = O52 ~37.24 = 9.48 LeViFFRF=25.00m 29.48 = Ne Q=0.000727 m’/s or 2.617 m'/h v= Q/A =0.000727/{()(0.030)"/4] = 1.028 m/s Na pdv/u = (917)(0.030)(1.028)/0.290=98 (laminar) 208 0 CHAPTER 9 Pa =200KPa 4230mm Mig. 9.15 9.57 Repeat Prob. 9.56 ifthe pressures are the same but there is a pump between A and B which adds a 10-m head rise inthe flow direction. Is the flow still laminar? By =HGL 4 —HGL + hyp. Using values of HGL, and HGL, from Prob. 9.56, 128uL.Q hy = 6.72 ~ 37.24 + 10= 39.48 m= aed L=25.00m (from Prob. 9.56) 128)(0.290)(25.00)(0) ()(917)(9.807)(0.030)" v= Q/A = 0,000974/{(2)(0.030)'/4] = 1.378 m/s 'Nq = pv] = (917)(0.030)(1.378)/0.290= 131 (laminar) = 0.000974 m"/s or 3.51m"/h 9.88 Water at 40°C flows from tank A to tank B as shown in Fig. 9-16, Find the volumetric flow, neglecting entrance losses to the capillary tube as well as exit losses. T paly + vileg + z4=Poly + ¥¥/2R+ 20th, 0+04(0.2240.1)=0+0404h, hy =0.32 ‘Assume laminar flow. fy = P2BHLQ 4 3p _ (128N651 x 10-9022 + 0.080) ped ()(992)(9.807)(0.001)* v= Q/A=3.912 x 10-7/{(4)(0.001)?/4] = 0.4981 m/s ‘Na = pau/ = (992)(0.001)(0.4981)/(6.51 x 10~*) = 759 (laminar) Q= 3.912 107 m'/s=1.41L/h TT. Fig. 9:16 9.59 In Prob. 9.58, what should the internal diameter of the tube be to permit a flow of 2.16 L/h? Assuming laminar flow, Q « d* or d= Q™; hence a= 288)"m) = 15340 ‘Computation of Ng shows the flow to be indeed laminar. 3.60 961 saa 968 364 FLOW IN CLOSED CONDUITS 9 209 ‘A hypodermic needle has an inside diameter of 0.3 mm and is 60 mm in length, as shown in Fig. 9-17. Ifthe piston moves to the right at a speed of 18 mm/s and there is no leakage, what force Fis needed on the piston? ‘The medicine in the hypodermic has a viscosity of 0.980 x 10~* Pas and its density p is 800 kg/m’. Consider flows in both needle and cylinder. Neglect exit losses from the needle as well as losses at the juncture of the needle and cylinder. Ht For cylinder: Q = Av = [()(0.005)"/4](0.018) = 3.534 x 10-7 m/s —_ Q/A =3.534% 1077/()(0.3/1000)7/4]= 5.000 m/s. 800)(0.3/1000)(S.000)/(0.980 x 10-9)= 1224 (laminar) 1 (128)(0.980 x 10-%)(0.060)(3.534 x 10-7) _ . a 03/1000)" wresasP (AP )(A pin) = (104 525 — 1.129)|(1)(0.005)°/4] = 2.05N Fig. 9:17 In Prob. 9.60, suppose that medicine is drawn from a bottle at atmospheric pressure. What is the largest flow of fluid if the fluid has a vapor pressure of 4.8 kPa abs? Neglect losses in the cylinder. 128)(0.980 x 10-*)(0.060)(Qrnus) (@G x10 ' 4p =101 400— 4800 = 96600 Pa = 78-296 600 = Qaue= 3.27 X10? m/s = 0.327 mL/s In Prob. 9.60, i ook a force of 2.05 N to move the piston to the right ata speed of 18 mm/s. What should the inside diameter be forthe cylinder ifthe force needed is only 1.2N forthe same piston speed? Neglect losses in oxlinder. Dn =61116Pa Fl 12 TPBMLQ gy 41g. (128)( 0.980 x 1079(0.0609(3.534 x 107) A” GN0.005)74 dt = xd 4=0,343mm N= pd/jt = (800)(0.343/1000)(0.018) /(0.980 x 10 =5 (laminar) Water at 70°F flows through a new cast iron pipe at a velocity of 9.7 ft/s. The pipe is 1200 ft long and has a diameter of 6 in. Find the head loss due to friction. ' y= (PULIAKv128) Nu = du/v = (8)(9.T)/(1.05 x 10-4) = 461.905 From Table A-9, ¢ ~ 0.00085 ft for new cast iron pipe; «/d = 0.00085/() = 0.0017. From Fig. A-5, f= 0.0230; n= 0.0230(1200/()}(9.77/[(2)32.2)]} = 80.6 ft. ‘A.96-in-diameter new cast iron pipe carries water at 60°F. The head loss due to friction is 1.5 ft per 1000 ft of pipe. What isthe discharge capacity of the pipe? ' fy = (GLIA yv7l2g) 1.5 = (F)[LOO0/C)](w7/[(2)32.2))}—_ fu? = 0.7728 ‘Assume f = 0.0150; (0.0150)(v2) = 0.7728, w = 7.178 ft/s; Ny = du/v = ($9(7.178)/(1.21 % 10-*) = 4.75 > 10" 210 0 CHAPTER 9 9.66 From Table A-9, ¢ = 0.00085 ft for new east iron pipe, ¢/d = 0,00085/($) = 0.000106. From Fig. A-5, {f=0.0124. Evidently, the assumed value of fof 0.0150 was not the correct one. Try a value of fof 0.0124. (0.0124)(v") = 0.7728, v = 7.894 ft/s; Ne = (%2)(7.894)/(1.21 x 10°*) = 5.22 x 10°. From Fig. A-5, f = 0.0124. Hence, 0.0124 must be the correct value of f, and u = 7.894 ft/s. Q = Av = [()(#)"/4](7.894) = 397 ft/s. ‘Water at 70°F is being drained from an open tank through a 24-in-diameter, 130-ft-long new cast iron pipe, as shown in Fig. 9-18. Find the flow rate at which water is being discharged from the pipe. Neglect minor losses. ' Pil + Vil2g + 21= poly + vi/2g + 22+ hy, a= hy = (PML /A) 0" 2g) = (1130/4 WHA)G22 040+ 150.5=0+ vi/[(2)G2.2)] + 984+ 1.009f04 .009f03 Assume f = 0.0240. 150.5 = vi/{(2)(32.2)] + 98.4 + (1.009)(0.0240)(02) vs = 36.21 ft/s Ng = du/v = (8)(36.21)/(1.05 x 10) = 6.90 x 10° From Table A-9, € = 0.00085, ¢/d =0.00085/(3) = 0.000825, From Fig. A-S, f= 0.0162. Evidently, the assumed value of fof 0.0240 was not the correct one. Try a valve of fof 0.0162. 150.5 = v3/[(2)(32.2)] + 98.4 + (1.009)(0.0162)(u%) vs = 40.43 ft/s ‘Na = (H)(40.43)/(1.05 x 10-4) = 7.70 x 10" From Fig. A-5, f = 0.0162. Hence, 0.0162 must be the correct value of f, and v = 40.43 ft/s. Q = Av {G3 /41(40.43) = 127 7. Blew. 1908 Q@ Bev nate Mig. 9:18 Gasoline is being discharged from a pipe, a shown in Fig. $-19. The pipe roughness (¢) is 0.500 mm, and the pressure at point 1 is 2500 kPa. Find the pipe diameter needed to discharge gasoline at arate of 0.10m*/s. Neglect any minor losses. W ply + ug +2 =paly + vi/28 + 2+ he hy = hy = PLIA(0? 12g) = (F)(965.5/d){v3[2V9.807)]) = 49.23f0d 2.500/7.05 + vi/2g + 82.65=0+ vi/2g +66.66+49.25f03/d — vi/2g=vi/2g foi/d = 0.3320 v, = Q/A,=0.10/(nd?/4) =0.1273/d? (fF )(0.1273/d")*/d = 0.3320 d = (0.04881f)"* *)= 1.25 x 10. From Table A-9, € = 0.00050 m. €/d = 0.00080/0.2500 = 0.0020. From Fig. A-5, f = 0.0235. Evidently, the assumed value of fof 0.0200 was not the correct one. Try @ value of fof 0.0235. {(0.04881)(0.0235)}!* =0.2582m v= 0.1273/0.2582* = 1.909 m/s ‘Na = (719)(0.2582)(1.909)/(2.92 x 10-4) = 1.21 x 10° €/d = 0,00050/0.2582= 0.00194 f =0.0235 Hence, 0.0235 must be the correct value of f, and d = 0.2582m. 96 9.68 98 om FLOW IN CLOSED CONDUITS J 211 Ele. 265 on Fig. 9:19 Water at 20°C flows through a new cast iron pipe at a velocity of 4.2.m/s. The pipe is 400m long and has a diameter of 150 mm. Determine the head loss due to friction. ! fy = (SUL Id)(v7/2g) Nu = dv /v = (48%)(4.2)/(1.02 x 10-*) = 6.18 10° From Table A-9, € = 0.00026 m. ¢/d = 0.00026/0.150 = 0.00173. From Fig, A-5, f =0.0226. hy = (0.0226,400/(35)1{4.27/{(2\(9.807)]} = 54.20 m. ‘SAE10 oil at 68°F isto be pumped at a flow rate of 2.0 /s through a level -in-diameter new wrought iron pipe. Determine the pressure los in pounds per square inch per mile of pipe and compute the horsepower lost to friction. ' fy = (fYLIdv7l2g)— v= QIA = 20/I(#)()"/4] = 10.19 fe/s [Na = pdu/u = (1.68)(8)(10.19)/(1.70 x 10") = 5035 (turbulent) From Table A-9, € = 0.00015 ft. ¢/d =0.00015/(t (0.038[5280/ (f)]{10.19"/{(2)(32.2)]) = 647 ft of oi 0.00030. From Fig. A-5, f = 0.038. hy = ‘= (64.2)(647)/144 = 244 psi/mile. ‘Water at 20°C flows in a 100-mm-diameter new cast iron pipe with a velocity of 5.0 m/s. Determine the pressure drop in kilopascals per 100 m of pipe and the power lost to friction. ! hy =(VLIA)(U"/28) Na = dv/v = (185)(5.0)/(1.02 x 10-*) = 4.90 x 10° From Table A-9, € = 0.00026 m. €/d = 0.00026/({'8) = 0.0026. From Fig. A-5,f = 0.0252. ‘hy = 0.0252{100/(#88)]{5.07/(2)(9.807)}} = 32.12m p= (9.79)(32.12) = 314 KN/m* per 100 m of pipe Q = Av = (8/4 6.0) = 0.03927 m’/s Power lost= Oh, = (0,03927)(9.79)32.12) = 12.35 KW per 100m of pipe Determine the discharge capacity of a 150-mm-diameter new wrought iron pipe to carry water at 20°C if the pressure loss due to friction may not exceed 35 kPa per 100 m of level pipe. ' [Ny = du/y = (338)(U)/(1.02X 10-9) = 1.47 x 10% ‘Trial No. 1 Assume v = 3.0 m/s: Nx = (1.47 x 10°)(3.0) = 4.41 x 10°, €/d = 0.000046/(38) = 0.000307. From Fig. A-5, J .OUSh y= NULLA") = py = 319.79 = 3.515m, 3.575 = 0.06400 / HB) o"N2V.8TT), 2.53mi/s. ‘Trial No. 2 ‘Assume v= 2.53 m/s: Nq = (1.47 x 10°)(2.53) = 3.72 x 10, f = 0.0166, 3.575 = 0.0166[100/ 8) {u*/[(2)(9.807)]}, wv = 2.52 m/s; Q = Av = [(x)(Hi)/4](2.52) = 0.0445 m’/s. SAE30 oil at 68°F isto be pumped ata flow rate of 3.0/5 through level new cast iron pipe. Allowable pipe fiction losis 10 psi per 1000 ft of pipe. What size commercial pipe should be used? ' = yh (10144) =55.4h, hy =26.0ft of oll per 1000 of pipe 212 0 CHAPTER 9 on 973 94 ‘Trial No. 1 Assume v =5.0ft/s: Q=Aly 3.0=(nd'/45.0) d=0.8748 Na = pat /u = (1.72)(0.874)(5.0)/(9.2 x 10"*)= 817 (laminas) hy = (LIA )(0"/28) f= 64/Nq = y= 0.0783 26.0 = (0.0783)(1000/0.874)(v°/[2\32.2) v= 4.32ft/s, ‘Trial No. 2 Assume v= 4.32 ft/s: 3.0= (2ed?/4)(4.32) d= 0,940 Nq = (1:72)(0.940)(4.32)/(9.2 107%) = 759 (laminar) f= = 0.0843 26,0 = (0.0843)(1000/0.940){v7/12)32.2)]} v= 4.325 Hence, a pipe diameter of 0.940 ft, or 11.28in, would be required. A 12-in-diameter commercial pipe should be used, which would result in a pipe friction loss somewhat less than the allowable 10 psi per 1000 ft of pipe. ‘SAE10 oil at 20°C isto flow through a 300-m level concrete pipe. What size pipe will carry 0.0142.m'/s witha pressure drop due to friction of 23.94 kPa? ' pHvh 2394=8.52h, hy =281m Trial No. 1 ‘Assume v= 1.5 m/s Q=Alv 0.0142 =(nd?/4)(1.5) d= 0.110m Ne = pdv/u = (869)(0.110)(1.5)/(8.14 x 10-7)= 1761 (laminar) fy = (P\LIA\(v" 128) f = 64a = es 2.81 = (0.0363)(300/0.110){v/(2(9.807))) ‘Trial No. 2 ‘Assume v = 0.746 m/s: (xd?/4)(0.746) d= 0.156 m_ Ny = (869)(0.156)(0.746)/(8.14/10") = 1242 (laminar) JH =0.0515 2.81 = (0.0515)(300/0.156)(07712).807)]} v= 0.746 m/s Hence, a pipe diameter of 0.156 m, or 156mm, would be required. ‘Compute the friction factor for flow having a Reynolds number of 5 x 10° and relative roughness (¢/d) of 0.08 ((ransition zone). Use the Colebrook formula, the Swamee-Tain formula, and the Moody diagram. 1 Colebrook formula: Vf 1.14 -2.010g [e/d +9.35/(N,VF)] = 1.14 2.01og 0.015 + 9.35/65 x 10°VF)] J =0.0515 (by tial and error) ‘Swamee—Jain formula: f =0.25/{log (€/3.7d) + (S-ATING)F =0.25/ log (0.015/3.7) + [5.47/(5 x 10°] = 0.0438 Moody diagram (Fig. A-5): fo0si2 Repeat Prob. 9.73 for flow having a Reynolds number of 4 x 10° and relative roughness (¢/d) of 0.0001 (rough-pipe zone). I Colebrook formule: f= 1/[l.14—2.0 og (¢/d)} = 1/(1.14 - 2.0 log (0.00017 = 0.0120 Swamee-Jain formula: {f =0.25/llog (€/3.7d) + (5.74/N3°)} = 0.25} log (0.0001/3.1) + [5.74/(4 x 10°°°)}? = 0.0120 Moody diagram (Fig. A-5): f=0.0125 on FLOW IN CLOSED CONDUITS 1 213 ‘We have oil of kinematic viscosity 8 x 10-*t'/s going through an 80-t horizontal pipe. If the initial pressure is, 5.0psig and the final pressure is 3.5 psig, compute the mass flow if the pipe has a diameter of 3in. At a point 10ft from the end ofthe pipe a vertical tube is attached to be flush with the inside radius ofthe pipe. How high will the oil rise inthe tube? p = 50 bm/ft. Pipe is commercial steel (€ = 0.000145 ft). ' Duly +vil2g + 2,= aly + 032g + 25+ hy (5.0)(144)/(50/32.2) + ui/2g +0 = (3.5)(144)/(50/32.2) + vi/2g + 0+ hy Ui/2g = vi/2g fay = 139.1 = (f)(Lidv"/2)_ 139.1 = (f)[80/(4)v*/2) ‘Try f = 0.020: 0.0200? = 0.8694, v = 6.593 ft/s; Na = du/v = ((4)(6.593)/(8 x 10°) = (.000145/(:) = 0.000580. From Fig. A-5, f= 0.0265. Try f = 0.0265: 0.02650" Na = (8Y5.728)/(8 X 10°*) = 1.79 x 1 0267, 0.02670" = 0.8694, v = 5.706 ft/s; M = pA 50{()(3)*/4)5.706) = 14.0 lbm/s. To find the pressure at the point 10 ft from the end of the pipe (call it point ‘A), apply the Bernoulli equation between point 1 and point A: (65.0)(144)/(50/32.2) + v2/2g + 0 = pl(S0/32.2) + v3/2g + 0+ hy hy = 463.7 ~ 0.6440p = 0.0267(70/(3)|(5.706"/2) = 121.7 ft 121.7=463.7-0.6440p. pa =S31-LIbm/f® = p/p = 531.1/50= 10.628 How much water is flowing through the pipe shown in Fig. 9-20? Take v = 0.114 x 10-* m'/s and. ‘¢ =0,0000442 m. ! pily+vil2g-+2,—paly +02 2g + 2th, 1.64 Ui/2g+0=0.34 v3/2g 404K, veg = vig hy = 13m= (AYLI) v*/28) Tryf=0.015: 1.3=0.015(10/(0.150)](v?/[2X9.807)}}_ v= 5.050.m/s Ng = du/y = (0.150)(5.050)/(0.114 x 10-4) = 6.64 10 _¢/d =0.0000442/(0.150) = 0.000295 From Fig. A-S,f = 0.016. 13=0.016(10/(0.15){v°/2)9.80)]} v= 4.889 m/s -M = pAv = 1000{()(0.150)/4](4.889) = 86.4 kg/s a) 10 m- Fig. 9-20 Whiskey (s.g. = 0.6, v= 5.0 x 10-7 m’/s) is drawn from a tank through a hose of inside diameter 25 mm (see Fig. 9-21). The relative roughness forthe hose is 0.0004. Calculate the volumetric flow and the minimum pressure in the hose. The total length of hose is 9m and the length of hose to point A is 3.25 m. Neglect minor losses at head entrance. Fo ply + vilagtn=pily +uitgt nth, 040+ (5-15)=0+ 03/[2)9.807] +0+h, hy =3.5~ 0.0508803= (\LId)(v?/28) 214 0 CHAPTER 9 Try f =0.016: hy =0.016(9/(0.025)]{v4/[Q)(2.807)]} = 0.293702 0.293703=3.5— 0.050980} v =3.187 m/s [Ng = du/v = (0.025)(3.187)/(5.0 % 1077) = 1.59 x 10° From Fig. A-S with ¢/d = 0.0004, f = 0.019. Try f =0.019: fy, = 0.019(9/(0.025)](v/(29.807)]} = 0.348703 0.3487vI =3.5-0.0509803 v= 2.959 m/s Na = du/v = (0.025)(2.959)/(5.0x 10") = 1.4810" f = 0.019 Q = Av =[()(0.025/5]2.959) = 1.45L/s— puly + vig += paly + URI2E + Eathe 0404 (51.5) =pall(0.6)9.79)]+2.95F/[2\9.807)]+ 5+, pa = 11.43 ‘hy =0.019{3.25/(0.025)}2.9597/1(2)(9.807)]} =1.103m p= ~11.43 ~ (5.874)(1.103) Fig. 9:21 9.78 musing the Darcy-Weisbach equation for flow in a pressure conduit, what percentage error is introduced in Q ‘when fis misjudged by 20 percent? ' hy = (fM(LId)(v?/2g) = KifQ? (where K, is a constant) Assume hy is constant. Q = K,/Vf= K.f-"*, dQ = -M(K.)(f~*\df) dQ_ WK IE“A) _ df. 2 Kf 9.79 For the system in Fig. 9-13, find the flow rate ifthe liquid is water at 68°F. I Assume smooth-wall turbulent flow. pa/y + u3/2g + z= oly + V5/2g+ zx +h, 0+0+10= 0+ vb/{(2)(32.2)] + 0+ hy, hy = 10 ~ 0.0155305 = (f(L/d)(v"/2g). Try f = 0.02: ‘i, = 0.026 /(0.5/12)}(v3/[(2)(32.2)]} = 0.044720 10-0.01553v5= 0.044720 vp = 12.88 4/5 [Ng = pdv/ y= (1.930.5/12)(12.88)/(2.04 x 10-4) = 5.08 x 10" (turbulent) From Fig. AS, f = 0.0208. Try f = 0.0208: ny = 0.0208(6/(0.5/12)}(¥/{2)(32.2))} = 0.046513, 10-0.01553v} = 0.04651v5 vg = 12.708t/s [Ny = pdv/u = (1.93)(0.5/12)(12.70)/(2.04 x 10-*)= 5.01 x 10" (turbulent) (0.20) = -0.10 or 10 percent f= 0.0208 Q=Av=[(x)(0.5/12)1/4](12.70) = 0.0173 #P/s 9.80 If 1 mile of }in-diameter wrought iron pipe carries water at 68°F and v = 23{t/s, compute the head loss and te pressure drop fi hig = SMLIA)(07128) Na = par/u = (1.93)(3)(710.3048)/ (2.04 10-*) = 5.43 > 10° €/d = 0.00015 /(2) = 0.000600 sat oat FLOW IN CLOSED CONDUITS J 215 From Fig. A-5, f = 0.0182. ty, = 0.0182{5280/(i)}{(23)*/[2)(32.2)]} = 3157 p= yhy = (62.4)(3157)/144= 1368 tb/in? Mercury at 20°C flows through 3 m of 6-mm-diameter glass tubing with average velocity 2.0 m/s. Compute the head loss and the pressure drop. t Ing = (FMLIA)(0"/2R) Ny = pdu/p = (13 570)(0.006)(2.0)/(1.56 x 10-%) = 1.04 x 10° From Fig. A-5, f = 0.0180 (assuming glass to be “smooth”). ‘hy = 0.01803 /(0.006)]{2.07/[(2)(9.807)]} = 1.835 mp = yh, =[(13.6)(9.79)|(1.835) = 244 kPa Gasoline at 20°C is pumped at 0.2 m’/s through 16 km of 180-mm-diameter cast iron pipe. Compute the power required if the pumps are 5 percent eficient U hy = (ULI (W128) v= Q/A = 0.2/{(4)(0.180)/4] = 7.860 m/s Ng = pdu/ = (719)(0. 180)(7.860)/ (2.92 x 10°*) = 3.48 x 10° €/d = 0,00026/(0.180) = 0.00144 From Fig. A-5, f = 0.0216. +h, = 0.02164(16)(1000)/(0.180)](7.860"/[(2\(9.807]} = 6048 m P= pgQhy/n = (719)(9.807)(0.2)(6048)/0.75 = 11.37 x 10°W or 11.37MW ‘Vinegar (s.g. = 0.86, v = 0.00003 f'/s) flows at 1 ft/s through a 6-in asphalted cast iron pipe. The pipe is 2000 ft Jong and slopes upward at 10" in the flow direction. Compute the head loss and the pressure change. U (AULIA(W7I28) v= OLA = (a )( 1°14] = 5.098 fs Ne = dv/v = ({5)(5.093)/0.00003 = 8.49 x 10* <«/d = 0.0004/() = 0.000800 From Fig. A-5, f = 0.0219. sy, = 0.0219{2000/(()](5.093°/(2)(32.2))} = 35.28 ft, = 35.28 + 2000 sin 10° = 382.6 ft P= hy = [(0.86)(62.4)](382.6)/144 = 142.6 fin? ‘The pipe flow in Fig, 9-22 is driven by pressurized air in the tank. What gage pressure p is needed to provide a flow rate of 50 m'/h of water? Assume a “smooth” pipe. De pily + vil2g + n= pay + uil2g + 2:4 he v2 Q/Az= (50/3600) /[()(0.050)*/4] = 7.074 m/s he=hy = (f\LIa)(v'/28)——_Ne= pu /u = (998)(0.050)(7.074)/(1.02 x 10) = 3.46 x 10° From Fig. A'S, f = 0.0140 +h, = 0.0140[(40 + 80 + 20)/0.050}{7.0747/[(2)(9.807)]) = 100.0 m 1/9.79-+0 + 10= 0+ 7.0747/((2)(9.807)] + 80+ 100.0 p, = 1689 kPa gage Sco ipe ey d= 50mm: ae Open jet 80m | an-| neo 216 0 CHAPTER 9 985 9.86 937 In Fig. 9-22 suppose the fluid is methanol at 20°C and p, = 900 kPa gage. What flow rate Q results? H pily + vil2g + 21=paly + vil2g + 24h, 900/7.73-+ 0-4 10= 0+ v3/{(2)(9.807)] +80 +h, i, = 46.43 — 0.050880} = hy = (f\(L/d)(v"/2g) ‘Try f = 0.02: ‘h, = 0.02{(40 + 80 + 20)/0.050}{vi/[(2)(9.807)]} =2.855u3 2.855u3=46.43-0.05098u3 — v, = 4.000m/s ‘Na = pau = (788)(0.050)(4,000)/(5.98 x 10-4) =2.64 x 10° From Fig. A-5, f = 0.0150. Try f = 0.0150: ‘z= 0.0150{(40 + 80 + 20)/0.050}{v3/[(2)(9.807)]} =2.141v} 2.141v3=46.43-0.0500803 v= 4.602 m/s ‘Na = (788)(0.050)(4.602)/(5.98 x 10°*) = 3.03 x 10° Try f =0.0145; +h, =0.0145{(40-+ 80+ 20)/0.050}(03/[(2)(9.807)}} =2.070v3 2.070u}=46.43—0,0509803 [Ne = (788)(0.050)(4.679)/(5.98 x 10) =3.08 x 10? f = 0.0145 [At this Reynolds number, Q = Av = [(}(0.050)*/4)4.679) = 0.00919 m/s or 33.1 m'/. In Fig. 9-22 suppose the fluid is carbon tetrachloride at 20°C and p, = 1300 kPa. Calculate the pipe diameter needed for a volumetric flow of 5.555 L/s. Bo pil vile = ply vig + eth, ply +04 2,=0+ vig +2 + (NL/AvI/28) pp CaM pul + 212) _ 2)(9.807)(1300/15.57-+ 10-80) _ 264.7 BO T+flld 1+ (G)40+ 8042/4 T+ Opa = (QIA,) = [6.555 x 10-)/(d"/4)F = 0.00005004/d" Y= 0.007074/? 0005004 264 = (1+ 140 ay” i+ i40f/d a7. ‘Try d = 50mm, or 0.050 m: Na = pdu/ = (1588)(0.050)(0,007074/0.050")/(9.67 x 10-*) = 2.32 x 10%. From Fig. A-S, f = 0.0151. d = [1 + (140)(0.0151/0.050)]!/47.96 = 0.0535 m. Try d = 0.0535 m: ‘Na = (1588)(0.0535)(0.007074/0.0535°)/(9.67 x 10°) =2.17 x10" f =0,0155 = [1+ 140)(0.0155/0,0535)]""/47.96 = 0.0529 m ‘Try d=0.0529m: ‘Ny = (1588)(0.0529)(0.007074/0.0529°)/(9.67 x 10°) =2.20% 10° f = 0.0155 d= {1 + (140)(0.0155/0.0529)}"/47.96 = 0.0531 m Try d= 0.0531 m: ‘Ng = (1588)(0.0531)(0.007074/0.0531°)/(9.67 x 10-*)=2.19 x 10" f= 0.0155 d= [1 + (140)(0.0155/0.0531)]"/47.96 = 0.05301 m Hence, use d = 0.053 m, or 53 mm. a ‘The reservoirs in Fig. 9-23 contain water at 20°C. Ifthe pipe is smooth, with L=7km and d= 50mm, what will the flow rate be for Az =98 m? T pily+ vite + 2=pily tilt t nth, O+0+100—0+040+h, hy = 98m=(FY(L/d)(v7/24) 98 = (/)7000/0.050}(v7/[(2)(9.807)]} v=0.1172/VF ‘Try f = 0.02: v= 0.1172/VO.02 = 0.8287 m/s, Ne = pau = (998)(0.050)(0.8287)/(1.02 x 10-*) = 4.05 x 10% From Fig. A-5, f = 0.022. Try f = 0.022: v= 0.1172/VOG2 = 0.7902 m/s Nq = (998)(0.050)(0.7902)/(1.02 x 10°) = 3.87 > 10" f=002 (OK) = Av = {(x)(0.050)?/4](0.7902) = 0.00155 m’/s or 5.58 m°/h 390 sot Fig. 9:23, Repeat Prob. 9.87 to find Q if L= 2500 ft, d = 3in, and Az = 82. 4H pily + vite +2.=paly + vig + nth, OF0+82=04040+h, hy = 82ft= (F(LIA\(v#/24) = (/PSOO/EIV*A)G22)) v= 0.7267/VF Try f= 0.02: 9=0.7267/VO.03=5.13948/s Ny = pdv/u = (1.93)(8)(5.139)/(2.04 x 10-4) = 1.22 10° ASS, f = 0.0175. Try f = 0.0175: v =0.7267/VO.01TS =5.493 ft/s Nx Tryf =0.0170: v= 0.7267/VO.0TT0 = 5.574 ft/s Nq= (1.93)(8)(5.574)/(2.08 x 10-*) = 1.32 x 10" From (1.93)()(5.493)/(2.08 x 10° 30x10" f =0.0170 F= 0.0170 (OK) Q=Av=[(x)(8)'/415.574) =0.27368/s or 985th Repeat Prob. 9.88 ifthe pipe has a roughness of 0.2mm. From Prob. 9.88, » = 0.7257/VF. Try f = 0.02: Np=1.22%10 fd =0.2/(3)(25.4)] = 0.00282 From Fig. A-S, f = 0.0265. Try, v= 0.7267/ VO.0265 = 4.464 ft/s Nq = (1.93)(()(4.468)/(2.04 x 10-4) = 1.06 x 10* F00265 (OK) — Q=Av=[(n\4)"/4\(4.464)=0.2191 27s or 789 f'/h ‘This is (985 ~ 789)/985 = 0.199, or 19.9 percent less than when the pipe is smooth. ‘Water at 20°C flows through a 598-m pipe 150 mm in diameter at 60 L/s. Determine the pipe roughness if the head loss is 49 m. ' f= LIA ye*/28) Q/A = 0.06/{(%)(0.150)"/4] = 3.395 m/s 49 = (f)[598/(0.150)}{3.3957/[(2)(9.807)]}_f = 0.0209 [Na = pd /u = (998)(0.150)(3.395)/(1.02 x 1079) = 4.98 x 10" From Fig. A-S with f = 0.0209 and Nq = 4.98 x 10°, €/d = 0.0012; € = (150)(0.0012) = 0.180 mm. ‘A 4in-diameter commercial steel pipe is to be sloped so that 198 gpm of water at 20°C passes through it in sxavity flow. Find the declination @ of the pipe. 1 Q=(198)(0.002228) = 0.4411 f/s v= Q/A = 0.4411/[(")(4)"/4] = 5.055 ft/s [d= 0.00015 /(i4) = 0.00045 Nq = dv/v = (44)(5.055)/(1.08 x 10%) = 1.60 x 10° 218 0 CHAPTER 9 From Fig. A- hyIL = (f(A/d)(v?/2g), sin 6 = 0.0190f1/($)]{5.055°/[(2)(32.2)]}, On 130, 9.92 In Prob. 9.91 find the volume flow corresponding to @= 3°, From Prob. 9.91, (f)(1/d)(v*/2g) = sin 8, (FY[1/()M¥*M[2)(32.2)]} = sin 3°, v = 1.060/ VF. Try f = 0.02: v= 1.060/VO02 = 7.495 ft/s (Ng = du/'v = (i)(7.495)/(1.05 x 10°*) = 2.38 x 10° 10", €/d = 10,0003 /0.8434 = 0.000354. From Fig. A-S,f = 0.0177; hy = (0.0177)(100/0.7500){44.44/[(2\32.2)]} = 72.37, ‘Ap = pay = (0.00237)(2.2(72.37) = 5.52 b/f. Find the head loss in 1 ft of 6-in-diameter pipe (¢ = 0.042:in) when turpentine (s.g. = 0.86) having a viscosity of 0.0008 Ib «s/f? flows ata rate of 16 cfs. Also find the shear stress at the wall of the pipe. 1! hy = LIA 2g) v= Q/A = 16/{(a)(8)'/4] = 81.49 ft/s Ng = pdu/ = {(0.86)(1.94)|(8)(81.49)/0.0008 = 8.50 x 10 ¢/d = 0.042/6 = 0.00700 ony 91m san 9am 93 FLOW IN CLOSED CONDUITS J 225 From Fig. A-S, f =0.0328. hy, = 0.032441) ENLAF Q)32.2}T} = 6.68. to fpv"/8 = 0.0324{(0.86)(1.94) (81.49)*/8 = 44.9 1b/f* ‘When water at 150°F flows in a 0.7S-in-diameter copper tube at 1.0 gpm, find the head loss per 1000. Also find the centerline velocity. BQ =(1.0)(0.002228) =0,002228 8/5 v= Q/A = 0.002228,/{()(0.75/12)"/4] = 0.7282 A/s hy = (PULIA)(U7I2g) Nu = dv = (0.75/12)(0,7262)/(4.68 x 10) = 9.70 10° ‘eld = 0,000005/(0.75/12) = 0.0000800 From Fig. A'S, f= 0.030. (0.030){1000/(0.75/12)]{0.7262/(2)32.2)1) = (u)(0 + 1.339") = 0.7262{1 + (1.33)(0.030)""] = 0.893 fs 8, = 32.BvI (uf) = B2.8)(4.68 x 10-°)/{(0.7262)(0.030)'7} = 0.00122 ft Refer to Prob. 9.119, How thick is the viscous boundary layer for a flow of 20 gpm? I The Reynolds number is now (20/1.0)(9.70 x 10°) = 1.94 x 10°, From Fig. A-S, f = 0.016. Then, 8, = 32.8v1 (Uf) = (32.8)(4.68 x 10°*/{(20)(0.7262}(0.016)""| = 8.36 x 10° ft Find the maximum velocity in a 100-mm-diameter pipe (€= 0.00085 m) when ol (6g. = 0.84), of viscosity 0.0052 Pa -s flows at a rate of 40 L/s. * 2 U v= Q/A = (40x 10-)/[()(0.100)*/4] = 5.093 m/s [Na = pv = {(0.84)(1000)(0.100)(5.093)/0.0052 = 8.23 x 10" €/d =0,00085/(0.100) = 0.0085 From Fig. A-5, =0.0365. Hence, aac = (W)(U-+ 1.339") = 5.09311 + (1.33)(0.0365)""] = 6.387 m/s ‘The head loss in 240 fof 6-in-diameter pipe is known to be 30 ft when oil (s.g. = 0.90) flows at 2.0/s. Determine the shear stress at the wall of the pipe. ' v= Q/A =2.0/{(a)(8)'/4] = 10.19 ft/s Nq= pdv/4 = [(0.90)(1.94)1(8)(10.19)/0.0008 = 1.11 x 10" (turbulent) hy = (LIA y(u"/2g)_ 30= (/)1240/(4)}(10.19°/@2)(32.2)]) f= 0.0388 T= fov?/8 = 0,0388{(0.90)(1.94)](10.19)*/8 = 0.879 1b/te If, ina I-m-diameter pipe, velocities are 5.03.m/s on the centerline and 4.85 m/s at r = 100 mm, what is the flow rate? ! = thar ~ (5:75)( Fol 9)" log [rd (to— 1) 4.85 = 5.03~ (8.75)(to/p)"* bog [500/500 — 100)]_ (ro/p)"*=0.3230 V = oan ~ (3)(2.5V0)p) = 5.03 — (3)(2.5)(0.3230) = 3.819 m/s Q = Av ={(x)(1)'/4](3.819) = 3.00 m'/s 226 0 CHAPTER 9 9.104 9.125 9.126 9.27 9.128 9.29 ‘The velocities in a 30-in-diameter pipe are measured as 15.0 and 14.5 ft/s at r = 0 and r = 4in, respectively Approximate the flow rate. ' 1 tenes ~ (5.75)(Falp)"* og [rl(to—P)] 14.5 = 15.0~(5.75)(t0/)" log [15/(05 ~4)] (rola) = 0.6455 tal =fu"l8 —(tolp)"" = (fu"/8)"* = 0.6456 f = 3.3344/ 0" dae = (U)(L+ 1334") 15.0 (W)(4 1.33f"%) 15.0 (Wyld + (1.33)08.3344/07)9 15.0=0 +2486 v=125TH/s Q=Av 22)(8)°/4](12.57) = 61.7 F/s With turbulent flow in a circular pipe, prove that the mean velocity occurs at a distance of approximately 0.78 from the centerline ofthe pipe. Bea 1.337Y0) = 2ONYFM(W) log frol(o=)]_— 0 1.33f"7v = 2.04f"7v) (Log [rol — DI log frol(y—1)]= 0.65196 rl (ro~ 7) = antilog 0.65196 = 4.487 716%, ‘The flow rate in a 12-in-diameter pipe is 8fs. The flow is known to be turbulent, and the centerline velocity s 12.0 fps. Determine the velocity profile, and determine the head loss per foot of pipe. He y= Q/A= S/N) IA= 10.19 8/5 tne = (OIF 133) 12.0= (10.19)(1 41.33") 0.01784 hy = SM L/d)(v"/2g) = 0.01784(1/(4)}(10.19"/{(2)(32.2)}) = 0.0288 ft per foot Ho= (F/4)(p)(0*12) = (0.01784/4)()(10.19"/2)(ra/p)"* = 0.4812 = tae 5.75(Fal 9)" op rl ("o~ 7] 0, u= 12.0 ~ (6.75)(0.4812) log [6/(6~ 0] = 12.0 ft/s. For r=2, u = 12.0 (5.75)(0.4812) log (6/(6- S is. For r= 4, w= 12.0 ~ (5.75)(0.4812) log (6/(6 ~ 4)] = 10.7 tes. For r= 5, w= 12.0~ (6.75)(0.4812) log (6/(6~ 5)] =9.85 ft/s. For r= 5.5, w= 12.0 (5.75)(0.4812) log [6/(6 ~ 5.5)] = 9.01 fs. For 1r=5.9, u=12.0~(5.75)(0.4812) log [6/(6 ~ 5.9)] = 7.08 ft/s. For r= 5.99, u = 12.0 ~ (5.75)(0.4812) log [6/(6- 5.99)] = 4.31 ft/s. ‘Tung oil (s.g. = 0.82) lows at a temperature of 80°F (v= 2.21 x 10° ft/s) in a 2-in-diameter smooth brass pipeline at 10 gpm. Find the head loss per mil. ' = (10)(0.002228) = 0.02228 ft'/s. v= Q/A = 0.02228/|(x)(8)*/4] = 1.021 fe/s fy IMLIAM07 12g) N= duly = (8Y(1.021)/02.21 x 10-4) From Fig. A-S for smooth pipe, f = 0.0333. h, = 0.0333}5280/()|(1.021°/[(2)(32.2))) = 17.1 per mile. 1.70 x 10" Water at 40°C flows in a 20-cm-diameter pipe with v = 5.1 m/s. Head loss measurements give f = 0.022. Determine the value of ¢ and find the shear stress at the wall of the pipe and at r = 3m, BN = duly = (0.20)(5.1)/(6.56 x 10-7), From Fig. A-5, €/d = 0.0015, € = (20)(0.0015) = 0.0300 em; t= Sov? [8 = (0.022)(992)(5-1)'/8 = 71.0 Pa; t = (ra)(r/1) = (71-0)(0.03) = 21.3 Pa. Water at 15°C flows through a 200-mm-diameter pipe with ¢ = 0.01 mm. (a) If the mean velocity i 3.6 m/s, what is the nominal thickness 6, of the viscous boundary layer? (6) What willbe the boundary layer thickness if the velocity i increased to 6.0 m/s? 1@ = 32.8v/(uf"9)——_Nq=du/v = (0.2009(3.6)/(1.16 x 10°*) = 6.21 x 10° €/d = (0.01/10)/20 = 00000500 From Fig. A-5,f = 0.0133. 5, ~ (32.8)(1-16 x 10-*/[(.6(0.0133)""] = 9.16% 10-* m, oF 91.6 um. Oy Na=(0.200/(6.0)/(1.16 x 10) = 1.03% 10° f= 0.0126 (62.8)(1.16 x 10-4/[(6.0)(0.0126)""] =5.65 x10-*m oF 56.5 um saat 9132 FLOW IN CLOSED CONDUITS J 227 When water at 50°F flows at 3.2cfs in a 2t pipeline, the head loss is 0.0004 ft per foot. What will be the head loss when glycerin at 68°F flows through the same pipe at the same rate? (ML /ayv*!2g) v= Q/A=3.2/| (a) GDI] = 1.0186 f/s 0.0004 = (f)[1/(2)]{1.0186"/[(2)(32.2))}_f = 0.04966 [Nq = pus = (1.94)(2)(1.0186)/(2.72 x 10) = 1.45 x 10° (turbulent) From Fig. A-5, ¢/d=0.021, For glycerin: ‘Np = (2.44)(2)(1.0186)/3.11 x 10") = 160 (laminar) $= 64/Ne=64/160=0.4000 hy = 0.4000{1/(2)}(1.0186"/{(2)(32.2)] Air flows at 501b/min in a 4in-diameter welded steel pipe (€ = 0.0018 in) at 100 psi head loss and pressure drop in 200 ft ofthis pipe. Assume the air to be of constant density. Hy = (LIA \W7128) Ne = pdvlu p= pIRT = (100)(144)/[(1716)(460 + 60)] = 0.01614 slug/ tt WeayAv=pgdv 8 = (0.01618) (32.2)((a)(4)'/4(0) v= 18.37ft/s Ng = (0.01614)(45)(18.37)/G.74 x 107) =2.64% 10" €/d = 0.0018/4 = 0.00045 From Fig. A-S, f = 0.018, hy = 0.018{200/(()](18.37°/[2)32.2)}) = 56.6 tof air, Ap = pghy = (0.01614)(32.2}(56.6) = 29.4 Ib/ft, oF 0.204 Ib/in® Air flows at an average velocity of 0.5 m/s through a long, 3.2-m-diameter, circular tunnel. Find the head-loss gradient ata point where the air temperature and pressure are 16°C and 109 kPa abs, respectively. Assume = 2mm. Find also the shear stress atthe wall and the thickness ofthe viscous sublayer. ' hy/L=(f1d)(v'12g) p= pIRT = (109)(1000)/[(287)(273 + 16)] = 1.314 kg/m? Ng = p/n = (1.314)(3.2)(0.5)/(.79 10-4) = 1.17 x10" e/d = (0.002)/3.2 = 0.000625 From Fig. A-S, f = 0.021. Ay ML (0.021/3.2}{0.54/(29.807)]} = 8.36 X 10°F m/m or 83.6 p/m To fpv"/8= (0.021)(1.314)(0.5)'/8 = 0.862 mPa 5, = 32.8v/(uf") = (32.8)(4/p)/(uf™) = (32.81.79 x 10°*/1.314)/{(0.5)(0.021)""] = 0.00620 m or 6.20 mm Repeat Prob. 9.132 forthe average velocity .0 m/s. I From Fig. A-S, for Ng = (5.0/0.5)(1.17 x 10°) = 1.17 x 10° and €/d = 0.000625, f = 0.018. Ay/L = (f/d)(v7/2g) = (0.018/3.2){5.07/{(2)(9.807)]} = 7.17 x 10-* m/m_ ‘To™ fov?/8 = (0.018)(1.314)(5.0)'/8 = 73.9 mPa 32.8)(u/pd/Cof") = (32.8)(1.79 x 10°*/1.314)/{(5.0)(0.018)'"] = 6.66 x 10-* mor 0.660mm_ Air at 20°C and atmospheric pressure flows with a velocity of 6 m/s through a S0-mm-diameter pipe. Find the head loss per meter of pipe if ¢ = 0.0025 mm. i hylL = (f\A/d\(v?/2g) Na = dv/v = (0.050)(6)/(1.51 x 10°*) €/d = 0.0025/50 = 0.000050 From Fig. AS, f = 0.26, hy/L = 0.026(1/0.050)(6?/{(2)(9.807)]) = 0.954 m/m. 9 x 10° 228 0 CHAPTER 9 9.135 9.136 9.37 9.138 9.138 9.40 ‘What is the head loss per foot of pipe when oil (s 2in-diameter welded steel pipe at 0.15 cfs? ' IylL=(f1d\v'l2) v= Q/A=' £8) having a viscosity of 1.9 101b- s/f? flows in a 15/2) 8)?/4] = 6.875 fe/s Ny = patu/e = (0.88) 1.94)](8X(6.875)/(1.9 x 10") = 1.03% 10" 0.00015/() = 0.00090 From Fig. AS, f= 0.083. hy/L= [0.088/(8)](6.875'/[(2)2.2))) = 0.145 ft ‘Water at 50°F flows in a 36-in-diameter concrete pipe (€ = 0.02 in). For a flow rate of 202 cfs, determine Nx and He v= Q/A= 202/{()(8)/4] = 28.58 ft/s Nu dv/v = (3)(28.58)/(1.40 x 10°) = 6.12« 10" Tom fov'l8 «ld = 0.02/36 = 0.000556 From Fig. A-5,f = 0.0175. To= (0.0175)(1.94)(28.58)"/8 = 3.47 1b/f ‘What isthe flow regime in Prob. 9.136? ' 5, =32.8v/(uf") = (82.8)(1.40 x 10-*)/[(28.58)(0.0175)"7] = 0.000121 ft Since [8, = 0.000121] <[0.3€ = (0.3)(0.02/12) = 0.000500}, regime is “rough.” Find the flow rate if water at 60°F experiences a head loss of zi ft/ft in 6-n cast iron pipe. ' AIL= (NU) d= (ND MAE22}—_v = 0.2809/ VF Try v = 2 ft/s: Nq = dv/¥ = (4)(2)/(1.21 x 10°) = 8.26 x 10*, €/d = 0,00085/(f2) = 0.00170, From Fig. A-S, {f= 0.0245. 1 =0.2808 010285 = 1.79 f/s. Try v = 1.79 ft/s: Nq = (8)(1.79)/(1.21 x 107) = 7.40 % 104, = 0.025, v = 0.2809/V0.025 = 1.78 ft/s. Try v= 1.78 ft/s: Na = ()(1-78)/ (1-21 X 10° $f =0.025 (0.K.); O = Av = [(x)(H)"/4](1.78) = 0.350 40 10, Kerosene with kinematic viscosity 5.1 x 10-? m’/s flows in ¢ 30-cm-diameter smooth pipe. Find the flow rate when the head loss is 0.4 m per 100m. ' hy = (LIA (v"/28) 0.4 = (FOO) (NAVO.BOD) v= 0.1534/VF Try v= 1 m/s: Ne=dv/y = ()(D/(S.1 x 10” 0,1534/VO.0128 = 1.36 m/s. Try v = 1.36m/ 0,1534/V0,0122 = 1:39 m/s. Try: = Av = [(x)( "4101.39 5.88 x 10°. From Fig. AS, fe = (BYU-36)/(5.1 x 10™ 39 m/s: Ng = (HB(U39)/(61 x 107) = 8.18 x 0983 m/s. A pipe with € = 0.00015 ft is required to carry fluid of kinematic viscosity 0.00021 ft'/s at the rate of 8.0 cfs If the head loss is to be 0.004 fr/ft, calculate the pipe diameter. He hylL=(flaXv'[2g) v= Q/A= 801 aa /4) = 10.19/d? 0.004» (F/4)((10.19/42F1VE2D 4=3.3209" Ny dv/v= (d)(t0.19/47/0,00001 = 48 24/4 (0015/1 =0.00015. From Fig. A-5, f =0.0215. d= 524/1.54 = 3.15 x 10, €/d = 0,00015/1.54 = 0.000974, }950/1.57 = 3.25 x 10%, €/d = 0.00015/1.57= set 918 sat FLOW IN CLOSED CONDUITS 1 229 (a) Find the shear stress on 40-in-diameter asphalted iron pipe ifthe fluid is water at 72°F and the average ‘velocity is 10 fps. (6) What will be the shear stres if the average velocity is reduced to 5 fps? ! to=fvl8 — Na= aviv @ Ng = (H9)(00)/(1.02% 10-4) = 3.27 x 10% fd =0.0004/(H8) = 0.00012 From Fig. A-S, f = 0.0127. To™ (0.0127)(1.93)(10)"/8 = 0.306 1b/f? o Na = (BYS)/(1.02 X10") = 1.6310 f= 0.0132 a= (0.0132)(1.93)(5)'/8 = 0.0796 1b/ft* A straight steel pipeline (€ = 0.00015 ft) slopes downward at a small angle 6, where sin @ ~ 6 = 0.01523. For xavity flow of il (v = 0.0006 ft/s) at 10 ft/s, what pipe size is needed? ' sin = hy/L = (f/d\(v'/2g) v= Q/A = 10/(ed*/4) = 12.73/a? 0.01523 = (f/d){(12.73/4¥2)132.2))) d= 2.771f Num duly = (d)(12.73/d")/0.0006 = 21 217/4 ‘Try d = 1 ft: Nq=21217/1=2.12 x 10", €/d =0.00015/1 = 0.00015. From Fig. A-5, f = 0.0265. d= (2.771(0.0265)"* = 1.34 ft. Try d = 1.34 ft: Nq = 21 217/1.34= 1.58 x 10", €/d = 0,00015/1.34 = 0.00011, (0.0284, d = (2.77)(0.0284)"" = 1.36 ft. Try d = 1.36 ft: Ny = 21 217/1.36= 1.56 x 10", e/d = 0.00015/1.36: 0.00011, f = 0.0284 (0.K.). Hence, d = 1.36 ft, oF 16.3, ‘Water at 140°F flows in a 0.824-in-diameter iron pipe (« = 0.00015 ft) of length 400 ft between points A and B. ‘At point A the elevation of the pipe is 104.0 ft and the pressure is 8.50 psi. At point B the elevation of the pipe 100.3 ft and the pressure is 9.00 psi. Compute the flow rate. 1 pale + Val2g + 24= pal + Ual2g + 0+ hy (8.50)(144)/61.4 + v4/2g + 104.0 = (9.00)(144)/61.4 + v3/2g + 100.34+h, 05/29 = v3/2g =252M hy =(FMLIAKV/29) 2.52 (F91400/00.824/12)}(07/12VG2.2)}) v= 0.1669/VF ‘Try v= 1 ft/s: Nq = dv/v = (0.824/12)(1)/(5.03 x 10°*) = 1.37 x 10%, €/d = 0.00015/(0.824/12) = 0.0022. From Fig. AS, f= 0.0325, v = 0.1669/ V0.0325 = 0.926 ft/s. Try v = 0.926 ft/s: Nn = (0.824/12)(0.926)/(5.03 X 10° = 1.26 10%, f = 0.0330, v = 0.1669/ V0.0330 = 0.919 ft/s. Try v= 0.919 ft/s: Ne = (0.824/12)(0.919)/(5.03 x 10-4) = 1.25 x 10%, f = 0.0330 (0.K.); 0.00339 fs. 19 = [(")(0.824/12)7/4](0.917) = ‘Air at 50 psia and 160°F flows in a 1-ft by 1.5-ft by 1000-ft duet at the rate of 1 Ib/min. Find the head loss if 0005 in. ' hy =(MLIA(W"/28) d= 4Ry Ry = Alp = (AY(.5)/(1 + 141.54 1.5) =0.300 ft d= (800300) = 1.20 = W=yAv——_y =p/RT = (50)(144)/[(53.3)(460 + 160)] =0.2179 10/8 1/60 = 0.2179{(1)(1.5)(v) v= 0,05099/s hy = (F)(1000/1.20)(0.050997/1(2)(32.2)]) = 0.03364 Nq= dv/v = (1.20)(0.05099)/(2.06 x 10"*) = 297 (laminar) f= 4INg = 64/297 = 0.2155 (0.03364)(0.2155) = 0.00725 ft Find the approximate rate at which 60°F water will flow in a conduit shaped in the form of an equilateral ‘tiangle ifthe head loss is 2ft per 100 ft. The cross-sectional area of the duct is 120 in’, and ¢ = 0.0018in. 1 First, ind the length ofeach side () ofthe cross section (see Fig. 9-29): AmbR2 1205 (¢ +x)(VE)/2 ABS 5 =2e= 2Y(RI2) = 16.6510 (NULIDWs) R= Alp. (40.2002) = 0,808 230 0 9.146 9.47 9.188, 9.149 9.150 CHAPTER 9 “Try v= 10 ft/s: Ng = du/ = (0,8008)(10)/(1.21 x 10-*) = 6.62 x 10°, €/d = (0.0018/12)/0.8008 = 0.000187 From Fig. A-S, f = 0.0150. v = 1.016/V0.0150 = 8.30 ft/s. Try v = 8.30 ft/s: Ne = (0.8008)(8.30)/(1.21 x 1079) =5.49 x 10°, f = 0.0155, wv = 1.016/V0.0155 = 8.16 f/s. Try v = 8.16 t/s: Nq = (0.8008)(8.16)/(1.21x 10-9) =5.40x 10°, f = 0.0155 (O.K.); Q= Av = (18)(8.16) = 6.80 7s. x % a“ Fig. 9:29 When fluid of weight density 50 1b/f? flows in a 6-in-diameter pipe, the frictional stress between the fluid and the pipe walls 0.5 1b/f. Calculate the head loss per mile of pipe. If the flow rate is 2.5efs, how much poweris lost per mile? De Rim dldm (y/4= 0.12508 y= (r(LIRay) = 0.5(5280/{(0.1250)(50)]} = 422.4 ft per mile P= Oph, = (2.5)(30)(422.4) = 52 800 b/s per mile = 52 800/550 = 96.0 hp per mile Prove that for a constant rate of discharge and a constant value of f the friction head loss in a pipe varies inversely as the fifth power of the diameter. Bue QiA=4Qlad® hy = (f\(LId)(v*/28)= GMLIA[(4Q/ a") 2g] = FMLIE O78) ‘Thus for constant fand constant Q, ji © 1/4 ‘Two long pipes are used to convey water between two reservoirs whose water surfaces are at different 1 elevations. One pipe has a diameter twice that of the other. If both pipes have the same value of f and if minor losses are neglected, what isthe ratio of the flow rates through the two pipes? t hy = (f(LId)(v?/2g) = 4 elevation hy=Q?/d° (from Prob. 9.147) (= (tye Therefore, Qi/di= Qi/d, Q2IQ.= (da/d,)" = 2 = 5.66. Thus the flow in the larger pipe willbe 5.66 times that in the smaller one. Points C and D, at the same elevation, are S00 ft apart in an 8-in pipe and are connected to a differential gage bby means of small tubing. When the flow of water is 6.31 cfs, the deflection of mercury in the gage is 6.43 ft Determine the friction factor f. H pcly + u2/2g + 2c= poly + vbl2g + zn + hy. Since vi/2¢ = v}/2g and zc= zp, Pl ~ Pol¥ =hy (643)(13.6~1)= 81.02, v= 0/A =6.31/{(a)(4P/4] = 18.08 Rs by = LIS) R)= (F)|S00/(4)] (18.08"/[(2)(32.2))) = 3807f, 81.02 = 3807F, Oil flows from tank A through 500 ft of 6-n new asphalt-dipped cast iron pipe to point B, as shown in Fig. 930. ‘What pressure in pounds per square inch will be needed at A to cause 0.450 cfs of cil to flow? (s.g. = 0.840; y= 227x 10" ft/s; € = 0.0004 ft.) ' paly + val2g + 24=PalY + B/2g+ Z0+h, — Ve= Q/Ay=0.450/[(x)(8)*/4] = 2.292 ft/s | = hy = (fL/d)(0* 2g) = (f)1500/(8)]{2.2927/1(2)(32.2)]) = 81.57F all(0.840)(62.4)] + 0 + 80.0 = 0 + 2.292%/[(2)(32.2)] + 100.0+81.57f px = 1053+ 4276f Nq=du/v = ((5)(2.292)/(2.27x 10) =5.05% 10" €/d = 0,0004/({8) = 0.000800 (235. pq = 1053+ (4276)(0.0235) = 1153 1b/f, or 8.01 Ib/in?, From Fig. A- sass 93st 955 FLOW IN CLOSED CONDUITS 2 231 BL to08 ++ Fig. 9:30 ‘An old 12-in by 18in rectangular duct carries air at 15.2 psia and 68°F through 1500 ft with an average velocity of 90.75 ft/s. Determine the loss of head and the pressure drop, assuming the duct to be horizontal and the size of the surface imperfections is 0.0018 ft ' Ry = Alp. = GDR + B+ + H) = 0.3008 IR, = (4)(0.300) = 1.20f¢ hy = (PL Ld)(v? 2g) = (F1500/ 1.20){9.75/1(2)(32.2))) = 1845 Ne = dv /v = (1.20)(9.75)/{(14.7/15.2)(0.64 x 10°] = 7.38% 10" (turbulent) €/d = 0.0018/1.20= 0.00150 From Fig. A-5, f= 0.024 hh, = (1845)(0.024) = 44.28 ft of air Ap = y= [(15.2/14.79(0.0750)] 44.28)/144 = 0.0238 tb/in* What size of new cast ron pipe, 8000 ft long, will deliver 37.5 fs of water at 70°F with a drop inthe hydrate grade line of 215 07 ‘ paly + U3/2g + 2n= poly +ubl2g + ze+hz — ((pal¥ +24)~ (Poly +20) = hu [pal +24) - (poly + 2s)] = HGL = 215 ft v=Q/A=37.5/(ad"/4) = 47.75/d” Ia, = hy = (f(L Iau" /2e) = (F (8000/d){ (47.75/42) (2)32.2)]} = 283 238f /a? 215 = 283238//d° d= 4.207F"" Ng = dv/y = (d)(47.75/d2)/(1.05 x 10-*) = 4.55 x 10°/d “Try d = 2 ft: Ne = (4.55 % 10")/2 = 2.28 x 10, €/d = 0,00085/2 = 0.000425. From Fig, A-5,f = 0.0164. d = (4 207)(0.0168)"* = 1.85. Try d = 1.85 ft: Ny = (4.55 % 109)/1.85 = 2.46 x 10", e/ = 0.00085/1.85 = 0.000459, 7=0.0164 (O.K.). Hence, d= 1.85, or 22.2in. What rate of flow of air at 68°F will be carried by a new horizontal 2-in-diameter steel pipe at an absolute ‘pressure of 3 atm and with a drop of 0.150 psiin 100 ft of pipe? Use © = 0.00025 ft. I Av 68°F and standard atmospheric pressure, 7 = 0.0752 lb/ft and v = 1.60% 10°*ft'/s. At a pressure of 3atm, y = (0.0752)(3) = 0.2256 Ib/f and v = (1.60 x 10"*)/3 = 5.333 x 10° ft/s. hy = (FYLIA)(0"/28), (6.130)(14) 0.2056 = (f){100/(#)](v*/[(2)G2.2)}, v = 3.206/VF. Try v= 10 ft/s: Ne (B) C0) /(5.333 x 107) = 3.13 10", «fd =0,00025)(4) = 0.00150. From Fig. A: 027 2067 V0.027 = 19.51 ft/s. Try v = 19.51 fils: Nq = (8)(19-51)/(5.333 x 10 *) = 6.10 x 10%, f = 0.0248, '206/'V0.0248 = 20.36 ft/s. Try v = 20.36 ft/s: Nq = (§)(20.36)/(5.333 x 10°) = 6.36 * 10", f=0.0248 (O.K.); Q = Av = [(2)(4)?/4](20.36) = 0.444 fe Determine the nature of the distribution of shear stress at a cross section in a horizontal, circular pipe under steady flow conditions, I For the free body in Fig, 9-31a, since the flow is steady, each particle moves to the right without acceleration. Hence, the summation of the forces in the x direction must equal zero. (ps)(2r’) ~ (p:)(ar)~ (2)@arL) = 0 or (Ps PNICL) @ ‘When r= 0, the shear stress ris zero; and when r= ry, the stress ty at the wall is a maximum. The variation is linear and is indicated in Fig, 9-318. Equation (1) holds for laminar and turbulent flows as no limitations, concerning flow were imposed in the derivation. Since (p, ~ p2)/ represents the drop in the energy line, or the lost head fr, multiplying Eq. (1) by y/y yields + = (yr/2L)I(Ps—Pa)/] oF = (rh. /2L)(0) @ Develop the expression for shear stress at a pipe wall. (HeelaXo'ng). From Prob 9136 hay = 2toL yro=4toL I Yd, AtoL/ 7d = (ALIA) 128), To = fov'/8. 232 0 CHAPTER 9 oat ead 9.186 For steady, laminar flow (a) what isthe relationship between the velocity at a point in the cross section and the velocity at the center of the pipe, and (b) what is the equation for velocity distribution? to = —(u\dv/ér)= (pr paMrV@L) (from Prob. 9.154) =(u)(dv/dr) = (Pp, ~ P2Mr)/(2L) Since (p.~p2)L is ota function ofr "py aPiZPe 1 -P=P2y, “w-v9=' ~[oen Plot tone BPee 01g = PP pipe van OO 0 ‘But the lost headin L fet is hg = (p,— psy: hence, yur vengEe @ (6) Since te velocity atthe boundary is zero, when r=, 0=O in Ea, (2), we have aor te Sarudal (at centerline) @ ‘Thus, in general, { o= CSP a-r) o | 9.157 Develop the expression forthe loss of headin pipe for steady, laminar flow ofan incompressible fd. Refer tore. 931d, | t © ‘Thus for laminar flow, the average velocity is half the maximum velocity v, in Eg, (3) of Prob. 9.156. Rearranging Eq. (1), we obtain fy Bt soe ° ‘These expressions apply for laminar flow of all luis in all pipes and conduits. | 9.188 Determine (a) the shear stress at the walls ofa 12-in-diameter pipe when water flowing causes a measured head loss of 15 ft in 300 ft of pipe length, (6) the shear stress 2in from the centerline of the pipe, (c) the shear velocity, (d) the average velocity for an f value of 0.50, and (e) the ratio v/v, 9.159 9.160 9.161 9.162 9.163 FLOW IN CLOSED CONDUITS 0 233 Bo) som phref2L = (624, A5N(B)/2V 12) (00) = 0.780 oF O.O0882 fi. (6) Since + varies linearly from centerline to wall, r= (0.00542)(3) = 0.00181 Ib/i (0) v, = Vt_/p = V0.780/1.94 = 0.634 ft/s. (@) 1 = fov"/8, 0.780 = (0.050)(1.94)(v7V/8, v = 8.02 fs. © = (WNuly), ¥= Hl, to pv(vly), Fol = (Muly) = v3 = (oN/y), U/vd = y1y, w/v, = vay! fin Prob. 9.158 the water is lowing through a 3-ft by 4-ft rectangular conduit of the same length with the same lost head, what isthe shear stress between the water and the pipe wall? i Ry = Alp. = BMA)/GB+3+4 +4) = 0.8571 ft €= (phi/L)(Ry) = [(62.4)(15)/300](0.8571) = 2.67 1b/f? or 0.0186 1b/in? Medium lubricating oil (s.g. = 0.860) is pumped through 1000 ft of horizontal 2in pipe at the rate of 0.0436 cfs. If the drop in pressure is 30.0 psi, what isthe absolute viscosity of the oil? Assuming laminar flow, Pixps_ 3uLve ye QIA = 0.0436/1()(&)/4] = 1.998 ft/s 32)(1)(1000)(1.998)/{((0.860)(62.4)1(2)") = 0.00188 1b - s/t (0.860)(1.94)](&)(1.998)/0.00188=296 (laminar) (from Prob. 9.157) (30.0)(144)/{(0.860)(62.4) Na = paul. ‘A horizontal wrought iron pipe, 6-n inside diameter and somewhat corroded, is transporting 4.50 Ib of air per second from A to B. At A the pressure is 70 psia and at B the pressure must be 65 psia. Flow is isothermal at (68°F. What is the length of pipe from A to B? Use € = 0.0013 ft. ' B 22 In (ve/vs) + LIA) \(vi/2) % 1¥palp: Y= (0.0752)(70/14.7)= 0.3581 Ib/f? y= (0.0752)(65/14.7) = 0.3325 Ite We yAv 450=(03581)[( (8) 4K.) vs = 64.005 4.50=0.3325(a)(4)7/41(05) vs = 68.93 ft/s [Ng = du/v = (§)(64.00)/{(14.7/70.0)(1.60 x 10°] =9.52 x10? /d = 0,0013/({4) = 0.0026 From Fig. A-S, f = 0.025. (20~65)(444)_ 2(21m (68.93/68.00) + 0.02512 /¢8)) {64.00°/[232.2))) ws SS”*~*~«*d Loot Heavy fuel ol flows from A to B through 3000 ft of horizontal 6-n steel pipe. The pressure at A is 155 psi and at Bis S.Opsi. The kinematic viscosity is 0.00444 ft/s and the specific gravity is 0.918. What is the flow rate? Assuming laminar flow, from Eq. (2) of Prob. 9.157, 32uL vn G2)(vo\Lvn) (155 ~$.0)(144) _ 32((0.00444){(0.918)(1.94)]) (3000), wed! (0.918)(62.4) ((0.918)(62.4)](8)" Uy = TAMAS Ng =du/y = (5)(.11)/0.00448 = 808 (laminar) Q= Av = [(a)()/4)(.11) = 1.40 0/5 Pe ‘What size pipe should be installed to carry 0.785 efs of heavy fuel oil (¥ = 0.00221 f'/, s.g. = 0.912) at 60°F if the availabe lost headin the 1000 length of horizontal pipe is 2.0? 1 Assuming laminar fow, m= py = [(0.912)(1.94)}0.00221) = 0.003910 1b s/t v= Q/A=0.785/(xd"/4)=0.9995/d? 22.0 (32)(0.003910)(1000)(0.9995/d")/({(0.912)(62.4)](4*)} 4 =0,562f or 6.75in Ne =du/v =(0.562)(0.9995/0,562")/0.00221 = 805 (laminar) 234 0 CHAPTER 9 9.164 9.165 9.166 9.67 9.168 9.169 9.70 Determine the head loss in 1000 ft of new, uncoated 12-in-ID cast iron pipe when water at 60°F flows at 5.00 ft/s. Use €/d = 0.0008. ' fy = (PULIA Kv 2g) Ng = dv/v = (48)(5.00)/(1.21 x 10-4) = 4.13 x 10° From Fig. A-5, f= 0.0196. h = 0.0194{1000/(2){5.00°/(2)(32.2))} =7.53 Rework Prob. 9.164 if the liquid is medium fuel ol at 60°F (v = 4.75 x 10-*f'/s) flowing at the same velocity i hy = (F(LIa)(u°2g) = du/v = (18)(5.00)/(4.75 x 104) = 1.05 x 10° From Fig. A-S,f = 0.0213. hy =0. aie 007/12)(32.2)}} = 8.27 fe. Points A and B are 4000 ft apart along a new 6-in-ID steel pipe. Point B is 50.5 ft higher than A and the pressures at A and B are 123 psi and 48.6 psi, respectively. How much medium fuel oil at 70°F will flow from A to B? Use s.g. = 0.854, y= 4.12 10"*fe/s, € = 0,0002 ft. 1 Daly + URl28 +24 Pal + UbI28 + 25+ hi (123)(144)/[(0.854)(62.4)] + v3 /2g + 0= (48.6)(148)/[(0.858)(62.4)] + v9/2g + 50.54 hy Vil2g=vil2g hy = 130.5 f= hy = (P(LIAN(W?/28) 150.5 = (f)[4000/(G)](¥°@VG22)))— v= 1.101/VF ‘Dry v= 104R/s: Nq = du/v = (f)(10)/(4.12 x 10) = 1.21 x 10%, €/d = 0,0002/(f) = 0.000400. From Fig, AS, 0195. w = 1.101/-V0.0195 = 7.884 ft/s. Try v = 7.884 ft/s: Na = ({)(7.884)/(4.12 x 10°*) = 9.57 x 10, 0195 (OK); (()(8)°/4](7.884) = 1.55 £075. How much water (60°F) would flow under the conditions of Prob. 9.166. ! aly + alg + 24= poly + vb/2¢ + 29+ hy, (123)(148)/62.4 + vi /2g + 0= (48.6)(148)/62.4-+ vb /2g + 50.5 +h, vil2g=val2g hy = 121.2ft= hy = (f)(L/d)(v*/2g) 121.2= (/)A000/()(VAVE2D}}—-v =0.9878/VF “Try v= 10ft/s: Nq = dv/v = (f)(10)/(1.21 x 10%) = 4.13 x 10%, €/d = 0.0002/(f) = 0.000400, From Fig. AS, f=0.0172. v = 0.9878/O.OI72 = 7.532 ft/s. Try w = 7.532 ft/s: Nq™ (f)(7.532)/(1.21 x 10°*) = 3.11 x 10% f= 0.0176, v = 0,9878/V0.0176 = 7.446 ft/s. Try u = 7.446 ft/s: Nq ™ (#5)(7.446)/(1.21 x 10") = 3.08 x 10°, F=0.0176. (O.K.); = Av = [(a)(8)/4](7.446) = 1.46 £7. ‘To transport 300 cfs of air, p = 16 psia, T'= 68°F, with a head loss of 3in of water per 1000 ft, what size galvanized pipe is needed? (€ = 0.00085 ft.) t d = 0.66{(€'*)(LO*/ghy)** + (VOY (Ligh, YP Y= PIRT = (16)(144)/[(53.3)(460 + 68)] =0.081871b/fe? hy = (3)(62.4/0.08187) = 190.5 fof air sf (40003(300)° -¢300)°-f —1000_ Jy 4 = 0.66{ 0.005 (any + (1.64 x 10°*)(300)" mas Pr =2e4n ‘Two tanks of a solvent (j= 0.05 Pas, y= 8kN/m:) are connected by 300 m of commercial steel pipe. What size must the pipe be to convey 50 L/s, if one tank is 4m higher than the other? t d= 0.66{(€'*)(LO*/gh,)* + (vQ)*(L igh, P* v= nlp = gly = (0.05)(9.807)/[8 x 10°] = 6.129 x 10-* m*/s (300) iis)" (9.807)(4) Calculate the diameter of a wooden conduit (€ = 0.006 ft) that is to carry 300 ft'/s of water at 60°F a distance of 1000 fe with a head loss of 1.1 ft ' 4 = 0.66[(€'*(LO7/ gh + (MOY (LighyP™ (1000)(300)* 1000 ry @2.2)(1.1) 200. =022m, a= 0.66{ 0.000046 T+ 612910968053 2. oa [OMEN sat x10-y 00 -1598 sam san sans sam uns 116 FLOW IN CLOSED CONDUITS 0 235 ‘Ano pipe 2m in diameter has a roughness of ¢ =30 mm. A 12-mm-thick lining would reduce the roughness to€=1mm. How much would pumping costs be reduced per kilometer of pipe for water at 20°C with 10%, f Water at 20°C is to be pumped through 2 km of 200-mm-diameter wrought iron pipe at the rate of 60 L/s. Compute the head loss and power required. ' hy = (PULIA)(W?/28) v= Q/A = (60 10™/{()(0.200)'/4] = 1.910 m/s [Ny = du/v = (0:200)(1.910)/(1.02 10-4) =3.75 x 10° e/d = 0,000046/(0.200) = 0.000230 From Fig. A-, f = 0.016 ‘yg = 0.016{2000/0.200]{ 1.9107/{(2)(9.807)]} = 29.76m P= Qyh, = (60 x 10°*)(9.79)(29.76) = 17.48 kW An industrial ventilation system contains 4000 ft of 12-in-diameter galvanized pipe. Neglecting minor losses, ‘what head must a blower produce to furnish 3 ton/h of air at p = 14 psia, T = 90°F? Bhp (Lay vI28)—Ng= pdvlu p= pIRT =(14)(144)/{(1716)(460 + 909] ~ 0.002136 suglt? M=pAv 6000/3600 = [(0.002136)(32.2)1()()'/4](0) — v =30.85 4/5 (0.002136) )(30.85)/3.90 x 10-7) = 1.69 10" ¢/d = 0,0005/¢42) = 0.000500 +h, = 0.019{4000/(13)](30.85*/[(2)(32.2)}) = 1123.2 ft of air oF 1.24 t of water 9.196 FLOW IN CLOSED CONDUITS 0 239 ‘A.2.0-m-diameter pipe of length 1560 m for which ¢ = 1.5 mm conveys water at 12°C between two reservoirs at arate of 8.0m"/s. What must be the difference in water-surface elevations between the two reservoirs? Neglect ‘minor losses. 1 Pily + il2g +2 =pily+vY2gt+nth, 0+0+2=04042+h, a~ hy = (F(L/d)(v"2g) 546 m/s Ny = du/v = (2.0)(2.546)/(1.24 x 10™ «ld = (1,5/1000)/2.0 = 0.000730 v=Q/A 41x 10" On (2.07/4 = From Fig. A-5, f = 0.018. hy = (0.018)(1560/2.0){2.546?/[(2)(9.807)]} = 4.64 m. Hence, the water-surface elevations between the two reservoirs is 4.64 m. Water flows from reservoir 1 to reservoir 2 through a 4in-diameter, 500-ft-length pipe, as shown in Fig. 933. Assume an initial friction factor (f) of 0.037 and a roughness (¢) of 0.008 ft for the pipe. Find the flow rate. H puly +0llg + 21=paly + vi2g + 2+ hy 0+0+700.6=0404655.5+h, hy =45.2f0= hy + he (D Friction loss: hy = (f)(L/d)(v?/2g) = 0.037[500/(4)](v/2g) = 55.50v2/2e. (ID) Minor losses: (a) Due to entrance: From Fig. A-7, take K, = 0.45. (b) Due to globe valve: From Table A-11, Kage, ™ 5.7. From Table A-12, take Kz/Kayea 1.75. Hence, K,=(5.1)(1.75) =9.98. (©) Due to bend: R/D = #=3.0, /D = (2) Due to elbow: From Table A-11, K, (©) Due to exit: From Fig. A-7, Ke= 1.0. Thus, os, 003/(i8) = 0.00900, From Fig. A-12, K, 0.23. iy + hn = (V2/[(2)(32.2)]} (55.50 + 0.45 + 9,98 + 0.45 + 0.23 + 1.0) = 1.05007 1.0500" = 45,2. uv =6.561 ft/s Nu = Du/v = (i)(6.561)/(1.90 x 10%) =1.15 x 10" _€/D = 0,003/(1$) = 0.00900 From Fig. A-5, f = 0.037. (Assumed value of f 0.K.) Q = Av = [(r)(#)*/4](6.561) = 0.573 f/. SE 0. 10581 12m bend radius lew. 65554 Sharpredged entrance Globe valve, 25%, closed 4 Sharpedged exit 90" long radius elbow Fig. 9.33 240 0 CHAPTER 9 9.197 Determine the head oss in the system shown in Fig. 9-34 and the discharge in the pipe. 9.198 9.199 i ily + 022g +2 paly + vi/2g + 24h, — 0+0+100=04 vi/[(2)(32.2)] 40+ hy Iya hy hy hy = (FMLID)(U*/28) = f1500/(S)U3/2)32.2)}} = 15.53/03 hm =(KY(U4/28) K=O ig = 0.5 (U3/[2)(32.2)]} = 0.007640}, = 15.S3fu3 + 0.00776403 100 = 0.0155303 + 15.53fu3 + 0.00764v3 v= V1007(0.02329 + 15.53f) Try v2 = 10 ft/s: Nq = Dv/v = (f4)(10)/(1.21 x 104) = 4.13 x 10°, €/D = 0.00015/(i) = 0.00030. From Fig. A-S, f = 0.0165. v.= V100/{(0.02329 + (15.53)(0.0165)] = 18.91 ft/s. Try vz= 18.91 ft/s: Ny = (§)(18.91)/(1.21 x 10-*) = 7.81 x 10%, f = 0.0157, vs = V1007[0.02329 + (15.53)(0.0157)] = 19.35 ft/s. ‘Try v_= 19.35 ft/s: Ne = (§)(19.35)/(1.21 x 10°) = 8.00% 10°, f = 0.0157 (O.K.); hy (15.53)(0.0157)(19.35) + (0.007764)(19.35)" = 94.2 ft of water, Q = Av = {(22)(#)'/4](19.35) = 3.80 fs. 1008 Gin-diameter new wrought icon pipe on | Fig. 954 Figure 9-35 shows two reservoirs containing water at 60°F. The water in the upper reservoir isto be drained to the other reservoir at a lower level as shown. The total length of commercial steel pipe is 100 ft, and the diameter ofthe pipe is 4 in. What will be the flow rate of water through the pipe when the water surface elevations are as shown in Fig. 9-357 ' ily + i/2g +2, =pily +VH2g+ 2th, 0+0+335=0+0+30+h, hy hy + him hy = MEIDY(0*/2g) = FLOO/ EMV AE22YY} = 4.658f03 has = (KY(W?/28) Duc to entrance, take K, = 0.45, Due to elbow, K; = 0.64. Due to gate valve, Kyyeg = 0.11, Take Ky/Kuyer = 17. Hence, Ks= (0.11)(17) = 1.87. Due to exit, K,= 1.0. Jin = (0.45 + 0.64 + 1.87 + 1.0){07/[2)32.2))) = 0.061490", = 4.65807 + 0.061497 335-=300+ 4658/0" + 0.061490" v= WISAESEF FO.06TH) ‘Try f = 0.019: v= V3SII(E658)(0.019) + 0.0614] = 15.28 ft/s, Nq = Dv/v = (4)(15.28)/(1.21 x 10-9) = THO Prom Bg, Asef = 00178 Try 0017S: v= VRC RNOATI TOO = 15.640, Na (4)(15.64)/(1.21 x 1079) = 4,31 x 105, f= 0.0175 (O.K.); = Av = [()(4)°/4](15.68) = 1.36 07/5 ‘A 15.in-diameter new cast iron pipe connecting two reservoirs as shown in Fig. 9-36 carries water at 60°F. The pipe is 120 ft long, and the discharge is 20 ft/s. Determine the difference in elevation between water surfaces in the two reservoirs. 1 pily +vi[2g+2.= poly +UV2gt nth, 04042204042 +h, Since 2,— 22 =H and hy = hy + hy H = hy + Ins hy = (S(LID)(U?/28). v= Q/A =20/|(a)(8)'/4] = 16.30 /s [Ne = Duly = (§9)(16.30)/(1.21 x 109) = 1.68 10 _€/D =0,00085/($8) = 0.00068 From Fig. A-5, f = 0.018. hy = 0.018[120/(#)]{16.30°/[(2)(32.2)]} =7.13f, hm = (K)(v*/2g). For entrance, take Ky =0.45. For exit, K;=1.0. hy = (0.45+ 1.0){16.30'/[(2)(32.2)]} = 5.98 ft, H = 7.13 + 5.98 = 13.11 9201 FLOW IN CLOSED CONDUITS 0 241 Blew. 335% 90" regula ebow oreo Gate valve, stewed type, 50% closed Fig. 9.35 Water OF Pent Fig. 9.36 Repeat Prob. 9.87 by including losses due to a sharp-edged entrance, the exit, and a fully open flanged globe valve. I For entrance, K, = 0.5. For exit, K, = 1.0. For globe valve, K; =8.5. From Prob. 9.87, hy = hy + hy = [f)(L/a) + K, + K+ K,](0"/2g), 98 = (f{7000/0.050] + 0.5 + 1.0-+ 8.5) (v?/[(2\(9.807)}, 1922 = (140 000f + 1.0}, = [1921 ad o0f + 10.0). Ta f =0.02 v= (1922/{040000N0.0) + 10.0) 0.8270 m/s, [Nj = pdv/u = (998)(0.050)(0.8270)/(1.02 x 10-*) = 4.05 > 10*. From Fig. A-5, f =0.022. Try 2 (1922 {Ga 000\0.022) + 10.0} = 09887 ls Ne = (08)(0.05040.76873/(.02> 10") = 3.86% 1, f=0.022 (O.K.); Q = Au = [(}(0.050)?/4(0.7887) = 0.001548 m"/s, or 5.57 m'/h. Repeat Prob. 9.95 by including losses due to a sharp entrance and a fully open screwed swing-check valve. I For sharp entrance, K,= 0.5. For swing-check valve, Ks=5.1. hn = (Ky + K,)(v/2g). From Prob. 9.95, ¥= 14.67 ft/s. hn = (05 +5.1)(14.67 /(2)(32.2)]} = 18.71 ft. From Prob. 9.95, 0+0-+h=0+ 14.67,[(2)G2.2)] + 0+ (236.6 + 18.71), h = 258.7. ‘Two lakes (Fig. 9-37) communicate via two 20-f-long wrought iron pipes joined abruptly. The entrance and exit are sharp-edged. Including minor losses, compute the flow rate if lake 1 stands 59 ft higher than lake 2, and the average temperature of the system is 20°C. i Pil + vil2g + 2 = paly + vi/2g + 22+ hy Ayah thy fy = (F (LI D)(v?/2g) = (fa){20/G4)]{ v2/(232.2)]} + f120/ (BY ve/{(2)32.2)]) Auta Ast (Cay /AK0.)=(AMBV ITs) v5 = vl hy = 3. T21f.vi + (1.863)(f, (04/4)? = 3.727f.0s + 0.011645, 242 0 CHAPTER 9 9.203 9.208 For sharp entrance, K, = 0.5. For sharp exit, Ks = 1.0. For sudden expansion, Ka = (1 ~ 0.5)" = 0.5625. ‘i= (0-5 + 0.5625) (v3) [(2)(32.2)}) + (1.0)(02/{(2)(32.2)]} = 0.016500 + (0.01553)(v./4)? = 0.017470, 0+ 0+ 59 = 0+ 040+ [(3.721f.u5 + 0.1164f.02) + 0.0174702]. Try f. 020: 3.727)(0.020)(02) + (0.1164)(0.020)(v2) +0.0174703 uv, =25.01ft/s vy =25.01/4= 6.253 ft/s Ng= Duly (Nq)u=(h)(25.01)/(1.11 x 10-4) = 1.8810 (€/D) = (8)(6.253)/(1.11 x 10-4) = 9.39 x 10%, (€/D), = 0.00015/() = 0.000900, WD: 3.727)(0.024)(v2) + (0.1164)(0.022)(0) +0.0174703 vy =23.21ft/s vy =23.21/4 = 5.803 ft/s (Wada = (2)(23.41)/(L11 X10") =1.76 10 f, = 0028 (Wee = (BYG-852)/(1.11 x10) = 8.79% 10" fy = 0.022 ‘Therefore, f,= 0.024 and fy = 0.022 is O.K. Q = Av = [(a)(4)"/4(23.21) =0.127 f/s. SH oom tion LZ w At is pein L=20ft Fig. 9-37 ‘Two reservoirs containing water at 20°C are connected by 800m of 180-mm cast iron pipe, including a sharp entrance, a submerged exit, a gate valve 75 percent open, two I-m-radius bends, and six regular 90° elbows. If the flow rate is 9 m°/min, find the difference in reservoir elevations. 1 pil +viltg + a= psly + viet ath, hah + lm — hy =(fML/D\v*/2) v= Q/A = (9/60)/{()(0.180)"/4] = 5.895 m/s Nq= Du/v = (0-180)(5.895)/(1.02 x 10~ ‘€/D = 0.00026)0.180 = 0.00144 = 0.0217. hy = 0.0217[800/0.180}(5.895'/[(2)(9.807)]} = 170.9 m. For sharp entrance, K, =0.5. 0, For gate valve 75 percent open, Ky= 0.3. For bends, K, = (2)(0.15) = 0.30. For elbows, .62. hy = (0.5 + 1.0-+ 0.3 + 0.30-+ 1.62)(5.895"/[(2)(9.807)]} = 6.6m, hz = 170.9 +6.6= £042, 4177.5, 1-25 177Sm. From Fig. A-S, Forexit, K,= (0.2 175m, 0+0+2, The system in Fig. 9-38 consists of 1000 m of $0-mm cast iron pipe, two 45" and four 90° langed long-radivs elbows, a fully open flanged globe valve, and a sharp exit into a reservoir. What gage pressure is required at Point Ito deliver 5L/s of water at 20°C into the reservoir, whose free surface lies 100-m above point 1? 1 pil + vig += poly +UE2B + 2th, shy thy hy = (FXLIDYV*/28) v= Q/A=0.005/{()(0.050)"/4] =2.546 m/s Nq = v/v = (0.050)(2.546)/(1.02 x 10°*) = 1.25 x 10° <«/D = 0,00026/0.050 = 0.00520 }.0315{1000/0.050]{2.546"/[(2(9.807)]} = 208.2 m. For 45° elbows, Ky = For exit, K,= 1.0. 11.9 m, py/9.79 + From Fig. A'S, f = 0.0315. (2)(0.20) =0.40. For 90° elbows, K; = (4)(0.30) = 1.20. For the open valve, K; a= (0.40 + 1.20 + 8.5 + 1.0){2.5467/[(2)(9.807)]) = 3.7 m, hy = 208.2 + 3. 2°546/{(2)(9.807)] + 0= 0+ 0+ 100 + 211.9, p= 3056 kPa gage. as se FLOW IN CLOSED CONDUITS 1 243 9.205 A LLin-diameter smooth water pipe slopes upward at 30° (Fig. 9-39); the flanged globe valve is fully open. Calculate the flow rate. i lt + Vide + z= poly + VIA th, vil2g= V2 hy =hy + hm Therefore, pily + 21= pal + z2+ hy + My (Pipa y= 22— 2, hy + hw o From manometer, Pi Pa ne ~ Yo)(B) + (Poe 2) @ Combining Eqs. (1) and (2) [(Y~ 7140) tol) = hy + han by = F(LIDY"/28) = (f)[(91008 3071 (A){v7/{2)G2.2)]} = 1.936 fv, hy, = (K)(v*/2g). For globe valve, 13. Ain = 13{v°/[(2)(32.2)]) = 0.2019v%, {((13.6)(62. }/62.4}(35 fu? + 0.2019v2, v = V7.350/(1.936f +0.2019).. yf = O08 v= VTE NEE TAD = 5.527 RN = Dus ylss27/(.08 > 10°) = 4.39% 10, From Fig. A-S,f = 0.0217. Ty f = 0.0217: v= V7.350/[(.936,0.0017) +0.2019] = 5.489 fs, Nx = (1h)(5.489)/(1.05 * 10 1.36 x 10°, f = 0.0217 (O.K.); Q = Av = [()(h)'/4](5.489) = 0.0299 ft'/s. Fig. 939 9206 A pipe system carries water from a reservoir and discharges it asa free jet, as shown in Fig. 9-40, How much flow isto be expected through a 200-mm steel commercial pipe with the itings shown’? 1 pily + v}/2g+2= poly +vi/2g+ nth, hy =hy + he hy = (FL/D)(v?/2g) = f[(60 + 20 + 60)/(FB)1{v3/[(2)(9.807)]} = 35.69fv3 in = (KY(8*/28) = [0.05 + 20.40) (v3/[219.807)} = 0.043343 0+ 04 30=0+ vi/[(2\9.807)] +20 + (35.69f03 + 0.043408) vs = VIOICS.F FORD) ‘Try f =0.014: Vi0/{(35.69)(0.014) + 0.09432] = 4.103 m/s, Nz = Du/v = (7t)(4.103)/(0.0113 x 10 7-26x 10, €/D = 0,000046/ (#3) = 0.000230. From Fig. AS, f = 0.0182. Try f =0.0132: {07/{(35.69)(0.0152) + 0.09432] = 3.963 m/s, Nx = (7i0)(3.963)/(0.0113 x 10°*) = 7.01 x 10°, f= 0.0152 (O.K.); Q = Av = [()(8)"/4](3.963) = 0.125 m"/s. steel commercial pipe treughout 2.90" elbows ‘R040 = .0113 x 10-4 mPa 990 ko/m? Fig. 9-40 9287 A pipe system having a given centerline geometry as shown in Fig. 9-41 isto be chosen to transport a maximum ‘of 1°75 of ol from tank A to tank B. What i a pipe size that will do the job? ' ely +0228 + 2.= Ppl ¥ +U22G + 2% +h, hy = hy + mw Ay = (f(L/ Dv" /2g) = f{(100 + 130 + 300)/D]{v5/{(2)(32.2)]} = 8.230fv,/D Mag = (K)(07/2g) = (0.05 + 0.5 + 0.5 + 1){u3/f(2)(32.2)]} = 0.03183u3 244 0 CHAPTER 9 (100)(144)/50 + 0 + 130 (50)(144)/50 + v3/[(2)(32.2)] + 0-+ 8.230f02/D + 0.031830; 0.047363 +8.230f03/D = 274 v3 = 274/(0.04736+8.230f/D) vp = Q/A, = 1/(aD*/4) = 1.273/D* (1.273/ D9) = 274 (0.04736 + 8.230f/D) ‘Try f = 0.015: (1.273/D7)* = 274/{0.04736 + (8.230)(0.015)/D}. By trial and error, D = 0.240 ft. = 1.273/0.2407 = 22.1 ft/s Nq= pDulu = (y/g)(Du)/u = ($0/32.2)(0.240)(22.1)/(50 x 10-4) = 1.65 10 £€/D = 0.00015/0.240 = 0.000625 From Fig. A-5, { = 0.0285. Try f = 0.0285: (1.273/D*}? = 274/[0.04736 + (8.230)(0.0285)/D}. By trial and error, D=027K. Ve 1273/0272 = 1724s Ny (80/32.2)(0.272)(17.2)/(50% 107) = 1.48 x 10" mss (OK) Hence, D = 0.272 ft, or 3.26in, m Feono-*m ane 50 y= S0tbyn” Ib/in? goge Fig. 941 9.208 What gage pressure pis required to cause 5 ft/s of water to flow through the system shown in Fig. 942? ‘Assume that the reservoirs large, that minor losses are negligible, and that v = 2.11 x 10"*ft/s. ' ily + Vil28 + 21= poly U2 + +h, V2= O/Ar= S/[ HY) /4] = 25.46 fe/s h Y(LID\(u#/2g) Ng = v/v = (8)(25.46)/(2.11 x 10°) = 6.03 x 10° €/d = 0.00015 /(4) = 0.00030 From Fig. A-5, f= 0.016. ‘hy, = 0.016{(400 + 160 + 250)/()]{25.467/[(2)(32.2)]} = 260.9 ft (P:)(144)/62.4 + 0+ 120 = 0-4 25.467/[2\(32.2)] + 160+ 260.9 py = 135tb/in* gage 9.209 In Prob. 9.208, let 6 in be the nominal diameter of the pipe. For the entrance fitting, r/d = 0.06. Calculate the pressure p,. The elbows are screwed elbows and there is now an open globe valve in the pipe system. Include ‘minor losses. FE Use d=6.065in aly + Vil2g + 2. pal + Ui/2g + 22 hy by thin hy = (FY(LI)(U"/28)——_Nq= du/v = (6.065/12)(24.92)/(2.11 x 10) «/d = 0.00015/(6.065/12) = 0.000297 A FLOW IN CLOSED CONDUITS J 245 From Fig. A-S, f = 0.016. h, = 0,016{(400 + 160 + 250)/(6,065/12)](24.92"/[(2)(32.2)]} =247.3 ft hn = (K)(0"/2g). For entrance, K, = 0.15. For elbows, K, = (2)(0.45)= 0.90, For globe valve, K, iy = (0.15 + 0,90 + 5.1){24.927/[2Y32.2)]} = 59.3% hy = 247.3 + 59.3 = 306.6 (P.(144)/62.4 + 0-+ 120 = 0+ 24.92°/[(2)(92.2)] + 160+ 306.6 py = 154 1b/in® gage ‘The industrial scrubber B of Fig. 9-43 consumes water (v = 0.113 x 10°? m'/s) atthe rate of 0.1 m/s. Ifthe pipe {is 150-mm commercial pipe, compute the necessary tank pressure Be paly + vi2g +24 =Paly + Val2g + 20th, Un = Q/An=0.1/{(x)(0.150)'/4] = 5.659 m/s aby the Ihe =(fL/d)(v7/2g) Np = du/v = (0.150)(5.659)/(0.113 x 10°°) = 7.51 x 10° 10"*) = 1.15 x 10°, e/d = 0,000046/0.196 = 0.000235. From Fig. AS,f-=0.015 (O.K.). Therefore, d=0.196m, or 196mm. A 12-in-diameter pipe with a friction factor of 0.02 conducts fluid between two tanks at 10 fps. The ends of the pipe are flush with the tank walls. Find the ratio of the minor losses to the pipe friction loss if the length of the pipe is (a) 10 ft, (6) 250 ft, and (e) 1500 ft ' hy =(MLIA)(W"I28) hy = (K(U*/28) @ hy =0.02[(10/58)\0/2g) = 0.2007/2g Ing = (0.5 +1.0)(0"/2) = 1.507/2g indy = (150*/28)/(0.200'/22)=75 (78:1) o y= 0.02(250/()|v7/2g) = 5.0072 hm = (0.5 + 1.0)(07/2g) = 1507/2 nally = (1.50/28)/(5.00/2g)=0.3 (03:1) © ‘hy = 0.02{1500/(18)|(v*/2g) = 30v*/2g im = (0.5 + 1.0)(0°/2g) = 1.5¥7/2g JIiglhy = (1.50*/2g)/(30v"/2g)= 0.05 (0.05:1) 248 0 CHAPTER 9 9.07 9.218 9.219 9.200 9221 92m ‘A smooth pipe 30 cm in diameter and 100m long has a flush entrance and a submerged discharge. The velocity is 3m/s. If the fui is water at 15°C, what is the total loss of head? ' hy = LIA) 0128) Nq= v/v = (0.30)(3)/(1.16 x 10-4) = 7.76 x 10° .0122{100/0.30}{3*/{(2)(9.807)]} = 1.87 m, ha = (KYv*/2e) = 69 m, hy, = 1.87 + 0,69 =2.56m. ‘The water of Prob. 9.217 is replaced by oil with kinematic viscosity 9.3 x 10~* m*/s and specific gravity 0.9, Determine the pressure loss. ' hyahy thy hy =(f(Lid)(v*/2g) Np = dv/v = (0.30)(3)/(9.3 x 10-4 From Fig. A-S,f = 0.031 ‘hy = 0.031[100/0.301{37/{(29.807)]} = 4.74mi = (K)(v?/2g) = (0.5 + 1.0)(32/[(2\9.807)]) = 0.69m =A 14+ 09=5.43mofoil — Ap= yh, =[(0.925)0.79)|5.43) = 49.2 kPa = 9.68 10” ‘A smooth pipe consists of 100 ft of 8-n pipe followed by 200 ft of 24-in pipe, with an abrupt change of cross section at the junction. It has a flish entrance and a submerged exit. If it carries water at 60 °F in the smaller leg with a velocity of 18 fps, what i the total head loss? hy = PMLIA(07I28) Ne = duly (Ne 0117 Ama Ay A, =[((H/4) = 0349108 A, = [Cay 4D'/8) =3.0028€ (0:3491)(18)=(3.142)(v,) —v,=2.00080/s ——_(Naanaa= (2, 000)/(1.21 x 10) = 3.31 10" (ene 0.0142 fy = 0.01171100/)}(18/(29(32.2)]} + 0.0142{200/(¥)](2.0007/1(2)(32.2))} = 892K ha = (KY(0"/28) For entrance, K, 0.5, For abrupt change, Ks= (1 ~ ,/A;) = (1 ~ 0.3491/3.142)? = 0.7901. For exit, K,= 1.0. ‘= (0.5 + 0.7901) (18"/[(2)(32.2)]} + 1.0(2.000"/[(2)(32.2)]} = 6.55 ft, fr, = 8.92 + 6.55 = 15.47 ft. 8)(18)/(1.21 x 10°) = 9.92% 10" ‘A Gin-diameter pipe (f = 0.032) of length 110 ft connects two reservoirs whose water-surface elevations diflet by 10, The pipe entrance is flush, and the discharge is submerged. (a) Compute the flow rate. (6) Ifthe last 10 fof pipe were replaced with a conical diffuser with a cone angle of 10°, compute the flow rate. ' pily + v2 +2,= pal y + ¥i/28 + 2+ he @ diy = (PML Ia)(v? 2g) = 0.032[110/(8)1{07/232.2)}) = 0.109307 a= (KY(07/28) = (0.5 + 1.O}{7/{@)322)Y) = 0.023297 ‘Ay, = hy + by, = 0.10930" + 0.023290" = 0.132607 040+ 10=0+0+0+0.13260 v=8.684ft/s = Av [(x)(H)°/4] 6.684) = 1.71 7s (8) by =0.052I(110 ~ 10)/€8){0°12N32.2)P) = 0.099380? hy = (0.5+ 0.40)(v*[(2)(32.2)}) = 0.013980" ‘hy, = 0.099380" + 0.013980 = 0.113407 0 +04 10=0+0+0+40.11347 V=9I1H/s — Q=Av=[(x)(4'/4](9.391) = 1.84 10/5 Given two pipes in series with a diameter ratio of 1:2 and flow velocity of 15 fps in the smaller pipe, find the loss of head due to abrupt (a) contraction and (6) enlargement. i hy = (Kv /28) @ 4D =0S — K=033 hy =0.33(15"/1(2)(32.2))) = 1.15 ft o Kn055 hy =0.55(15/(2)(32.2))) = 1.92 In a 50- length of 4-in-diameter wrought iron pipe there are one open globe valve (K = 10), one 45° regular elbow (K = 0.75), and one pipe bend with a radius of curvature of 40 in (K = 0.10). The bend is 90°, and its ons san FLOW IN CLOSED CONDUITS 7 249 length is not included in the 50 ft. No entrance or discharge losses are involved. Ifthe fluid is water at 72°F and the velocity is 6 fps, what is the total head loss? ! Aeshy the hy =(PLIa\u*?28) Ng duly = (4)(6)/(1.02 x 10°) = 1.96% 10°“ ¢/d = 0,00015/() = 0.00045 From Fig. A-S, f = 0.0185, L=50+HQ\IDIH=S.2A hy =0.0185155.2/(4 (6 M2V32.2)}) = 1.71 ag = (K)(07/2g) = (10 + 0.75 + 0.10)(64/[2)32.2))) = 6.07 hy = 1.714.607 = 7.788 ‘Compute the friction head per 1000 ft of pipe for laminar flow at Reynolds number 50 000 (the empirical upper limit). Consider two situations: one where the fuid is water at 60°F, the other where the fluid is SAE10 oil at 150°F (v = 0.00016 ft/s). Pipe diameter is .0in, 1 y= (MLIA) W124) f = 64/Nq = 64/50000= 0.00128 Ne =dv/v Foe water: $0000 = 2.0/12\uy/(1.21 x10") v= 3.6308/8 ‘hy = 0.00128{1000/(2.0/12)](3.630"/1(2)(32.2))) = 1.57 50000 = (2.0/12)(v)/0.00016 v= 48.00 ft/s, hy = 0.00128{1000/(2.0/12)]{ 48.00" /{(2)(32.2)]} =275 te Repeat Prob. 9.223 ifthe flow is turbulent in a smooth pipe. Hy = (L/d)(v"12g). From Fig. AS, f = 0.0207. or water: From Prob. 9.223, v = 3.630 ft/s. hy = 0.0207{1000/(2.0/12)]{3.6307/[(2)(32.2)}) = 25.4 ft. For oll: From Prob. 9.223, u = 48.00 ft/s. fy = 0.0207[1000/(2.0/12)]{ 48.00" /[(2)(32.2)]} = 4443 f. Repeat Prob. 9.223 ifthe flow is turbulent in a rough pipe with «/d = 0.08. Hb, = (f(L1d)(u" 2g). From Fig. AS, f = 0.072. For water: From Prob. 9.223, v = 3.630 ft/s. hy = 0.072{1000/(2.0/12)]{3.6307/[(2)(32.2)]) = 88.4 ft. For olt: From Prob. 9.223, v= 48.00 ft/s, fh, = 0.072{1000/(2.0/12)}(48.00*/[(2)(32.2)]} = 15.455 #. Water at 60°F flows through 10 000 t of 12-in-diameter pipe between two reservoirs whose water-surface ‘elevation difference is 200 ft. Find the flow rate if € = 0.0018in. t Pily + vil2g + 2 =pily + vY2gt mth, 0+0+200=0+0+0+h, Ay = 200 ft = hy = (f)(L/d)(v?/2g) = f[10 000/(8)]{v7/[(2)(32.2)]} = 155.3fv? 20= 155.307 v= 1A35/VG Try f= 0.03: v= 1.135/VO0B = 6.553 ft/s, Ne = du/v = (H)(6.553)/(1.21 x 10%) = 5.42 x 10%, /d = 0.0018/12 = 0.00015. From Fig. A-5, f = 0.015. Try f = 0.015: v = 1.135/V0.015 = 9.267 ft/s, ‘Ny = (B)(.267)/(1.21 x 10°) = 7.66% 10, f = 0.0145. Try f = 0.0145: v = 1.135/V0.0145 =9.426 ft/s, ‘Na (H)(9.426)/(1.21 x 10-*) =7.79 x 10%, f =0.0145. (O.K.); = Av = [(=)(B)'/4](9.426) = 7.40 fs. Repeat Prob. 9.226 if e is twenty times larger than in Prob. 9.226. H v= 1.135/VF (from Prob, 9.226). Try f = 0.03: v= 1.135/ VOUS = 6.553 f/5, Na = dv/y = (8)(6.553)/(1.21 x 10%) = 5.42 x 10°, e /d = (20)(0.0018)/12 = 0.00300. From Fig. A-S, f = 0.027. ‘Try f = 0027: v = 1.135/V0.027 = 6.907 f/s, Na = (B)(6.907)/(1.21 x 10-*) =5.71 x 10%, f = 0.027 (O.K.); = [(=)(8)°/4]6.907) = 5.42 ft"/s. Q=av 250 0 CHAPTER 9 9.228 How large a wrought iron pipe is required to convey oil (sg. 9.229 9231 9.232 = 0.0008 Ib s/f’) from one tank to another ata rate of 1.0 cf if the pipe is 3000 ft long and the difference in elevation of the free liquid surfaces is on? ' pily + vig tam pily + VU2g+nth, 0+0+40=040404h, fy 4Oft= hy = (PLId\(v"/28) v= Q/A=1.0/(aed*/4) = 1.273 /* ‘hz = (F) 3000/4) ((1.273/a°F/2)G32.2))) = 75.49f [a> 40=75.49f/d° d= 1.135 f° ‘Try f = 0.03; d = (1.135)(0.03)!* = 0,5629 ft, Ne = pdv/u = [(0.9)(1.94)](0.5629)(1.273/0.5629°)/0.0008 = 4936, «fd = 0,00015/0,5629 = 0.00027. From Fig. A-5, f = 0.0375. Try f = 0.0375: d = (1.135)(0.0375)"* =0.5886 ft, ‘Na = [(0.9)(1.94)](0.5886)(1.273/0.5886?)/0,0008 = 4720, f = 0.038. Try f = 0.038: d = (1.135)(0.038)"* = 0.5901 ft, Na = [(0.9)(1.94)](0.5901)(1.273/0,5901°)/0,0008 = 4708, f = 0.038 (O.K.). Therefore, d = 0.5901 ft, oF 7.08in, Ifthe diameter of a pipe is doubled, what effect does this have on the flow rate for a given head loss if the flow is laminar? ' he = 32)(v)(Ligd?\(u) = (constant)(v/a?) v= kd? Q=Av=k'd* ‘Thus, doubling the diameter will increase the flow rate by a factor of 2', oF 16. Ifthe diameter of a pipe is doubled, what effect does this have on the flow rate for a given head loss if the flow is turbulent? 111 =constant (complete turbulence): hy = kyv"/d, v= kad", Q= will increase the flow rate by a factor of 2°, or 5.66. For smooth pipe with Ny < 100 000:f = 0.316/N3%, h, = (f)(L/d)("/2g) = (ky)(v"4/d%), v= kad”, Q=Av=kyd™”. Thus, doubling the diameter will increase the flow rate by a factor of 2”, of 6.56 nd. Thus, doubling the diameter ‘A 150-mm-diameter pipeline 100 m long discharges a 50-mm-diameter jet of water into the atmosphere at a point 60 m below the water surface at intake. The entrance to the pipe is a projecting one, with K = 0.9, and the nozzle loss coefficient is 0.05. Find the flow rate and the pressure head at the base of the nozzle, assuming f= 0.03. 1 pily + Wil2g-+ 2,= poly + vi/2g + 20th o hy = (PMLId)(v"[2g) = 0.031100/0.150]{0*/[2)(9.807)}} = 1.0200" hn = (KY(U*I28) = (0.9)(v° 29-507) + 0.05){ vz (2)(9.907)]} = 0.045890" + 0,00254905. 1h, = y+ hg = 1.0200? + 0.085890 + 0,0025490, = 1.0660 + 0,0025490,, 0+0+60=0+ uja/[(2)(9.807)] + 0+ 1.0660" + 0.025490}. Since velocity varies asthe square ofthe diameter, vjg = (2)°(v) = 9u, 60 (9u)/[2)(9.807)] + 1.0660" + (0.002549)(9u), v = 3.333 m/s; Q = Au = [(x)(0.150)"/4](3.333) = 0.0589 m/s. Applying Eq. (1) to the nozzle, paJ9-79 + 3.333*/[(2)(9.807)] + 0= 0 [(9)(3.333)F /[(2)(9.807)] + 0 + 0.05(((9)3.333)F/12)(9.807)}), pr= 46.9kP a, ‘A.2.0-m-diameter, 1600-m-long concrete pipe (€ = 1.5 mm) carries water at 12°C between two reservoirs at 8.0m’/s, Find the difference in water-surface elevation between the two reservoirs, considering minor losses at entrance and exit. ! pal + Wiig +25 paly + 2g + 2h, em hy tlhe y= (FLIA\(U*/28) v= Q/A=8.O/{(x)(2.0/4]=2.546m/s__Nq= du/v = (2.0)(2.546)/(1.24 x 10-9) = 4.11 10° «/d=0.0015/2.0 = 0.00075 From Fig. A-S, f= 0.0185. y= (0.0185)(1600/2.0){2.546*/[(2)(9.807)]} = 4.89 m a= (RY(0*/2Q) = (05 + 1.0)(2.5464/1(2)(9.807))) = 18940.50=5.39m 04042 =040+2+539m 9.233 934 925 9.236 9231 9.238 FLOW IN CLOSED CONDUITS 0 251 A pipe of mean diameter 5ft and length 6000 ft delivers water to a facility 1300 ft below the water surface at intake. Assume f = 0.025. When the pipe delivers 300 cfs, what is the horsepower delivered? ' ¥2= Q/Ar= 300/[()(8)/4] = 15.28 ft/s ‘hy = (PML 1d)(v"[2g) = 0.025{6000/ ((9)](15.28°/l(2)(32.2)}} = 108,76 ft hg = (Kv 2g) = 0.50(15.28"/[(2)(32.2)}) = 1.81 ft a, = hy + hg = 108.76 + 1.81 = 110.57 ft P= Qy(Az ~h,) = (300)(62.4)(1300 ~ 110.57) = 2.227 x 10" ft Ib/s= (2.227 x 10°)/550 = 40.491 hp Find the kilowatt loss in 500 m of S0-cm-diameter pipe for which ¢ =0.05 mm when dye at 45°C (s.g. = 0.86, v= 4.4% 10° ft/s) flows at 0.22 m/s. Neglect minor losses. PML/av?/2g) v= Q/A = 0.22/f(x)(B)'/4] = 1.120 m/s Ng = du/y = (0.50)(1.120)/(8.4% 10°) =1.27 x10" fd = (5x 10-)/(5 x 10") = 0.0001 From Fig. A-S, f = 0.018. hy = 0.018{500/0.50]{1.120°/[(2)(9.807)]} = 1.151 m, P = Qyhy = 0.22{(0.86)(9.79)](.151) = 2.13 kW. ' y= Linseed oil, of kinematic viscosity 0.0005 ft/s and weight density 59.8 Ib/ft, is pumped through a 3-in pipe (€ =0.001 in). (a) At what maximum velocity would the flow still be laminar? (b) What would then be the loss in energy head per 1000 ft of pipe? HG Assume laminar flow exists for Ny = 2000. Ny = v/v, 2000 = (2)(v)/0.0005, v = 4.00 is. (0) f= 64/Nq = Sig = 0.032 hy = (F)(L/d)(v/2g) = 0.032{1000/( 3) }{4.00/{(2)(32.2)]} = 31.80 ft = (69.8)(31.80)/144 = 13.2 psi per 1000 ft Pp Repeat Prob. 9.235 ifthe velocity is three times the maximum velocity for laminar flow. 1 v= (3)(4.00)=12.00f/s hy =(F{LIa)(v"/28), [Nq = du/v = (3)(12.00)/0.0005 = 6000 ¢/d = 0.001/2 = 0.00050 From Fig, A-5, f = 0.036. +, = 0.036{1000/ 33)}(12.007/{(2)(32.2)]} = 322.0 P= Yh, = (59.8)(322.0)/144 = 134 psi per 1000 ft Water flows upward at 3 m/s through a vertical 150-mm-diameter pipe standing in a body of water with its lower end 1.0 m below the surface. Considering all losses and with f = 0.022, find the pressure at a point 3m above the surface of the water. 1B pily + viltg + 2.= pil + vi[2g +.2,+ hy. Let point 1 be atthe water surface and point 2 be 3m above the water surface, hy = hy + hy, hy = (f)(L/d)(v"/2g) = 0.022[(3 + 1.0)/0.150]{3*/1(2)(9.807)]} = 0.269 m, hn = (K)(v'/2g). For entrance fos, assume K = 0.8. fn =0.8(3"/[(2)(9.807)}) = 0.367 m, fh, = 0.269 + 0.367 = 0.636 m, 0+ 0-+0= p3/9.79 + 3'/(2)(9.807)] +3 + 0.636, p= ~40.1 KPa. Work Prob. 9.237 ifthe flow is downward. ! pay + vil2g + 2. pil + vile + 2,4 hy, hy =0.269m — (from Prob.9.237) hw For exit loss, assume K = 1.0 hg = 1.0(3/[2)(9.807)]} =0.459m fh, 0.269 + 0.459 = 0.728m 29.79 + 34/(2)(9.807)] +3=0+0+040.728 — p,=—26.7kPa (KY? /2g) 252 0 CHAPTER 9 9.20 9240 9.281 9.282 9.083 ‘A horizontal pipe 20m in diameter and for which f = 0.030 projects into a pond 1 m below the surface. Considering all losses, find the pressure at a point 41m from the end of the pipe if the low is at 3 m/s from the pipe into the pond, Wily + vil2g+ 21 paly + v3/2g + 2,+ hy. Let point 1 be 4m from the end ofthe pipe and point 2 be atthe water surface. f= hy + my My = (f)(L/d)(v*/2g) = 0.030 4/0.20](3"/[(2)(9.807)}} = 0.275 m, Jan = (K)(0/28). For exit loss, K= 1.0. hy = 1.0(3°/[2(9.807)]} = 0.459 m, hy, = 0.275 + 0.459 = 0.734 m, i/9.19 + 3°/[(2N9.807)] + 0=0-+0+ 1 +0.734, p, = 12.5 KPa, Repeat Prob, 9.239 if the flow is from the pond into the pipe. ' paly + 02g +22= ply + vilag + thy h iy tn 1275m (fom Prob. 9.239) ha = (KY(v"/26) For entrance, K=0.8. hn = 0.8(5°/[(2)(9-807)]} = 0.367m hy = 0.275 + 0.367 0404 1= p4/9.79+3°/[2)9.807)] +0+ 0.642 ‘A pipe runs from one reservoir to another, both ends of the pipe being under water. The intake is ‘nonprojecting; the length of pipe is 480 f; its diameter is 10.25 in; and the difference in the two water levels is 106 ft. Iff = 0.02, what will be the pressure at a point 300 ft from the intake and 120 ft below the surface of the water in the upper reservoir? 1 ily + vile +21 pily + U2g +2 thy, o ‘Let points 1 and 2 be at the water surface in the upper and lower reservoirs, respectively. hy = ({MLla)(v? (2g) = 0.02{(480)/(10.25/12)H{v"/[(2)32.2)]} = 0.17450" ‘i = (KY(07/2g) = (0.5 + 1.0}{v*/[(2)G2.2)]) = 0.023290" 1745 u? + 0.023290" = 0.19780? 040+ 106=0+0+0+0.19780 vm 23.15 fis ha hy the [Now apply Eq. (1) between the upper reservoir (point 1) and the point 300 ft from the intake (point 2). iy = 0.024(300)/(10.25/12)}(23.15°/{2(32-2))} = 58-46 a = 0.5(23.15%/[(2)G2.2)]) = 4.16 ny = 58.46 + 4.16= 62.624 0+ 04120 (p,)(144)/62.4 + 23.157/[(2)(32.2)] + 0+ 62.62 pa =21.3Ib/in? ‘A 9.S:in-diameter pipeline runs from one reservoir to another, both ends being under water, and the intake end is nonprojecting. The difference in water levels between the two reservoirs is 110‘t, and the length of pipe is 1000 ft. What isthe discharge if f= 0.062 1 pily + vil2g + 2,= pal + Ug + 2+ hy (A LId){v? 12g) = 0-06{(1000)/(9.5/12)v°/{(2)(32.2)]) = (0.5 + 1.0){0"/1(2)(32.2)]} = 0.023290" 17690 a = (KY(U7/28) .1769u? + 0.023290" = 1.2000? 0+0+ 110. +0404 1.20007 uth va9SThis — Q=Av=[(x)(9.5/12)/41(9.57) = 4.71 10/5 A jet of water is discharged through a nozzle at a point 200 ft below the water level at intake. The jet is 4 in in diameter, and the loss coefficient of the nozzle is 0.15. Ifthe pipeline is 12in in diameter, 600 ft long, with a 9.24 9.246 FLOW IN CLOSED CONDUITS J 253 ‘nonprojecting entrance, what isthe pressure at the base of the nozzle? Assume f = 0.0125. 1 pil + vRl2g + 2,= poly + VU 2g + 2th, o ny = M(L/d)(¥2/2g) = 0.0125{600/(13)}(v2/[(2)(32.2)]} = 0.11650 ha = (KY(0" 2g) = 0.507 2NB2-2)} + (0.15) vpall(232.2))} = 0.007764 + 0,0023250 n= hy + hy = 0.11650" + 0.007640" + 0.00232902, = 0.12430" + 0.0023290%, 040+ 200=0+ viu/(2)(32.2)] + 0+ 0.12430" + 0.002329, Since velocity varies withthe square of diameter, Ujq = ()*(v) = 9v, 200 = (9v)?/{(2)(32.2)] + 0.12430? + (0.002329)(9v)?, v = 11.28 ft/s. Applying Eq. (1) to the nozzle, (p,)(144)/62.4 + 11.287/[(2)(32.2)] + 0= 0+ (9911.28) /12)(32.2)] + 0 + (0.15){L)(11.28)F 12)G2.2))}, ps = 78.9 bin” Compute the losses due to flow of 25 m"/min of air, p = 1 atm, 7'= 20°C, through a sudden expansion from 300-mm pipe to 900-mm pipe. How much head would be saved by using a 10° conical diffuser? I For sudden expansion: 4. = {1 — (D,/D,)"P(W3/24), 1 = Q/As = B)/(() (388) /4] = 5.895 m/s, ‘a= [1 — (BBY (5.895°/{(2)9.807)]} = 1.400 m. For conical diffuser: h,, = 0.152{(v, — v3)*/2g], vs = O/Az = (B)M()(GE)*/4] = 0.655 m/s, + = 0.152 (5.895 ~ 0.655)'/[(2)(9.807)]} = 0.213 m. Saving in head = 1.400~0.213= 1.187m or 1.187N-m/N or 1.1875/N Calculate the value of H in Fig, 9-47 for 125 L/s of water at 15°C through commercial stee! pipe. ' pily + ViI2g+ 2= ply + Ug + mth, hrs hy tlm hy = (PM L/ANv7/28) = Q/A = (125 x 107)/[()(0.30)"/4] = 1.768 m/s Ng = dv/v = (0,30)(1.768)/(1.16 X 10-4) = 4.57 x 10" €/d =0,000046/0.30 = 0.00015 From Fig. A-S, f = 0.015 +h, = 0.015{50/0.30(1.768°/[(2)(9.807)]} = 0.398 m fa = (K)(v*/2g) = (0.5 + 1.0){1.768"/[(2)(9.807)]} = 0.239 m_ hy, =0.39840.299=0.637m 04042504042 40687 —2 1.637 m In Fig. 947, for H =3m and a fluid with s.g, = 0.8 and 1 = 0.007 Pa -s, calculate the discharge through smooth Pipe. f pil + vil2g +2." poly + vie + 2+ hy ‘hy = (FMLIA)(02 2) = (F)150/0-30](¥*/12)(9-807)]} = 8.497f* rn = (K)(0?/28) = (05+ 1.0){07/[2).807)]) = 0.076880? = hy + he = BAIT O" + 0.076480" 04043—0+040+8497f0" + 0.076480" v= VIIRATF + 0076S) ‘Try f = 0.02: v = VIIEATTHO.OD) + O.G7EAB] = 3.489 m/s, Nq = pdv/ = [(0.8)(1000)|0.30)(3.489)/ QUGra LI Bom Fag Af -O0NT yf =O, 0 Va EEO FUME ~ 26x 10%, f =0.017 (O.K.); Q=Av= 3.685 m/s, Nx = {(0.8)(1000)|(0.30)(3.685)/0.007 [()(0.30)'/4}(.685) = 0.260 m/s. 254 0 CHAPTER 9 9.247 Evaluate K for a valve that, placed in the line in Prob. 9.246, would reduce the discharge by 50%. 9.248 9.249 9.250 92st I From Prob. 9.246, H = 3 = [(/)(50)/0.30 + 0.5 + 1.0+ Kye} ((3.685/2)"/[(2)(9.807)}}, Ke = 15.83 ~ 166.7f, Nx = pdv /u = {(0.8)(1000)|(0.30)(3.685/2)/0.007 = 6.32 x 10*, From Fig. AS, f = 0.0197. Ku 15.83 ~ (166.7)(0.0197) = 12.5. ‘A waterline connecting two reservoirs at 70°F has 4860 ft of 24-in-diameter steel pipe, three standard elbows (=0.5), a globe valve (K = 10), a re-entrant pipe entrance (K= 1.0), and a submerged pipe exit (K = 1.0). ‘What is the difference in reservoir elevations for 20 cfs? ! Duly + vig +2 = paly + vide + ath, heh; thm y= (fMLId)(0"/28) v= Q/A=20/{(™)(H)/4] = 366 ft/s Nu= dv /v = (#)(6.366)/(1.05 10-*) = 1.21 x 10° é/d =0.00015/(3) = 000075 From Fig. A-S,f =0.013. ‘hy = O.013{4860/ (3) {6.366 /[(2)(32.2)}} = 19.88 ft ‘hn = (KY(U7/28) = [(3)(0.9) + 10 + 1.0 + 1.0}{6.366"/{(2)(32.2)]} = 9.25 ft 19.88 +9.25=29.13f 0+0+=04042 429.13 3-2 =29.13f For the conditions given in Prob. 9.248, determine the discharge ifthe difference in elevations is 40 ft. From Prob, 9.248, 40 hy + ha = (0/[(2)32.2)}){(/)14860/()] + [G)(0.9) + 10 + 1.0 + 1.0)}, 1 AISTONU AT) yf 2001 oe WS AOU 7 sot, Ne dal (4)(7.46)/(1.05 x 10°*) = 1.42 x 10%, €/d =0,000075 _(from Prob. 9.248). From Fig. A-5, f =0.0125. Try f = 0.0125: v = V2576/[(2430)(0.0125) + 14.7] 4)(7,560)/(1.05 & 10 14.4 x 10°, f = 0.0125 (O.K.); Q = Au = [(a)(#)7/4](7.560) = 23.8 ft/s. ‘What size commercial steel pipe is needed to convey 200 L/s of water at 20°C a distance of 5000 m with a head ‘drop of 4 m? The line connects two reservoirs, has a re-entrant (K = 1.0), a submerged outlet (K = 1.0), four standard elbows (K =0,9), and a globe valve (K = 10). 1! pily+uil2g tna pily+ui2gtnth, OF0+4=040404h, hy V= QA = (78)/(d"/4) =0.2546/4 hy = (FY L/d)(v*/2g) = (F (5000/d){ (0.2546 a") [(2)(9.807)]} = 16.52f fd ha = (KY(U"/28) = [1.0-+ 1.0 + (4)(0.9) + 10}{(0.2546/4")?/(2)(9.807)}} = 0.05156/d* 4= 16.52f a? + 0.05156/a* ‘Try f = 0.02: 4 = (16.52)(0.02)/d° + 0.05156/d*. By trial and error, d = 0.619 m. 2546 0.619" = 0.6645 m/s Nq= dv/v = (0.619)(0.6645)/(1.02 x 10°) = 4.03 x 10° «e/d = 0.000046/0.619 = 0.000074 0145, Try f = 0.0145: 4 = (16.52)(0.0145)/a + 0.08156 /d", d = 0.588, v = 7364 m/s, Nq = (0.588)(0.7364)/(1.02 x 10"*) = 4.25 x 10%, f= 0.0145. Therefore, d= 0.588m, or $88 mm, Find the equivalent lengths of 167-mm-diameter pipe, f = 0.024, for (a) a re-entrant pipe entrance (K = 1.0), (6) a sudden expansion from 167 mm to 334 mm diameter, and (e) a globe valve and a standard tee? 1 Le=KDIE @ (1.0)(0.167)/0.024 = 6.94 Oy K=[1~-(D/D,)P =[1- (FP = 0.5625 L, = (0.5625)(0.167)/0.024 = 3.91 m © K=10+18=118 — L,=(11.8)(0.167)/0.024=81.9 m 9.282 9.283 9.284 9.285 FLOW IN CLOSED CONDUITS J 255 Find H in Fig, 9-48 for 200 gpm of oil (= 1 cP, y = 60 lb/ft’) for the angle valve wide open (K = 5.0). 1 pily + vig +2= pay + UHR + rath, he hy thm hy = (F(LIA\(v"/28) = (200(0.002228) = 0.4456f'/s v= Q/A = 0.4456/[()(4)'/4] = 9.078 ft/s By Prob. 9.40, 4 = 0.002089 Ib s/t; Na = (y/g)(dv)/1 = (60/32.2)(i)(9.078)/0.0002089 = 2.02 x 10", <€/d = 0,00015/(¢) = 0.000600. From Fig. A-5, f = 0.0275. sy =0.0275{212/(3)9.078*112)G2.2)]} = 29.84 ft nn = (K)(v"/2g) = [0.5 + 5.0 + 1.0](9.078"/[(2)(32.2)}) = 8.32 t bh =T844832= 38.16 04042, 20404243816 ym e Hm 3816 ft Find K forthe angle valve in Prob. 9.252 for a flow of 10L/s at the same H. 1 u = QUA = [(s)/0.3048° (7) (i3)"/4] = 7.194 ft/s Ne = pdu/u = (yg)(du)/4e = (60/32.2)(7)(7-194)/0.0002089 = 1.60 x 10" €/d = 0.000600 (from Prob. 9.252) From Fig. A-5, f =0.0285, From Prob. 9.252, H = ((f)(Li/d) + Ky + Ka+ K:|(v"/2g), 38.16= {(0.0285)(212/()] + 0.5 + K, + 1.0} (7.198 /[(2)32.2)]}, K2=21.8. Calculate the discharge through the system of Fig. 9-48 for water at 25°C and H = 8m. ' puly + vile + In British Engineering units, we have = IML IA og) = 22MM AVS22)N hn = (KY(07/2g) = (0.5-+5.0+ 1.0)(v2/[2)(32.2))) =0.10080" hy 11fe? + 0.10050" 040+ 8/0,3088-=04+0-+0-+ 13.17f0°+ 0.100502 w= VI6ISTTR I ¥ 0.1009) Try f = 0.02: v = ¥26.25/{(13.17)(0.02) + 0.1009] 489 ft/s, Nx = dv/v = (j3)(8.489)/(9.56 x 10° 2.22 10°, €/d = 0.00015/( 3) = 0.000600. From Fig. A-S, f = 0.019. Try f = 0.019: v= ‘V26.25/{(13.17)(0.019) + 0.1008] = 8.646 ft/s, Ne = (3)(8.677)/9.56 x 10) =2.27 x 10%, f= 0.019 (OK); O= Av = [()(3)°/418. 646) = 0.4244 f0/s = (0.4244)(0.3048)’ = 0.0120 m/s, oF 2.0L. poly + vile + 22+ hi Find the discharge through the pipeline in Fig. 9-49 for H = 10 m, as shown. Use minor loss coefficients for the entrance, elbows, and globe valve of 0.5, 0.9 (each), and 10, respectively. ' Pal + vig + 2,= aly + vY/2e + 2+ hy di, = (ULL) (v2 g) = 10 + 12+ 60)/ (48K) WFMLQ)(9-807)]} = 34.67f03 ng = (K)(v/24) = [0.5 + (2(0.9) + 10}(v/[2)9.807))} = 0.627103 i + hg = 3461fuE+ 0.627103 0-4 0+ 10= 0+ vi/(2)(9.807)] +0+ 34.67/03 + 0.627103 ViI0/G4.67F + 0.6781) du/v = (7i)(2.700)/(1.02 x 10°*) ‘Try f = 0.02: v = V10/[(34.67)(0.02) + 0.6781] 3.97 x 10°, €/d = 0.00026) (tin) 00173. From Fig. A-5, f = 0.023. Try f = 0.023: ‘Vi0}[($4.67)(0.023) + 0.6781] = 2.603 m/s, Nq = (if)(2-603)/(1.02 X 10-*) = 3.83% 10", (O.K.); Q = Av = [(7)(R)'/4](2.603) = 0.0460 m’/s, or 46.0 L/s. 256 0 CHAPTER 9 150.mm-dham clean east won pipe H=10m Sua: edged entance Fig. 9-49 9.256 Rework Prob. 9.255 to find H if = 60L/s. ' Pily+vi2g+2,=prly + vig t nth, hrahythe — hy=(f\LId\(u"/2¢) w= QUA = (s85)/[()(488)"/4] = 3.395 m/s Ng = du/'v = (1H8)(3.395)/(1.02 x 10-4 €/d=0.00173 (from Prob. 9.255) 99 10° ‘hy = 0.0225|(30 + 12 + 60)/(15%)]{3.395°/[(2)(9.807)]} = 8.99 m in = (KY0?/2g) = [0.5 (2)(0.9) + 10](3.3957/1(2)(9.807)]} = 7.23 m 89947.23=16.22m — 040421 =043.395/(2)0.807)] +4162 4-4 =H=16.81m 9.287 Assume that water at 10°C is to be conveyed at 300 L/s through 500 m of commercial steel pipe with a total head drop of 6 m. Minor losses are 12v?/2g. Determine the required diameter. 1 Nq= RJD w lin (Ry/D + RaINRYP @ Z=Re+R DIF @) D= (Rat + Ra o where Ro = (0.66)(€'*Q"*)"%, Ry = v/e™Q", Ry= 5.74, Ry= €/3.7, Re= K/ghy, Rs=4Q/av, Re= Ligh, Ry= 1.325, € = 0.000046 m, Q = 300L/s, oF 0.300m'/s, v= 1.30 x 10-6 m’/s. K=12 y= 6mm ——_Ry=(0.66)(0.000046'0,300"4)°* = 0.25351, Ry = 1.30 x 10°4/(0.000046'0.300°") = 0.38707 Ry = 0,000046/3.7 = 1.2432 x 10-* Rex 12/[(9.807)(6)] = 0.20394 R= (4)(0.300)/[()(1.30 x 10-4] = 2.9382 10" R= 500 /[(9.807)(6)] = 8.4973 ‘Assume D = 1 m. Substituting into Eqs. (1), (2), (3), and (4), [Ny = 2.9982 X 10°/1 = 2.9382 x 10? f= 1.325/{Im (1.2432 x 10°°)/1 + 5.74/(2.9382 x 10°)°"}? = 0.014938, += 8.4973 + (0.20894)(1)/0,014938 = 22.150 D-= (0.25351)(22. 150" + (0.38707)(22.150"))° = 0.47418m ‘Try D = 0.47418 m: Nq = 2.9382 x 10°/0,47418 = 6.1964 x 10°, f= 1.325/{ln (1.2432 x 1079)/0.47418 + 5.74/(6.1964 x 10°)"7]}? = 0.014086, x = 8.4973 + (0.20394)(0.47418)/0,014086= 15.363, D = (0.25351)f15.363** + (0.38707)(15. 363°?) = 0.44063 m. Try D = 0.44063 m: Nx = 2.9382 x 10°/0.44063 = 6.6682 x 10%, f = 1.325/{In [(1.2432 x 10"7)/0.44063 + 5.74/(6,6682 x 10°)°*]}? = 0.014075, x = 8.4973 + (0.20394)(0.44063)/0.014075 = 14.882, D = (0.25351)|14.882*” + (0,38707)(14,882°*)]*° = 0.43783 m. Try D = (0.43783 m: Nq = 2.9382 x 10°/0.43783 = 6.7108 x 10°, f = 1.325/{In [1.2432 x 10-*)/0.43783 + 5.74/(6.7108 x 10°)°*})? = 0.014074, x = 8.4973 + (0.20394)(0.43783)/0,014074 = 14.842, D = (0.25351)[14.842"" + (0.38707)(14.842°*))°° = 0.43759 m. Therefore, D = 0.438 m, or 438 mm. 9.258 One equation for determining the fiction factor is 1/Vf=0.869 In (NqVf) ~ 0.8. Compare the smooth pipe curve on the Moody diagram (Fig. AS) with results from the equation above for values of Reynolds number of 10°, 10°, and 10”, = 10%, from Fig. A-5, = 0.0178, From the equation, 1/V7=0.869{In [10° Vf)]} ~ 0.8. By tril and error, f = 0.0183. For Nq = 10°, from Fig. A-5, f = 0.0116. From the equation, 1/Vf=0.869{in [(10*)(Vf)]}) — 0.8. By trial and error, f = 0.0116. For Nz = 10’, from Fig. A-5, f = 0.0082. From the equation, UVF =0.869{In (107(Vf)}} ~ 0.8. By til and errr, f= 0.00810. 9.289 9.200 9.261 9.262 9.268 9.264 FLOW IN CLOSED CONDUITS 0 257 ‘An equation for determining the friction factor developed by Colebrook is 1/Vj = ~0.869 n (¢/D)/3.7-+ 2.523/(NaVf)]- Check the location of line €/D = 0.0002 on the Moody diagram (Fig. A-5) with the equation above for a Reynolds number of 10° From Fig. A-5, f = 0.0190. From the equation, 1/Vf = ~0.869 In {0,0002/3.7 + 2.523/{(10°)(VA)]}- By trial and error, f = 0.0192. Find the head loss ina pipeline consisting of 200ft of 4-n steel pipe, a 90° bend on 24-n radius (K = 0.15), 4in gate valve (fully open) (K = 0.20), 100 fof 4in stel pipe, expansion to 6in with a 20° taper (K'=0.4), 300 ft of Gin tel pipe, abrupt contraction to 3. diameter (K=0'35), and 50 tof 3.in steel pipe. The discharge rate is 15efs. 1, (f)(L1d)(v"/28), hy = (K){0?/2g). For 200 ft of 4in pipe: v Ng = duy/v= (4)(47.19)/(1L05 x 10") = 5.46 x 10%, €/d = ‘hy = 0.0175{200/(8)]{17.19°/[(2)2.2))) = 48.2 ft For bend: h,, =0.15(17.19/[(2)(32.2)]} = 0.78 For gate valve: fy = 0.20(17.19°/[(2)32.2)]} = 0.9 ft For 100 ft of -in pipe: fh, =0.0175{100/(4)}{37.197/((2)(32.2)]} = 24.1 8. For expansion: 5/l()(8)/4] = 7.639 fs, hy = 0.4(17.19 ~ 7.639) /[(2)(32.2)]} = 0.6, For 300 fof 6-in pipe: Nx = (#)(7.639)/(1.05 x 10%) = 3.64 x 10°, €/d = 0.00015/(#) = 0.00030, f = 0.0170, ‘i, = 0.0170{300/ (3) {7.639"/{(2)(32.2)}} = 9.2 ft For abrupt contraction: vs =1.5/{(7)(i)'/4] = 30.56 ft/s, ha = 0.35(30.567/{(2)(32.2)]} =5.1 ft. For 501 of 3in pipe: N= ()(30.56)/(1.05 x 10-4) =7.28 x 10°, €/d = 0.00015/(i) = 0.00060, f = 0.0180, hy = 0.0180(50/ (&)]{30.56"/|(2)G2.2)]} =52.2 ft, hy = 48.2 +0.7 + 0.9 + 24.1 +0.6+9.2+ 5.14 522= 141.08. A= 1.5/{()(4)"/4] = 17.19 /s, (00015/(i$) = 0.00045, From Fig. A-5, f = 0.0175, Using the Darey-Weisbach formula, find the head loss in 1000 tof 6-t-diameter smooth concrete pipe carrying Sct of water at 50°F. t hy = (f(LIDYu"/2g)__-v = QA = 80/{(x)(6)"/4] = 2.829 ft/s [Ng = Du/ = (6)(2.829)/(1.40% 10"4) = 1.21 x 10" €/D =0.001/6 = 0:000167 From Fig, A-5,f = 0.014 hy = (0014)(288)(2,829%/((2)(32.2)]} = 0.290 ft Solve Prob, 9.261 using the Manning formula ! 10 = (1.486/n)(R)"(6)!2=2.829 ft/s (from Prob. 9.261) n= 0.013 (from Table A-13) .486/0.013)(1.500)°%(8)"* 5 = 0.003567 hy }.0003567)(1000) = 0.357 fe Solve Prob. 9.261 using the Hazen—Williams formula, ' v= 1.3I8CR's"* = 2.829 ft/s (from Prob. 9.261) C=120 (from Table A-14) R= 1.500ft (from Prob. 9.262) 2.829 = (1.318)(120)(1.S00)°%)°™ 5 = 0.003618 fy = (1000)(0.0003618) = 0. Using the Darcy-Weisbach formula, find the head loss in 100 ft of 3-t-diameter welded steel pipe carrying 1S cfs of water at 60°F. ' hy = (PMLID\v?2g)— v= Q/A = 15/{(=)(3)*/4] = 2.122808 Nu = Dv/v = (3)(2.122)/(1.21 x 10-* .26 x 10° €/D =0.00015/3 = 0.000050 From Fig. A-5, f = 0.0135. hy = (0.0135)(1%!){2.122/{(2)(32.2)}} = 0.031 ft 258 0 CHAPTER 9 9.268 9.266 9.267 9.288, 9.20 9270 9m 92m 9.273 9274 9.208 Solve Prob. 9.264 using the Manning formula with n = 0.012. ' 1.486 /n)(RY"(6)""= 2.122 ft/s (from Prob. 9.264) R=D/4=2=0.7500f 2.122 = (1.486/0.012)(0.7500)""(6)" ‘iy = (0.0004309)(100) = 0.043 ft 0.004309 Solve Prob. 9.264 using the Hazen-Wiliams formula with C = 120. 1 .SISCR?“S°=2.122ft/s (from Prob. 9.264) .7500ft (from Prob. 9.268) 2.122 (1.318)(120)(0.7500)°(s)" (0004769, = (100)(0.0008769) = 0.048 ft A.36-in-diameter concrete pipe is 4000 ft long and has a head loss of 12.7 ft. Find the discharge capacity of water for this pipe by the Hazen-Williams formula ! BI8CR°Ms** = (1.318)(120){(3)/4]° (12.7/4000)° = 5.906 fe/s Q= Av = [(-)(H8)/4)(5.906) = 41.7 0/5 Solve Prob. 9.267 using the Manning formula. ! v= (1486/m)(RP%)" Q=Av= 1.486/0.013){(8)/4P (12.7 /4000)"" = 5.317 f/s 9) (8) /4].317) = 37.6 F075 ‘A L-m-diameter new cast iron pipe is 845 m long and has a head loss of 1.11 m. Find the discharge capacity of water for this pipe by the Hazen Williams formula. ' v= 0.8492CR® "4 = (0.8492)(130}(4)°(L 11/845)" = Q= Av = [((1)'/41(1.281) = 1.01 m/s 281 m/s Solve Prob. 9.269 using the Manning formula. ! 10 (1.0/n)(R)*%6)"2 = (1.0/0.013)(2)*9(.11/885)"? = 1.106 m/s Q = Av =[()(1)"/4](1.106) = 0.869 m"/s ‘A riveted steel pipe must transport 2.4 ft/s of water a distance of 190 ft with a head loss of 2.7 ft. Find the necessary pipe diameter using the Hazen-Williams formula. ! = 1.3I8CR*S** = Q/A =2.4/(xD*/4) =3.056/D* 3.056/D? = (1318) 110)(D/4°"(2.7/190" — D=0,7695f or 9.23in ‘A square concrete conduit must transport 4.0 m"/s of water a distance of 45 m with @ head loss of 1.80 m. Find the necessary conduit size using the Hazen—Williams formula, Hv = 0,8192CR°*s**. Leta = length of conduit side. v = Q/A = 4.0/0, 4.0/a*= (0.8492)(120)(@?/4a)?*(1.80/45)°*, @ = 0.788 m. Solve Prob. 9.267 using the Hazen-Williams pipe diagram. Let hy = unit head loss. hy = 12.7/4000 = 0.003175. From Fig. A-13 with h, = 0.008175 and D =3 ft, or 36in, Q=415'/s, Solve Prob. 9.269 using the Hazen-Williams pipe diagram, Fh =1.11/845 = 0.001314, From Fig. A-14 with hy = 0.001314 and D = 1m, or 100mm, Q =0.91 m*/s. This value of @ is for C= 120 (since the pipe diagram is for C = 120). Inasmuch as C= 130 for new cast iron (Table ‘A-I4), this value of Q must be adjusted. Since Q varies directly with C, 0.91/(Q)c-im0= #8, (Q)e-130™ 0.99 m/s, ‘Water is lowing in a 500-mm-diameter new cast iron pipe at a velocity of 2.0 m/s. Using the Hazen—Williams pipe diagram, find the pipe friction loss per 100 m of pipe. 976 san samy 9.280 FLOW IN CLOSED CONDUITS 1 259 1 C=130 for new cast iron pipe. In order to use the pipe diagram (Fig. A-I4) for which C= 120, the given velocity must be adjusted for a value of C of 120. Since velocity varies directly with C, 2.0/(v)caim0= Hb; (erm = 1.85 m/s. From Fig. A-14 with v = 1.85 m/s and D = 500 mm, hy = 0.0067 m/m, or 0.67 m per 100m. p= ph = (.79)(0.67) = 6.6KPa per 100”. ‘Annew cast ion pipe must cary 30cfs of water at ahead loss of 19.0 per mile of pipe length. Find the required pipe diameter using the Hazen-Williams pipe diagram, I hy =19.0/5280 = 0.003598. C = 130 for new cast iron pipe. In order to use the pipe diagram (Fig. A-13) for which C = 120, the given discharge must be adjusted fora value of C of 120. Since discharge varies directly with ©, 30/(Q)c-s0 = #8, (Q)emio0= 27. Tels. With Q = 27.7 cfs and h, = 0.003598, a pipe diameter of 30in is ‘determined trom Fig. A-13 Solve Prob. 9.270 using the Manning pipe diagram with n = 0.012. y= 1.11/845 = 0.001314. With D = 1 m, or 1000 mm, and hy =0,001314, a discharge of 0.88 m"/s is determined from Fig. A-16. Inasmuch as Fig. A-I6 s for n =0.013 and nis 0.012 inthis problem, this discharge ‘ust be adjusted. Since discharge varies inversely with 1, 0.88/(Q),-nora =0.012/0.013, (Q)nuaoa=0.95 m"/8. A concrete pipe must carry 80 cfs of water at a head loss of 1.5 ft per 100 ft of pipe length. Find the required pipe diameter using the Manning pipe diagram. 5/100 = 0.015. With Q = 80 efs and h; = 0.015, a pipe diameter of about 36 in is determined from Solve Prob. 9.278 for the'same given conditions except that the pipe has an n value of 0.015. 1 h,=0.015 (from Prob. 9.278) ‘The Manning pipe diagram (Fig. A-15) is for a value of n of 0.013. The given discharge (80 cfs) is for a value of n of 0.015. In order to use Fig. A-15, the given discharge must be adjusted for a value of n of 0.013. Since discharge varies inversely with n, 80/(Q),q201s = 0.013/0.015, (Q)y-nors = 92.3 cfs. With Q = 92.3 cfs and ‘hy =0.015, a pipe diameter of about 38 in is determined from Fig. A-15. ‘A capillary tube of inside diameter 6 mm connects tank A and open container B, as shown in Fig. 950. The liquid in A, B, and capillary CD is water having a speciic weight of 9780 N/m? and a viscosity of 0.0008 kg/(m 8). The pressure p, = 34.5 kPa gage. Which direction will the water flow? What isthe flow rate? Assume laminar flow from B to A. Vig oy PA MA a, 128uL-Q _ (128)(0.0008)(4.3)(Q) y 2g ty tg tea the Dt (H)9780) Ea)" 0404 (1.44 4.3 sin 45") = 34,5/9.79+0+1 + 110580 Q=-755x10'm'/s or -7.55% 10° L/s (7.55 Since Q is negative, the flow must be from A to B. Check for laminar flow: v 1997 (laminar). 1. 10°*)/{(2)(xtia)*/4] = 0.2670 m/s, Na = pDu/y = (9780/9.807)(zths)(0.2670)/0.0008 14am Fig. 9.50 260 0 CHAPTER 9 9.281 9.282 9.283 9.288 ‘An equivalent length of pipe is one whose head loss for the same value of flow is equal to that of some other system of different geometry for which it is equivalent. Consider a steel pipe of inside diameter 10.02 in having init an open globe valve (K = 4.8) and four screwed 90° elbows (K = 0.42 each). The length of the pipe is 100 ft and 5 ft'/s of water at 60°F flows through the pipe. What is the equivalent length of pipe with inside diameter 13.00in? Hor the given pipes f= hy + has hy = \L/d)( 012g), v = Q/A = 5/[(x)(10.02/12)/4] = 9.131 fs, Ng = dV /v = (10.02/12)(9, 131)/(1.21 % 10°) = 6.30 x 10°, €/d = 0.00015/(10.02/12) = 0.00018. From Fig. A-5, f = 0.015. fy = 0.015{100/(10.02/12)]{9. 1317/{(2)(32.2)}} = 2.33 ft, hy, = (K)(v?/2g) = [4.8 + (4}(0.42)}(9.13°/(2)G2.2)}) = 8.39, = 2.38 + 8.39 = 10.72. For the equivalent pipe: v= 5/{()(13.00/12)?/4] = 5.424 ft/s, Nq = (13.00/12)(5.424)/(1.21 x 10~ 10%, e/d = 0.00015/(13.00/12) = 0.000138, f = 0.015, 10.72 = 0,015[L, /(13.00/12)](5.4247/[(2)(32.2)1}, Le= 1695 ft ‘A 100-f-Long duct transports sir at 50°F (assumed constant) atthe rate of 8000 ft"/min. The duct is made of talvanized iron and has a cross section of 2t by ft. Find the outlet pressure, ifthe inlet pressure is 16.7 psa. I Assume incompressible, isothermal, fully turbulent flow. pily + vil2g+2,= poly + vig tah, hom hy = (ALIA M028) Ne= edule P= PIRT = (16.7)(144)/[(1716\460 + 50)] = 0.002748 slug/t?” —_ y = pg = (0.002748)(32.2) = 0.08849 Ib /f? d= Dy=4Alp.=AQMIVAVZ+ D=1.333 v= Q/A= CBPY/I(2\(1)] = 66.67 fe/s (0.002748)(1.333)(66.67)/(3.68 x 10°”) = 6.64 10° (O.K.)_¢/d =0,0005/1.333 = 0.000875 Ne From Fig. A-S, f =0.0165. Jn, = (0.0165)(100/1.333) 66.67 1(2)32.2)]) = 85.43 ft ‘p,/0.08849 + vi/2g + 0 = p./0.08849 + vi/2g + 85.43 VE2g=v2g — pi~pp=7S6Ib/R? py = 16.7 -7.56= 9.1 pia Ina heating system, there is a run of insulated duct of 50 m carrying air at a temperature of 35°C at a pressure at the inlet of 100 kPa. The duct has a rectangular cross section of 650 mm by 320 mm. If there is a pressure drop from inlet to outlet of Simm of mercury, what is the volumetric flow? For such a small pressure drop in the duct, treat the flow as incompressible. Take R = 287 /(kg K). The duct is galvanized iron. Consider that the flow is entirely turbulent. ' pile + vi/2¢ +2 =paly + vi/2g+ 2th, hem hy=(F(L/A\v7I2)—Nx= pdvl P= pIRT = (100)(1000)/{(287)(273 + 35)] = 1.131 kg/m? y= pg = (1-131)(9.807) = 11.09 N/m? d= Dy =4A lp. = MCS) BSI QN(H + #2B)] = 0.4289 my, = (F(50/0.4289){v?/[(2)(9.807)}} = 5.944f0? UY2g + 2+ he Pe P= rdsl(13.6)9.79)] = 0.6657 kPa oF 665.7Pa pul poly + vi/28 +2 Vig =v/2g — 665.7/11.09+0—0+5.944f0> v= 3.178/VF ‘Try f=0.016: v=3.178/VOOT6 = 25.12 m/s, _Nq= (1.131)(0.4289)(25.12)/(1.88 x 10-9) = 6.48 10", €/d= (0.00015/0.4289 = 0.000350. From Fig. A-5, f = 0.0165. Try f = 0.0165: v = 3.178/V0.0165 = 24.74 m/s, ‘Nx = (1.131)(0.4289)(24.74)/(1.88 x 10™*) = 6.38 10%, f = 0.0165 (O.K.); Q = Av = {(8)(i85)|(24.74) = S.15m'/s. ‘Water at 60°F flows through a smooth pipe of 12in diameter at 9f’/s. Estimate the shear stress at the wall and the thickness of the viscous sublayer. ' v= Q/A=9/{(a\(BP/4] = 11.46 ft/s to = 0.03525 pu*(v/ Ru)" = (0.03325)(1.94)(11.46)*{(1.21 x 10°*)/[()(11.46)])" = 0.323 lb/ft? 81= Sv /vy = SvVp/t9= (51.21 x 10°*)VI,9470.323 = 0.000148 ft or 0.00178 in i | t t | t 9.286 FLOW IN CLOSED CONDUITS 0 261 Solve Prob. 9.284 if the shear stress is given by ty = (f/4)(pv"/2). ' Nu = dv/v = (3)(11.46)/(1.21 x10) = 9.47 x 10° From Fig. A-5, f = 0.0118. to= (0.0118/4)[(1.98)(11.46)*/2] = 0.376 Ib/f 6, = Sv/v, = SvVp/ = (5)(1.21 x 10-*)V1.94/0.376 = 0.0001374 ft or 0.00165 in ‘What head H is needed in Fig. 9-51 to produce a discharge of 0.3 m'/s? Use minor loss coefficients of 0.5 forthe entrance and 0.12 for the diffusers. W pal + VAG + 24= Dol + UNI2G+ tt hy I= hy thm — hy = (PULIAv*28) —Na=dv/(ulp) Y= Q/A, = 0.3/{(7)(0.210)'/4] = 9.549 m/s__(Nq) = (0.2103(9.549)/{(0.04)0.1)/1000] = 5.01 x 10° From Fig. A-S, f:= 0.013, U2 Q/Aa=0.3/{()(0.305)/4] = 4.244 m/s (Naa = (0.305)(4.244)/{(0.04)(0.1)/1000} = 3.24 x 10° fi=0.014 hy = 0.013130/(0.210)]{9.549°/[(2)(9.907)]} + 0.014{60/(0.305)] (4.244"/[(2)(9.907)}} = 11.64 m For entrance: fh = (K)(07/2g),(ha)s= 0.5(9.5492/[(2)(9.907)]} = 2.32:m, or frst diffuser: = (K)(0y —¥3)*/28, (Ih): ™ (0-12)(9.549 ~ 4.244)*/[(2)(9.807) Foc econd ter: v= Q/As= O:3/(5)(0.450//4]= 886m ha) = (0428.24 — 18867100.) = For exit: (n)4= 1.0{1.886'/[(2)(9.807)]} = 0.18m, Hh, = 1164 + 2.32 + 0.17 + 0.03 + 0.18 » 14.34, ‘Square edge Smooth ipe Fig. 51 Calculate the discharge through the siphon of Fig. 9-52 (H = 4ft) without the conical diffuser. Use minor loss coefficients of 1.0 for both the entrance and exit, and 0.9 for each elbow. ' Daly +a/2g +24 Daly + Vol2@ + 20+ he hy = PULIA) 0" (2g) = f{A0 + 18+ 129/21 (0A)E22)} = 0.9817/0* ag = (K (02/24) = [(.0 + (20.9) + 1.0}{02/12)G2.2)]} = 0.059010" hy = hy + hn = 0.9317fu" + 0.059010" 0+0+4=0+040+40.9317fu7+0.05901u? v= V4/(O.9317F + 0.05901) ‘Try f = 0.016: v = VAI(O.931 710.016) + 0.05901] = 7.356 ft/s, Na = dv = ({5)(7.356)/(1.21 x 10°*) = 4.05 x 105, ¢/d = 0.00015/(%) = 0.000225. From Fig. A-5,f'= 0.016 (O.K.). Q= Av = [(a)(8)"/410.356) = 2574s. _ Standaedeloow — x D ~ 8.n-dam Stee’ poe 262 0 CHAPTER 9 9.288 Calculate the discharge in the siphon of Fig. 9-52 for H = 8 ft and with diffuser included, Use minor loss coefficients of 1.0 for both the entrance and exit, 0.9 for each elbow, and 0.15 for the diffuser. ! aly + Va/2g + 24= Pal + Vb/2g +25 hy 0 ty thy y= FMEIA)(Y/25) = F110 + 18 + 12)/E)MV/(Q)32.2))) = 0.9317f4 te (K)(v?/2g) for entrance, exit, and elbows. h, = (Kv, — v:)/2¢ for diffuse. AW = Avs — (AEFI) = (COLNE) v2 = 0.44440, ‘hn = [1.0 2) O.9)I(WiN|2N32.2)) + 0.15(05 ~ 0.4444) 2)(32.2)} + 1.0((0.44440 9*/1@)G2.2))} = 0.047260 hy = 0.9317fvj + 0.047260; 0+ 0+ 8=0+0+0+0.9317fv7 + 0.04726" v= VENOITF + 0.04726) Try f = 0,015: v = VET{OLDSIT)O.OIS) + 0.04725] = 11.43 ft/s, Ny = du/y = (§)(11.48)/(.21 x 10 6.30 x 10°, €/d = 0.00015/(j5) = 0.000225. From Fig. A-! 1.0155: v ‘VaI{(0.9347)(0.0155) + 0.08726] = 1.39 fs, Ny 28x10", ‘f=0.0155 (O.K.); = 98 fs. 9.289 Find the discharge through the siphon of Fig. 9-53, as well as the minimum pressure inthe system. Use minor Joss coefficients of 1.0 fr the entrance, 0.1 forthe nozzle, and 2.2 for the bend F Let v= velocity of water inthe pipe and v, = velocity through the nozzle. Since velocity varies with the square of diameter, v:=4v. pil + Vile + 2.—paly + Re + 25+ hy o diy = MLA) W128) = f1(6-+ 1.8 + 3.69/0.100](0"/[2)9.807))} = 5.812F0* in = (KYU 12g) = (1.0 + 2.2){02VO.807)}) + 0.1 v3/(2N9-807)]} = (1.0+2.2)(v'NN2N9.80) + (0.1 (C4v)°12VO.807)) = 0.24477 i + hg = 58128 + 0.24470 040+ (6-1.8)=0+ (4v)°(2)9.807)] + (6-18-3.6)+ 5.8130" + 0.24470? v= VEGIERII + 1.060) Try f = 0.03: v = VIBE B12 O03) + 1.06] = 1.708 ms, Ne = pd/ = [(0.81)(1000)](0.100)1.708)/0.01 = 1,38 x 10%. From Fig. A, = 0,028, Try f =0.028: v = V3.6/{@ 812)(0.028) + 1.060] = 1.716 m/s, Ne = {(0.81)(1000)}(0. 100)(1.716)/0.01 = 1.38 x 10°, f = 0.028 (O.K.); Q = Av =[()(0.100)7/4](1.716) = 0.0135 m’/s, or 13.5 L/s. Minimum pressure would occur at point 3. Apply Eq. (1) between points 1 and 3: 0.02360. 100](1.716/[(2(9.807)}} 0.2522 m, hy = (1.0-+2.2)(1.716/[(2\9.807)}) — 0.4804 m, hy 0.2522 + 0.4804 = 0.7326 m, 0+0+1 13/{(0.81)(9.79)] + 1.716*/{(2)(9.807)] + 1.8 + 0.7326, py = —21.3 kPa. Close return ben 10 me dam ‘moo Bbe Ftuidsye 08% ee olpas 150 mm sommdam nome ng, 9.53 9290 9a 9.298 9.298 FLOW IN CLOSED CONDUITS 9 263 With reference to Fig. 9-54, what is the maximum height of point 4 for no cavitation? Barometer reading is 29.5in of mercury. Use minor loss coefficients of 1.0 for both entrance and exit, and 4.5 for the globe valve. I First calelate the flow velocity v Duly + vil2g + 2.= ply + V3/28 + 22+ hy o hy = (F(L1d)(v"[2g) = F100 + 70 + 105)/(8)Iv/(2)132.2))) = 6.405fu® Jing = (KY(0?/2g) = (1.04 4.5 + 1.0){v?/[(2)G2.2)]} = 0.100007 hy = hy + hy = 6AOSf" + 0.10090" 0-404 12=0+0 +0 + 6.405f0" + 0.10090" v= ViZ/(6.405F + 0.1009) Try f = 0.013: v = VI2I[@-M5)(0.013) + 0.1009] = 8.072 t/s, Ny = dv/v = (8)(8.072)/(1.21 x 10-*) = 4.45 x 10°, From Fig. A'S, f = 0.0135, Try f = 0.0135: v= Vi2/{(6 405)(0.0135) + 0.1008] = 8.003 ft/s, {8)(6.003)/(1.21 x 10-4) = 4.41 10%, f 0.0135 (O.K.). Now, to find the maximum height of point A apply Eq. (1) between points 1 and A: hy = 0.0135{100/()](8.003"/[(2)(32.2)]} = 2.014 fh = 0, f= 2.014-+0=2.014 ft, From Table A-L, py = 36.510/E, Pam = [(13.6)(62-4)](29.5/12) = 2086.2 Tb/f. At cavitation, p = (2086.2 ~ 36.5)/62.4 = 32.85 ft (vacuum), oF 32.85 ft, 0+ 0-+ 0 = —32.85 +0,995 + 2, +2014, 247 29.81. (za isthe height of point A above the water surface in the left reservoir.) = = - B.insdameter smooth pipe Se WWatera oF Fig. 9-54 ‘What diameter smooth pipe is required to convey 8 L/s of kerosene (v = 1.93 x 10°* m"/s) at 32°C, 150m with ‘ahead of Sm? There are a valve and other minor losses with total K of 7.6. ' ily + Vile + 21=paly + UP2G + 2th, v= QA = (adn)/(xD*/4) = 0.01019/D* hip = (PME Id)(W?28) = (F1050/D){(0.01019/D")/[(2)(9.807)}} = 0.0007941f/D* Jin = (K)(0"/2g) = 7.64(0.01019/ D*¥/[(2)(9.807)}} = 0.00004023/D* hip = hy + hy = 0.0007941f /D* + 0,00004023/D py +24 — pal — 22 vi/2g = vi/2g ‘5 = 0.0007941f/D* + 0.00004023/D* ‘Try f = 0.02: 5 = (0,0007941)(0.02)/D* + 0.00004023/ 4, By trial and error, D = 0.0826 m. Ne = Du/y = (0.0826)(0.01019/0.0826/(1.93 x 10°) = 6.39 x 10%, From Fig. A-5,f = 0.0195. Try f = 0.0195: (0.0007981)(0.0195)/D* + 0.00004023/*, D = 0.0822.m, Ne = (0.0822)(0.01019/0.0822°)/(1.93 X 10-4 6.42% 10, f = 0.0195, Therefore, D = 0.0822 m, or 82.2 mm. Find the value of the Hazen-Williams coefficient for the water flow in Prob. 9.130. ' L3IBCR® SS" = Q/A =3.2/{(a)H)/A] = 1.0186 ft/s 1.0186 = (1.318)(CMC)/41° (0.00081) 5.5 Find the value of the Hazen—Williams coefficient for the case where water flows at 0.20 m"/s in a L-m-diameter pipeline with a head loss of 0.0012 m/m. ' y= 0.8492CR°™s" = Q/A = 0.20/{(-)(1)*/4] = 0.2546 m/s 0.2546 = (0.8492)(C)[1/4)°°(0.0012)"* C= 27.1 Manning's n equals 0.0132 when water at 50°F flows at a Reynolds number of 1.10 x 10” through a tunnel that is 16 tin diameter. Determine the average value of Fon Same (cae) 264 0 CHAPTER 9 Egquating sand hy/L gives mJ a(t oor (ra) - (OE aaa) ~(slagaa} 208 ).00240 fr, or 0.0288in, 1.0128 and enr 1, ed 00015; « = (16)(0.00015) From Fig. A-S with 9.295 Prepare a computer program that will determine for a single pipe with constant diameter either the flow rate or the required conduit diameter for closed conduit, incompressible flow. The program must be usable for problems in both the USCS and the SI. {THIS PROGRAM DETERMINES THE FLOW RATE OR THE REQUIRED CONDUIT DIAMETER FOR CLOSED CONDUIT, INCOMPRESSIBLE FLOK. IT CAN BE USED FOR PROBLEMS IN BOTH THE ENGLISH SYSTEM OF UNITS AND THE TRTER- LIMITED 70 CASES INVOLVING A SINGLE PIPE WITH CONSTANT DIAMETER. {THIS PROGRAM 1S BASED ON THE SOLUTION OF THE BERNOULLT EQUATION; ACCORDINGLY, CERTAIN DATA HUST BE ENTERED FOR EACH OF WO POINTS. 1A PIPE SYSTEM. HOWEVER, OTH THE VELOCITY AT POINT 1 AND THE VELOCETY AT POINT 2 ARE NOT CONSIDERED AS AON VALUES INITIALLY. IP EITHER VALUE OF VELOCITY I5 ESSENTIALLY ZERO (SUCH AS AT A RESERVOIR OR TANK SURFACE) , ENTER O (ZERO) FOR THAT VELOCITY (OR FOR BOTH VELOCITIES IF BOTW ARE ESSENTIALLY ZERO). OTHERWISE, INPUT DATA WRIST BE SET UP AS FOLLOKS. caRD 1 comme 1 JBeTER 0 (ZERO) OR BLAM IF ENGLISH SYSTEM (OF UNITS 5 TO BE USED. ENTER 1 (ONE) I DESIRED. CARD 2 COLUMNS 1-10 BNTER NUMBER INCLUDING DECIMAL CIVING (GAUGE PRESSURE AT POINT 1 (IN POUNDS PER SQUARE INCH OR KILOPASCALS) . CouMS 11-20 ENTER NUMBER INCLUDING DECIMAL GIVING (GAUGE PRESSURE AT POINT 2 (IN POUNDS PER SQUARE. INCH OR RILOPASCALS) « Ccoumats 21-30 ENTER O (ZERO) OR BLANK TP VELOCITY AT Ccovvneis 31-40 ENTER 0” (ZERO) OR BLAME TP VELOCETY AT POINT 2 15 ESSENTIALLY ZERO; OTHERNISE, ENTER 1.0. ENTER WMBIER INCLUDING DECIMAL, GIVING ELEVATION AT POINT 1. (10 FEET OR METERS) . [BWTER MUMBER INCLUDING DECIMAL GIVING ELEVATION AT POINT 2 (30 FEET OR METERS) . Couviais 61-70 ENTER NUMBER INCLUDING DECTRAL GIVING [ACTUAL EMERGY ADDED (IN HORSEFOWER OR KILOWaTS) Covers 71-80 ENTER NUMBER INCLUDING DECIMAL GIVING ACTUAL ENERGY REMOVED (IN HORSEPOWER OR KILOWATTS) (caRD 3 coLUINIS 1-10 ENTER NUMBER INCLUDING DECIMAL GIVING ‘TOTAL "MINOR LOSSES" (IN FEET OR METERS) . (COLUMNS 11-20 ENTER NUMBER INCLUDING DECIMAL GIVING DIAMETER OF CONDUIT (IW INCHES GR MELLaMETERS) COLmS 21-50 ENTER NOMDER INCLUDING DECIMAL GIVING LENGTH OF CONDUIT (IN FEET OR METERS). Coneeis 31-40 ENTER RUWER INCLUDING DECIMAL GIVING XINERATIC VISCOSITY OF FLUID” (IN SQUARE PEET PER SECOND OR SQUARE METERS PER SECOND) . {EATER NOMBER INCLUDING DECIMAL GIVING ROUGHNESS OF CONDUIT MATERIAL (IN FEET OR METERS). ENTER A VALUE CP 0 (ZERO) FoR "SMDOT" CONDUITS. cous FLOW IN CLOSED CONDUITS 0 265 coLMNS $1-60 XTER NUMBER INCLUDING DECINAL GIVING ‘SPDCIFIC (OR UNIT) WEIGHT OF FLUID (IN POUNDS PER CUBIC FOOT OR KILO~ IMEWTONS. PER CUBIC METER). cous 61-70 ATER NUMBER INCLUDING DECIHAL GIVING PLOW RATE OP FLUID (IN CUBIC FEET PER SECOND OR CUBIC METERS PER SECOND). ccaRD 4 comtweis 1-24 ENTER TYPE OF FLUID COLURIS 25-88 ENTER TYPE OP CONDUIT. NOTE WELL... EITHER THE FLOW RATE (COLUMIS 61-70) OR THE CONDUIT DIAMETER. (COLUMS 11-20), WHICHEVER ONE 1S TO BE DETER- MINED BY THIS PROGRAM, SHOULD BE’LEFT BLANK ON CARD 3. MOUTTPLE OATA SETS FOR SOLVING ANY NUMBER OP PROBLEMS YAY BE INCLUDED FOR PROCESSING. DIMENSION TITLE(13) ,FLUID(4) ,PEPECA) SNTEGER UNITS 14159265, or FoRMAT(! 1", 13R6,///) FREAD(5, 102)P1,2,V1,V2,21,22,HA,MR, 1M,D,1,VIS,E,SW,Q,FLUID, PIPE 102 FORMAT (GF10.0/7P10.0/8K6) PISW=P1/SH*PACTOR*=2 Te (UNITS. bp. 1)PISW=PI/SH Ge32.2 48 (UNITS. 89. 1)6°9.807 25w=P2/Sw*FACTOR=*2 EP (UNITS. Bp. 1) P25W=P2/SH #r=0,02, TF (Q.6.0.0001)00 70 117 IP (Vi-GF.0.0001) vi=1.0/2.0/6 AP(2.Gr-0,0001)¥2=1.0/2.0/6 105 n= HAT=550,*HA/SH 2 (uNITS.ED. 1) HATOHA/SH HR =550,*HR/SH IP (UNITS. BD. 1) HRT=HR/SH 40.001 ‘YeR=(Q/(PI* (0/FACTOR) **2/4.01) 942 [ERY LaPLow: VERE *VL 92 -HAE /Q-HRC /Q- (P2SW+VTRYAV24226HF VERY) a6 Qe9%0:001 ‘Yon (Q/(Pr* (O/eACTOR) *#2/4.0)) 42 ‘TRY2-P SW VTREAVI421 4HAT/Q-HRT/Q- (B2SWOVTRYAV2"22¢HE*VTRY) Ee CaRYLOTRY?) 114,114,115, 115 TRvi=rRy2 70 116 114 9+9-0.0005 ag/ (Pr#(0/eACTOR) **2/4.0) uso /FACTOR"V/VIS ED-E/DePACTOR DEFF=aBS(e-FF) Te(DEFP.17,0,00019C0 70 104 > 70 105, 104 IF(V1-cr-0.0001)¥1 SP (v2.6r.0.0001)v2=¥ TeCONTTS,89.0)HRITE(6,106)P2 22,71 ,22,HA,1R,NM,D,L,PLUID,PIPE,2, wiv 106 Pofaat (1X, "GIVEN DATA FOR A CIRCULAR CLOSED CONDUIT CARRYING TNCOM ‘SPRESSEBLE FLOW" //5K, "PRESSURE AT POINT 1 =" F7.1," PST! ,//9K," PRE 266 0 CHAPTER 9 . sure AT POINT 2 =1,P7.1,* P51", //SX, ELEVATION AT POINT 1 =", SPV.1, FD" //5K, "ELEVATION RT POINT 2 21,8701," FT", //5K,ACTAL E SNERGY ADDED BETWEEN POINTS 1 AND 2 =" 85/1," fi //5K, ‘ACTUAL ENER GY RENOVED BETWEEN POINTS 1 AKD 2.=",F5.i,1 HP", //5X, "MINOR HEAD 1 sOS5ES BETWEEN POINTS 1 AND 2.=",PS.1,? ET", //SK, °DIAMETER OF CONDU SIT #*,76.2," 10° ,//5X, "LENGTH OF CONDUIT =!,F0.1,° FT", //5X, “FLUID Srudwing 8 yanb,//9k, *CONDUIT MATERIAL, 18°" ,8KG,////48, "HE. PLOW RATE WILL BE!,P7.1," CU P/S" //Sk," VELOCITY AT POINT 1'=",6.2, Fr/S" //5k, "VELOCITY. AP POINE 2 =! P6.2," FT/8") ae UNETS 9. {)WRETE(G, 107) P22) 122, HA AR HM, DL PLUTO PIPE, ova 107 FORWAT! LX, ‘GIVEN DATA FOR A CIRCULAR CLOSED CONDUIT CARRYING ICOM ‘SPRESSIBLE FLOW //5K, PRESSURE AP POINT 1° F7.1," EPA", //5K, "PRE SSSURE AT POLNT 2 =°)57.1," KPA',//S%, ELEVATION RE POINT 12", SETSU 0M 1 //5K, RLEVARION AB POIND 2 =1GE7.10" M1 4//5K, ACTUAL E SWERGE ADDED BETWEEN POINTS 1 AND 2 =*,P5.1," Ki)" ,//5X, ACTUAL ENER SOY REMOVED BETWEEN POINTS 1 AND 2 =" 25.1, KR", //5K, “MINOR READ 1 sOSSES BETWEEN POINTS 1 AND 2 =",PS.1," M "4 //SK, “DIAMETER OF CONDU AYP 21 97.3," MH, //5%, "LENGTH OF CONDUIT = F804," "4 //SK, “FLUID S FuawING 1S" yane,//5, (CONDUIT MATERIAL 18°" 46, ////0, PE. FLOM RATE WILL BE ,P7.3,* CU M/S" 4//5K, "VELOCITY AT POINT 1 "46.2, SW/s "4 //9x, WELOCETY AT POINT 2 °,F6.2," W/S.") 117 vievis(g/Pre4.09 v2ev2"(Q/PI+4.0) 103 wesPeL#(g/21"4.0) HAT=550. *HA/SH/Q 2F (ONITS.BQ. 1) HAT=HA/SW/O RT=550. +HR/SW/Q IF (ONITS. BQ. 1)URT=HR/SH/Q 0.001 [TRY =HF/0*45» (V2442/2.0/6) /0¥*4>(VI4#2/2.0/6) /D**4-PISH-2I-HAT 110 DeD0.001 "TRy2F/D**5¢(V2442/2.0/6) o**a~(V1442/2.0/6) /D**4-PISH-Z1-HAT snnnep2eeea2si IPUTRYI*7RY2} 208, 108, 109 og tevi=r¥2 70 110, 108 B-0-0.0005 Ru=D*Q/ (PI*D"*2/4.03 VIS ED-E/D ‘CALL Rov DiFF-Ans(P-FF) IF (DIFP-L-0.0001)60 70 311 Fee fo 103 nin viewisorez vaev2/or"2 IF (WUITS.e9.0) RITE (6,112) PL ,P2, 21,22, HA, HR HM, 0,1 FLUID, PIPE,D, 112 FORWAT(IX,"GEVEN DATA FOR A CIRCULAR CLOSED CONDUIT CARRYING INCOM “PRESSIGLE’ FLOW", //5K, "PRESSURE AT POINT 1 =1,F7.1,* PSE", //3%, PRE SSSURE AT POINT 2 =*,£7.1," PSI',//5K, ELEVATION AP POINT’? SPT, Fy //5K, RUEVATION RE POINT 2 £8,87.1," FE //5K, ACTUAL € SNERGY ADDED BETWEEN POINTS 1 AND.2 =" PS.1," HDs //5k, "ACTUAL 2NER SO REMOVED BETWEEN POINTS 1 AND 2 =°,P5.1," MP" ,//5E, MINOR LOSSES {BETWEEN POINTS 1 AND 2 =',€5.1," FT! //5%, ‘Flow RATE =",£7.1, 5 U FT/S", //5K, LENGTH OF” CONDUIT ="/PB.1," FP", //5k, 'PLUID FLOWE NG 15 *)4R6, //Sk, ‘CONDUIT MATERIAL 15"*,4N6,///1%,°THE CONDUIT DIA METER REQUIRED WILL BE" 6.2," 0",//5K, "VELOCITY AE POINS 1 =", 2/2.0/6 986.2," FT/S", //Sk, "VELOCITY AT POINT 2 "476.2," FT/S") TF (UTS. ED. 1) WRITE(6,113)P1,P2,21 22, HA, HRM, QL, FLUID, P2PE,D, 113. FORMAT, ‘GIVEN DATA FOR A CIRCULAR CLOSED CONDUIT CARRYISG INCOM ‘SPRESSIBLE FLOW ,//SK, "PRESSURE AT POINT T=! GPT. 1," RAN 4s 3X, "PRE 9SURE AT POINT 2 4*,87.1," XPR",//5K, "ELEVATION AT POINT’ |<", SPT1/ 9 M7 //5k ELEVATION AT FOIE 2 9°)87.1," H°//5% ACTUAL E ‘SERGE ADDED BENWEEN POINTS 1 AND 2 ="PS/1,4 R#',//SR, ‘ACTUAL EER GY RENOVED BETWEEN POINTS 1 AND 2 =",F5.1," KW", //5X,/MINOR LOSSES BETWEEN POINTS 1 AND 2 = P5.1,1 5,//5K, "PLOW RATE =",77.3, SUNS ",//5K, "LENGTH OF” CONDUIT =*/P0.1," 6" //5%, FLUID FLOM FLOW IN CLOSED CONDUITS J 267 suc 15 + 4N6,//Sk, "CONDUIT MATERIAL 18 *4N6,///1X, "THE COMDOTT DIA ‘METER. REQUIRED WILL BE*,F7-1," MM", //5K, "VELOCITY AT FORT t=", $76.2," M/S. //SE, VELOCIPY AP POINT 2 =*,76.2,' M/S.) SUBROUTINE ROUGH TF (R4.LE.2000.01P=64.0/701 IF (RN-LE,2000.0)RETURN IF (RUT, 4000.0) WRITE(6, 103) 103 FORAAT(LK,"® REMIOLDS NUMBER 1S IN THE CRITICAL ZONE, FOR WSICH TH ‘HE PRIGTION PACTOR 18 UNCERTAIN. HENCE, PROGRAN EXECUTION 1AS TERA vmware.) IP (RN. LP. 4000.0)T0P 006 TRY +1. O/SQRE(P) #2.0*RLOG10(#0/3.742.51/R1/SQRT(F) 10.0000 "TRY2=1.0/SQRE(F) #2.0*RL0610 20/3. 7+2.51/RW/SQRTCE)) EP (aRYi*TRE2) 100, 100,101 101 TRvi=TRY2 70 102 100 FeF-0.000005 102 92% For the data given in Prob. 9.65, find the flow rate at which water is being discharged from the pipe, utilizing the computer program of Prob. 9.295. INPUT OSAMPLE ANALYSIS GF INCOMPRESSIBLE FLOW 1.0 150.5 98.4 24.0 130.0 0.000105 0.00085 62.4 waver NEW CAST IRON oureur SAMPLE ANALYSIS OP INCOMPRESSIBLE PLOW GIVEN DATA FOR A CIRCULAR CLOSED CONDUIT CARRYING INCOMPRESSIBLE FLOW PRESSURE AT POINT 1 = 0.0 PST PRESSURE AT FOIWT 2 = 0.0 FSI ELEVATION AT POINT 1 = 150.5 FT LEVATION AT POINT 2 = 98.4 FP ACTUAL BNERGY ADDED BETWEEN POINIS 1 AND 2 = 0.0 HP ACTUAL EWERGY REAOVED BETWEEN POINTS 1 AND 2 = 0.0 HP MEWOR HEAD LOSSES BETWEEN POINTS 1 AND 2 = 0.0 FT DIAMETER OF CONDUIT = 24.00 IN LewarH OF cowDurT = 130.0 FF FLUID FLOWING 15 MATER [THE FLOW RATE WILL BE 127.0 CU FT/S vevocrty an POINT 1 = 0,00 F1/S, veLociTY ar POINT 2 = 40.44 F1/8 268 0 CHAPTER 9 9.2977 For the data given in Prob. 9.66, find the pipe diameter, utilizing the computer program of Prob. 9.295. INPUT ISAMPLE ANALYSIS QF INCOMPRESSIBLE FLAW 25 1.0 1.0 82.65 66.66 965.5 .000000406.00050 7.05, dO 5 GASOLINE (ROUGHNESS = 0.500 o) ourruT GIVEN DATA FOR A CIRCULAR CLOSED CONDUIT CARRYING INCOMPRESSIBLE FLOW PRESSURE AT FONT 1 = 2.5 KPA PRESSURE AP POINT 2= 0.0 KPA RLEVATION AD POINT 1 = 02.7 H ELEVATION AT POINT 2-= 6.7 4 ACTUNL ENERGY ADDED BETWEEN POINTS 1 AND 2 = 0.0 KH ACTUAL, ENERGY REMOVED BETWEEN FOINTS 1 AND 2 = 0.0 XW MINOR LOSSES BETWEEN FOIWTS 1 AND 2= 0.0 H FLOW RATE = 0.100 CU M/S LENGTH OF CONDUIT = 965.5 FLOID FLOWING 15 GASOLINE (CONDUIT MATERIAL 15 (ROUGINESS = 0.500 mx) {WE CONDUIT DIAMETER REQUIRED WILL BE 257.5 mt veLocity ar PoIWr 1 = 1.92 m/s vewociTy ar pomNt 2+ 1.92 w/s ma 2 04 [] CHAPTER 10 Series Pipeline Systems Fora 12in-diameter concrete pipe 12000 ft long, find the diameter of a 1000-t-long equivalent pipe I Assume a flow rate of 3.0 cfs. (The result should be the same regardless ofthe flow rate assumed.) From Fig. Ac, with D = 12in and Q-= 3.0¢'/s, b= 0,0052,/f. Therefore, hy = (0.0052)(12 000) = 62.40 ft. For a 1000-f-1ong equivalent pipe with the same head loss, h, = 62.40/1000 = 0.06240 ft/ft. From Fig, A-13, with ‘n,=0.06240 f/f and Q=3.0f¢/s, D=7.3in. ‘A.480-flong, 18in-diameter concrete pipe and a $90-t-long, 12-in-diameter conerete pipe are connected in series. Find the length of an equivalent pipe of iin diameter I Assume a flow rate of Sefs through the two given pipes. For the 18-in-diameter pipe, from Fig. A-13, 0.00180 ft. For the 12in-diameter pipe, hy = 0.0137 f/f. The total head loss for both pipes is (0,00180)(480) + (0.0137)(590) = 8.947 ft. For a 10-in-diameter pipe with Q = 5.0cfs, from Fig. A-13, 4h, =0.032 60. Since the equivalent pipe must have the same head loss as that ofthe system it replaces (ie, 81947 f), the required length of a 10-in-diameter equivalent pipe can now be determined by 0.032L = 8.947, ‘L= 280 t. Note that the required length of pipe can be determined in a single computation as follows: L= (0.00180)(480)/0.032 + (0.0137)(590)/0.032 = 280. ‘A.225.m-long, 300-mm-diameter concrete pipe and a 400-m-long, $00-mm-diameter concrete pipe are ‘connected in series. Find the diameter of a 625-m-long equivalent pipe. Assume a flow rate of 0.1 m/s, For the 300-mm-diameter pipe, from Fig. A-14, hy = 0.0074 m/m. For the '500-mm-diameter pipe, hy = 0.00064 m/m. The total head loss for both pipes isfy = (0,0074)(225) + (0.00064)(400) = 1.921 m. For a 625-m-long equivalent pipe with this head loss, ft, = 1.921/625 = 0.00307 m/m. From Fig. A-14, D = 360mm. ‘Water flows at arate of 0.020 m/s from reservoir A to reservoir B through three concrete pipes connected in series, as shown in Fig. 10-1. Find the difference in water-surface elevations in the reservoirs. Neglect minor losses. pale + 52g +24 =Poly + Val2g ttn th, OF0424=0404 2 +h, hy = hy = 24 With Q = 0.020 m?/s and D = 160 mm, from Fig. A-14, h, = 0.0082 m/m. With Q =0.020m'/s and D 200 mm, f= 0.0028 m/m. With Q = 0.020m°/s and D = 180 mm, hy = 0.0046 m/m. H = hy = (0,0082)(1000) + (0.0028)(1600) + (0.0046)(850) = 16.59 m. Fig. 10-1 270 0 CHAPTER 10 10s 10.8 ‘Compute the flow rate of water through the three concrete pipes connected in series as shown in Fig, 10-2 by the equivalent length method. Hh, =z4~ 29 =20Mt/ft. Assume Q = 1 fs. From Fig. A-13, (Ps)in = 0.00067 f/f; (A )oin = 0.0016 t/t; (hs)uia = 0.00475 f/f. ‘Consider a 10-in-diameter equivalent pipe: L, = 120 + (0.00475)(150)/0.0016 + (0.00067)(100)/0.0016 = 607 f. With D = 10iin and h, 0.0329 ft/ft, from Fig. A-13, Q = 5.1 cf. Fig. 102 Solve Prob. 10.5 by the equivalent diameter method. Using values obtained from Prob. 105, hy = (0,00067)(100) + (0.00475)(150) + 0.0016)(120) =0.972 8. With Q= I cfs and hy = 0.972/(100 + 150-+ 120) = 0.09263 R/f, D =89in, With D = 8.9in and h, = = 0.0541 ft, Q = 5.1. ‘Two concrete pipes are connected in series. The flow rate of water through the pipes is 0.14 m"/s with a total friction loss of 14.10 m for both pipes. Each pipe has a length of 300 m. If one pipe has a diameter of 300 mm, ‘what is the diameter of the other one? Neglect minor losses. For fest pipe: With Q =0.14 m/s and D = 300 mm, hy =0.014 m/e. hy =(0,014)(300) = 420m. For second pipe: = 14.10 4.20 = 9.90 m, hy = 9.90/300 =0.033 m/m. With Q-=0.14m°/s and h, 0.033 m/m, D = 250 mm. ‘Three concrete pipes are connected in series, as shown in Fig. 10-3. Determine the length of an 8-in-diameter ‘equivalent pipe. Assume Q = 1 cfs. (hs)aia = 0.00475 f/f; (:)eia = 0.0195 f/f; (hs) rose = 0.0016 f/t5 Le = (0.0195)(1000)/0.00475 + (0:0016)(2000)/0.00475 = 5979 f. , a diameter 1000, 64a diameter 2000 f, 10 diameter For the three pipes in Fig. 10-3, determine the diameter of a 4200-ft-long equivalent pipe. Using values obtained from Prob. 10.8, for a 4200-f-long equivalent pipe, hy = (0.00475)(1200) + (6.0195)(1000) + (0.0016)(2000) = 28.4 fe, = 28.4/4200 = 0.00676 ft. With Q = 1 fs and hy = 0.00676 f/f, D=1: 0.10 wot m2 103 mas SERIES PIPELINE SYSTEMS 1 271 For three pipes in series, the total pressure drop is p~ p» = 150.000 Pa, and the elevation drop is. 247 Zq= 5m. The pipe data are wo | s | 02 | 0.003 0 | 6 | 012 | 0.002 wo | 4 pipe | £,m [dem | emm | 10°) = 3.67 10° From Fig. A-5 f= 0.021, f= 0.021, and f= 0.020. ‘hy = 10 = 0.021[300/ #8) ]{v7/[2)(9.807)]} + 0.021{150/(#8)]{v3/[2)(9.807)]} + 0.020[250/()]{v3(2)19.80M) 10 = 1.0713 + 0.803003 + 1.02003 = 1.07104 + (0.8030)(2.250v,)? + (1.020)(1.4400,)* A74m/s_— Q = Au=[()(98)*/4](1.174) = 0.0830 m"/s Fig. 104 Solve Prob, 10.14 by the equivalent length method. Using data from Prob. 10.14 and choosing 30-cm pipe asthe standard, (L.)z = (150)(0.021 /0.020)(38)* = 1196 m of 30-cm pipe (L,)s= (250)(0.020/0.020)(3)° = 622 m of 30-em pipe (Cina =300-+ 1196 + 622=2118 m of 30-cm pipe = 10 = 0.020(2118/(7B)]{v*/(@).907)) v= L179 m/s Q = {(%)(3B)/4] 1.179) = 0.0833 m/s In Fig. 10-4 pipes 1, 2, and 3 are 500 ft of 3.068:in, 200 ft of 2.067-in, and 400 ft of 2.469-in wrought iron pipe. Given a head loss of 19.5 from A to B, find the flow of water at 60°F. U (€/d), = 0.00015/(3.068/12)= 0.000587 (€/d)_= 0.00015 /(2.067/12) = 0.000871, (€/4)s = 0.00015 /(2.469/12) = 0.000729 From Fig. A-5, assume fi=0.0172, f;= 0.0190, and f= 0.0181. y= (fML1d)(v"/2g) = 19.5 = 0.0172{500/ (3.068/12)}(vF/[(2)(32.2))} + 0.0190{200/(2.067/12)} (w3/[2)(32.2)) + 0.0181[400/(2.469/12)]{3/[2)32.2)) 19.5 = 0.522303 + 0.342603 + 0.54643 From the continuity relation, v= (di/d;)*(v1) = (3.068/2.067)*(uy) = 2.2030, vs= (di/ds)*(u) (3.068/2.469)"(v,) = 1.544u,, and = (258) a= ey ed = (480M) Ove = (28) 0 = [Oey a= (248) 20 = 0.522302 + (0.3426)(2.208v,)? + (0.5464)(1.544,)? vy = 2.365 ft/s Na=dv/y (Ng), = (3.068/12)(2.365)/(1.21 x 10-*) = 5.00 10* (Na)o = (1.484)(5.00 > 10") = 7.42 x 10" (Nu) = (1.243)(5.00 x 10") = 6.22 x 10° war 10.8 1019 SERIES PIPELINE SYSTEMS 0 273 From Fig. A- 0228, f;= 0.0225, and f= 0.0225. ‘hy = 19.5 = 0.0228{500/(3.068/12)](v3/[(2)(32.2)}} + 0.0225 200/(2.067/12)}(v3/[2)(32.2)0, ++ 0.0225[400/(2.469/12){v3/1(2)(32.2)]} 19.5 = 0.69240 + 0.405703 + 0.679203 = 0.6924ui + (0.4057)(2.200,)? + (0.6792)(1.544u,)? vy = 2.134 ft/s Q = Av= {(7)(3.068/12)'/4](2.134) = 0.110 £2 /s ‘An additional iteration (not shown) produces an insignificant difference. ‘Suppose that 100 ft of I-in (€ = 0.000005 ft) pipe and 500 ft of 6-in (e = 0.0008 ft) pipe are connected in series. For a total head loss of 24 ft, find the flow of water at 60°F. i (€/d), = 0.000005 /(73) = 0.0000600 (€/d)s = 0.0008/(‘) = 0.00160 From Fig, A'S, assume f,= 00110 nd f= 0.0215. fy = UNLIA) 0/29) =25 = 0.0110{100/(A)CvFN2VG2.2) + 0.02151500/(8) V¥/ENG22)) 25 = 0.20800 +0.333903 From the continuity relation, vs (/d(0») = (Co) = 0.02778, and (veda (2292) evan = [OPENS |, = (0.196710) 24 = 0.2050v} + (0.3339)(0.02778u,)" uv, = 11.04 ft/s Na= duly (Nas = (M(11.04)/(1.21 & 10-*) = 7.60 x 10" (Nq)2 = (0.1667)(7.60 x 10*) = 1.27 x 10° From Fig. AS, f.= 0.0195 and f= 0.02. 900/89} vi @N2.2)) + 0.0321500/¢8)} 42X22) 24 = 0,3634vj + 0.496903 = 0.3634] + (0.4969)(0.02778v,)? uv, = 8.122 ft/s (ads = N18 122) 11.28 x10") = 5.59% 10" (p)a= (1667)5.59 104) =9.32 x 10" From Fig. A, f= 00203 and f.= 0.033, y= 24 00203100/(5)] 0222.2) + 0.033500/(f)](¥42)32.2) 2h OSTESu} + 0St24v} = 0.37850! + (0.5124)(0.027780,)° uy = 7.961 fs o- ()(8)°/4](7.961) = 0.0434 Fs ‘An addiona iteration (9ot shown) produces an osignifiant diference. Repeat Prob. 10.16 for the case where the uid has s.g.=0.9 and j= 0.0008 Ib - s/f I First apply hy = (F)(L/d)(v"/2g), using values from Prob. 10.16 and assuming f = 0.03 forall pipes. 19.5 = 0.03{500/(3.068/12)}(vi/{(2)(32.2)]) + 0.03{200/(2.067/12)}(v3(2)(32.2))) +0.03{400/(2.469/12)](v3/[@)G2.2))} = 0.911003 + 0.540903 + 0.905603 = 0.911003 + (0.5409)(2.2030,) + (0.9086)(1.5440,)* j= LBS0f/s Nem pdv/n (Na), = [(0.9)(1.94)]3.068/12)(1.850)/0.0008 = 1032 (Wada = (1-484)(1046) = 1532 (Na)s = (1.248)(1046) = 1283 ‘Therefore, the flow is laminar and hy = (32)(u/y(L/a"\(v). 19.5 = 32(0,0008/{(0.9)(62.4)})[500/(3.068/12)"(%) ++ 32(0.0008/(0.9)(62.4))}[200/(2.067/12)"(2.2030,) + 32(0.0008/{(0.9)(62.4))}[400/(2.469/12)"\(1.5440,) Uys LOMAE/s —— Q = Av = {()(3.068/12)7/4](1.074) = 0.0551 f/s ‘One end of a 150-m-Long, 300-mm-diameter pipe is submerged in a reservoir; the other end abuts on a ‘90-m-long, 200-mm-diameter pipe at a point 30m below the reservoir surface. Water discharges freely (K = 1.0) {from the free end of the shorter pipe, which is 15 m below the junction. [This implies a 2° bend at the junction.} 274 10.20 10.21 0D CHAPTER 10 Determine the pressure heads just above and just below the junction, if f 24 for the contraction-bend at the junction. ! pily + Vi/2g + 2,= poly + V3/2g + 25h o np= (F(LLd)(v"[2g) = 0.04{150/0.300} v/{(2)(9.807)}) + 0.04490/0.200}{v3/[(2)(9.807)]} = 1.02003 +0.917703 (K)(v7/2g) = 0.8(vi(2)(9.807)}} + 0.24(v3/{(2)(9.807)}} = 0.04079uF + 0.0122403 hh, + hn = (1.102003 + 0.917703) + (0.04079u3 + 0.01224v3) = 1.06103 + 0.929903 Av Ass [()(0.300)"/4](v,) = [()(0.200)*/4I(v.) vy = 0.44460, .061)(0.4440,)? + 0,929903= 1.13903 0+0+ 45 =0 + vis{(2)(0.807)] + 0+ 1.13903 v= 6.149 m/s v, = (0.4444)(6.149)=2.733m/s 20.200)" /4](6.149) = 0.193 m/s ‘Now apply Eq. (1) between the water level at intake and a point just above the junction. 0+0 + 30 = aly + 2.733/[(2)(9.807)] + 0+ (1.0200; + 0.040790), 30 = ps/y + 0.3808 + (1.020 + 0.04079)(2.733" 21.70m., Finally apply Eq, (1) between the water level at intake and a point just below the junction. 0+ 04 30= poly + 6.149'/[(2)(9.807)] +0 + (1.0200 + 0.040790} + 0.012240), 30 = pa/y + 1.928 + [(1.020+ 0.04079)(2. 733°) + (0.01224)(6.149")], p2/y = 19.69 m. 04, K=0.8 for the entrance, and pily= Repeat Prob. 10.19 neglecting minor losses. 1 pil + Vil2g + 21 = poly + v/28 + 2+ hy, a Ii hy thy tg 0 Using data from Prob. 10.19, hy = 1.02003 + 0.917708, hy, =(1.020)(0.4444u,)? + 0.9177v3 = 1.11903, 040+ 45=0 + vi/[(2\(9.807)] + 0+ 1.11903, ve = 6.202 m/s, vy = (0.4444)(6.202) = 2.756 m/s; Q = Av, = {(2)(0.200}'/4](6.202) = 0.195 m'/s. For the pressure head just above the junction, 0+ 0-+ 30 = p,/y + 2.756'[(2\(9.807)] + 0+ 1.02002, 30=p;/y + 0.3873 + (1.020)(2.756"), pa/y = 21.87 m. For the pressure head just below the junction, 0-+ 0+ 30= pa/y + 6.202"/[(2)(9.807)] + 0-+ 1.02003, 30 = ps/y + 1.961 + (1.020)(2.756"), paly = 20.29 m, ‘Three new cast iron pipes, having diameters of 30 in, 24in, and 18 in, respectively, each S00 ft long, are ‘connected in series. The 30-in pipe leads from a reservoir (flush entrance), and the 18-in pipe discharges into the air at a point 11.5 ft below the water surface in the reservoir. Assuming all changes in section to be abrupt, find the rate of discharge of water at 60°F. : pur + uiag + z= pily + vit ath, hiahyth, — hy=(f\(Lia)(v*/28) Assume f =0.016 for each pipe. vs = velocity for 18-in pipe, velocity for 4-in pipe = ()*(v,) = 0.562502, velocity for bin pipe = (8) (us) = 0.36000, ‘hy = 0,016(500/ 8) (0.36000)/[(2)32.2)]} + 0.016{500/(¥)]{(0-56250,)°/12)32.2)} +.0.016{500/(18)] {02/[(2)(32.2)]} = 0.108903 ha = (R028) For entrance, take K = 0.5 (Fig. A-7). For sudden contractions, with d/D = %, or 0.80, K=0.15 and with 4d/D = #, 0F 0.75, K =0.18 (Fig. A-9). For exit, K = 1.0 (Fig. A-7). 5)(0.3600v,) + (0.15)(0.5625u,)*+ 0.18v3=0.292303 fh, = 0.108903 +0.292303= 0404 115=0+ v31Q)G221+0-40.401203 y=5.253/s a= duly (Na) son = #3((0.3600)(5.253)]/(1.21 10°) = 3.91 X10" (Na)oesn = H8{(0.5625)(5.253)]/(1.21 x 10°*) = 4.88 x 10° (Na eeia = (B)(5.253)/(1.21 10%) = 6.51% 10° (€/d)poa4= 0.00085/(78) = 0.000340 (€/d) iq ™ 0.00085/(#) = 0.000825 (€/d) n= 0.00085/( 8) = 0.000567 From Fig. A'S, fue = 0.0168, farg=0.0172, and fn ‘i, = 0,0168{500/(3)]{(0.36000,)*/(2)(32.2)}) + 0.0172{500/ 4) (0.562502)*12)32.2)0) + 0.01761500/(1)](v3/[(2)G2.2)]} = 0.119003 0.1190v; +0,2923u3=0.41130 OF0411.5=0+ vI/2VG2L2]+0+0.41133 vp = 5.191 f/s ‘An additional iteration (not shown) gives no significant change in v:- Q= Av= [(a}(H)*/4]5.191)=9.17 88/5. 401203 1.0176. 2 woz 044 ‘SERIES PIPELINE SYSTEMS 275 InFig. 10-5, K,=0.5, L,=300m, D, = 600mm, e,=2mm, L;=240m, D,= 1m, €=0.3 mm, %10"Smi/s, and H = 6m. Determine the discharge through the system, ! Ply + Vie += paly +VY2g+ th, he=hy thm hy = (f(LIA\(v*/28) Assume f, = 0.026 and f= 0.015. v2 [(f8%)/1P°(0,) = 0.36000, hy = 0.026{300/($88)]{v?/(219.807)}) + {(.015)(240/1)((0,36000,)/(2)(9.807)} = 0.685605 For entrance: fa = (K)(v*/2g) = 0.5(wi/(2)(9.807)]} = 0.025494 For exits fy = 1.0(v3/{(2),9.807)]} = 0.050880 = (0.05008)(0.3600v,)? = 0.006607? For sudden contraction: = v2) 22g = (v ~ 0.36000,)*112)(9.807)] 1020880 at = 0.025490 + 0.006077 + 0.020880 = 0.052980? 1, 0.686603 + 0.0529803 =0.7396u3 0+0+6=0+40+0+0.739603 v, =2.848 m/s 3600) (2.848) = 1.025 m/s tu/v (Nu) = ({885)(2-848)/(3 x 10°*) = 5.70 x 10° (Ng) = (1)(1.025)/(3 x 10%) (€/d), = d= 0.00333 (€/d),= (0.3/1000)/1 = 0.000300 From Fig. AS, f= 0.0265 and f, = 0.0168. ‘i, = 0.0265300/ (8) {v#/[2)(9.807)]} + (0.0168)(240/1){(0.3600)*/[2)(9.807)}) = 0.70220 ng = 0.702203 + 0.052980} = 0.755203 0+0+6=0+04040.755203 v= 2.819m/s ‘An additional iteration (not shown) gives no significant change in v4. Q = Av = [(=)(82)'/4](2.819) = 0.797 m'fs. ‘A Ne 2x10" Solve Prob. 10.22 by means of equivalent pipes. Expressing the minor losses in terms of equivalent lengths gives for pipe 1: Ky = 0.5 + [1 ~ ({88)"F = 0.9096, (L).= K:Dslf = (0.9096)(§83)/0.026 = 20.99 m; and for pipe 2: K2= 1.0, (L.)2= KzDs/f,= (1.0)(1)/0.015 = 66.67 m. The values off, and fy are selected for the fully turbulent range as an approximation. The problem {snow reduced to 300 + 20.99, or 320.99 m of 600-mm pipe and 240 + 66.67, oF 306.67 m of I Expressing the I-m pipe in terms of an equivalent length of 600-m pipe, Le = (fulfi(Ls)(Dy/D,)*= (0.015/0.026)(306.67){#85)/1}’ = 13.76m. By adding to the 600-mm pipe, the problem is reduced to finding the discharge through 320.99 + 13.76, or 334.75 m of 600-mm pipe, €, = 2mm, H = 6m: hy = (f)(L/D\(u"/2a), 6= (f)1334.75/ (@)](v°/[(2)(9.807)}}, v = 0.4593/VF. Try f = 0.026: v = 0.4593/ 0.036 = 2.848 m/s, Nq = Duly = (#83)(2.848)/(3 x 10°) = 5.70 x 10°. From Fig. A-5, with Ny = 5.70°x 10° and €/D = 0.00333, 0265. Try f = 0.0265: v = 0.4593/ V0.0265 = 2.821 m/s, Ne = ($8)(2.821)/(3 x 10°*) = 5.64 x 10°, 0265 (O.K.); Q = Av = {()(i8)"/4](2.821) = 0.798 m/s ‘Two reservoirs are connected by three clean cast iron pipes in series: L = 300m, D,= 300 mm; L, = 1200 m, Ds = 450mm. Ifthe low is 360 m'/h of water at 20°C, determine the difference in clevation ofthe reservoirs. ' v= QIA v= (360/3600)/[()(0.200)?/4] = 3.183 m/s ¥s = (360/3600)/[()(0.300)7/4] = 1.415 m/s v4 = (360/3600)/{(72(0.450)"/4] = 0.6288 m/s Ne= Duly (Na), = (0.200)(3.183)/(1.02 x 10-*) = 6.24 x 10° (Nada = (0.300)(1.415)/(1.02 x 10-9) = 4,16 x 10° (Ns = (0.450)(0.6288)/(1.02 x 10 (€/D),= 0.00026/0.200 = 0.00130 (¢/D): = 0.0026/0.300 = 0.000867 (1D), = 0,00026/0.450 = 0.000578 = 0.0215, f, = 0.020, and f,= 0.0185. H = hy = (/)(L/D)(v"/2g) = (0.0215{300/0, 200 (3. 183/(2)9.807)]} + 0,020{400/0.3004(1.$157/((2)9.807))) + £0.0185(12007/0.450](0.6288"/[2)(9.807)]} = 20.37 m. 2.77% 10" 276 0 CHAPTER 10 10.25 Solve Prob. 10.24 by the method of equivalent lengths. I Express pipes 2 and 3 in terms of pipe 1: Le= GAME DIDY (Las (0.020/0.0215)(400)(38)* = 49.00m (Ls = (0.0185/0.0215)(1200)(38)* = 17.91 m (Lo) = 300 + 49.00 + 17.91 = 366.9 m H = hy = (JM{L/D)(v"2g) = 0.02154366,9/0.200]{3.189/[(2)(9.807)]} = 20.37 m 10.26 Air at atmospheric pressure and 60 °F is carried through two horizontal pipes ( in) in series. The "upstream pipe is 360 ft of 24in diameter, and the downstream pipe is 120 ft of 36in diameter. Estimate the ‘equivalent length of 18-in (€ = 0.008 in) pipe. Neglect minor losses. i €/D,=0.06/24= 0.0025 €/D,= 0.06/36 = 0.00167 From Fig, A-S, assuming high Reynolds numbers, f, = 0.025 and f, = 0.022 = (0.025/f,)(360)(M)* + (0.02216 )(120)G8)* = 2.218/f, 6D, =0.003/18 = 0.000167 f= 0.013 L, =2.218/0.013 = 171 ft 10.27 What pressure drop is required for flow of 6000 "/min in Prob. 10.26? Inctude losses due to sudden expansion. ' hea hy thm hy = (PMLID\(v? 124) From Prob. 10.26, €/D, = 0.0025, ¢/D; = 0.00167. = QUA, = CBP) AGIA = 31.8348 vy = O/A,= (BIR) 4] = 14.15 f/s Ng =Dulv (Ny) = (39)(31.83)/(1.58X 10-4) = 4.03 x 10° (Naa = (#)(14.15)/(1.58 x 10-4) = 2.69 x 10° (025 and f= 0.022, ‘hy = 0.025360 /(3)](31.83"/[(2)(32.2)}} + 0.022{120/(8)]{14.15°/[(2)(32.2)]} = 73.53 ft = (Us —v3'/2g = B18 — 14.15)7/(2(32.2)} =4.85m_ hy =73.53-4 4.85 =78.38 ft Ap= yk y=p/RT 7 (14.7)(144)/{(S3.3)(460 + 60)] = 0.07637 /f? Ap = (0.07637)(78.38) = 5.99 Ib/ft? From Fig. A-5 f 10.28 Two pipes, D, = 3in, L, = 300ft, and D, = 2in, L, = 180 ft, are joined in series with a total pressure drop of 3ibfin’. What is the flow rate of syrup (p = 1.78 lugs/ft°, = 0.00606 Ib - s/f?) at 20°C? ' y= Aply = Sp/pg = (3) U44)/[(. 7882.2] =7.537 ‘Assume laminar flow Y2BULO 5 55> _ (128)(0.00606)(300\(Q) , (128)(0.00606)(180)(0) “pED* ETRE) VLTAGZAEY ‘Computation of Ng (not shown) indicates the flow is indeed laminar. D 2 = 0.00564 ft/s 10.29 Convert the piping system shown in Fig. 10-6 to an equivalent length of 6-in pipe. ' pal + VBI28 + 24= Puly + Vid2¢ +20 +h, hes hy + hm hy = (PL 1d)(V 7/24) = 0.0251150/ BK Vis/{2NG2.2)) + 0.020{100/(8)K VEM2)32.2)1) = 0,05823V3, + 0.062112 rm = (KY(V?/28) = [8.0-+ (2)(0.5) + 0.7 + LO](VI/(2)32.2)) +10.7+6.0+ (2900.5) +3.0+ 1.0K VE/12)G2.2))) = 0.1661}, +0.1817V3 hn, =0.05823V%, + 0.06211¥3 + 0.1661V%, + 0.1817V3 = 0.2243V3, + 0.2438V3 0+0+h=0+0+040.2283Vi +0.2438V3 Va= ()*(Ve)=0.2500% += (0.2243)(0.2500¥,) + 0.2438V3 = 0.2578V3 For a 6:n equivalent pipe, = 0.020{L,/( 8)]{V2/[(2)(32.2)}} = 0.00062111,V2, 0.2578V3, L.=Als ft .00062111,V3, 030 SERIES PIPELINE SYSTEMS 0 277 Factors K Strainer B 80 12” Bends C, F (each) 05 12” Tee D =07 12” Valve E 10 12” x 6” Cross G (x Vi/2g) = 0.7 6” Meter 6.0 6” Bends J, K (each) 05 6” Valve L = 38.0 Fig. 10-6 For the compound piping system described in Prob. 10.12, what flow will be produced for a total head loss of M08? I From Prob. 10.12, 13 540 of 16-in pipe is equivalent to the compound system. For a head los of 70.0ft, ‘y= 70.0/13 540 = 0.00517. From Fig. A-13, Q = 6.3 cfs. Ths is for C= 120. For new cast iron pipe (C= 130), = (18)(6.3) = 6.8cfs. For the system shown in Fig. 10-7, when the flow from reservoir A to main D is 3.25 mga, the pressure at D is 20.0 psi. The flow to D must be increased to 4.25 mgd with the pressure at 40.0 psi. What size pipe, 5000ft long, should be lai from B to C (shown dashed) parallel to the existing 12-n pipe to accomplish this? I. The elevation of reservoir A can be determined as follows: Q = (3.25 x 10°)/[(7.48)(86 400)] = 5.03 ef. From Fig. A-13, (I) = 0.0038 ft/ft,(h,),2 = 0.0150 ft/ft. These values of hy are for C = 120; they must be adjusted for C (hi ro (B)(0.0038) = 0.0046 f/f (hy) ra = (180.0150) = 0.018 f/f (hy) (0-0046)(8000) = 36.8ft (hy a= (0.018)(5000) = 90.0 (hy )ua = 36.8-+ 90.0 = 126.8 ft ‘The hydraulic grade line drops 126.8 ft to an elevation of 46.2ft above D (the equivalent of 20.0 psi). Thus reservoir A is 126.8 + 46.2, of 173.0 t above point D. For a pressure of 40.0 psi, the elevation of the hydraulic grade line at D will be 92.4 ft above D, oF the available head forthe flow of 4.25 mgd will be 173.0—92.4, or 80.6. In the 16:n pipe with Q = 4.25 mgd, or 6.58 fs, (H,)is = 0.0062 t/t for C = 120. or C= 100: (h;):e = (18)(0.0062) = 0.0074 f/f, (Hye (0.0074)(8000) = 59.2 ft (Fp) wc= 80.6 — 59.2 = 2.4t For the existing 12-in pipe: (hs). = 21.4/5000 = 0.0043 fi/ft, Qua = 2.7 cfs for C = 120. For C= 100, Qra= (18)(2.7) = 2.3efs. The flow in the new pipe must be 6.58 ~ 2.3, or 4.28 fs with an available hhead (drop in the hydraulic grade line) of 21.4 from B to C. hy = 21.4/5000 = 0.0043 fi/t. From Fig. A-13, D=14in, approximately. CHAPTER 11 Parallel Pipeline Systems. ua 2 m3 Figure 11-1 shows a looping pipe system. Pressure heads at points A and E are 70,0 m and 46.0 m, respectively ‘Compute the flow rate of water through each branch of the loop. Assume C = 120 forall pipes. F Giy)y-¢~ 70.0 46.0 = 24.0 m. From Fig. A-14, for pipe ABE, with h, = 24,0/3000, of 0.0080 m/m, and 300 mm, Oye = 0-105 m"/s. For pipe ACE, with, = 24.0/1300, oF 0.0185 m/m, and D = 200 mm, Quce™ 0.056 m/s. For pipe ADE, with h, = 24.0/2600, or 0.0082 m/m, and D =250 mm, Qxox= 0.070 "fs. 1300 m, 200-mm diameter 2600 m,250"mm diameter Fig. 1 A looping concrete pipe system is shown in Fig. 11-2. The total flow rate of water is 18.0 cfs. Determine the division of flow and the loss of head from point B to péint E. 1 Assume a head loss of 20 from point B o point E. With (hy)ace™ sis, 0F 0.0040 f/t and D = 15in, Qrce= 4.75 cfs. With (hy)noe = a, oF 0.0050 f/f and D = 18in, Qane = 8.60 cls. Fraction of fow through pipe BCE = 4.75/(4.75 + 8.60) =0.356, Fraction of flow through pipe BDE = 8.60/(4.75 + 8.60) = 0.644 Qrce= (18.0)(0.356) = 6.4.cf, Onne = (18.0)(0.644) = 11.6cf. With Qgce=6.4efs and D = 15in, (yee O.0070 f/f. (hy) y-e = (0.00703(5000) = 38.0. 5000 13: diameter 3000 f, 30. diameter re | 4000 f, 2449 diame O= stm 1000 8 i diameter Fig. 12 ‘The discharge of water in the eonerete pipe looping system shown in Fig. 11-3 if 15.0/s. Compute the head Joss from point A to point G. H With Qu.» = 15.0fs and D = 30in, (hi)a-n = 0.00116 f/ft. With Ong = 15.0 cf and D = 24:0, (hy) po = (0.00345 f/f (,)an = (0.00116)(2500) = 2.90 fe, (h-)nc = (0.00345)(3000) = 10.35 ft. Assume (Mor =30 ft With D = 18 in and (h:)ncy = vx = 0.020 ft/ft, Qncy™ 18.1 cfs. With D = 12 in and (h; yop = rie = 0.080 ft/, Quop=7.7 ef. With D = 15in and (h)par™ soe ~ 0.015 t/ft, Oner=9.6 cfs. Fraction of flow through pipe BCF =18.1/(18.1 + 7.7 + 9.6) =0.511, fraction of flow through pipe BDF = 7.7/(18.1 +7.7 + 9.6) = 0.218, fraction of flow through pipe BEF = 9.6/(18.1+7.7 + 9.6) = 0.271; Once (15.0)(0.511) = 7.67 efs. With Qace=7.67 and D = 18in, (hy) cy = 0.0040 f/f. (hy n.2™ (0.0040)(1500) = 6.00 ft; (4-0 Cyan + Cave (i Jno =2.90 + 6.00 + 10.35 = 19.25 f PARALLEL PIPELINE SYSTEMS 0 279 1500, Hin diameter 2500 f, 30 diameter 7 Qntse B 2000 We diameter Fig. 11-3 WA Ifthe flow rate of water through the pipe system shown in Fig. 11-4 0.050 m"/s under total head loss of 9.0m, determine the diameter of pipe C. Assume a C coefficient of 120 forall pipes. I With Q, = 0.050 m/s and D, = 200 mm, (ts) = 0.015 m/m and (h,)4 = (0.015)(300) = 4.50m. With Qo = 0.050'm/s and Dp = 250 mm, (ho = 0.0049 m/m and (,)p = (0.0049)(510) = 2.50 m. (ya = (hyde = 9.0 ~ 4,502.50 = 2.00 m. With (:)» = 2.00/266 = 0.0075 m/m, Qx = 0.019 m’/s. Qc = 0.050 ~ 0.019 = (0.031 m'/s. With Qc =0.081 m'/s and (h:)e=2.0/190 =0.0105 m/m, De= 180mm. Pipe B Pipe A 300 me, 300-mm diameter Pipe D S10 wm, 250-mm iamek 190 m Fig. 11-4 115 For the looping pipe system shown in Fig, 11-5, the head loss between points A and Fis 160.0 tof water. Determine the flow rate of water in the system. Assume C = 120 forall pipes. I Assume a head loss of 12 ft from point B to point E. Bo = 0.0030 fi/ft and D = 15in, Qace™ 4.0. With (hy) so = sis = 0.0040 f/f¢ and D = "2 2.6 cfs, Fraction of flow through pipe BCE = 4.0/(4.0+ 2.6) = 0.606; fraction of fow through pipe BDE = 2.61(4.0 + 2.6) = 0.394; Qan= Orr = 4.0+2.6= 6.6 cfs. With Og» = 6.6¢fs and D = 24 in, (hs) an =0.00075 10 (ty) an = (0.00075)(8000) = 6.0 ft. With Qer =6.6.fs and D = 18 in, (h:)er = 0.0030 f/f (ar = (0.0030)(6000) = 18.0 ft. Fraction of total head loss through pipe AB = 6.0/(6.0+ 12.0+ 18.0) =0.167, fraction of total head loss from B to E = 12.0/(6.0+ 12.0+ 18.0) = 0.333, fraction of total head los through pipe EF = 18.0/(6.0+ 12.0+ 18.0) =0.500. Actual head losses are, therefore, (I) qa» = (160)(0.167) = 26.72 ft (hy) ne = (160)(0.333) = 53.28 ft (yer = (160)(0.500) = {80,00 f. With (ty) a» = 26.72/8000 = 0.00334 ft/ft and D = 24in, Oxy = 14.5 cfs. With (h,)er = 80.00/6000 0.0133 fi/ft and D = 180, Qer (14.5)(0.606)= 8.8 cls, One = (14.5)(0.394) Tels. 8000 f, 24 diameter a a 6000 f, 18a diameter Fig. 11-5 116 A flow of 570 L/sis proceeding through the pipe network shown in Fig. 11-6. For a pressure of 690 kPa at node ‘A, what pressure may be expected at node B? Neglect minor losses. Take p = 1000 kg/m’. W pal + v4l28 + 24= pal + Ub/2e + 20+ hx. Assume that a flow Qi of 170 L/s is proceeding through branch 1. y= (MLID)(u"12g) = Qi = (1B) {C)(BY'/4] =2.405 m/s Na= Dv (Na): = (2B)(2.405)/(0.0113 x 10°) = 6.38 10° —_€/D, = 0.00026/ (884) = 0.00087 From Fig. AS, f = 0.0198. fh, = (ty), = 0.0198{600 (3) (2.405?/[(2)(9.807)}) = 11.68 m. For branch 1, Paly + (vi)4/2g + 6 = poly + (vi)a/2g + 15+ 11.68 — (vi)a/2g =(V7)n/28 [pa —P)/ Yh = 20.68 m_ 280 0 CHAPTER 11 Using tis pressure head, which must be the same for each loop, for branch 2, 20.68 + (v2)4/2g +6 = (vi)e/2g + 15+ (hyo (WDal2g=(U2)0/28—(hy)2=11.68m 11.68 = {460 / (185) {v3/[(2)(9.807)]} = 49.9010 (€/D). = 0.00026/ 88) = 0.00055 Estimate f,= 0.018. (49.903(0.018)(03) v= 3.606m/s = Aqva= [()(488)"/4]@.606) = 0.6256 m/s or 625.6L/s Now the desired actual ows Q; and Q; may be computed so as to maintain the ratio Qi/Q3 and to satisfy continuity 0 that Q,+Q,= S7OL/s. Q, = [170/(625.6 + 170)|(570) = 121.8 L/s. = [625.6/ (625.6 + 170)}(570) = 448.2L/s 1% = (1218/1000) (88) /4] = 1.723 m/s (48.2/1000) (8) =2.583 m/s (Pn) = (8)1.723)/(0.0113 x 104) = 4.57 10° (Ny) = (ABR). 583)/(0.0113 x 10-4) = 1.07 « 10" f=0.0198 — fe= 0.0180 (hg), = 0.0198{600/(388)]{1.7234/|(2)(9.807)]} = 5.99 m pal¥ + (Wi)al2+6™ poly +(vi)al2¢+15+599 — [(pa-pa)/yh= 149m (y= 0.0180(460/($B)](2.583°/12)(9.807)}) = 5.991 Pal + (3)al28 +6=paly + (WDal2g + 1545.99 ((ea-Pa)/yh=14.99m — [(690—pa)/9.79]= 14.99 pa = S43 kPa Cast iron pine ‘rougheut 1 200113.x 10-4 ms Fig. 11-6 1.7 The system shown in Fig. 11-7 receives at A 66 ft’/min of water at a pressure of 95 psig. Neglecting minor loses, find the pressure at B. The pipe is commercial steel and = 2.11 x 10™*Ib - s/f ' Pal + Ua/2g + 24= Pel + 812g + 20+ hy. ‘Assume that a flow Q{ of 30t"/min =0.5 f/s is proceeding through branch 1. hy = (FULIDU7/2g) vy = D/A, = O.5/{(H)(H8)'/4] = 2.546 ft/s Ne = pDu ly. (a). = (1.98) (8)(2.546)/(2.11 x 10-4) = 1.17% 10" €/D, = 0.00015/() = 0.00030 From Fig. A-S, f = 0.0195. hi = (hy). = 0.0195{(35 + 1200 + 35)/(f)]{2.546"/[(2)(32.2)]) = 4.985 ft For branch 1, ply + (v3)a/28 +0 poly + (v%)o/2g +0 4.985, (vi)q/2g = (vi) /28, [(P4~Pa)/th = 4.985 ft. Using this pressure head, which must be the same for each loop, for branch 2, 4.985 + (WDa/2g+0= (VD a/2g+04(h)s (WDa/2=(Dal2¢— (hy)2 = 4.985 4.985 = fil(35 + 1200 + 35)/(){v3/[(2)G2.2)]} = 29.5803 (¢/D) = 0.00015/(8) = 0.000225, 4.985 = (29.58)(0.018)(v2) v= R.060H/s Qt = Avs =[(a)(8) 413.060) = 1.068 F's [Now the desired actual ows Q; and Q may be computed so as to maintain the ratio Qi/Q: and to satisy ‘continuity so that Q, + Qs = 66 ft/min = 1.1 ft/s. Q, = [0.5/(0.5 + 1.068)](1.1) = 0.3508 ft/s a. .068/(0.5 + 1.068)](1.1) = 0.7492 fr'/s = (0:3508)/[(x)(8)?/4] = 1.787 ft/s (0.7492)/[()(8)'/4] =2.146 fs (Nn )s = (1.94)( (1. 787)/(2.11 10-*) = 8.22 x 10 — (Nq)z ™ (1.94) 48)(2.146)/(2. 11 x 107%) = 1.32 x 10° ‘f= 0.0205 ‘f= 0.0190 (hy), = 0.02051 (35 + 1200 + 35)/(#8)]{1.787°/{(2)(32.2)]} = 2.582 ft PARALLEL PIPELINE SYSTEMS 0 281 Paly + (vi)al28 +0™ ely + (vi)cI28 +0+2.582 — [(Pa—pa)/yh= 2.582% (hy)a= 0.01904(35 + 1200-+ 38)/(f)]{2.146/(2)(32.2)}) = 2.588 ft aly + (W2)al2g-+0= poly + (WDcl2g +0+2.588 — [(pa~Pe)/ye= 2.588% [095—pc)(144)/62.4] =2.588 pe =93.91b/in? rcv, v= Q/A=1.1/[(")(5)/4]=3.151 ft/s Ng = (1.94)(4)(3.151)/(2.11 x 10~ 93x10" f = 0.0178 ‘hy = 0.0178[3200/(4)]{3.1517/[(2)(32.2)]} = 13.17 ft Pa = 93.9 ~ (13.17)(62.4)/144 = 88.2 psig ca c SH mens Repeat Prob. 11.7 using nominal pipe sizes (6.065 in and 7.981 in diameters) and supposing an open globe valve in the &-in pipe just before B. The fittings are all screwed fitings. Use K =0.45 for a 6-n standard tee, 0.30 for elbows, and 4.8 for the globe valve. 1H paly + vil2g-+ 24=pely + vbl2g + ze + hy. Assume that a flow Q; of 0.5 ft/s is proceeding through ‘branch 1. hshy the hy =MLID\WI28) Na=pDvlte — (Nq)s= (1-94)(6.065/12)(2.492)/(2.11 x 10° From Fig. A-5, f = 0.0198, (hy) = 0.0198{(35 + 1200 + 35)/(6.065/12)}{2.492"/[(2)(32.2)]} = 4.798 ft 4h = (KMO28) (hm) =[2MO.30) + (2)(0.45)]{2.492/(2(32.2)]} = 0.145 ft (hy), = 4.798 + 0.145 = 4.943 /A, = 05/I(H)(6.065/12)/4] = 2.492 4/5 16 10° €/D, = 0,00015/(9§*) = 0.00030 For branch 1, Daly + (oDal28 +0= poly + (WDel2e+0+4983 — (WDal2e=(Ds/28 [Pa Po)/h=4.948 ‘Using this pressure head, which must be the same for each loop, for branch 2, 4943+ (WD2g +0= (Ui)a/2 +04 (hi)s — (WBal2E=(WDal24 (he e= AE 4,943 = ff(35 + 1200 + 35)/(7.981/12)]{v3/{(2)(32.2)]} + [(2)(0.30) + (2)(0.45)] {v3/{(2)(32.2)]} = 29.65f,3 + 0.023290; (€/D),= 0.00015/(7.981/12) = 0.000226 Estimate = 0.018, 4.943 = (29.65)(0,018(0%) + 0.023293 v= 2.979ft/s O5= Agus = [()(7.981/12)"/4]2.979) = 1.085 £75 Now the desired actual flows Q, and Q, may be computed so as to maintain the ratio Q;/Q; and to satisfy ‘continuity s0 that Q, + Q5= 1.1 f€/s. Qs =(0.5/(0.5 + 1.035)](1.1) = 0.3583 /s—Q,=[1.035/(0.5 + 1.035)\(1.1) = 0.7417 fe/s ‘vy = (0.3583)/{()(6.065/12)'/4] = 1.786 ft/s v= (0.7417) /[("\(7.981/12)7/4] = 2.135 fels (Nq)s= (1:94)(6.065/12)(1.786)/(2.11 X10") = 8.30 x 10" (Na)a= (1.947.981 12)(2.135)/(2.11 x 10%) = 1.31 x 10" f= 0.0205 f,=0.019 282 0 CHAPTER 11 ns (4), =0.02054(35 + 1200 + 35)/(6,065/12)} (1. 786/[(2)(32.2)]} = 2.551 ft (20.30) + 2)(0.45)](1.786'/(2)(32.2)]} = 0.074 (,),=2.551 + 0.07 paly + (WDal2g +0 pely + (vI)cI2R+0+2.625 —[(pa—pa)/v= 2.6258 (hy)2=0.019{(35 + 1200 + 35)/(7.981/12)] 2. 135*/{(2)(32.2)]} = 2.568 ft (hada = [(2)(0.30) + (2)(0.45)]{2.135°/[(2)(32.2)]} = 0.106 ft (hey =2.368-+0.1065= 2.674 pal + (UDal28 +0™ pel + (Uc/2g +04 2.674 [ea-podlrh=2.674 — ((95—pe\144)/62.4]=2.674% pe =93.81b/in* For pipe CB, vm Q/A = 1.1/[()(7.981/12)"/4]=3.166 f/s Ng = (1.94)(7.981/12)(3.166)/(2.11 x 10-4) = 1.94 x 10° f= 0.0178 hy =0.0178(5200/(7.981/12)]13.166%/1(2)(32.2))) nu = 4 8(3.NGEF IT) G2.2)}) = 0.75 ft Po = 93.8 ~ (14.08)(62.4)/144 = 87.7 1b/in? ‘A two-branch, commercial stel pipe system (Fig. 11-8) delivers 0.42 m'/s of water at 5°C. The pressure at B is ‘22KPa gage. What is the pressure at A? Neglect minor losses. 1B paly + v5)08 + 24= aly + 0/28 + 24+ hy. Assume that a flow Q of 0.20/m°/sis proceeding through branch 1 y= (DULIDMv'l28) (an = Qi A, = 0.20/{()(0.250)'/4] = 4.074 m/s (oan = QY/A, = 0.20/[()(0.200)/5]=6.366m/s Ne= Duy [0p shw = (0.250)(4.074)/(1.52 x 10-4) =6.70% 10° (€/D,) y= 0.000046/0.250 = 0.000184 From Fig. A-5, (f)xe=0.015. Me) shan = (0.200(6.366)/(1.52 x 10-4) = 8.38% 10° (€/D,) qo =0,000046/0,200 = 0.00023 (fiw = 0.018 ‘a, = 0.015{(60 + 110 + 40)/0.250}{4.074/[(2)(9.807)}} + 0.015{(110 + 40)/0.200](6.366"/[(2(9.807))} = 33.91 m For branch 1, aly + (Wi)al28+0= poly + (vipa!28 +0+33.91 (Ui)a!28=(vi)a/28 [(Pa—Pa)lyh=33.91.m ‘Using this pressure head, which must be the same for each loop, for branch 2, 33.914 (WDal2g +0= (WDa/2e +04 (hy), (WDal2E=(WDa/24 —(hp)2=33.91m 33,91 = fif(20 + 220)/0.200}{v3/[(2(9.807)]} = 61.18/03 __(€/D)s = 0.000046/0.200 = 0.00023 Estimate f= 0.015. 38.91 = (61.18)(0.015)(03) —vs= 6.079 m/s Qs Asus = [(4)(0.200)"/4]6.079) = 0.1910 m"/s Now the desired actual flows Q, and Q; may be computed so as to maintain the ratio Q{/@: and to satisfy continuity so that Q, + Q, = 0.42 m’/s. = [0.20/(0.20 + 0.1910)](0.42) = 0.2148 m’/s. Q. = [0.1910/(0.20 + 0.1910)](0.42) = 0.2052 m'/s (vs)aso = 0.2148/{(1)(0.250)*/4] = 4.376 m/s (ws )amw= (0.2148)/{()(0.200)7/4] = 6.837 m/s. ‘v2 ™ (205.2/1000)/[()(0.200)?/4] = 6.532 m/s Since the assumed value of Q, is close t0 0.2148 m’/s, the values of fof 0.015 should be lose enough. (yp), = 0.015[(60 + 110 + 40)/0.250] (4.3767/[(2)(9.807)]} +0.015{(110 + 40)/0,200](6.837°/(2)(9.807)]) = 39.11 m Daly + (vi)al2g +0= paly + (vi)a/2g + 0439.11 (es-Padyh (hp)2 = 0.015{(20 + 220)/0.200]{6.5327/[(2)(9.807)]} = 39.16 m aly + (Wi)al28 +0= Pal + (WDal26 +0+39.16 — [(Pa~Pe)/v=39.16m ‘These values of [(p —Ps)/1/] are close enough, so take an average value of (39.11 + 39.16)/2, of 39.14 m: (Pa=2)/9.79 = 39.14, px = 405 kPa, ut PARALLEL PIPELINE SYSTEMS 0 283 Branch | , = 250 mm! 1D, = 200 men Broneh I 220m Fig. 11-8 ‘The vertical, commercial steel pipe system shown in Fig. 11-9 (D = 300 mm throughout) carries 0.82 m'/s of water. Find the pressure drop between A and B if B is 100m higher than A. Neglect minor losses. The water is at sc. 1 Daly + Ual2g + 24™ poly + Vb/2¢ + 20+ hy For branch I: Estimate Qj = 0.25 m/s, hy = (f)(L/D\(v"/2g). Let f = 0.0 Oi: 0.25/{()(0.300)*/4} = 3.537 mis, h, = 0.014[(230 + $50 + 250)/0.300}(3.537°/1(2)(9.807)) va/2g +0 paly + va/2g + 100 + 31.25, va/2g = va/24, (Pa~Pa)/¥ = 131.25. For branch Ii: Let f = 0.015: hy = 0.0151550/0.300}{(vin)3/[2)(9.807)}) = (1.402)(0.), pal y + ¥4/2g + 0= Pal y+ vol2g + 100+ (1.402)(wi)*, (pa ~ Pa)/¥ = 100 + (1.402)(ui)*. Use (Pp —Pa)/¥ = 131.25 m. 131.25 100 + (1.402)(ui)?, vj 4.721 m/s. For branch III: Let f = 0.013: h, = 0.013{(380 + $50 + 350)/0.300]{(vin)?/(2)(9.807)9} = (2.762)(vin), pal + Ual2g + 0 pal y + V5/28 + 100 (2.762)(vin), (pa ~ Pa) ¥ = 100 + (2.762)( wn)” Use (Pa ~ Pa)! = 131.25 m, 131.28 = 100 + (2.762)(viy)*, vin = 3.364 m/s. The new values of Q are Q, = 250L/s; {Qu = [(} 0.300) /4] 4.721) = 0.3337 m/s, of 33.7 Lis: Qnu= [()(0.300)'/4](.364) = 0.2378 m'/s, or 237.8 Ls. Find new values of [250/(250 + 333.7 + 237.8)|(820) = 249.5L/s, Q, 333.7 + 237.8)|(820) = 333.1 L/s, Qin = [237.8/(250 + 383.4 + 237.8)|(820) = 237.4 Lis For branch I: v= (249.5/1000)/{(7)(0.300)*/4] = 3.530 m/s, Ne = Du/' 6.97 10, €/D = 0.000046/0,300 = 0.00015. From Fig. A°S,. 1.25 m, paly + f=0.0141 For branch IML: vn = (237.4/1000)/{()(0.300)/4] = 3.359 m/s, Nq = (0,300)(3.359)/(1.52 x 10-*) = 6.63 x 10°, £=0.0146. Compute (p4 ~ pa)/y for each branch For branch I: hy = 0.0146[(250 + $50 + 250)/0.300}{3.530"/[(2)(9.807)}) = 32.46 m, pa/y + vs/2g +0= paly + i /2g + 100+ 32.46, (p,— pa)! = 132.46 m. For branch Hi: /y = 0.0141{550/0.300){4.712"/[(2)(9.807)]} =29.26 m, paly + v3/2 +0=paly + va/2¢ + 100 + 29.26, (pa ~ ps)! = 129.26m. For branch III: /, = 0.0146 350 + 550 + 350)/0.300}(3.359"/[(2)(9.807)]} = 34.99 m, paly + U3/2g + 0= Pal + UB/2g + 100 + 34.99, (Px —pa)/7 = 134.99 m. ‘We could go through another cycle. Instead, we stop here and take an average value of (p~ Pe)/: (132.46 + 129.26 + 134.99)/3, oF 132.24m. (px pa) = (132.24)(9.79) = 1295 kPa gage. In Fig. 11-10, 0.001 ft; Ln = 200 ft, Dy €, = 0.0008 ft; 00003 ft'/s, pa = 80 psi, determine the flow through each pipe and the pressure at B Why = (fYLID)(v*/2g). Assume Qi = 3 cfs. Cy(1)/4 Div 1)8.820)/0,00003 = 1.27 10° .001/1 = 0.0010 From Fig. A-5, f= 0.022. (h)j = (0.022)(F"){3.8207/{(2)(32.2)]} = 14.95 ft. For pipe 2, assume f; = 0.020. 14,95 = 0,020{2000/(#5)}{us"/[(2)(32.2)]}_vj = 4.006 ft/s (Na) = (#4)(4.006)/0.00003 = 8.90 x 10° €_/Dz=0.0001/(%)=0.00015 f= 0.019 14.95 =0.019(2000/(8)](w!/1(2)(32.2))) 4.110 8/s = 1() (8/41 4.110) = 1.43 £75 in, €: = 0.0001 ft; Ly = 4000 ft, D, = 16in, 00 f, z» = 80ft. For a total flow of 12 cfs, Ne 204 0 CHAPTER 11 m2 Mig. 119 For pipe 3, assume f; = 0.020. 14.95 = 0.020(4000/(H)]{w#/12)(32.2)]} v= 4.006 ft/s (Nas = (8)(4.006)/0.0000 = 1.78 x 10" €,/Ds=0,0008/(H2) = 0.00060 f=0.020 5 = (n)(1)*/4](4.006) = 5.59 40/5 EQ =34+1.43-+5.59= 1002%5 = (8/10.02)(12) = 3.59 '/s Qs = (1.43/10.02)(12) =1.71 8/8 Qy= (6.59/10.02)(12) = 6.69 8°75 ‘Check the values off, fh, and hy v= 3.59/{(R)AP VA] = 4.571 ft/s (Nq)y= (1)(4.571)/0.00003 = 1.52 x 10° O21 (hy ).= (0.021) EY (4.5711) {22))) = 20.44 v2= TGV] = 4.899 5 (eds ()(4-899)/0.00003 = 1.09% 10" f= 0.018 (hs =0.018(2000/(i8)](4.899"/[(2)(32.2))) = 20.12 5 6.69/[()(H)"/4]= 4.791 fe/s (a= (¥)(4.791)/0.00008 = 2.13 x 10° f= 0.019 (hy) =0.019{4000/(H6)](4.7917/[2)(32.2)]} = 20.32 ft Since these values of h are close, accept the ow values determined above and take an average value of (20.44 + 20.12 + 20.32)/3, of 20.29 ft for calculating py: paly + 03/28 + 24 ol + Vs/2¢ + Ze +: (80)(144)/62.4-+ v3/2g + 100 = (po)(14)/62.4-+ v3/2g +80 + 20.29, vi/2g = 03/28, pp = 79.9 b/ [os Prepare a computer program to solve for the head drop and flow distribution through N pipes in parallel. Check it out with the data in Prob. 11.11. Tso nen 9: ‘SOLUTION OF PARALLEL PIPE PROBLEMS 38 BEEING tame ber evocos 02,03, 0n0 bi sBieLontbzeb3/00 m3 m4 mas a6 PARALLEL PIPELINE SYSTEMS 1 285 ‘Two pipes are connected in parallel between two reservoirs: L, = 2600 m, D, = 1.3m, C= 90; L, = 2400m, D,=0.9m, C= 100, For a difference in elevation of 3.8m, determine the total low of water at 20°C. ' y= 0.8492CR"s"™ 1¥, = (0.8492)(90)(1.3/4)"(3.8/2600)"* = 1.109 m/s. ‘v5 = (0.8492)(100)(%2)"(3.8/2400)" = 1.020 m/s = Av = {(x)(1.39°/44(1.109) + [()(0.9)7/4](1.020) = 2.12 m/s For 5.4 m°/s of flow in the system of Prob. 11.13, determine the difference in elevation of reservoir surfaces. ' = Av=0.8492ACR°s"* 5.4 = 0,8492{(-)(1.3)7/4](90)(1.3/4)°s° + 0.8492{(27)(0.9)°/4](100)(%2)° 5° 5 =0,008455 m/m A elevation = (0.008455)(2400) = 20.3 m ‘Three smooth tubes are connected in parallel: L, = 45 ft, D, = in; Lz=55 ft, D.=1in; Ly = 60ft, Dy = tin. Given Q = 33 gpm, y= 551b/ft, and y= 0.13 Pas, what is the drop in hydraulic grade line between junctions? = 28uL@ : ay" ‘Therefore, L,Q,/Di = LQ3/D$= LsQs/D3. = 2, +0, +,= (33(0.002228)=0.07352A/s _450,/{()/12}"= 550,/(8)" = 0.076390, 55Q,/(8)'= G0Qs/(G)/12}¢ — Qs=0.29000; 0.076390, + 0; + 0.29000, = 0.07352 Q,= 0.08381 f°/s (0.13)(0.3048)? __(128(0.002716)(55)(0.05381) 448 @cxiy Ae =0.0027161b- s/f? hy IH = 11m, 4 =8X 10 Pa-s, and s.g. =0.9 in Fig. 11-16, find the discharges through pipes 1, 2, and 3. ' y= MLIDW"I28) (ED), =0.006/0.050=0.120 f= 0.114 (€/D),=0.009/0.120= 0.075 f=0.088 —(€/D),=0.012/0.100=0.10 — f=0.118 (y= (ye = Ql, = Qu((0.050)"/4] = 509.30, v= Qa A= Qa/{(-)(0.120)'/4] = 88.420, (0.114{70/0.050}{(509.30,)"/[(2)9.807)}} = 0.08880/0.120}{(88.420;)*/(2\(9.807)}) 2:=9.5002, —-Qs= 0, + 2.= Q, +9.5002, = 10.5000, U5 = QsfAs= 10.500Q,/[()(0.100)7/4]=13370, H= (y+ (ys 11 =0.114{70/0.050}{(509.3Q,)7/[(2\(9.807)]} + 0. 114{110/0. 100}(13370,)7/[(2)(9.807)]} 000901 m'/s=0.901 L/s Q,=(9.500}(0.901)=8.56L/s Qs = (10.500)(0.901) = 9.46 Lis = 402 _ (4(0:00546)(0.9(1000) Du ~ (x)(0.100)8 x 107) le 13550 Hence, both f’s are O.K. 286 0 CHAPTER 11 ma7 118 Mig. 1-11 Find H in Fig. 11-11 for Q,= 20L/s, w= 5x10" Pa-s,.g.= 009. 1 0, and Q, will be in the same proportion asin Prob. 11.16. Hence, Q, = (0.901/9.46)(20) = 1.90 L/s, Qs (9.500)(1.90) = 18.0.L/s. H=(t)+ (is hy = PMLID\'28) 1¥, = Qs/A, =0.00190/{()(0.050)*/4] = 0.9677 m/s v5 = Qs/As = 0.020/[(=)(0.100)"/4] = 2.546 m/s Using values of ffrom Prob. 11.16, -H =0.114{70/0.050} {0.9677 /(2)(9.907)}} + 0-114[110/0. 100] (2.546?/[(2)(9.907)]} = 49.06 ‘What length of 300-mm-diameter, clean cast-iron pipe could replace the system of Fig, 11-12? I (€/D); =0.00026/0.200= 0.0013 (€/D), = 0.00026/0.300 = 0.00087 (€/D); = 0.00026/0.500 = 0.00052 (€/D)«=0,00026/0.300 = 0.00087 (€/D); = 0.00026/0.300 = 0.00087 Try fi= 0.023, f= 0.021, fi 0.018, r= 0.021, and f= 0.021: Assume a head loss in pipes 1 and 2 of 3m. hy = (F(LIDYu7/2g) 3 = 0.023[310/0.200}{v7/[(2)(9.807)]} v= 1.285 m/s Q1= Ayu, = [()(0.200)?/4](1.285) = 0.04037 m'/s 3 = 0.021[310/0.300]{v3/[(2)(9.807)]} vs = 1.647 m/s Oz = Azvs = [(-2)(0.300)?/4](1.647) = 0.1164 m?/s To find equivalent for 1 and 2, v, = (0.04037 + 0.1164)/[(2r)(0.300)"/4] = 2.218 m/s, 3= (0.021[L, /0.300]{2.218%/{(2)(9.807)]}, L, = 170.9m (for pipes 1 and 2). eee oN 00714) g(t) LOM. 3014) (2)(9.807) 0. 300, (209.807) L,=53.1m (Cor pipe 3) ‘Assume a head loss in pipes 4 and S of 3m. ‘3 =0.021(620/0.300]{vi/{(2\9.807)]) v4= 1.164 m/s Qa= [(")(0.300)?/4](1.164) = 0.08228 m"/s_ 3 =0.021{790/0.300]{v2/[(2)(9.807)]} y= 1.032 m/s ‘Qs = [(-2)(0.300)*/4](1.032) = 0.07295 m°/s To find equivalent for 4 and 5, v, = (0.08228 + 0.07295)/{()(0.300)?/4] = 2.196 m/s 3 = 0.021, /0.300](2.1967/{(2)(9.807)]} L,=174.3m (for pipes 4 and 5) cc 170.9 + 53.7 + 174.3 = 398.9 m_ Fig. 1-12 a9 1.20 m2 PARALLEL PIPELINE SYSTEMS J 287 For a velocity of 1.1 m/s in pipe 1 of Fig. 11-12, calculate the flow through the system and the head H required. ‘The fluid is water at 15°C. Using values from Prob. 11.18, hy = (f)(L/D)(u*/2g), (by)s=0.023[310/0.200]{1.1°/[(2)(9.807)}) = 2.199 m, (hy)a= (hy) = 0.021[310/0.300} (v3/[(2)@.807)]) = 2.199 m, v= 1.410 m/s; Q = Au = [(x)(0.200)*/4](1.1) + {(2e)(0.300}'/4](1.410) = 0.134 m/s. (This is the flow through the system.) v5 = Qs/As = 0.134/[()(0.500)'/4] = (0.6825 m/s, (Fs = 0.018{290/0,500}{ 0, 6825*/{(2)(9.807)}} = 0.363 m, The flow of 0.134 m/s will split in pipes 4 and 5 in the same proportions as determined in Prob. 11.18. Hence, Q4 = {0.08228/(0.08228 + 0.07295)](0.134) = 0.0710 m"/s, v4 =0.0710/{(x)(0.300)*/4] = 1.008 m/s, (a= (0.021{620/0.300}{1.004"/{(2)(9:807)}} = 2.230 m, H =2.199 + 0.363 + 2.230 = 4.792 m. ‘Three pipes are in parallel with a total head loss of 20.3 m. The pipe data are 100 150 30 0.24 | 0.003 oz | 0.002 0.20 | 0.005 pipe | Z,m | dem | ¢,mm | e/d 1 2 3 ‘The fluid is water, p = 1000 kg/m’, and v = 1.02 10-* m*/s, Calculate the total low rate in cubic meters per hour, neglecting minor losses. Hh, = (LIDY 28), (4) = (ty) ly). Guess fly rough flow in pipe 1: 4 =20.3 = 0.0262(100/(8)1(04/1@)2.80T]) vy =3.487 m/s Ne=Dv/v (Ne)s = (the)(3.487)/(1.02 x 107%) = 2.73 x 10° From Fig. A-5, f; = 0.0267. 20.3 = 0.02671100/(i) VI 2VO.80D]) v4 =3.454m/s Qh = Ayu = [(e)(hs)"/4](3. 454) = 0.01736 m/s = 0.0262. Guess f,= 0.0234: (hy), = 20.3 = 0.0234{150/(f5){v2/[2)(9.807)]} v2 = 2.609 m/s (Nada = (if5)(2-609)/(1.02 x 1 53X10 f= 0.0286 20.3 = 0.0246(150/ (16) {¥3/12)0.80))) v:=2.544m/s = [()(ia)*/41(2.544) = 0.007193 m"/s Guess f, = 0.0304: (hy)s = 20.3 = 0.0304{80/ (aia)]{v3/[(2)(9.807)])} vs = 2.559 m/s (We) = (5)(2.559)/(1.02 x 10°8) = 1.00 x 10" H=0.0813 20,3 = 0.0313480/(ri) vi/[2)9.807]} vs =2.522m/s (2, = [() Gis) /4)(2.522) = 0.003169 m/s ‘This is satisfactory convergence. Hence, Q.nu:= 0.01736 + 0.007193 + 0.008169 = 0.02772 m’/s, or 99.8 mh For the parallel pipe system of Fig. 11-13, each pipe is cast iron, and the pressure drop p, — ps equals 2.5 psi ‘Compute the total flow rate between 1 and 2if the fluid properties are y = 57.321b/ft, m = 0.00606 Ib -/ft. Hh, = (p.—pdy = (25)(144)/57.32 = 6.281 ft, Assume laminar flow. 1284LO yD" For the 3-in-diameter pipe: (128)(0.00606)(225)(0.) E732 5 Qs/Ay = 0.02531/[()(3)?/4] =0.5156 4/3 (Ng)s = (57.52)(3)(0.5156)/(0.00606)(32.2) =38 (laminar) yDv 6.281 = = 0.02531 f0/s Nye 2 mg 288 0 CHAPTER 11 m2 n23 ‘For the 2in-diameter pipe: = (128)(0.00606)(185)(05) - 08 era (2,= 0.006081 fs vs = 0.006081/{()(4)?/4] = 0.2787 fs (Na)s = (57.32)(3)(0.2787) (0.00606)(32.2)= 14 (laminar) Qui = 0.02531 + 0.006081 = 0.0314 "7 Mig. 11-13 ‘The system of Fig. 11-14 consists of 60-mm-diameter, wrought iron pipes containing water at 20°C. If the total flow rate from 1 to 2 is 13 L/s, compute the total pressure drop p, ~ p:. Neglect minor losses. 1H Forpipe cz 2, = Qc/Ac=0.013/[(x)(0.060)/4] =4.598 m/s Ne=dv/v (Wn) = (0.0609(4.598)/(1.02 x 10"9) = 2.70% 10° ¢/d = 0,000046/0.060 = 0.00077 From Fig. AS, f= 0.020 hy = QULIANWI28) (by). =0.020(175/0.060){4.598*/12)9.807)}) = 62.88. (Ap). = (7Mhy). = (-79)(62.88) = 616 kPa u/ A= Qul( N14] = 383.70. Us = Osa = OuM a) 4] = or pipes a and b: (hy). = (os Ve 353.7 £.[240/0.060) (353.70.)*/[2)(9.807)]} = fa120/0.060) ((353.72.)*/4(2)(9.807)]} Y Iff,=fy, OQ, =0.7071Qs, Q. + Qs = 1.70710, = 0.013 m/s, Q, = 0.007615 m°/s, Q, = (0.7071)(0.007615) = 0.005385 m/s; (Nr)e = s(353.7)0.005385)](1.02 x 10-*) = 1.12 x 10 f, = 0.0219; (Np)p = sfel(B53.7(0.007615}]/(1.02 x 10-*) = 1.58 x 10°, f, = 0.0208. Substituting these values of finto Eq. (1), 0.0219{240/ )]{353.70.)"/{2)(9.807)}) = 0.0208{120/(sf)] {(353.70s)"/1@)(9.807)}), O = 0.68910, 0. + 2, =0.68910, + 2x = 0.013, Q, = 0.007696 m"/s, Q, = (0.6891)( 0.007696) = 0.005303 m'/s. ((¥x)x = 0.060((353.7)(0.005303)/(1.02 x 10-4) = 1.10 10° (Wa = 0.060{(353.79(0.007696)]/(1.02 x 10-*) = 1.6010" fy Hence, Q, = 0.005303 m/s and Q, = 0.007696 m'/s. (hy)a= (hyo = 0.0219(240/0.060]{[(353.7)(0.005303)} /12)(9.807)]} = 15.71 m (Ap)ax = (0-79)(15.71) = 154kPa (Ap) = 616-+ 154= 770 KPa =0.0219 (O.K.) ons (OK) Reconsider Prob. 11.22 with the flow rate unknown but with the total pressure drop given as 850 kPa. Find the resulting flow rate Q. I Using data from Prob. 11.22, guess 0. (0.008707 = 0.014707 m'fs, v (0.060)(5.202)/(1.02 x y= LIA 28), 0.006-+ 5.202 m/s, Nu = dv/, (Na). = .00046/0,060 = 0.00077. From Fig. A-5, f= 0.0195: 5 202/12),9.807)]} = 78.47 m, PARALLEL PIPELINE SYSTEMS 289 Fore eam jem the ¥. OulA, = 0.006/(5)(0.089'/4) =2.122./5 Up = Qn/An = 0.008707 /{(2)(0.060)'/4] = 3.079 m/s (Wa)a = (0.060(2.122)/(.02 x 10%) =1.25 x10" f= 0.0205, 81x 10° ‘f,= 0.020 ,0205{240/0.060} (2.122?/{(2)(9.807)}} = 18.83 m .020{120/0.060] {(3.0792/[(2)(9.807)]} = 19.33 m Since these are fit close, forego anther iteration and take an average value of (yes: (18.83 + 19.33)/2 = 19.08 m, (hy ous = 78.47 + 19.08 = 97.55 m, (SPuna = (YN nce thi i sighty larger than the requied 850 KPa, reduce Qa small amount, 0.00565 m’/s: Qs = 0.00565/0.6891 = 0.008199 m*/s_ Q. = 0.00565 + 0.008199 = 0.013849 m*/s 1.1. = 0.013849/|(*)(0.060)"/4] = 4.898 m/s (Na). = (0.060)(4.898)/(1.02 x 10-*) = 2.88 x 10° 0195. (hy )e = 0.0195[175/0.060] 4.898" /[(2)(9.807)]} = 69.57 m_ (9.79)(97.55) = try Q41A, = 0.00565/[(2)(0.060)7/4] = 1.998 m/s vp = Qy/Ay = 0.008199/{(7)(0.060)°/4] = 2.900 m/s (Wa)u = (0.060)(1.998)/(1.02 x 10°) = 1.18% 10 f, = 0.0205 (Wado = (0.060)(2.900)/(1.02% 10") =1.71 x10" fy = 0.020 (Ju 0.0205{240/0,060) {1.998"/[(2)(9.807)]} = 16.69 m (hy a= 0.020{120/0.060}{2.9007/[(2)(9.807)}} = 17.15 m Since these are fairly close, forego another iteration and take an average value of (hy), (Jaw (16.69 + 17.15)/2= 16.92 m, (oy = 69.57 + 16.92 = 86.49 , (AP ana = (9) a= (9.79)(86.49) = 847 KPa (close enough). Hence, Q = Q, = 0.0138 m'/s= 13.8Ls. 11.24 For the piping system of Fig. 11-15, all pipes are concrete with roughness of 0.03 in. If the flow rate is 23 cfs of ‘water at 20°C, compute the pressure drop p, ~ p; and the three volumetric flows. 1 For pipe 4 = QA, = 23/[(A)BY/4] = 29-28 ft/s, Ne = Div (Ne)e = (13)(29.28)/(1.08 x 10") =2.71 x 10° €/D = 0.03/12 = 0.0025 From Fig. A-S, f, = 0.0249. hy = (f)(L/D)(v"/2g), (hy a= 0.0249{1200/(12)}{29.28°/[(2\(32.2)]} = 397.8 ft. For pipes b, ¢, and d: (ae=(de= (ya 9 = Qul Ay = Qo/I(H)(H)"/4] = 2.8650, 0. QdAc= Q.(HVEY 4) = 1.2730. QulAy = Qull()(R) 14] = 0.81490, AR Gee Peers An) we Q,= 1.4530. Qs + Q. + Oy = 0.22090, + Q. + 1.4539, = 23 Qp = (0.2209)(8.602) = 1.900 ft’/s Q, = (1.453)(8.602) = 12,50 ft/s (q)o= U2. 865)1.900)]/(1.08 x 10") =3.36 x10" (€/D)y =0.03/8 = 0.0038 {[(1.273)(8.602)]/ (1.08 x 10-*) = 1.01 x 10% (€/D). = 0.03/12 = 0.0025 . (vq)a H1(0.8149)(12.50)/(1.08 10-7) = 1.1810 (€/D),= 0.03/15 = 0.0020 f,=0.0235 Substituting these values off into Eq. (1), 0.028(1400/()]{(2.8650,)*1(2)(32.2)}} = 0.0251900/()]((1.2730.)'M12)32.2))) = 0.0235{1300/(1]((0.81490,)/12)62.2)1} Q, = 0.27499, Q, = 1.4999, Op + Q. + Q, = 0.2749Q, + Q, + 1.4999, = 23 Q. = 8.292 fe /s Qs = (0.2749)(8.292) = 2.279 ft/s Qa = (1.499)(8.292) = 12.43 ft/s. 290 0 CHAPTER 11 1.28 11.26 (Nn)o = Bl(2.865)(2.279)}/(1.08 x 10") =4.03 x10" f=0.028 (O.K.) (N)e= #{(1.273)(8.292)]/(1.08 x 10-*) = 9.77 x 10° 0S (OK) (Me)a= Hl (0.8149)(12.43)]/(1.08 x 10-4) = 1.17 10" 035 (OK) Hence, Q, = 2.28 f/s, Q. = 8.29 t/s, and Oy = 12.43 ft/s Cipn= (hy e= (tye (x= 0.028(1400/(4)]{{(2.865)(2.28)F (2)(82.2)]} = 39.0 Cpt = 397.8 + 39.0 436.8 ft (Pana = (7) (ym = (62.4)(436.8) = 27256 b/f or 189 Ibi? Drain eI Chadon Pisce Fig. 11-15 For the system of Fig. 11-15, with vegetable oil (v = 4.62 x 10 *ft'/s, y =42.51b/f?) as the fluid, compute the flow rate in all pipes ifthe pressure drop p, ~ pz is 44 psi. Neglect minor losses. I Using data from Prob. 11.24, note that pipe a takes 397.8/436.8 = 0.9107, or 91.07 percent of the total pressure drop. Hence, take (Ap), = (#4)(144)(0.9107) = 5770 lb/fP, (i). = (Ap)e/y = 5770/42. f= (f (LLaNo? 29) 135.8 1200/2) (02 2)32.2)), ve VF-28 Tey f= 005 7.07 ft/s, Ne= dv/v, (Na)a= (HB)(17.07)/(.62 x 10°*) = 3.69 x 10, (¢/d), = 0.03/12: . Oats yf 020% 097 DHIO OOS 17 Doe (Mel AHN 25/e4dB x 10") ~ 9.79% 18, 0245 (O.K.); ()(H)*/4](17.25) = 13.55 ft/s. Ths flow of 13.55 ft/s for ©. will divide ‘among the three parallel pipes inthe same proportions as found in Prob. 11.24. Hence, Qs = [2.28/(2.28 + 8.29 + 12.43)](13.55)= 1.343 0/5 Qc =[8.29/(2.28 +8.29 + 12.43)](13.55) = 4.984 fs Qe [12.43 /(2.28 + 8,29 + 12.43)](13.55) = 7.323 ft/s .343/[()(1)°/4] = 3.847 fs (Cld)p=0.03/8=0.0038 (Ny), = (AGATA. X10) =5.55x10° f= 0.028 pone 0.028{1400/ (413.847 /[232.2)]} = 13.51 ft (APYo a= (42.5)(13.51) = 974 Ib/AE (Ap ani ™ 5770 + 574 = 6344 Ib/f? or 44.1 Ib/in® (0.K.) 3h.chs, O, = 4.88 fs, and Oy = 7.32 cfs Hence, Q, = 13.55 efs, In Fig. 11-16 all pipes are cast iron and p, ~ p= 49 psi. Compute the total low rate of water at 20 ! hy = 49)(148)/62.4 = 113.1 = (FY(L/A\*/28) For pipe e: (hy)e= 113.1 = f(2000/()l(v2/{(2)(32.2)]}, ve = VO.BI057f. Try f. = 0.03: v.= VO.9105/0.03 = 5.509 /s, Ny = dv/, (Na). = (3)(5-509)/(1.08 > 10-*) = 1.28 x 10%, (€/4). = 0.00085/( 78) = 0.0036. From Fig 0.910570. 672 fs, (Nq)e= (3N(5:672)/(1.08 x 10-4) =1.31 x (2) (3) 145.672) = 0.2784 #75, 113.1 = f,[1200/(2)]{wi/(2)(32.2))} + f1300/(4)](02/[@)G2.2)). From san tes tCNS22) + E3008) Co AQI22, 74 = 7284 (vi)[(7200)(0.03) + (243.8)(0.03)] vy =5.71fR/s (Nya (BMS.TIN)/(.08 x 10-4) = 8.81 > 10" (€/d),=0.00085/() = 0.0051 f,=0.031 uy = S.711/4 = 1.428 /s (as = (40.428) /(1.08 « 10) = 4.41 > 10" (€/d)y=0.00085/(4) = 0.0026 f, = 0.028 ‘Try f, = 0.081 and f, = 0.028: 7284 = (v2)[(7200)(0.031) + 243.8)(0.028)]_v, = 5.627 f/s (a). = ()(5.627)/(1.08 X10") = 8.68 x10" f,=0.081 (OK) Y= S.627/4= LAOTEAIS——(Na)o= (4)(1.407)/(1.08 10") =4.34x 10" fy = 0.028. (OK) 2. = [(2)(8)/4]6.627) = 0.1228 10/5 Qua = 0.2784 + 0.1228 = 0.4012 f0/5 contin 1/4 MBA (v2)(7200f, + 243.84). Try f, PARALLEL PIPELINE SYSTEMS J 291 syat3007, aintaoon. ¥ eo in 20008 Fig. 11-16 1.21 Refer to Prob. 11.26. Given Que = 0.7 f?/s, compute the pressure drop p, ~ p; and the division of flow between the two paths. I The flow will divide between the two paths in the same proportions as found in Prob. 11.26. Hence, Q, = [0.2784/(0.2784 + 0.1228)|(0.7) = 0.4857 fe/s Oy = Op = [0.1228/(0.2784 + 0,1228)](0.7) = 0.2143 ft/s hy = (LIU 12g) ve = OIA, = 0.4857/[(A VAY /4] = 9.895 ft/s Nq=dv/y (Nnde= (3)(9.895)/(1.08 x 10%) =2.29 x 10 (€/d),™ 0.00085/() = 0.0034 From Fig. A-S, f. = 0.028. (fy). = 0.028{2000/( #3)}{9.895*/[(2)(32.2)]} = 340.6 ft, ps — p2=(Y(hye= (62.4)(340.6) = 21 253 lb/ft, or 147.6 1b/in®. 11.28 Two cisterns are connected by cast iron pipes as in Fig. 11-17. Neglecting minor losses, compute the flow rate in cach pipe for water at 20°C. B Assume (4,).= 201. aaah, |L a, = (32.2)((4)°(20)/[(1300)(1.08 x 10-*)"] = 1.573 x 10" (Ne)e= —V@(LSTIX AP) og [0.00255/3.7 + 2.51/ VOLS IP) 1.093 x 10° = 49. /[()(1.08 x 10)(4)]_-Q. = 0.3090 ft/s (hy) &, = (32.2)(4)°(39)/[(900)(1.08 x 10-*)"] = 1.869 x 10° ,00085/ (7) = 0.00340 (Na)e= -V@}C-BH X10} Jog [0.00340/3.7 + 2.51/ VENT. 869 x 10] = 1.152 X 10° 1.152% 10°= 40, /[(2)(1.08 x 10-4] Q, = 0.2463 /s «x5 = (82.2)(8)*(9)/(600)(1.08 x 10-*)] = 8.308% 107 _(c/d), = 0,00085/(8%) = 0.00510 (Nao = VEE HBX 1) log [0.00510/3.7 + 2.51/ VQYEIOEX TV) = 7.227 x 10" 1.221 x 10" 4, /I(4)(.08%10-*V()] Qs = 0.102245 Qu+ Oy = 0.2443 + 0.1022 = 0.3465 f0/5 3465 ft/s is not equal to Q. = 0.3090 0/s, the assumed value of (hy). = 20 tis -Vfarlog [(€/d)/3.7 + 2.51/20} .00085/ (i) .093 x 10° Since this value of Q, + Q, incorrect. Try (Iy)-=23.3 ft: 1 = (82.2)(4)*(23.3)/1(1300)(1.08 x 10°*)"] = 1.833 x 10" (Ne). —V@EBIIN AP log [0.00255/3.7 + 2.51/V@)T RES TOF) = 1.182 10° 1.182 x 10" = 4Q,/{(4)(.0810"°)(4)] Qe = 0.3341 f/s Ciyda= (iyo = 99=23.3= 35.7 4, = (32.2)(3)°85.7)/1(900}(1.08 x 10™))] = 1.711 x 10" (Na)e = —V(BML.THI & 107) log (0.00340/3.7 + 2.51/ V(2)(L.TIL x 107] = 1.101 x 10° 1.101 x 10° = 4Q,/{(4)(1.08%10"4)(3)]Q, = 0.2336 4/5 4% = (32.2)(8)*(8S.7)[(600)(1.08 x 10°*)"] = 7.604 x 10° (Na )e= -V@VUT-HAH AV log 0.00510/3.7 + 2.51/ VQNTBOTX TO} = 6.908 x 10" 6.908 x 10" = 4Q,/[(=)(1.08% 10°2)] = O.0977F'/s y+ p= 0.2336 + 0.0977 = 0.3313 f0/5 ‘This value of Q, + Q, = 0.3313 t'/s is practically the same as Q, = 0.3341 t0/s. Hence, take Q, = 0.235 fs, 098 f"/s, O, = 0.333 ft/s. 292 0 CHAPTER 11 u29 11.30 131 1.32 Ain. goon 59" ZinGoot 4.43008 Fig. 11-17 For the pipe system in Fig. 11-18, the pressure head at Ais 120.0ft of water and the pressure head at Eis 7208 of water. Assoming the pipe are ina horizontal plane, what are the flows in each branch of he loop? Net hy=1200-720=480f — Q=1.318ACR*SS# Qn = 1.318 (-")(H8)"/4](100)|(8)/4]°(48.0/12 000)" = 2.19 ft/s. Qe = 1.318{())/4](100){(8)/4P°(48.0/4000)" = 1.37 £07 Qn = 1.3184 %)(H3)*/4](100)[(18)/4]°*(48.0/8000)"* = 1.69 ft/s Troon eee D. FoR = toe Fig. 11-18 In Fig. 11-18, ifthe total flow is 6.50 mgd, how much head loss occurs between A and E and how does Q divide in the loop? Tre total low wil divide inthe loop inthe same proportions as in Prob. 11.29. Hence, Qs = [2.19/(2.19 + 1.37 + 1.69)](6.50) = 2.71 mgd, or 4.20 cf8; Qc = [1.37/(2.19 + 1.37 + 1.69)](6.50) = 1.70 mgd, or 2E2efs; Op =|1.69/(2.19 + 1.37 + 1.69)\(6.50) = 2.09 mgd, or 3.24 chs; Q = 1.318ACR™*:°™., For pipe B, 4.20 = 1.318f()(8)"/4](100)[(2)/4P-°s", s = 0.013345 (iy) = (0.01334) (12.000) = 160 ft. This isthe head loss between A and E. It should, of course, be the same when computed by pipes C and D. To confirm this, for pipe ©, 2.62 = 1.318{(4)(8)'/4] LOO)[(E)/4 Ps", s = 0.04010; (4) = (0.04010)(4000) = 160. For pipe D, 3 ab 1 3162 (197/41 Q0OICR)/AP SE, s = 0.02005; (n= (0.02005)(8000) = 160 fe. For the system shown in Fig. 11-19, what flow will occur when the drop in the hydraulic grade line from A to B is 20087 H Assume (Hy)wz = 30ft. Using Fig. A-13: For pipe 2, with d = 12in and hy = ss = 0.0060, Q,~= 3.2 ft/s. For pipe 3, with d = 16in and hy = xis = 0.0100, Q, = 9.0 ft/s. For pipe 1, with d = 24 in and Q =3.2+ 9. 12.2 ft/s, (hy): .0024 ft/ft; (iy )aw = (0.0024)(10 000) = 24.0 ft. For pipe 4, with d = 20in and Q = 12.2 t'/s, (hse= 0.0060 f/f; (hy) 2» = (0.0060)(8000) = 48.04, (,)en = 24.0+ 30 + 48.0 = 102.0. This value of aw 0 + 30 + 48.0 = 102.0 ft is not equal to the given value of 200 ft, but the actual head losses will be in the same proportions as those above. Hence, (t,)4w = (24.0/102.0)(200) = 47.06 ft. For pipe 1, with d = 24in and h, = 47.06/10 000 = 0.0047, Q = 18 cfs. Fig. 11-19 In Prob. 11.31, what length of 20-in pipe with C= 120 is equivalent to section A-B? From Prob. 11.3, itis known that a flow of 18cfs is produced when the drop inthe hydraulic grade line fom A to Bis 200 t. For 18efsin a 20-n pipe, from Fig. A-3, h, = 0.012 R/f, 0.0122, = 200, Ly = 16700 : : m3 ust PARALLEL PIPELINE SYSTEMS 0 203 In Fig. 11-20, when pump YA delivers 5.00 cfs, find the pressure heads at A and B. Draw the hydraulic grade lines. I Reduce loop BC to an equivalent pipe, 16in in diameter, C = 100. By so doing, a single-size pipe of the same relative roughness is readily handled for all conditions of flow. Assume a drop in the grade line of 22f¢ fiom B to C. Q = 1.3184R°%s"*, For the 10-in pipe: Qo = 1.318{( 7) 8)*/4](90)[(1)/4)° (22/10 000)°* = 0.8843 cfs. For the 8-in pipe: O, = 1.318{(-)()"/4](100){()/4]° 22/11 000)" = 0.5190 cfs. Qu = 0.8843 + 0.5190 = 1.408 cfs. For a 16-in-diameter equivalent pipe with C= 100, 1.403 = 1.318{(-)(#)*/4](100)[(8)/4y"5°, 5 = 0.0004313 ft/t; 0.0004313L, = 22, L, = 51 000 ft. For a I6in-diameter pipe from A to C with a length of 16 000 + 51 000 = 67 000ft and carrying 5.00 cfs, 5.00 = 1.318{(=)(H8)*/4](100)[(H)/4]°*s°™, s = 0.004537 t/t; ‘hy = (0.004537) (67 000) = 304.0 ft, Thus the elevation of the hydraulic grade line at Ais 217.0-+ 304.0 = 521.0 ft, as shown in the figure. The drop from A to B is (0,004537)(16 000) = 72.6ft and the elevation of the hydraulic ‘rade line at B becomes 521.0 = 72.6 = 448.4 ft. Pressure head at A = 521.0 ~ 50,0 471.0 ft, pressure head at B= 48.4 ~ 50.0= 398.4. Fig. 11-20 In Fig. 11-21, which system has the greater capacity, ABCD or EFGH? Use C= 120 forall pipes. HF Assume Q =3 cfs in ABCD. Using Fig. A-13, O14 A/F (hy)an= (0.0014)(9000) = 12.6 0.0053 ft/ft (hac = (0.0053)(6000)= 31.8 ft (hea = 0013 f/f (han = (0.013)(3000) = 39.0 (uuu =12.6+ 31.84 39.0" 83.4f¢ (for ABCD) For EFGH, assume (hy)rc= 24 ft. (hdc v= 0.00480 ft/Ft (Jno ss = 0.00343 t/t Ore Hence, pipe FIG carries 0.97/(0.97 + 1.6) = 0.3774, or 37.74 percent of the flow and pipe FIG carries 1.6/(0.97 + 1.6) = 0.6226, or 62.26 percent. For Q = 3.0 fs in pipe EF, (der 0.00074 f/f (hen = (0.00078)(11 000) =8.1F Ona = (0.3774)(3.0) (dre = 0.0060 8/8 (hydric (0.0060)(5000) = 30.0 ft For Q = 3.0cfsin pipe GH, (hon = 0.013 81 Chon (0.013)2500)= 32.51 (hun = 8.1 +30.0+32.5= 70.64 (Cor EFGH) Since EFGH carries the assumed flow of 3.0 fs with a lesser head loss than ABCD, it (EFGH) has the greater capacity 11,000'—18" 2500'—10" 204 0 CHAPTER 11 11.38 1136 ‘Three pipes A, B, and C are interconnected as shown in Fig. 11-22. The pipe characteristics are as follows: pee | Doin | GR | S a | 6 | 2000 | oom 2 | 4 | 10 | aos c | 8 | a | oom Find the rate at which water wil ow in each pipe. Find also the pressure at point P. Neglect minor losses. ' ily + Vil2g + 21= poly + VH2g +224 hy Q Ing = hy = (L fd? 2g) = 0.020(2000/(#) (v5/12)G2.2))) + 0,024[4000/(8)]{v2/{(2)(32.2)]} = 1.242v4 + 2.23602 0-+0+ 200 = 0+ wi/[(2)(32.2)] + 50 + 1.2420, + 2.23602 150 = 1.242v% + 2.25202, (2 Qt Qn= Oe [HNN U4) + [NEA = (NEY) 0.19630, + 0.08727v» = 0.349106 @ Gy)a= Cyn 0.020{2000/( {v5 [(2)32.2))) = 0.032[1600/(4)}(U5/[2)32.2)]) ve = 0.721704 Substituting into Eq. (3), 0.1963t1, + (0.08727)(0.7217u4) = 0.3491ve, v= 1.346ve. Substituting into Eq. (2), 242) 1.460) + 2.2520, Yom 5.7ILHIS; Qe = Acve= [(A)(E) 145.772) =2.01 fP/s, Ua = (1:346)(5.772) = 7.769 ft/s, Qa = [(a)((9/4](.768) = 1.53 f/s. Substituting into Eq. (3), (0.1963)(7.769) + (0.3491)(5.772), vu = 5.614 1/5; On = [(n)(i)*/4](.614) = 0.490 ft/s. To find pp, apply Eq. (1) between points P and 2. (p-)(144)/62.4 + v3/2g + 1200 + vi/2g + 50-+ 0.024{4000/(B)]{S.772°/[2N32.2)}) vb/2g = v2/2g, pe= 1.95 Ib/in*. he Fig. 11-22 Figure 11-23 shows an element of a sugar refinery. Smooth brass pipes 1, 2, and 3 are 550 ft of 2-in diameter, 350 tof 3-n diameter, and 600 ft of 4-in diameter, respectively. If the net flow of molasses (v = 6.5 x 10"* ft/s) is 0.7 f0/5, find the head loss from A to B and the flow in each pipe. ' hy = (PULIAv7I28) y= Oy )a= (ts S(550/( BH i/{(2)32.2))) = £3501 v2 (2)2.29} = LL600/ (v3 @NS2.291} @ Ith=ha he SL.24u?=21.7403=27.95v3 v= 1.535y, vy = 1.354 Quon = Qi + Qs + Q3= AsV + ABs + Adds 0.7 = (C2) (BY/4] 00) + [C(14]CL 53509) + (C2) 4)"/4]01.3540,) uy = 3.251 ft/s v,=(1.535)(3.251) = 4.990 ft/s vy = (1.354)(3.251) = 4.402 ft/s Ng= duly, (Na)s = (8)G.251)1(6.5 x 10°*) = 8336, (Nao = ()(4.990)/(6.5 x 10°*) = 19 192, (Nr)s= (4)(4.402)/(6.5 « 10°*) = 22547. For Reynolds numbers in this range the Blasius formula can be used to determine friction factors: f = 0.316/N™. f= 0316/8336 =0.0331 f= 0.316/19 192°" = 0.0268 f,= 0316/22 547°* = 0.0258 Substituting these values of finto Eq. (1), 0.0331550/(&)]{wi/(2)(32.2))) = 0.0268(380/()}(v3/[2)32.2)]) = 0.0258(600/()](¥3/[2)32.2))) 1.6960 = 0.5826u= 0.721103 v,= 1.706, v5 = 1.5340, PARALLEL PIPELINE SYSTEMS 0 295 0.7 = [CyB /AICU) + U6)" /ACD. 7060.) + [V/A 5340) vy = (1.534)(2.918) = 4.476 fs v=ZOIB Ets va= (1.706)(2.918) = 4.978 fs (Ma = BN291B) (6510-9) = 7482 Na = BYA97HV(G.S X10 = 1946 (Na) = (8)(4.476)1(6.5 X 10°) = 22.954 f= 0316/19 146" = 0.0269 f= 0.316/22 954" = 0.0257 1(2)(32.2)}) = 0.02691350/ HDI U3MIG220 fi=0.316/7482 = 0.0340 (0,0340{850/(B)1{04/ = 0.02571600/(4)}(¥3/(2)G2.2))} 1.742y3 = 0.584803 =0.718303 v= 1.726) y= 15ST Uy /4](0.5570,) 0.7 = UC )BY/AI 0) + (NAT C-72600 + 1NEY wpe2ssss y= (L-T2OYZABR)=4.985Hs = (55TI2888) = 4.497 Rls (Na)s = (BY{2-888)/(6.5 x 10-*) = 7405, (Nada = (2)(4.985)/(6.5 * 10 *)= 19173 (Va)s = (AY4.497)1(6.5 * 10-*) = 23.062 0,316/7405° = 0.0341 (O-K.) fr=0.316/19 173° = 0.0269 f=0316/23 062" =0.0286 (OK) Qy = [Cr)(BY*/41(2.888) = 0.0630 fe) Q2=[(#)(AY'/4](4.985) = 0.245 fe/s Qs [Cay(y?/41 4.497) = 0.392605 .0341[550/(3)](2.888/2)2.2)) = 14.64 (OK) C)an= (y= Tee Fig. 11-23 11.37 Repeat Prob. 11.36 for a flow rate of 0.07 ft/s. I Assame this low will be laminar. hy = 32v)(L/gD*)(v). Since (t= (Hy)2= (ty)o» '550v,/( 4) = 35005/(8)? = 6000/4)? v= 3.536, 5 3.6670, Quant = Os + 02+ Os Avs + Arts + AY 0.07 = [Cap CYAN.) + [ea B) HAIG. 53600) + [IEEYIAG.66709) ‘v5 = (8.667(0.1358) = 0.4980 ft/s Na= du Ly|D}= LavsID¥= LavslD3 v= 0.1358 t/s vs = (3.536)(0.1358) = 0.4802 ft/s From Fig. A2, ¥=6.5 x 10° £0755 (Nada = (B)(0.1358)/(6.5 % 10 (Ng)a = (2){0.4802) (6.5 107) = 1847 (Nn) = (4)(0.4980)/(6.5 * 10"*) = 2554 a= [(e)(4)°/4 0.4802) = 0.0236 f'/5 348 (laminar) (aminar) (laminar) y= [()(B)*/4( 0.1358) = 0.00296 fs Os= [Gr (4)°/4](0.4980) = 0.0435 fe /s Cyan = (ty) = B2Y(6.S 10-4) {550 2.2)(4) 110.1358) = 0.174 ft meters are changed to: 90 m of 30-mm (pipe 1), 120m of 40-mm (pipe 2),90m 11.38 Rework Prob. 11.36 ifthe para ‘m/s; 6 L/s net flow. cof 50-mm (pipe 3); ¥= 4.8% 10 296 0 CHAPTER 11 Assume Reynolds numbers between 3000 and 100.000, in which case the Blasius equation can be used to determine f: hy = (f)(L/d)(v"/2g), f = 0.316/N%®, Ne= du/v. Therefore, 0316_Y/Ly/v") _0.316y¢%Ly' Lyi te=[aaerelDG) Soe = needs (20)(v47%)/(0.030)'* = (120)(03)/(0.040)"* = (909(u5")/(0.050)'* v= 0.9597, vs = 1.382, Qu = Qs + Qs Os = AWW, + Ards + Ads 6 x 107 = [(%}(0.030)'/4](0.95970) + [(0.040)"/4](v,) + [()(0.0509°/4](1.3820.) Up=1.29Lm/s v= (0.959)(1.291) =1.239m/s v4 = (1.382)(1.291) = 1.784 m/s (u)1 = (0.030)(1.239)/(4.8 x 10-1) = 7.74 10? (Nq)o= (0.040)(1.291)/(4.8 10-*) = 1.08% 10" (q)s = (0.050)(1.784)/(4.8 x 10-9) = 1.86 x 10" Hence, use of the Blasius equation is O.K. Q, = [(-")(0.030)7/4](1.239) = 0,00088:m"/s or 0.88 L/s Q,= [(2r}(0.040)"/4](1.291) = 0.00162m’/s or 1.62L/s Qs = [(}{0.050)*/4](1.784) = 0.00350 m*/s or 3.50L/s chp, = O3USNA.8X 10°9(00)(1.239)"* (aa = Od y9 a0TY(0.030)"= 11.39 Repeat Prob. 11.38 fora net flow of 0.5 L/s; other parameters asin Prob. 11.38. Assume laminar flow, in which case the following equation can be used to determine head loss: hy = Gov \Ligd'\(v), hy «Lvl Chyr=(hy)x= (ips 90v4/(0.0309" = 120v,/(0.040)'= 90v,/(0.050)? 1, = 0.75000, vs =2.083%, Quon Qs + Qa Qs= AY + AsV. + AWS 0.5 x 10° = [(1}(0.080)*/4] 0.75000.) + [(x)(0.040)/4](v:) + ()(0.080}/4](2.0830,) 1¥,= 0.08508 m/s v, = (0.7500)(0.08508) = 0,06381 m/s _v»= (2.083)(0.08508) = 0.1772 m/s (Wes = (0.030)(0.06381)/(4.8 x 10) = 399 laminar) (Wes = (0.040)(0.08508)/(4.8 x 10-4 (laminar) (We)s = (0.050)(0.1772)/(4.8 x 10") = 1846 (laminar) (Q,-= ((#)(0.080}/4]0.06381) = 0.0000451 m"/s or 0.0451 L/s ‘22 [(4)(0.040)/4](0.08508) = 0,0001069 m/s or 0.1069 L/s {25 ((%)(0.050)"/4](0.1772) = 0.0003479m"/s or 0.3479 Ls Cyan = (ty) = B2Y(A.8 10-4) (90/{(9.807)(0.030)"]} (0.06381) = 0,0999m or Lem =7.91m 11.40 Figure 11-24 shows a network of cast iron pipes. Given a low of 24 ef, find the head loss from A to D. Neglect minor losses. 1H Q,= AN3R GN aTde) 0 Qa= CVhz, where C,= AgV2daTfak. Assume f= f= f= 0.019. C= [EF /AIVQVG22VBMO.019) 300] = 1.170. y= (a) BY MAIVEEDE) (OOTY LOOO}] = 0.8967 a= [HF MIN@OEL ATH MOOI TN] = 1.659 [1.170/(1.170 + 0.8967 + 1.659)](24) = 7.537 ft/s. Q = [0.8967/(1.170 + 0.8967 + 1.659)](24) = 5.776 f/s. .659/(1.170-+ 0.8967 + 1.659)](24) = 10.687 f'/s V2 = Qo/ Az = 7.537/()(8)"/4] = 7.050 ft/s ‘vy = 5.776/{(x)(B)'/4] = 7.354 ft/s ‘v¢= 10.687/{()(19)*/4]= 7.654 ft/s Na =du/v. (Wes (49(7.050)/(.05 x 10°4) =7.83 x 10° (as = @N7.354)/(1.05 x 10°5) = 7,00 10° (nda (H9)(7.654) (1.05 x 10-5) = 9.72 x 10° (€/d), = 0.00085/({3) = 0.00073 (€/d), = 0.00085/({3) = 0.00085 —_(€/d), = 0.00085/($) = 0.00064 PARALLEL PIPELINE SYSTEMS [ 297 From Fig. A-5; f:=0.0185, f,= 0.0195, and f, = 0.0185. x= [GNC /AIVQ}EZ2VC.0185)3300}] = 1.186 C= [C4 )(B)/AIV@)E22VE)IMO.0195)(2600)] = 0.8852 Ca= [CyB /AIV@)EZ2VTDMO.0185S200)} = 1.585 Q,= [1.186 /(1.186 + 0.8852 + 1.585)](24) = 7.785 ft/s (Qs ~ (0.8852/(1. 186 + 0.8852 + 1.585)](24) = 5.811 f°/s, (Qu= [1.585/(1.186 + 0.8852 + 1.585)](24) = 10.404 f/s ¥5= QalAz=7.78S/{(a\($4)/4] = 7.282 ft/s vy=5.811/{()(1)"/4] = 7.399 tes 404/{(2)(H)"/4] =7.451 ft/s (Na)a= (18)(7.282)/(1.05 x 10%) = 8.09 x 10° (Na)a = (H)(7.399)/(1.05 x 109) = 7.05 x 10° (Na)a= (19)(7-451)/ (1.05 x 10°*) = 9.46 x 10° fir 0.0185 f=0.0195 and f= 0.0185 (OK) Quant = Whig(Cr+ Cy+C,) 24 = Vi Jacl. 186 + 0.8852 + 1.585) (idac= 43.09% hy =hy = (F(L/d\(u?/2g) v= 24/[(=e)(R)/4] = 7.639 /s (Ng) = (H)(7.639)/(1.05 x 10 (1d), = 0.00085/(3) = 0.00042, = 0.0168 (u,), = 0.0168{3300/(%)](7.639"/(2)(32.2)]) = 25.12 ft vg = 24/{(x)(1)"/4] = 4.889 fe/s (q)s= (3) (4.889)/(1.05 x 1079) = 1.16 x10 (€/d),= 0.00085/(#2) = 0.00034 f,= 0.016 (hic)s = 0.016{5200/(H8)]{4.889"/[2132-2)]) = 12.35 (Jan = 25.12 + 43.09 + 12,35 = 80.56 ft 4610" s3008t, 520018, 30-in, 7 Fig. 11-28 1141 Find the diameter ofthe series equivalent ofthe three parallel pipes in Prob. 11.40. Bh, = (LIdy(v" 12g). Assume f = 0.016; v= Q/A=24/(xd"/4) =30.56/d? 43.09 = (0.016)(2600/4)((30.56/6°)*/[2)32.2)) d= 1.695 ft [Ng = du/v = (1.695)(30.56/1.695*)/(.05 x 10-*) = 1.72 x 10 ¢/d = 0.00085/1.707 = 0.00050 From Fig. A-S, f =0.017, Try f = 0.017: 48.09 = (0.017)2600/d) (30.56/47 110)32.2))} d= 1.716 ft [Nj = (1.716)(30.56/1.7167)/(1.05 x 10") =1.70%10° f= 0.017 (OK) Therefore, d= 1.716 t= 20.6in. na For the network of Fig. 11-24, determine the five pipe flows, given the head loss from A to D is 91 ft and all Pipes have f = 0.017. 1 hy = (JY LId)(v" 12g) = (JL 1d)" 2g”) = (JL 29d NOI xd" /4)} = (Bf /x°gLO"/A*) Hence, h=LQld® — (hyr=(ys= (ye 3300Q3/(18)* = 260003/(18)* = 320003/(11)° 2= 1.3050, Q4= 1.8500, Qs = Ds= O,+ Qs+ Qs = 1.3050, + y+ 1.8500, = 4.1550; (haw = (iy + (iyo (lis 298 0 CHAPTER 11 91 = {(8)(0.017)/{()*(82.2)})[300)¢4.155Q,)/() + (2600)(5) HCD + (5200)°4.1550.)°/08) Q,= 6338/5 Q,=(1.305)(6.33) = 8.2645 1.850)(6.33) = 11.7105, 5 = (4.155)(6.33) = 26.308 11.43 In Prob. 11.42, what percentage increase in the capacity of the system would be achieved by adding another 12.in pipe 2600 ft long between B and C? I The additional pipe from B to C is identical to pipe 3. Asin Prob. 11.42, fy =(BfleLO'I@) — = 1.3050, Q.= 1.8500 Q1= Q.= 0,+20s+ Q4= 1.305Qs +205 +1.850Q5= 5.1550, (hy)an™ Gy) (p+ (ps 91 = {(8)(0.017)/[(a)*32.2)}}{(3300)(5.155Q)°/(E)* + (2600)(Q4)° (C8) + (5200)(5.155Q)°1()] Q.=S.1f/s — Q= Qs=(5.155)(5.61) =28.921/s Increase in capacity = (28.92 ~ 26.30)/26.30 = 0.100, or 10.0 percent, 1.44 Compute the flows in al pipes of the system shown in Fig. 11-25. Pipe AB is 800 t long, 6in. in diameter, = 0.03; pipe w is $00 ft long, 4 n. in diameter, f = 0.02; pipe /is 700 ft long, 2in. in diameter, f = 0.04; pipe ‘CDs 300 t long, 4in. in diameter, f = 0.02, The tank surface is 84 ft above the free outlet hy = PML Iay(v' 12g). For B to C, (hy)u= (hye 0.02{500/()|(v2/12)132.2))) = 0.04{700/(®)I(UF/2VGZ2)— v4 = 2.3660, (5) 78100.) + [YC RY Aw) = (Ca) /812. 3660) + [CHC AN) 2+ Avs =O. + [(x)()/4(Q/0.2283) 90440 1 = (1 — 0.9044)(Q) = 0.09560 U, = Qu Ag = 0.90440 /[()(4)'/4] = 10.369, | tan Q/Aan™ QMCVEFI]=5.093Q 0, = (10.36)(u49/5.093)= 2.034049 hy =(hy)an + (tye (den 4 = 0,03{800/()} (val {(2)32.2))) + 0.02{500/()](v2/[(2)32.2)}) + 0.02[300/(i)](véo/(2)G22)}) [e=)08) van) = (NR Mco) Yen = 2.250049 {84 = 0.03{800/(#)](wn/((2)(32.2))) + 0.02[500/(8)]{2-034v4n)*/(@)322))) +0,02{300/(4)}((2.250%149)°/1@)G2.2))) ‘Uap = 4.533 ft/s. = Qs =[(%)(4)"/4](4.533) = 0.890 f/s, Qe ™ Qaa = 0.890 ft/s Q, = (0.9044)(0.890) = 0.805 ft'/s_Q, = (0.0956)(0.890) = 0.085 ft'/s Fig. 11-25 11.45 Rework Prob. 11.44if pipe Ais 400 m long, 200 mm in diameter, and f = 0.03; pipe u is 300 m long, 100 mm indiameter, and f = 0.02; pipe 1's 200m long, 150 mm in diameter, and f = 0.025; pipe CD is 700m long, 300 mm in diameter, and f = 0.018; the tank surface is 94 ft above D. FGy)a= (hyde 0.02[300/(0.100)]{v2/[(2)(9.807)]} = 0.0251200/(0.150)}{vi/[2)(9.807))} 1, +O, = Au, + Ayv, = [(70)(0.100)7/4](v,) + [()(0.150)*/4](v») (2¢)(0.100)*/4](0.7454u,) + [(7)(0.150)*/4](u,) 1.02353, =O, + Avs = Q, + [(-)(0.150)"/4](Q/0.02353) 2, = 0.24900 Q,= (1 — 0.2490)(Q) = 0.75109 V4 = Qu/Ay = 0.24900 /{()(0.100)7/4] = 31.709 Van = Q/Agn = O/I()(0.200/4] =31.830 U4 = (31.70) /31.83) = 0.995904 hy = (idan + (tye + (iyeo PARALLEL PIPELINE SYSTEMS J 299 ‘94 = 0.03400/0.200}{v’»/{(2)(9.807)}} + 0.02{300/0.100}{v2/[(2)(9.807)}} + 0.018{700/0.300}{veo/{(2)(9.807)]} [()(0.2007°/4 (v2) = [()(0.300)'/41(0c0) Ven = 0.4444 Van ‘94 = 0.03{400/0.200} v’n/(2)(9.807)]} + 0.02300/0. 100} (0.9959%4»)/1(2)(9-807)]} +0,018{700/0,300}{(0.4444044)*/{(2)(9.807)}) tan =3.798m/s OQ = Qaw=[()(0.200)"/4]3.798) = 0.1193 m/s co = On Q. = (0.2490)(0.1193) = 0.0297 m'/s___Q, = (0.7510)(0.1193) = 0.0896 m’/s 01193 m’/s Here, as in Prob. 11.44, the question remains open as to whether the computed flows are physically realizable. It all depends on the elevations of junctions B and C. 11.46 Pump P in Fig 11-26 develops 27t of head, producing a velocity of flow in pipe C of 5 fps. Neglecting minor Tomes; find the flow rates in pipes A and B. The parameters are: pipe A—4200 ft long, 2ft diameter, f= 0.03; pipe B-4200 ft long, 1ft in diameter, f = 0.03; pipe C—4200 ft long, 2 fin diameter, f = 0.02 B= GLIA FI28) Oya = (0.08) PNA /@)E22)]) = 0.978304 (nde = (ye Rouen (yde= (0.02)()(57/12)(32.2)]} = 16.301 (Ha)e = 16.3027 = —10.70ft “Hence the energy is greater at the right end of C and flow will be to the left in A. 0.9783u,= 10.70 v4=3307fA/s Qa = Agua =[(x)(2V/4]G.307) = 10.39 '/s (to the lef) Qc = [(x)(2)7/4N5)=15.71 10/5 (to the right) a+ Qe= =10.394 15.71= -S.324"/s (to the right) Qn Bie 2158 Fig. 11-26 1147 In Prob. 11.46, find the elevation (El) of pipe B at discharge ‘ Ely = Bherwicsurice + pump ~ Ds hy — va/28 n/ Ag = 5.32/1()(1)/4] = 6.774 es (ypu = (003)(3)16.774/12)02.2)1) = 89.78 hy = 16.30+ 89.78 = 106.08 6 Ely = 21.5 +27 ~ 106.08 — 6.774/[(2)(32.2)] = ~58.3 ft 11.48 Repeat Prob. 11.46 given that the velocity in pipe Cis 3 fps and all other data remain the same. Dy = (MLA 712g) (ry a= (0.03)(F°H(V4/[(2)G2Z29]) = 0.978304 (e= (ye — Mra, ine = O.0NPLFNDE2DY=S87TH (hye 5.8727 = ~21.13 Hence the energy is greater atthe right end of C and flow will be to the left in A. 0.9783U, = 21.13 vg=4.GATE/S Qa = Aaa =[(a)2V/AI(4.647)= 14.6040 /5 (to the left) Qc = [(x)(2)'/4]G)=9.424/s (to the right) Qe= Qn + Qc=-14.6049.42=-5.18f0/s (to the left) 1149 In Prob. 11.48, find the elevation of pipe B at discharge. ' ewenicsartce + Hyume ~ Sy hy + Va/28 n/n = 5.18/{(21)(1)*/4] = 6.595 ft/s (pre = (0.03)(9)(6.595"/[(2)(32.2)]} = 85.10 Shy = 5.87 —85.10= ~79.23 ft Ely = 21,5 +27 — (—79.23) + 6.595"/{(2)(32.2)] = 127.8 300 0 CHAPTER 11 1.50 ust 182 153 Refer to Fig. 11-26. Assume the water surface in the reservoir is at elevation 94 m. Pipes A, B, and C are all £840 m long, and they all have diameter of 0.7 m, with f = 0.022. Neglecting minor losses, find the flow rate in all pipes, supposing that the pump develops 9 m of head when the velocity in pipe Cis 3.6m/s. Why =ULIAMW*I28) ya = 0.0224840/00.N(V2NIQVO.8OT]) = 1.3460, Cade= Ce hay (hy Je = 0.022[840/(0.7)]{3.67/{(2)(9.807)]} = 17.44 m_ (he = 17.44 -9=8.44m 1.3460, =8.44 v4 =2.50m/s { Qa = Aad, = [()(0.7)7/4](2.50) = 0.962 m/s (to the right) | Qe = [(#0.7)/4](3.6) = 1.385 ar/s (to the right) { Qn = Qa + Qc = 0.962 + 1.385=2347 m'/s (to the right) | In Prob. 11.50, find the elevation of pipe B at discharge. BF Elp = Eher ste + hump Shy — val28 (hin = 0.022184 /(0.7)](6.099°/[(2)(9.807)]} = 50.07 mE hy = 17.44 + 50.07 = 67.51. Ely =94+9 — 67.51 — 6.099"/((2)(9.807)] = 33.6m Qol Ay =2.347/[(x)(0.7)'/4] = 6.099 m/s Using n = 0.013 and neglecting minor losses, express the head loss through the pipe system of Fig. 11-27 in the form of hy = KQ". norL (0.013050) is rasa 8 aReyT IAC TAE ” °0%72 (0.013 79714000) __ : - deo TROTTER VL dare = Core Ci, | Oil (Q}y4000) ___ (03.5000) RPA REAL DMG Paar 0173260, = 0,+0,=3260,+0, — 0,=0.2300 - (0.013)(2,)°(5000) - 2 (hare = aa ATT ENTAR 22? (haare = (3.939)(0.23420)" = 0.054850? (han = 0.009770 + 0.090639" + 0.0548597= 0.1559" 40008 .¢_ 100 Toa dan Te dam FE Mig. 11-27 A pipe system, connecting two reservoirs whose difference in surface elevation is 13 m, consists of 320m of 60-cm concrete pipe (pipe A), branching into 640 m of 30 cm (pipe B) and 640 m of 45 em (pipe C) in parallel, Which join again to a single 60-cm line 1300 m long (pipe D). f = 0.032 for all pipes, calculate all flows. ' Cina (ie y= FLIX /28) ((A){640/0.30} wa /l(2).9-807)}} = f[640/0.45}{ve/[2)(9.807)]) ve = 1.225u_ Qn=Qnt+ Qe 1€)(0.609/4](v4) = [()(0.30)/4( v0) + [(2(0.45)*/4](¥e) 0.28274 = 0.07069U» + 0.15900. 0.282704 = 0.070650» + (0.1590)(1.225v») va = 1.06504 ‘Convert the parallel pipes toa single equivalent 60-cm-diameter pipe F(640/0.30} v5 /[2)9.807)1} = f1L/0.60}(v4/12)(9-807)}) 108.8V3 = 0.08497L.u% (108.8)(1.065u,)°=0.08497L.0%, Lem 452m AEL= Dy 13 = 0,032[(20 + 1452-+ 1300)/0.60}{v3/[2\9.807)]} v4 = 1.248 m/s Qa Aqua ((7)(0.60)'/4](1.248) = 0.353 m"/s— Qp = a= 0.353 m/s 1.065)(1.248) = 1.329 m/s Qn = [()(0.30)/4](1.329)=0.094m'/s c= Q.- 0, 094 = 0.259 m"/s 154 Refer to Fig. 11-27. Suppose p4/y =6.5 ft, po/y = 20ft, and z4= zp. A pump in the 4000-f pipe (flow from left to right) develops 30 of head. Find the flow rate in each pipe. Assume n = 0.013 for all pipes. ' Qua = Qssc— src= Qco 6.5 ~ (ya ly Pal y+ hyamg~ (hone = Pel Y Ply ~(hy)arc = PalY PelY~ (ty)co =20 hy =nPL/LABR?Y— Van = Qual Aas = Qaall(H)HF/4]= 0.3183 Qa0 Cy dan = (0.013)"(0.3183Q 44)"(500)/{(1.486)[()/4P"}* = 0.0097690'.n Vasc = Qnrcll()(H)/4] =0.5659Q p00 Crp)aec = (0.013)%(0.5659Q prc)*(4000)/{(1.486)[(1)/4P"}? = 0.362505 ac Vere = Qarel{(a)(H)"/4] = 1.273Qarc (hy)arc = (0.013)"(1.273Qnec)*(S000)/{(1.486)[(8)/4P*)* = 3.938Q'src Veo = Qcoll(a)()"/4] = 056590 co (eo = (0-013)"(0.5659Qc0)*(1000)/ {(1.486){(H)/4]"}? = 0.090630%0 Substitute Eqs. (6), (7), (8), and (9) into Eqs. (2), (3), (4), and (5), respectively, and then solve simultaneously between Eqs. (2), (3), (4), and (8), introducing also Eq. (1). Two equations result, such as Chee 2PrecOnre~3740Varc = -132 Shee + 10-7 Dre = 82 By trial and error, Qazc= 6.25 cf8 and Qarc=2.00ets. Hence, Qan =6.25 ~2.00= 4.25 ef | PARALLEL PIPELINE SYSTEMS 0 301 @ @ @ @ ©) (6) o @ ~ CHAPTER 12 Branching Pipeline Systems m2 302 In Fig. 12-1, find the flows for the following data: L, = 200m, 300 mm, €1/Dy i= 7 atm; Ly = 300m, Dz = 350mm, €:/D;= 0.00015, z= 400 m, p=2 atm; Ly £65/D5=0.0001, 25= 100m, p,=3.atm. The fluid is water with v =0.113 x 10°" m'/s. I We first caleulate Hy, Ha, and Hy. Thus, H, = 2+ (1)\Pan)/ = 700+ (7\(101 325)/9806 = 772.3 m, = 21+ 2)(Pam)/Y = 400 + (2)(001 325)/9806 = 420.7 m, Hy = 25+ (3)(Pam)/¥ = 100 + (3)(101 325)/9806 = 131.0. Now estimate H; = 380 m. We will hence use the continuity equation for test purposes. We can then say that (772.3 — 380) = (1/g)f(200/0.300)(V3/2), (420.7 ~ 380) = (1/g)f,(300/0.350)(V3/2), (380 — 131.0) = (A/g)(400/0.400)(V3/2). Estimating f,=0.014, f:= 0.013, and f,=0.012, we can compute velocities and the flows q,. We get Vi=28.72m/s qi (/4)(0,300°)28.72) =2.03 m/s Ve 8.465 m/s gz (/4)(0.3507)(8.468) = 0.814 m/s Vy=2018m/s — q5=(/4)(0.400")(20.18) = 2.536 m/s We see that (q1 + 4:) > 4s. Asa second estimate, we increase H, and use more accurate friction factors. Using the preceding velocities, we now find a second set of fiction factors: (Na), = (28.72)(0.300)/(0.0113 x 10™) = 7.625 x 10, f= 0.014; (Ny)a = (8.465) (0.350)/(0.0113 x 10°) = 2.622 x 10, f,= 0.0134; (Ne) (20.18)(0.400)/(0.0113 x 10~*) = 7.143 X 10, f= 0.012. Suppose we next choose H, to be 400m. Thus, we sill use the continuity equation. We get the following results: V;= 27.98 m/s, qi = 1.977 m/s; Vs= 5.95 m/s, q2= 0.5721 m'/5; Vs=20.97 m/s, qs= 2.635 m'/s. [Note that (1 + 42) 0' THEN ZMI“HIUN ELSE ZHAHILN 280 LPRINT"PIPE DISCHARGES ARE. {POSITIVE INTO THE JUNCTION)" BAO FOR To TO Ns “CPRINT*OUsI3")="F8 —— LPRINT USING" W@M.WNB "Z0U1); 250 NeXT Ts Lenin 106001 10076-0815 0002 33. Libiersz= to00 “*e “ooce 9 ELevaTion OF JuncTioN Ts” 24.8901 PIPE DISCHARGES ARE POSITIVE INTO THE SUNCTION BOL ye tet98 Gt 2 I -O-S29 BS )= 0.669 B.A Calculate the three volumetric flows in Fig. 12-10 ! (€1D)qy = 1/200 =0,0050(€/D)qy = 1/200= 0.0050 (€/D)cy = 3/300= 0.0100 fas = 0.082 fer 0.038 hy = (SY(LID)(0*/2g) = ((L/D*VBO"/gz*) .032{1000/(0.200)"}{803,/{(9.807)()"}) = 82650%, 0.032[300/(0.200)*](82/[(9.807)(7)"]) = 248003, 5 = 0.038{600/(0.300)"1(80%,/[(9.807)(x)"}} = 775.502, Try hy = 19m: 2B-19=82650%, Qa =0.03300m"/s 25-19 = 24800, Dg = 0.08919 m'/s 19-15" 775.50 — Qa=0.07182 m/s Q.o1~ Oram, = 0.03300 + 0.04919 — 0.07182 = 0.01037 m'/s 310 0 CHAPTER 12 Try hy = 20m: 2B-20= 826502, Qy = 0.03111 m/s 25-20= 248003, On, = 0.04490m'y/s 2-15=775.5Q% — Qcy= 0.08030 m'/s Quo) ~ Qvoms = 0.08111 + 0.04490 — 0.08030 = —0.00429 m"/s, Try hy = 19.7 m: 28 - 19.7 = 826503, 03169 m")s 2519.7 = 248093, ay = 0.04623 m"/s Qcy = 0.07785 m"/s Qrs1~ Qnoms = 0.03169 + 0,04623 — 0.07785 = 0.00007 m?/s Hence, Qc = 77.8 Lis from J, and Q4, = 31.7 L/s and Qyy = 46.2L/s toward J. Fig. 12-10 12.12 Calculate the five volumetric flows in Fig. 12-11 1 (ALIDY(v* 2g) = SMLIDV(8Q"e2") ‘hy ~ 18 = 0.025{600/ (0.600)"(80% /{(9.807)(2}) = 15.9450, ‘hy ~ 88 = 0,030{600/(0.450)"1(80'/[(9.807)(7)"]} = 80.6240% ‘h,— hy = 0.030(900/(0.450)°1{803,/(9.807)(x)"]) = 120.9493, 98 — h, = 0.030,300/(0.450)*]{8Q2/{(2.807)(x)"1} = 40.31202 {88 ~ hy = 0.030300/(0.450)"]{(80/[(9.807)()"]} = 40.3120 = 15.9450 Q,=0.5009m'/s 82-88 = 80.6240} p= -0.2728 m"/s ‘h,~ 82 (120.94)(0.5009 ~ 0.2728)? = 88.29 m 98-88.29= 4031203 Qc-= 0.4908 m"/s 88—88.29= 40.3120 — Qn = ~0.0848 m"/s Ifthe above values are correct, Q; (0.5009 — 0.2728 = 0.2281 m'/s) must equal Qc + Qp (0.4908 — 0.0848 = 0.4060 m/s). Since they are not equal, try hy = 83m and note that Q,, Qc+ Qn, the correct value of h, must be between ‘82m and 83 m, Try hy = 82.24 m: $2.24-78= 15.9450, Q4=0.515Tm'/s 82.2488 = 80.6240 Qp = 0.2673 m'/s h,~ 82.24 = (120.94)(0.5157 0.2673)" hy =89.70m 98-89.70= 40,3120 Qc= 0.4538 m/s 88-89.70=40.31203 p= -0.2054m"/s Ie these values are correct, Q, (0.5157 —0.2673 = 0.2484 m/s) must equal Qc + Qo 0.2490 m"/s BRANCHING PIPELINE SYSTEMS J 311 (0.4538 - 0.2054 = 0.2484 m/s). Since they are in fact equal, the correct values must be Q4 = S16L/s, Op = 267 Lis, Oy = 248 Lis, Oc = 454 L/s, and Qo = 205 Lis. 800m, 600-mm=p 7 f20025 Fig. 12-11 12.13 In Fig, 12-12, pipe 1 is 36-in smooth concrete, 5000 ft long; pipe 2 is 24in cast iron, 1500 ft long; and pipe 3 is 18-in cast iron, 4000 ft long. The elevations of water surfaces in reservoirs A and B are 300 ft and 250 ft, respectively, and discharge Q, is $0 f"/s. Find the elevation of the surface of reservoir C. 1 fy = (fMLIdyv7l2g)——_(€/d), = 0.001/(F8) = 0.000333 Ng = dv /y y= Qs As = 5O/{(2)(H8)"/4] = 7.074 ft/s (a) = (19(7.074)/(1.08 x 10°) = 2.02 10° From Fig. A-5, f,= 0.0157. (hy), = 0.0157{5000/(i)]{7.074°/{(2)(32.2)]} = 20.33 ft, hy = 300 — 20.33 = 279.67 fr, (hy x= 279.61 ~ 250 = 29.67 ft. Assume f, = 0.0162: 29.67 = 0.0162{1300/ (V2 2VG22))) ve = (Nes = (¥)(12.54)/(1.05 x 10-*) = 2.39 x 10° (€/d), = 0.00085/(3) = 0.000825 f,=0.0162 (0.K.) O:= Avs = (C2) I4K12.58) = 39.40 80/8 Qy=S0~39.40= 10.6 /s 10.6/{(2)(14)'/4] = 5.998 ft/s (Na)s = (8)(5.998)/(1.05 x 10-*) = 8.57 x 10° (€/d)s=0.00085/(18) = 0.000567 f,=0.0177 ()s=0.01774000/(1)1(5.998°/12)2.2)]} = 26.37 79.67 ~ 26.37 = 253.30 ft 54 ft/s Hydraulic Grade Line Fig. 12-12 12.14 In Prob. 12.13, suppose that the surface elevations of reservoirs A and C are 300 ft and 225 ft, respectively, and discharge Q2 is 20 cfs into B; other data are unchanged. Find the surface elevation of reservoir B. ' hy =(FMLId\(v?/2g) v2 = Qa/Az=20/|(n)(H8)*/4] = 6.366 ft/s Ne = duly (We) = (2)(6.366)/(1.05 > 10-4) = 1.21 x 10" (€/d),=0.000425 (from Prob. 12.13) 312 0 CHAPTER 12 From Fig. A-S, f= 0.0168. (h,), = 0.0168(1500/(#)] (6.3667/[(2)(32.2)]} =7.93 ft Q:~Qs=20 Q (hy) + (hy)s = 300 = 225 = 75 ft @ ‘We need to find an hy in line 1 or 3 such that both Eqs. (1) and (2) are satisfied. Try (,), = 64 ft and oe 4 =0.0175{4000/(ADIVI@G22) v5 =9.398 ts (Nas = 18(9.398)/(1.05 x 10-*) = 1.34 x 10° (€/d)s= 0.000567 (from Prob. 12.13) f=0.0175 (O.K.) Qs = Asus = [()(18)"/4](9.398) = 16.61 fE'/s Q,-16.61=20 9, =36.61ft'/s u, = 36.61/(()(3§)"/4] = 5.179 ft/s (Na)i = G9)(S.179)/(1.05 x 10°*) = 1.48 x 10° (€/d),=0.000333 (from Prob. 12.13) f,=0.0158 (hy), = 0.0158[5000/ #8)]{5.1797/[(2)(32.2)] 10.97 ft Zan 24~ (hy) ~ (hy) = 3010.97 ~7.93-= 281.18 12.15 Given, in Prob. 12.14, that discharge Q, is 20 cfs out of reservoir B, find the elevation of the surface of B. I Asin Prob. 12.14, (hy). = 7.93 ft, but Eq, (1) becomes 2,-Q,=20 o ‘Equation (2) is the same as in Prob. 12.14; that is, (hy). + (hy) = 300 ~ 225 = 75 ft (2) We need to find an hy in line 1 or 3 such that both Eqs. (1) and (2) are satisfied. Try (hy), = 77 ft and r= 0.0175: 77 =0.0175(4000/(8)1(03/(2)32.2)]} v= 10.31 f/s (ads = (1)(40.319/.05 x 10°9) = 1.47 x 10" (€/d)= 0.000567 (from Prob. 12.13) f=00175 (OK) Q,= Asus = ((ay(H/4](00.31) = 18.2260/s 1822-9, =20 Q.=-1.7BH/s (ies, flow is into reservoir A) = 1.78/[(a)Y/4] = 0.2518 As (a) = G9(0-2518)/(1.05 x 10%) = 7.19 x 10" (¢/d),= 0.000833 (from Prob. 12.13) 208 (hy) = 0.0208{5000/ (i) {0.2518"/[(2(32.2)]} = 0.08 fe Zn™ 244 (ly * (tyr 300 + 0.08 +7.93 = 308 ft 12.46 In Fig. 12-12, assume that 1, 2, and 3 represent 900 m of 60-cm, 300 m of 45-em, and 1200 m of 40-cm, respectively, of new welded steel pipe. The surface elevations of A, B, and C are 32m, 20m, and 2m, respectively. Estimate the water flows in all pipes. ’ (€/d), = 0.000046/(0.60) = 0.000767 (€/d), = 0.000046/(0.45) = 0.0001022 (€/d)s = 0.000046/ (0.40) = 0.001150 Assuming complete turbulence (high Reynolds numbers), friction factors for these values of (€/d) will be A, =0.0115, f, = 0.0122, and f, = 0.0126. hy = LIV 07/28) = IL/A)80*/e7) (hy) = 0.0115{900/(0.60)"}(802/{(9.807)(2)"}} = 11.000 (y)a™0.0122300/(0.45)"}8Q2/{(9.807)(2)]} = 16.3993 (hy)s = 0.0126{1200/(0.40)*]{803/{(9.807)(-x)"]} = 122.003 32- 11.0007 =h, o a7 a8 BRANCHING PIPELINE SYSTEMS J 313 hy -16.399=20 of 0-16.39? =h, @ hy ~ 122093 =2 @ Q=0:+0, or O,+0:=9; w ‘Assume no flow in pipe 2, in which case hy =20/m. 32 ~ 11.0003 =20, Q3 = 1.091; 20~ 122.093 = = 0.1475. Since Q,> Qs, flow must be into reservoir B, and h, >20'm. By trial and error, hy = 23.5 m. 32~ 11.009} =23.5, Q, = 0.879 m/s; 23.5 ~ 16.3903 = 20, Q,= 0.462 m/s; 23.5 ~ 122.0032, Qs= (0.420m"/s. Check Eq. (4): 0.879 = 0.462 + 0.420, 0.879 =0.882 (close enough). Check the Reynolds numbers for normal temperature: Na = duly vy=0.879/{(=)(0.60)*/4] = 3.109 m/s (Na) = (0.60)(3.108)/(1.02 10°) = 1.83 x 10° v= 0.462/{()(0.45)"/4] = 2.905 m/s (Nada = (0.45)(2.905)/(1.02 x 10%) = 1.28 x 10° 5 = 0.420/{(22)(0.40)7/4] = 3.342 m/s (aba = (0.40)(3.342)/ (1.02 10%) = 1.31 > 10° From Fig. A-5, f = 0.0135 approximately forall pipes; hence, the above flows should be multiplied by (0.012/0.0135), giving Q, = (0.012/0.0135)(0.879) = 0.781 m"/s, Q, = (0.012/0.0135)(0.462) = 0.411 m’/s, (Qs (0.012/0.0135)(0.420) = 0.373 m’/s. Further refinement of these approximations is not justified. ‘Suppose that, in Fig. 12-12, pipe 1 is 1500 ft of 12-in new cast iron, pipe 2 is 800 ft of 6-n wrought iron, and pipe 3 is 1200 ft of 8-in wrought iron. The water surface of reservoir B is 20t below that of A, while junction J is 40 ft below the surface of A. When the pressure head at J is 30 ft, find the flow in each pipe. i (hy), = 40 — 30= 10 ft (iy) = (40 ~ 20) — 30 = —10 ft Ay = (FXLId\(v?/2g) Assume f= 0.019 10 = 0.019{1500/(H)]{vi/[(2)(32.2)]} vy = 4.754 ft/s Na= duly (Na): = (8)(4.754)/(1.05 x 10°*) = 4.53 x 10° (€/d), = 0.00085/(#) = 0.00085 = 0.0198. Try fi = 0.0198: 10=0.0198(1500/)I(U@G22)) v1 = 4.657 fs From Fig. Ax! (Na): = (BY(4.657)/(1.08 x 10") =4.44x 10° f,=0.0198 (O.K.) 2, = Aiur = [ca 44.657) = 3.658 00/5 Assume f= 0.015: 10 = 0.0151800/(8)¥3/12}G2.2))—v.= 5180s (Naz = (B)(S.180)/(1.05 x 10-*) = 2.47 x 10° (€/d),= 0.00015/(#) = 0.00030 ‘f =0.0175 Try f= 0.0175: 10 = 0.0175{800/(8){03/1(2)32.2)}} v2 = 4.796 ft/s (Nada = (18)(4.796)/(1.05 x 109) =2.28 x 10° f= 0.0175 (OK) Q.= (YAP /4(4.796) = 0.942F/s y= Q, ~ Q,= 3.658 - 0.942 = 2.716 RVs In the reservoir system of Fig. 12-13, 24 = 210 ft, 25 =240ft, zc = 130 ft, 2 = 150 ft, BD = 3000 ft of ¢-in cast iron pipe, AD = 2000 ft of 1-in steel pipe, and DC = 500 ft of 6-in cast iron pipe. Using f = 0.025 and neglecting minor losses, determine the flow in each pipe. Hb, = MLIdy{v? 2g). Let poly = 15 (4))s0= 210 150 (—15) =0.02512000/(1)}(v0/{@)(32.2))) (by no = 240 — 150 ~ (~15) = 0.025{3000/()] 5o/l2)G2.2)D (yep = 130 — 180 — (15) = 0.025{500/(f)](veo/K2)(32.2))) If these velocities are correct, Qn + Quo must equal Qc: Qan + Qn [a )(4)"/416.482) = 0.4939 7/8, eo = [C)(8Y/413.589) = 0.7047 fs. 837 ls = 5.482 fs (away from D) ((#)G5)'/41(2.837) + ince they are unequal, try 9314 0 CHAPTER 12 poly=-17ft: (ryan = 210 150 — (17) = 0.025(2000/(4)] Wan/lVG22D) van = 2.875 Hs (yao = 240 — 150 — (~17) = 0.025{3000/()vao/(232.2)) van = 5.534 /s (hydeo = 130 — 150 — (—17) = 0.025500/()I{veo/{2)32.2)]} Veo =2.789ft/s (away from D) Qan+ Ono ™ [()(18)°/4](2.875) + 1) ()"/4](S.534) = 0.4986 f/s Qn = Ic) 8)*/8]@2.789) = 0.5476 1s Qao+ Orvis stil not equal to Qcoi try Po! y= ~17-A ft: (ty)an = 210 ~ 150 ~ (-17.5) = 0.025(2000/¢)] wo/(@)G22)1) van = 2.884 /s (hy ao = 240 — 150 ~ (17.3) = 0.0253000/(4)H(v3o/{(2032.2)1) Uno = 5.547 fs (Jeo = 130 ~ 150 ~ (~17.5) = 0.025{500/(#)1(¥o/{2)G22)1) veo =2.538 ft/s (away from D) Quo + Dan = [()(H)/4] 2.884) + [CY /AI15.547) = 0.4998 f/s : Qo = [()(4)"/4] 2.538) = 0.4983 f'/s Since Qan + Ono is close enough to Oco, take Qno™ {(a4)7/412.884)=0.0150/s (toward D) on = {(a)(8)°/4K5.547) = 0.483 107s (toward D) Qco=0.498/s (away from D) Fig. 12-13 12.19 Three reservoirs A, B, and C whose water-surface elevations are z, = 60 ft, 2» = SOft, and zo = 32, are interconnected by & pipe system with a common junction D, zp = 35 ft. The pipes are as follows: from A to junction, L = 800, d =3in; from B to junction, L = 500 ft, in; from C to junction, L= 1000 t, d=4in, “Assume J = 0.02 foF al pipes and neglect minor losses and velocity heads. Determine whether the flow is into or ‘out of reservoir B. I Assume no flow between B and D. hy = FALIA(W?I2g) (ya = 0.02800 /()}(04/[2)G2.2)]} = 0.99380 (hye = 0.02[1000/(8)]{v2/{(2)(32.2)]} = 0.93170 Anta Acte — (a(R /I 4) = DG) Ve=LTIBVC (yale = 0.993803 0.931702 = (0.9938)(1.778e)"/0.9317 ve = 3.372 Hence, (hy)a = [3.372/(3.372 + 1)|(60 — 32) = 21.60 ft, and zp = 60 ~ 21.60 = 38.40. Since our no-flow assumption makes D too high (by 3.40 ft) relative to B, the flow must be out of B into D. Ba B2 CHAPTER 13 Pipe Networks Develop the expression utilized to study flows in a pipe network (Hardy Cross method) I The method of attack, developed by Professor Hardy Cross, consists of assuming flows throughout the network, and then balancing the calculated head loses (LH). In the simple looping pipe system shown in Fig. 134, for the correct flow in each branch ofthe loop, LHyac=LHane or LHyac~ LH ane: @ In order to utilize this relationship, the low formula to be used must be writen inthe form LH = kQ". For the Hazen-Williams formula, this expression is LH = kQ"*: But, ince we are assuming flows Q,, the correct flow Q in any pipe of a network can be expressed as =, +A, where Ais the correction to be applied to Q,. Then, using the binomial theorem, KOMP = k(Q, +A) = k(QS" + 1.8503""* A+), Terms beyond the second can be neglected since A is small compared with Q,, For the loop above, substituting in expression (1) we obtain k(Q!**+ 1.85Q5" A) — k(Q*'+ 1.8503" 4) = 0, (QE ~ 8) + 1.85k(O2" — 05") A =0, Solving for 4, B= (QS = O2)/[1.85K(02" ~ @ In general, we may write for a more complicated loop, A=-DkOs*/(1.85 D kOe") ) But kQ!* = LH and kQ* = (LH)/Q,. Therefore A=-D (LHD/[L85 D (LH/Q,)]__for each loop of a network @ In utilizing expression (4), care must be exercised regarding the sign of the numerator. Expression (1) indicates that clockwise flows may be considered as producing clockwise losses, and counterclockwise flows, counterclockwise losses. This means that the minus sign is assigned to all counterclockwise conditions in a loop, ‘namely flow Q and lost head LH. Hence to avoid mistakes, this sign notation must be observed in carrying out a solution. On the other hand, the denominator of (4) is always positive. Fig. 13-1 In Fig, 132, for Q = 11.7 mgd total, how much flow occurs in each branch of the loop, using the Hardy Cross method? I Values of O12 and Ove are assumed to be 4.0 mgd and 7.7 mgd, respectively. The tabulation below is prepared, (note the —7.70 mgd), the values of S calculated by Fig. A-17, then LH =$ x L, and LH/Q, can be calculated. Note that the large 3: LH indicates that the Q's are not well-balanced. (The values were assumed deliberately to produce this large LH, to illustrate the procedure.) ‘assumed 5 D L Qomgd | f/i000n | LHM~ LH/Q, | A a xin | sooo 4.00 195) 7s m4 | -o8s | ais i6in | 30008 -7.70 163 -4.9 64 | -08s | -855 (|= 11.70 Ea+486 308 1.70 ~ (448.6)/{1.85(30.8) > LA/{1.85 ¥ (LH/Q) 315 316 0 CHAPTER 13 ‘Then the Q, values become (4.00 — 0.85) calculation produces 1S mgd and (—7.70 — 0.85) 8.55 mgd. Repeating the s LH a Q 12.5 25 0.06 3.09 -19.8 59.4 ~006 | -361 pasa 11.70 [No further calculation is necessary, since the slide rule or chart cannot be read to the accuracy of 0.06 mgd. Ideally, J LH should equal zero, but this goal is seldom attained. C= ite Sooo —12" Fig. 13:2 133 For the pipe network giving pipe diameters and lengths and external flows entering and leaving the network as shown in Fig. 13-34, find the flow rate in each pipe in the network. 1 Te frst iteration in carrying out the Hardy Cross analysis is given in the table below. A detailed explanation corresponding to steps in carrying out the procedure is given after the table. eration No. 1 Loop 1 o @ eo @ ) oO m pipe | length, | diameter, | flowrate (Q), | unit head loss, | head loss (hy), | hy/Q Ro. ft in fP/s ft ft sit? ‘estimated or from pipe given | given | given ‘assumed diagram axe | om 1 2000) 18 7.00 0.0038 2 900 m 3.50 0.000235 3 | 280 2 -219 0.0029 4 | i100 4 =7.0 0.00085, Loop it a @ 8) @ © o o pipe length, diameter, flow rate (Q), unit head loss, head loss (hy), hyIQ, 0. f i fis ft ff site ‘from pipe iven | given | given diagram Oo) s | 2200 2 0.0070 4.800 6 750 18 0.00078, 0.186 7 | 2600 4 0.00057 0.260 2 300 Py 0.00023 0.059 495 PIPE NETWORKS J 317 Step 1: Column (1), 2, and (3) in the ale above record given data, Flow rae recorded i coamn (4 are ial flow rate estimates. These are listed on the sketch of Fig. 13-3, but they are not “given” data. The flow rte of 14.00 cls entering joint A was extnated Yo separate sec that 7 Deft goes through pipe 1 and an equal amount goes through pipe 4. At joint F, since 7.00 cfs enters the joint from pipe 4 and 4.81 cfs leaves the joint externally, the flow rate in pipe 3 has to be 2.19 cfs (that is, 7.00 cfs ~ 4.81 cfs) in the direction away from joint Fin order to satisfy the principle thatthe total flow entering a joint must be equal to the total flow leaving. that joint. Remaining flow rates were estimated in a similar manner, making sure that for each joint the total flow entering equaled the total flow leaving. It should be noted that clockwise flows in each loop (such as in pipes 1 and 5) are indicated as positive, while counterclockwise flows (such as in pipes 3 and 6) are indicated as negative. It should particularly be noted that the flow in pipe 2 is clockwise with respect to loop I but counterclockwise with respect to loop Il; hence, itis indicated as positive when listed in loop T and negative when listed in loop I Step 2: Unit head losses in column (5) are determined from Fig. A-13, based on diameters [column (3)] and flow rates [column (4). For example, pipe 1 has a diameter of 18 in and flow rate of 7.00 cfs; hence, the unit head loss is determined from Fig. A-13 to be 0.0034 t/t. Head losses in column (6) are determined by ‘multiplying pipe lengths [column (2)] by unit head losses [columa (5)]. For example, pipe 1 has a length of 2000 and unit head loss of 0.0034 f/f; hence, the head loss is (2000 (0.0034 f/f), or 6.800. It should be noted that head losses are positive if their corresponding flow rates are positive and negative if their corresponding flow rates are negative. The hy/Q fractions in columa (7) are determined by dividing head losses {column (6)] by flow rates [column (4)). For example, pipe 1 has a head loss of 6.800 ft and a flow rate of 7.00 efs; hence, hy /Q is 6.800/7.00, oF 0.971 s/t. Step 3: Algebraic sums of head losses are observed from the table to be ~2.043 ft in loop I and 13.126 ft in loop IL. Since these are not zero, the original estimated flows are not correct. Step 4: A flow rate correction (AQ) can be computed for each loop: AQ = —F hyn E (hy/O)} (Since the Hazen-Wiliams formula is being used, 1 = 1.85.) (AQ) ang: =(~2.043)/[(1-85)(4.874)] = 0.23 es, (8Q) aon = (13-126) /{(1.85)(4.905)} = 1.45 ef, ‘Step 5: Adjusted flow rates for each pipe are determined by adding flow rate corrections to the previous rate for each pipe. These are as follows Loop 1 Pipe AQ, cfs new Q, cfs 1 +0.23 1B 2 +0.23-41.45 S18 3 $0.23 1.96 4 +023 -67 Loop tt Pipe AQ, cfs new Q, cfs $ = 145 2.05 6 =145 4.59 7 145 714 2 1450.23 5:18 ‘These adjusted flow rates are shown in Fig 133c. It should be noted that the flow rate in pipe 2 was adjusted using flow rate corrections for both loops, since ths pipe is common to both. It should be particularly noted that the sign of the flow rate correction for loop Il was reversed when it was applied in loop I (and vice versa). 318 0 CHAPTER 13 Step 6: Return to step 2 and repeat the entire procedure using adjusted flow rates. This is given in the table below. Iteration No. 2 Loop I a @ @ ® © © o pipe | length, | diameter, head loss (hy), | he/Qs 0. a in ft sit given | piven given axe ola 1 2000 18 7.200) 0.996 2 300 a 0.423 0.082 3 2800 2 -6.720 3.429 4 100 a 0.880 0.130 0.023 4.637 a @ @ © © o pipe | length, | diameter, ‘unit bead loss, | head loss (hy), | y/O 0. ft in ft a s/ft from pipe given | given ren isgram Oxo ola 5 7200 2 0.0026 5.720 2.790 6 750 18 0.00156 -1.170 0.255 7 2600 a 0.00088 2.288 0300 2 00 m 0.00087 0.423 0.082 1.839 3.447 ‘Since the algebraic sums of head losses are not both zero, new flow rate corrections must be computed. (0.023)/{(1.85)(4.637)] = 0.00 cfs, (AQ) nap = ~(1-899)/{(1-85)(3.447)] = ~0.29 cfs. Revised flow ‘are determined using these flow rate corrections. These are as follows: Loop 1 Pipe 49, cfs new Q, cfs 1 0.00 123 2 0.00 +0.29 SAT 3 0.00 196 4 0.00 67 Loop I Pipe old Q, ef AQ, ets new Q, cfs 5 2.05 0.29 1.76 6 459 0.29 4.88 7 =114 -0.29 -743 2 5.18 0.29 ~547 PIPE NETWORKS 0 319 ‘These adjusted flow rates are shown in Fig. 13-3d. Return to step 2 and repeat the entire procedure using these revised flow rates, This is given in the table below. eration No. 3 Loop 1 a @ e @ 6) o a pipe | length, | diameter, | flow rate (Q), | unit head loss, | head loss (A), | //Q, 0. ft in W/s yn ft site estimate or from pipe given | given given assumed iagram x) CO) 1 2000 18 123 7.200 0.996 2 900 Dy S47 0477 0.087 3 2800 2 1.96 6.720 3.429 4 1100 ey -677 0.880 0.130 om 4.682 Loop It @ @ © 6), © a length, | diameter, | flow rate (Q), | unit head loss, | ead loss (4), | hy/Q, ft in fe/s ate t site ‘estimate oF given | given sive assumed @xo or 5 200 2 1.76 4.180 2375 6 750 18 -4.88 1312 0.269 7 2600 ey -743 2.486 0.329 2 900 m 547 047 0.087 0.083 3.060 Since the algebraic sums of head losses are not both zero, new flow rate corrections must be computed, (BO )nop: = ~(0.077)/[(1.85)(4.642)] = =0.01 cfs, (AQ)ion 1 = —(~0.053)/{(1-85)(3.060)] = 0.01 cf. Revised flow rates for each pipe are determined using these flow rate corrections. These are as follows: Loop 1 Pipe AQ, cfs 1 0.01 2 37 0.01 - 0.01 5.45 3 1.96 0.01 -197 4 ~677 0.01 ~678 Loop pipe old @, cfs AQ, efs new Q, efs s 1.76 40.01 4.77 6 4.88, +001 ~4.87 7 -143 +001 =742 2 5.47 +0.01+0.01 ~5.45 ‘These adjusted flow rates are shown in Fig. 13-3¢, It would be appropriate to return to step 2 and repeat the entire procedure using these revised flow rates. However, an additional iteration (not shown) indicates that the next flow rate corrections would be no greater than 0.01 cfs, and further computation would appear to be wasted effort. Hence, the “new Q” values just before this paragraph are taken to be the correct flow rates for these pipes. On = 1400 10% 124 diameter aa = AHI Hs 2A. diameter (Pipe network Og = 2.55 10/6 219 () Initial ow rate estimates 2550 1400 0/8 2330 4a Oh 2350 (6) Adjusted low rates fer eration No, | 1400 6 A123 00 (4) Adjusted flow rates afte Heration No, 2 1400 0) 222 06 48108 255 8% (0 Adjusted low rates after eration No. 3 Fig. 13:3 Ba PIPE NETWORKS 0 321 For the pipe network shown in Fig. 13-4, find the rate of flow of water in each pipe. Assume C= 120 for all pipes. ' pre | Din | Lt | Quoc | Anti | mm | W/O | AO AB 1% | 1500 80 0.0046 oe | oss | —o1 Bc n 500 30 0.0053 265 | 088 | -o.11 oF 1s | 1500 -40 ~o.on29s | 442 | 110s | 223 FA 15 500 =10 =o | 41s | 0593 | -o.11 0.68 | 3406 FC 15 | 1500 40 ons | 442 | 110s | 223 | 17 cD 10 500 70 0.059 230 | dais | -236 | 466 DE n | 1500 20 0.0025 375 | 197s | -234 | -036 EF 2 500 30 0.0053 |_-265 | oss | -234 | -534 38.02 | 807 Qu = ~35.02/{(1.85)(8.077)] = ~2.34 cfs cts pipe Ay hy IQ aa | ooo 630 | 078 ac | cous ass | om cr | oor | -100 | oes ra | ones | —430 | 0.605 nas | 2816 Fe | 0.067 100 | 0.565 co | oo uo | 3.000 be | coos | -o1 | ost zr | oois =750 | 1408 7.36 | 5381 AQ, = ~3.45/(0.85)2.816) = 0.668 vine hy a ap | 00036 | sao ac | oomss | 148 cr | -0.0060 | -090 Fa | -o.010 | —s.00 098 Fe | 00050 | 09 | oss | 020 | 109 cp | 000 | 100 | 2s | om | 392 ve | -o00 | -120 | iin | om | -108 er | -oows | -975 | 164 | oo | —608 =o0s 5.799 AQ, = ~0.98/{(1.85)(2.588)} Qn = ~(—0.05)/[(1.85)(5.799)] = 0.00 cfs 922 0 CHAPTER 13 Pipe wIQ 4g 2 AB 0.0034 0.725 0.02 7.05 BC 0.0026 0.640 02 2.05 cF | -0,00075 0.587 aos | 186 FA | ~0.0108 0.678 oo | 795 2.630 FC 0.00074 0.587 0.03 1.86 cD | 0.0200 2551 0.01 3.91 DE | ~0.00080 Lin 0.01 -109 EF | -0.0195 1.604 0.01 6.09 5.853 AQc= ~(~0.11)/{(1.85)(2.630)] = 0.02 fs iste Qu = -0.16/{(1.85)(5.853)] = -0.01 cfs 1500 18 diameter 00 diameter diameter 1300 fi, 1S.n diameter 500 f 00 oie diameter diameter 150012 jn diameter sth sie Sih Fig. 134 135 The pipe network shown in Fig. 13-5 represents a spray rinse system. Find the flow rate of water in each pipe. Assume C= 120 for all pipes. ' vee | Dimm | Lm | Qumomls | Anmim | nm | &/Q | AO ‘ap | 300 | 600 0.200 0.027 1620 | 1.0 | oon ac | 250 | 400 0.100 o.or7s 7.00 | 70.0 | 0.003 cH | 30 | 60 0.100 -ooom | -446 | 444 | oon HA | 250 | 400 ~0.200 -o064 | -2560 | a0 | aon =6a4 | 323.4 ac | 30 | 60 0.100 oor | a | aca | oor] oats ce | 20 | 400 0.050 0.0049 196 | 392 | oo | oot Fa | 30 | 600 0.100 -ooors | 44s | 444 | o.ore | 0.086 op | 250 | 40 0.100 =0.0175 | _-700 | 700 | a.00s | 0.007 =s.04 | 198.0 PIPE NETWORKS 323 pipe | Dymm | Lym tymim | tom | w/@ | 32 | Om cD 30 | 60 0.050 0.0020 24.0 | 0.000 | 0.050 DE 250 | 400 0.050 0.0089 392 | 0.000] 0.050 EF 30 | 600 =0.050 0.0020 24.0 | 0.000 | -0.050 FC 250 | 400 =0.050 0.0089 30.2 | -0.014 | -0.064 126.4 AQ= ~Z ylin E (hy /O) -6.84)/{(1.85)(323.4)] = 0.011 5.04)/{(1.85)(198.0)] =0.014 AQ = =0.00/{(1.85)(326.4)} = 0.00 Pipe hy b HO 4g Que AB 0.0295 770 | 3.9 0.008 ois BG 0.017 680} 701 0.001 0.098 GH | -00059 | -a5¢ | 398 0.004 | -0.085, HA | -0058 | -2320 | ms 0.008 | 0.185 -2m | 3166 BC 0.0095 37 | 50.0 (0.003 0.07 cr 0.0079 316 | 494 | -o02 | 0.08 FG | -0.0s6 | -336 | 394 0.003 | -0.083 cB | 0017 -690 | 70.1 | -o001 | -0.008 =130 | 2086 cD 0.0020 12 | 240 0.005 0.055 DE 0.0049 196 | 30.2 0.005, 0.055, EF | ~0000 | -120 | 240 0.005 | -0.045 Fe | -o079 | 316 | 494 0.02 | -0.082 =120 | B66 (-2.24)/1(1.85)(316.6)] = 0.008 Qu = ~(~1.30)/[(1.85)(208.6)] = 0.003 8Qnn = —(-1.20)/[(0.85)(136.6)] = 0.005 Pipe hy by 410 so Qu AB 0.081 1860 | 865 0.000 | 0215 Bc | oon 688 | m2 | -0.003 | 0.085 GH | 0.0055 330 | 388 0.000 | 0.085 HA 0.056 =mao | ii1 0.00 | -o.185 -02 | 3166 Bc | 0010 60 | 513 0.03 [0.120 cr 0.0075 30 | 484 0.002 | 0.064 FG 0.0052 -32 | 376 0.003 | -0.080 cs | oom =688 | 702 0.003 | -0.095 =100 | 2075 cp | 0.0028 uu | 262 0.0 | 0.056 DE | 0.0059 235 | 429 o.001 | 0.056 EF 0.0017 -1@ | 27 o.01 | -0.044 FC 0.0075, =30 | 434 | -o002 | -o.064 02 | 1402 -0.22)/{(1.85(316.6)] = 0.000 Qn (-0.22)/{(1.85)(140.2)] (=1.00)/[(1.85)(207.5)] = 0.001 324 0 CHAPTER 13 Odms Om 0) om a mm A a ‘diametet___C. ‘ameter 400 m 400 m 250mm 250mm diameter diameter lc iF le Om Om 300. 300-mm rms diameter Ota diameter ON mn Fig, 135 Bs ‘Assume C'= 120 for all pipes. ‘The pipe network shown in Fig. 13-6 represents a small industrial park. Find the flow rate of water in each pipe. pie | Dymm | tm | Qunamls tom | 410 aB | 500 | 1000 0.250 340 | 136 BE | 40 | 1200 0.120 12 | 26.0 EF 300 | 1000 0.130 2.3 FA eo | 120 0.250 ¥ 67 716 | 138.6 ac | so | 100 0.130 12 | 78 ce | «0 | 120 0.030 024 | 80 bE | 40 | 10 0.100 -18 | 180 EB 400 | 1200 0.120 312 | 260 306 | 59.8 ED | 400 | 1000 0.100 180 | 180 | 0.050 | 0.030 DI 300 1200 0.080 5.76 72.0 | 0.017 0.063 a 300 | 1000 0.020 -037 | 185 | -0017 } -a0s7 HE | 300 | 1200 =0.050 ~240 | 40 | oor | -o08 479 | 1565 FE 300 | 1000 0.130 0.012 12.0 | 923 | -0.063 | 0087 EH | 30 | 1200 0.050 0.0020 240 | 480 | -o018 | aos xc | 4&0 | 1000 -0.000 | -o.000s | -0.10 | 5.0 | —0035 | -o.0ss GF | 40 | 120 =0.120 | -o.006 | -312 | 260 | —o03s | -o.1ss nas [4713 BQ =~ hy/ler (hy Q)] = ~(-7.16)/[(1.85)(138.6)] = 0. ~(~3.66)/{(1.85)(59.8)] = 0.033 ‘AQ = ~4.79/{(1.85)(156.5)]= 0.017 AQyy = ~11.18/](0.85)(171.3)] = ~0.088 Pipe A by HO 4g Qave AB 155 | -0.013 0.265 BE 240 | -0.020 0.095 EF 0.7 | -0.018 | -0.085 FA 39 | -0013 | -0.235 96.1 PIPE NETWORKS 2 325 pipe hy by HI 4g Dace BC 0.0016 10 98 0.007 0.170 rary 0.00078 094 149 0.007 0.070 DE | -0.00050 | -0.50 10.0 0.015 | -0.035 EB | 0.0023 2.76 24.0 0.020 | -0.095 ~072 387 ED (0.00050 0.50 100 0.035) DI 10,0031 37 59.0 0.055 1H ~0.001s | 1.15 31 0.045 HE | -0.00087 | -1.04 32.5 0.045 2.08 132.6 FE 0.0034 3.40 50.7 o.o18: (0.085) EH 0.00087 1.04 32.5 0.013, 0.045 HG | 0.0060 | -0.60 109) 0.005 | -0.050 GF | -0.004 4.92 317 0.05 | -0.150 = 1.08 125.8 2.34/{(1.85)(96.1)] =2.03/{(1.85)(132.6)] 0.72)/{(1.85)(58.7)] = 0.007 —1.08)/{(1.85)(125.8)] = 0.005 Pipe Ay hy wIQ ag Quen AB 0.0039 3.90 147 0,006 oz BE 0.00165 1.98 208 012 0.107 EF | -0.0055 5:30 a7 os | -0.071 Fa | 0.00125 | -1.30 64 0,006 | 0.229 112 106.6 BC 0.0017 1.70 10.0 | 0.006 0.164 cD 0.00094 113 161 | 0.006 0.064 DE |} -0.00065 | -0.26 7.4 | 0.008 | 0.083 EB | -00015 | 1.98 28 | -o012 | -0.107 0.59 543 ED 0.000265 0.26 74 0.008 0.083 DI 0.0024 2.88 24 0.002 0.057 TH | -0.0017 -1.70 318 0.002 | -0.043 HE | -0.0017 204 45.3 o.oo | -0.035 -0.60 1429 FE 0.0055 5.50 7 | -o0 oon EH 0.0017 2.08 453 | -0.010 0.035 HG } ~0.00051 | -0.51 10.2 | 0.008 } -0.058 GF | 0.0039 ~4.68 312 | 0.008 | -0.158 235 151.4 AQ: = —(-1.12)/{(1.85)(106.6)] = 0.008 AQn A Qu = ~(-0.60)/{(1.85)(142.9)] = 0.002 AQw= 0.59/{(1.85)(54.3)] = -0.006 2.35/{(1.85)(351.4)] = -0.008 926 0 CHAPTER 13 Pipe Ay hy IQ 4a Quen AB 0.0040 4.00 148 | ~0.006 0.265) BE 0021 252 23.6 | -0.009 0.098 EF | -0.0039 -3.90 54.9 | -0.008 | -0.079 FA 0.0012 hag 63 | 0006 | -0.235 118 9.6 BC 0.0016 1.60 9.8 0.003, 0.167 cD 0.00080 0.96 15.0 0.003 0.067 DE | -0.00038 | -0.38 a8 0.006 | -0.037 EB | ~0.021 252 26 0,009 | -0.098 0.34 S72 ED 0.00038 038 88 | 0.006 0.037 DI 10,0026 312 34.7} 0,003, 0.054 TA 0.0015 1.50 349 | 0.003 | -0.046 HE | -o.0010s | -126 36.0 | 0.005 | -0.040 0.74 144 FE 0.0039 3.90 34.9 (0,008 0.079 EH 0.0010 1.26 36.0 (0,005 0,040 HG | -0.00067 | -0.67 16 0,002 | -0.056 Gr | 0.0081 492 31d 0.002 | -0.156 0.43 133.6 AQ, = ~1.18/{(1.85)(99.6)] = ~0.006 0.34) /[(1.85)(57.2)] = 0.003, AQmn = —0.74/1(1.85)(134.4)] 05 mn ~(-0.43)/[(4.85)(133.6) 002 4 © “ 1000 1000 oe 0-mm s00-mm diameter diameter 1200 m 1200 m 600. 400-mm diameter diameter 1000 m 1000 m 300-mm mm lp diameter damier 10. 456 m9 Ol ms 1200 m 1200 0m 300-m diameter ameter ameter 1000 m 1000 m 0mm 0mm ic diameter diameter __|r Ol ms 005 mi 1m Fig. 136 BT PIPE NETWORKS J 327 Compute the flow rate of water in each pipe of the network shown in Fig. 13-7. Assume C = 120 for al pipes. pire | Dyin | 2, | Qumsets | hy ft/te HQ | 40 | Que ‘AB 24 2000 50 0.00045 ois | 029 | 5.29 BG 1s 3000 10 0.00022 oso | 042 | 142 GH 18 2000 -25 0.00051, 04s | 034 | -216 HA Pr 3000 -50 0.00045, ozo | 029 | 4.71 158 BC 4 2000 40 0.00030 0.150 3.87 CF 15 3000 10 0.00022 0.660 1:53 FG 18 2000 10 0.000095 0.190 1.48. GB 15 3000 -10 0.00022 0.660 142 1.600 cD 15 2000 3.0 0.0017 1.133) 2.34 DE 2 3000 1.0 0.00067 2.010 034 EF 18 2000 -03 0.000010 0.067 0.63 15 3000 -1.0 0.00022 0.660 | -053 | -153 3.870 FE 18 2000 03 o.oo | 0.02 | 0067 | 033 | 0.03 EL 2 3000 13 0.0011 330 | 2538 | -033 | 097 LK 15 2000 -17 0.00059 | 1.18 | 0.694 | -0.33 | -203 KF 15 3000 02 =0.000011 | -o.03 | 0.150 | -o68 | 0.88 au | 349 GF 18 2000 1.0 o.oo0s | 019 | 019 | 04s | 148 FK 15 3000 02 0.000011 0.03 | 0.150 | 068 | 08s wD 8 2000 ~15 0.0019 | -038 | 0253 | 035 | -115 IG 5 3000 -10 0.0022 | -066 | 0660 | 040 | -060 082 | 1.253 HG 18 2000 25 0.00081 102 | 0.408 | -0.34 | 216 a 15 3000 10 0.00022 0.66 | 0.60 | -040 | 060 I 2 | 2000 -05 0.000063 | -0.01 | 0.020 | -o05 | 055 1H 18 3000 “25 0.00050 | -1.50 | 0600 | -o0s | -255 017 | L688 = =4.73/{(1.85)(3.870)] = Ary = -2.11/{(1.85)(3.449)} 0.41/{(1.85)(1.660)} = AQy = ~(—0.82)/[(1.85)(1.253)] AQv: 17 /[(1.85)(1.688) pipe Ah 5 4/0 4g Que ‘AB | 0.00050 10 | a1 | 010 519 Ba | 0.00083 129 | 0908 0.00 182 cH | -o09 | -07 | o3a | -os | -297 HA | -o000 | -120 | 0255 | -o10 | - rr > hese eee Fig. 13-11 CCaleulate the pressure drop from h to fin Prob. 13.13. I Head loss from h to f (any path): hi = 0.716 ft ie= 0.719% SUL ft 1946 ft Hence Ap = (1.946)(62.4)/144 8 psi Fitan equation ofthe form h, = KQ" to flow of 60°F water through 1000 ft of 10-in cast iron pipe. ‘The equation should hold over the velocity range 2 1 8s. 1 hy = (PMLID\(w"/2)——_-€/D =0,00085/(18) = 0.00128 Na = Du/v. For v= 2 ps: Nq = (3)(2)/(I.21 x 10) = 1.38 x 10°, From Fig. A-S, f = 0.0230: hy = (0.0230 000/(§2))(27/12)(32.2)}) = 1.714 f, O = Av =[(2x)(19)/5]@) = 1.091 fs. For v = 8 ps: Ne = (8)C8)H(1.21 x 10-4) = 5.51 x 10%, f= 0.0213, hy, = 0.0213{1000/ (18)](8"/(2)32.2)}} = 25.40 ft, Q = Av = [Gay (8) = 4.363 fs. Given hy = KO", at 2 fps, 1.714-= (KY(ILO91)". At 8 fps, 25.4 = (K)(4.363)" Jog 1.714 = log K + log 1.091 o og 25.4= log K +n log 4.363 @ 396 0 CHAPTER 13 ‘Subtracting Eq. (1) from Eq. (2) gives 1.171 = 0.6022n, n = 1.945. Substituting into Eq. (1), log 1.714 = log K + (1.945)(00g 1.091), K= 1.447; hy = 1.4479". 13.16 The distribution of flow through the network of Fig. 13-12 is desired forthe inflows and outflows as given. For simplicity n has been given the value 2.0. I The assumed distribution is shown in Fig. 13-12a. At the upper left the term F rQo| Qe!" is computed for the lower cieuit number 1. Next to the diagram on the lefts the computation of E nr |QaI"" forthe same circuit. The same format is used for the second circuit in the upper right of the figure. The corrected flow after the first step for the top horizontal pipe is determined as 15 + 11.06 = 26.06 and for the diagonal as 35+ (~21.17) + (11.06) =2.77. Figure 13-126 shows the flows after one correction and Fig. 13-12c the values after four corrections. yo 0? x6 = 79.400 270 16-840 3x3 3078 2 95+ 3°210, Wyte 2m Dawa wD gO xar 24s 24382 sor ss-aain 7 903300 tan ens 0 tbane 90a 29,+ 738.10 20,2008 no 6 an 2 eee r+ om 2v2606x10 42 “sui sessgomp 2st 8 Test 30zaa demons a0,--128 20, + a0 % 20, =000" 39, -0108 2 coos 32, = 0003 eo ig 132 13.17 Calculate the flow through each ofthe pipes ofthe network shown in Fig. 1313 (n =2). ' 1x 6% = 3600 2x 1x 60= 120 2x3 = 1800 2x2x30=120 BxS= 7S ax3KS= 3 Bxse =7$2x3K5= 30 axa =m 2x2x40= 160 veasta ams 2x1 x45= 90 aa ae Fig 13138 5 30 200 0 9-15 a0 1xS8.P = 3422.25 2x1 x58S— 117 2x3P— 1922 2x2x31=124 1 Deas= tags “2x3xa5= 15 wf) axaezs= 13 ealPanBaans Dede HLS = 166 xine 2x41. 166, 1936 2x1x44= 88 Fig. 13.13¢ ee =3 2 a0 10-2 -ous ¥ 5 ns J» S15" 25 us ) © @ Bas Bas PIPE NETWORKS 0 337 Determine the low through each line of Fig. 13-14 (n = 2). a [2x m2 ax2x19- 76 4xaP= 5476 1x32= 1024 2x1x32= OF 1x32 = —1004 2x23) = 104 a 176 264 me 6, 19-28-061 738 x2 KIDAT= 768 24 36.8= 2044 1080.44 2x 1%32.87= 65.74 2x1X3287= 65.74 2x33" =-169962 2x2%3033= 121.32 2x 3%382= 229.2 a 265.74 Tan 50.3 4m yer 50 FG be Fiq.2 FNAL 9 98y 7 — => lie =, = ro 5 : 7 50 75 rs cf i) © @ Mig. 13.14 Determine the flow rates in Fig. 13-15. The fluid is water at 20 °C and all five pipes have f = 0.0201. =n(2)(%)-Pako: x= Sil s he W(5)(G) “Sep Kim Bie Cay __(@)0.0201)4000) __(8)(.0201),5000) _ eV e@r@ay "eyez ayer ~ 106 .~ (8)(0.0201)(4000) (8)(0.0201)(3000) _ Ko yG22) ont. ‘Two head loss loops Loop ABC: 15.3703+ 10.6603 - 48.5701=0 o Loop BCD: 10.6693 + 15.3702 — 155493 =0 a) 9398 0 CHAPTER 13 13.21 ‘Three junctions: Junction A: Q.+0.=20 @ Junetion B: Q:= 05+ Oy o Junction: 2+0.= 2. 0 Solving these simultaneous equations by trial and error gives Q, = 0.81 cfs, Q;~ 1.19 fs, Qs=0.99 cfs, Q.= 1.80 fs, Qs = 0.20. 4000 t,8-in 20 In Prob, 13.19 the pressure at A i given as 200 psig. Find the pressures at B, C, and D. ' Pa = Pa ~ pBK,O3= (200)(144) ~ (1.94)(32.2)(15.37)(1.19)? =27 440 1b/fe? or 190.6 psig Pe=Pa~ p&K,Q} = (200)(144) ~ (1.94)(32.2)(48.57)(0.81) = 26 8091b/f or 186.2 psig Po = Pe~ pgK.Qi = (186.2)(144) ~ (1.98)(32.2)(15.37)(1.80)?= 23702 b/t or 164.6 psig, Repeat Prob. 13.19 forthe boundary flows indicated in Fig. 13-16 ' ‘hy = (FM LI D128) = 8fLO*/ x°gD? = KQ? From Prob. 13.19, K, = 48.57, K,=15.37, K,= 10.66, K,= 15.37, and K,= 1554, ‘Two head-loss loops: Loop ABC: 15.37Q+ 10.6693 - 48.5703 =0 0 Loop BCD: 10,6693 + 15.3703 15540 =0 @ ‘Three junctions: Junction A +0:=20 0 Junction B 2:= 05+ 2.41.0 “ Junction C: Q1+0.=Q.+05 0 Solving these simultancous equations by trial and error gives Qy 2, =0.451 cfs, Q-= 0.049 cfs, 050 201% Lows PIPE NETWORKS 1 339 13.22 Repeat Prob. 13.20 for the flows found in Prob. 13.21. Fe _ pn= pa peK.O3 = (200)(144) ~ (1.94)(32.2)(15.37)(1.275)'= 27 239 b/f oF 189.2 1b/in* Pe= Pa p8K,Q% = (200)(144) ~ (1.94)(32.2)(48.57)(0.725)* =27 205 Ib/ft? or 188.91b/in* Po = Pe~ P&K.Qi= (188,9)(144) — (1.94)(32.2)(15.37)(0.451)? = 27 006 Ib/f or 187.5 Ib/in* 13.23 Compute the junction flows in Fig. 13-17, taking flow into the junction as positive. Each pipe is 30 m of 0-ram-diameter cast iron, with f = 0.0294. ' hy = Sply = 8{LQ"/2°gD" (800 — 5009/9. 79 = (8)(0.0294)(30)(Q,)*/()*(9.807)(0.060) y= +0.0181 m/s (500 —200)/9.79 = (80.0294) 30)(0.)7/()*9.807)(0.060)Q4= -0.0181 m’/s (S00 — 400)/9.79 = (8)(0.0294)(30)(Q.)°/(x)°(9.807)(0.060)* Q2= -0.0104 m'/s. (600 ~ 500)/9.79 = (8)(0.0294)(30)(Q3)?/()°(9.807(0.060)"Qy= +0.0104 m"/s pp=400KPa| 1y4:200KPa Fig. 13:17 1824 Determine the flow in each pipe of the network shown in Fig. 13-18, using f = 0.02 throughout. H Taking x =2, hy, = f(L/D)(V*124) = f(L/D)(1/28)(4Q/xD*? = (8(L/x7gD*(") = KO® Hence K = O.81fL/gD*, and the K value foreach pipe is Diameter, in 3 4 5 6 7 8 K 1030 | 368 | 160 | 804 | 224 | us ‘The assumed flows are indicated on the figure in parentheses. For loop AEDB, (By = ~[(4030% 0.5") + (11.5 x 0.17) ~ (22.4 x 0.24) ~ (368 x 0.7) 2f(1030 x 0.5) + (11.5 x 0.1) + (22.4 x 0.2) + (368 x 0.7)] = —0.05 ets and for loop BDC, Ay =[(22.4 x 0.2%) + (80.4 % 0.3") ~ (160 x 0.5°)]/2|(22.4 x 0.2) + (80.4 x 0.3) + (160 0.5)] = +0.15 cfs ‘The corrected flows appear on the figure below the first assumed flows. Recomputing A for each loop yields, ‘A, = +0.001 efs and A;= ~0.001 efs. Fig. 13.18 340 0 CHAPTER 13 13.25 Replace the quadrilateral network of Fig, 13-19 by a single SOO-ft-long pipe AC, Assume fis the same for all Pipes. ! (anc (have hy = (LID)(W"/28) (5001 8)(V i028) + fCENV aI28) =f CEM V 2/28) + 5 CPNVL28) ‘TSOV3 + 250V3, = 800V 3; +333V3, Va= Van Vin = OV (750 x 81 + 250)V3, = (800 x 81+333)V3, Vie = 0.96¥i4 D°V =2°Va5+ FVig= Vie + 9(0.96Vad)— D*V = 12.63¥au (500/18 V3 + V3, = (1000/D)¥" 750V3 + 250V3, = 61 000V3, = (1000/D)V* o But V=12.6¥y/D* @ Substituting (2) into (2) yields D* = 1.308, Hence D = 1.055 ft. A S00RBin ig eoott,1-ft Fig. 13:19 13.26 The pipe network in Fig. 13-20 consists of pipes as follows: AB, 5000 ft, 12 in; BC, 3000 ft, 6 in; CD, 8000 ft, in; DE, 7000 ft, Sin; EA, 4000 ft, 10in; BD, 7000 ft, 12 in. A flow of 4000 gpm enters the system at D, while outflow at the junctions is as follows: A, 500 gpm; B, 300 gpm; C, 1000 gpm; E, 2200 gpm. Find the fow in each pipe and the pressure at each junction if the head at D is 400 ft. Take f = 0.023, ' 1, = 8fL.Q*/*gD* = (8)(0.023)(LQ*)/()*32.2)D* = 0.000579L.0"/D" D vive | Dyin k 10 AB 2 209 oon | ac 6 56.0 2 cD a 0.143 1x2 | DE 8 206 10 BD 2 4.05 oa | Az 10 375 Loop 1: [—(2.99(3.62)" — (4.05)(3.40)? + (30.6)(2.43)" ~ (5.75)2.5)°V 2f(2.9(3.62) + (4.05)(3.40) + (30.6)(2.43) + (5.75)(2.5)] = -0.27 Loop 2: ‘4, = ~[~(56)( 0.89)? —(0.143)(3.13)° + (4.05)3.4)*/2{(56)(0.89) + (0.143)(3.13) + (4.05)(3.4)] = -0.01 Ay = [=(2.9)(3.89)* — (4.05) 3.66)? + (30.6)2.16)? ~ (5.752.777 2A(2.9)(3.89) + (4.05)(3.66) + (30.6)(2.16) + (5.75)(2.7)]=0 (=45,3 — 1.4 + $4.1)/2(50.5 + 0.45 + 14.8) = -0.056 44 ~ 52.5 + 142.5 —48)/2(11.3 + 14.6 + 66.1 + 15.9) = ~[-(56)(0.96)* = (0.143)(3.20)? + (4.05) 3.6)7/2(53.6 + 0.46 + 14.6) = +0.01 Bye 441-53.141402-442—=-12 — Ap= 505-1464 53.2= 41.2 PIPE NETWORKS J 341 Flow in 1000's gp £ = 0,023 mhtrd eet ay ate 4 hp 303! 1740 347" _, #303945" 3 | gem 3 960 asy 00" Fig. 13-20 342 0 CHAPTER 13 13.27 The pipe network in Fig. 13-21 consists ofthe following pipes: AB, 3000 ft, 8in; BC, 3000 ft, 12in; CD, 10.000 ft, 36 in; DE, 8000 fe, 24in; EF, 5000 ft, 6 in; FA, 4000 ft, 8in; BE, 10000 f, 6 in; BF, 8000 ft, 12in. Inflow at D =6.cfs. Outfiows are A, 2cts; B, cfs; E, 1.Sefs; C, 0.5; F, Lefs. Assume Manning n = 0.015, ‘and find the flow in each pipe and the pressure at each junction ifthe pressure at D is 120 ps Hn=001 S=n°¥7/72.21R =n°Q*/2.21(D/4)D"/16)] = ((0.015)°Q7(16)(6.4)}/(2.2127D") = 0.00106(Q*/D™) = kQ?=(0.00106L/D™)(Q")___-k = 0.00106L/D"" D Db 0.001061 k aa_| oa | oie 38 m4 sc | vo | 10 318 32 co | 30 | a5 0.6 08 ve | 20 | 8.48 o21 zr | os | aoms 530 | 26 ra | os | on6 am | 365 gr | io | 10 a8 8.48 ze | os | dass | 1.0 | a2 x OT AV.3Y? + (8.48)(1.6)° + (36.5)(0.7)* AP AT-ANA.3) + 48).6) + G6.5NO.T] = TOO Loop 2: o_O.) = (8.48)(1.67" M2" 519+ 1601) + BABB] P02” a ACT ATR) + (6.48). 36) + BESTA] 7°58 =1110+31-15.6 = SRY + 26038) + BABI] “PS Oe —C =~ 31G.2)(2.02) + (0.03)(2.52) + (0.21.48) + 432.) at SS FRT HEA + 6.48071) + CESNOTI T° A W14.14251~42 037 2" 2f432)(0.18) + (216)(1.08) + (B.48)(0.71)] “ a 242-034 1.64141 a = BG. 2)C.74) + (G0B)(3.24) + (0.21)(2.76) + 432)(0.18)] ~ * Aye- eB ggg [2T.AVG.AD) + (B.48)(1-18) + 6.50.89] 155+ 109 -11.8 AF ERAIU BO) + CIOKOTD) + BABI] “7 hee =23-03-+1.64 155 02s ARHEH)+ 00.19) + O22) + BHOH] =3749.5425.7 S 1[(27.4)(1.16) + (8.5)(1.06) + (36.5)(0.84)] = +0.01 PIPE NETWORKS 0 343 a= ~ B32) (0-28) + 216)(0.78) + BU. _ +1.44+338 - ME DV94) + (C.03)(3.44) + 0.212.56) + (32)(O-2B) ‘And after several more trials, final results are as shown on the last sketch of Fig. 13-21. Ae 0.03 pressures (psi) 875 104.5 119.8 120.0 119.5 98.0 amoAeh Firat ertal Second ertal | Third ertal 299 3. Fig. 13:21 944 0 CHAPTER 13 Fourth etal Lt 2.9 Fifth extal ost D_ Fig. 13-21 (continued) 13.28 Prepare a computer program written in Fortran to determine the flow in each pi 7 pipe in a pipe network by the Hardy Cross method. The program must be usble in both the English system of units and the International system of units. We STIS PROGRAM DETERMINES THE FLOW RATE IN EACH PIPE IN PIPE NET- WORK BY THE HARDY CROSS METHOD. 1 CAN BE USED FOR PROBLEMS IN BOTH THE ENGLISH SYSTEM OF UNITS ANO THE INTERNATIONAL SYSTEM OF UNITS. EACH LOOP MIST BE NUMBERED AND EACH INDIVIDUAL PIPE, MUST INPUT DATA MUST BE SET UP AS FOLLOWS. caso 1 conve 1 ITER 0 (ZERO) OR BLANK IF ENGLISH s¥sTEM (OF UNITS 18 70 BE USED. ENTER T (ONE) IF TRTERSATIONAL SYSTEM OF ITS 18 TO BE cone 2 ATER 0 (ZERO) OR BLANK IP ONLY THE BE PRINTED. ENTER 1 (ONE) IF THE RESULTS APTER EACH ITERATION ARE 0 BE. PRINTED. ‘commis 3-5 BARTER INTEGER NUMER (RIGHT ADSUSTED) NETWORK, CouNats 6-80 EXTER TITLE, DATE, AND OTHER INFORMATION, (camo 2 COLUMNS 1-2 ENTER INTEGER NUMBER (RIGHT ADJUSTED) ‘TELLING HOW RANY PIPES ARE IN THE FIRST COLUINS 3-4 ENTER INTEGER NUMBER (RIGHT ADJUSTED) ‘TELLING HOW DANY PIPES ARE IN THE SECOND oor. cous 5-6 ENTER INTEGER NUMBER (RIGHT ADJUSTED) ‘TELLING Wow MANY PIPES ARE IN THE THIRD (CONTINUE PATTERN FOR ALL LOOPS.) ccaRD 3 COLUMNS 1-3 ENTER INTEGER NUMBER (RIGHT ADJUSTED) TO IDENTIFY AND TO REPRESENT A PARTICULAR PIPE NETWORKS J 345 conus 4-6 TP THIS PIPE 18 16 COMMON WITH ANY OTHER too, ENTER INTEGER NUMBER (RIGHT AD= Jusnéo) IDENTIFYING THAT LOOP. OTHER Wise, ENTER 0 (ZERO) OR BLAM. [ENTER NUMBER INCLUDING DECINAL GIVING DIAMETER OF THIS PIPE (IN INCHES OR MILLIMETERS) - covets 17-26 ENTER NUWDER INCLUDING DECIMAL GIVING LENGTH OF THIS PIPE (IN FEET OR METERS) couuieis 27-36 ENTER NUMBER INCLUDING DECIMME GIVING INITIAL ESTIMATE OF FLOW RATE FOR THIS PIPE CIN CUBIC FEET PER SECOND OR CUBIC METERS PER SECOND). (ENTER CLOCKWISE Flow RATES WITH RESPECT 70 THIS LOOP AS POSITIVE. AND COUNTERCLOCKWISE FLOM RATES AS NEGATIVE.) ITER NUMBER INCLUDING DECIMAL GIVING AZEN-WILLTAMS ROUGHNESS COEFPICIENT FOR (WaerER ADDITIONAL, CARDS LIKE CARD 3 FOR EACH REXAINING PIPE iN THE FIRST LOOP, THEN FOR EACH PIPE IN THE SECOND Toor, ETC, FOR ALL 100PS.) MULTEPLE DATA SETS FOR SOLVING ANY NUMBER OF PROBLEMS MAY BE INCLUDED POR PROCESSING. DIMENSION TETLE(13) ,DIAM( 100,10) ,91100,10) , ROUGH (100,10), PN HLDSS(100, 10) ,Q0BL (100) REAL LENGT(100, 10) INTEGER UNTTS, LOOPS, PLPES(100) ,PPLOOP ,PIPENO(100, 10) ,OLO0P(100, 10) FREAD(5, 100, N0=2) UNITS, WRITE, LOOPS, TITLE 100 FORMAR(211,13,12R6,83) were (6,105) 77EE 305 FORMAT 1246,03,////) Conrr=t.318 ‘ERnoR=01 TP (UTI. Ep. 1 COEFF. 6492 BF (UNITS. »D_1) FACTOR=1000.0 IF (UNITS.Bp. 1) ERHOR=.001 BREAD(5,101) (PIPES (3) » 101 Foran (4012) o 200 Jet ,L00%s Pevoop-PrPes (3) 200 READ(S, 102) PEPEND(2,X) ;OLOOP (JX) ,DIAN(,X) ,LENGTH(S,K) ,Q13+K) OUGHT KD 102 FoRARE (213, 410.0) mr-l 208 TF QWRITE. 69.1) ¥RITE (6, 106) 106 FORDNT(//3K," ITERATION NOW" 12, /,20X, "===" PP COWRITE,£9.1.AND-UNITS.29-0) E2TE(6, 107) 107 FORMAT(" 100° NO.” PIPE NO. DIAMETER’ (IN) LENGTH (FT) ROUGIBVESS s'PuaW RATE (CFS) MEAD LOSS (PT)",/" see a {IF OWRITE. HQ. 1 AND. UNITS BQ. 1) HRITE(6, 108) 108 FoRNAT(" 100P NO. PIPE NO, DIAMETER (9a) LENGTH ( M) ROUGHNESS PUM RATE (CHS) HEAD LOSS CM)" ,/* ~ a 0 201 1, LooPs IF (3-59. 1)WITONTTL ‘sHL0ss*0.0 8990.0 PPLOOP=PIPES (3) 10 202 K=1,,pPLOOP 085354) =(ABS(Q(,K) )*4.0%41.63/3.14159265/ (IAM(3,¥) /ERCTORD ree Ne 63/OORFE /ROUGM (SK) )**(1,0/.54) LENGTHS XD “reigia,k) ,1NDCSOO> ,9¢1009 ,HC190> »86209 KC 2409 Pom Jet vo too irypecsoeds Neas=st008!F Next J reno Tivtes Upaint Ueaints ss riTuee READ Te,RK TOL, UNULDEFA "NISMUSC OR SI, KK=NO. OF ITER.) UNUSKIN. UISC. "Tou=TOL gwaice iN ITERATION, DEFAMDEFAIAT COCP.~ EITHER ¢ OR EPS Unt Teee. 729 onan 7a: rrinr® “G8 cusvarwer uur SPC uscosity IN F1-2/G=" 4s OTO 170 UeRINT! S1 WITS SPEC. .UISCOSITY IN, HP2/s—y NU CPRINT? DESIRED TOLERAICE®: TOL} |" NO. OF ITERATIONSS* KK Kerints "PIPE QUErS OR Hoa/s)" LCFT ORM) DUET" OR HD” HC OR EPS ReaD Net Tres TP Neto THEN 310, 1 TO Net READ 1,00,L,0,x31 IF X90! THEN XODEFA, ND: sO0=FLOU, L=LENGTH OWBIAETER X3=C OR EPS Tveet ECO (KP) =L/¢2"x040-Se,7054°2)¢ ou ELEMCKPYTJ=1/¢,708440mNU)# ELON KPH DOG, FADD Treeaners Beat 0521 | ELEMCKPDSUNITSHL/OC9"BXH0"4.8700) “14d Berit mint? vty CPRINT-USING® —waNANwaMNR 01D sL{0123¢ Neri READ NPS, TT TF NPSeO" THEN 260, FOR isi TO NPS: READ 1,041 ATYPECH #35 keeaecr-1et 950 0 CHAPTER 13 1g puso THEN 490 [True NO. ,O0-PLOM,DOWDEL Gy Hisi2,HS,HécEGUACLY ‘SPACED PTS ON PUP CURVE rrr h acinaas ELENCRP DSA Eleni 3 neie-3! #ch9-H2)-M1/ + TeeTt=0 THEN 700 Pom Jet To 111 Temmoctesay+ IF 1<0 THEN 570 it super coro 590 scpait GH NTY GoTo 610,620,400,470 Recumncarys oro 660 ReveELENCHP +19 sa@6¢061)> 9 1 REYCL! THEN REVEL! See cere ris eeoreiecan nae casa Rogenoa CELONKP #1) 62! MELENCKPe 2) #01949! ALLEMCRPO DD OCT) 2) Nex Por Jor rots TARBSCINOCIPAGD IF ITYPECI INS THEN 760 Ber=aci rests) x00 Tenteetiets sora 530 Terooactou THEN 830 Se ne ES ae ane rch Eo ne ee ee Spectacle erred iat eae i Lc ca cece te . Bana Bitte Recess eee ee I Pe, Mak RY omen ren ia secaggee Eid Te ixestipeaeo Ten 975 koe NET J Corants T°. pnts UbRINT USING" was wen HCO) noms ome Lome itm cai 150mm diane 1 325m le " oto 10m 150mm ameter T3-mm diamerer ome diamete] [000m 30m © c= 10 mss Far M1 some cane 1000 diameter 250mm diameter om ® w om @e 00 cnt ‘ , © sommdiancer ions [Sommer ome 10m" 506-mam diameter @ 200 m 400 om : ® cnt lcm 120 C= 100 500-mm diameter 500-mm diameter 500. diameter| |750-mum diameter om tom oan ama 2 Lome Cain Lome Caio Oma) BEV Coe ' “lise daneel 7a a mom ‘om na eog tte celat peace tn tan (0) Pipe eon Fig. 1322 196 7 SYHOMLAN Sdld rome Lome n9mp osm 20m oO 26 ms} 1 u Dhioo mys cana | hom 10m Q somts sqm Tis mis NR @ ars ml Lom Ww 20m] Om Lome Lops @ 075m oSma we cnn romaZa hsm y, osmal@ vu @fosms fon Lows @ " ® 10m O15 me (2) ita Now rate extimates Fig. 13:22 (continued) Os €l WAldVHO O zs 20mYs 1om'is 20m 2532 ms Lomys os 06s ® 1 2s7menl@ 0 Orem toms om Lom (6) Adjuied flow rates ser 16 iterations Fig. 13-22 (continued) 898 7 SYHOMLAN Jdid 354 0 CHAPTER 13 13.32 Use the computer program written for Prob. 13.31 to solve the network problem displayed in Fig. 13-23. The pump data are as follows: Qmis | 0 | oo | 006] 009 Am 3 | 26 20 ' Inpet oaTA ExaneLe Data $1,30,-001,-000001 ,100. bara 5/44 Data 1)-12,400.,0.3,.0 DATA 2) 03,300,513, -0 DaTa 3) 0,300.5 26,0 Data 41103, 400. bara 3,03, 300, Data 2,8 bata 710.0 bara 1,0. DATA 8, .06, .03,0.,27. 26.420. Dara 16, IND) DaTA 3)2414-3,3) Dare 1 ,NO0gs bata sy1i7. Dara 91x Dara 356,4,2,2)141,4,9 1357.8 316 )4 vipat APL SI_UNITS SEC. VISCOSITY IN M2/s= .000001 DESIRED TOLERAICE= 001 NO. OF ITERATIONS 30 PIPE QCCFS OR W378) LOFT ORM) OCFT ORM) Hu C OR EPS 1 ‘o-12000” 00-0000, 9.20000" 100,00000. 2 ©:03000 30000000, 2113000 100:00000, 3 8200000 50000000, 0.60000 L00“00000 ’ 703000 400:00000, 730000 490:00000, 5 8.03000 300-0000, 8/0000 40080000 RESERVOIR ELEV. DIFFERENCE 15 7 RESERVOIR ELeV. DIFFERENCE 18 8 Pump CURVE, DO= 409, He 90 2. 26 20 COEF. IN PUMP EO.= 30) W11.11112 ~839,5985 ~6172.041 Wor a2 tna 3 48 3 2 eee nl 8 8 7B ITERATION NO. 1 Su‘OF FLOW CORRECTIONS® "011988 ITERATION NO. 2 SUM OF FLOW CORRECTIONS= 0.1040 ITERATION NO. 3 SUM OF FLOM CORRECTIONS= 0.0972 ITERATION NO. 4 SUM OF FLOW CORRECTIONS= 0.0094 ITERATION NO. 5 SUM OF FLOW CORRECTIONS= 0.0006 ELEneNT Peau 2 3 4 lriaer 3 117/000 PIPE NETWORKS 0 355 B\o \ AN e117 m %y ® Fig. 13-23 CHAPTER 14 /] Flow in Open Channels 14.1 Water flows in a rectangular, concrete, open channel that is 1.0m wide at a depth of 2.5 m. The channel slope is 0.0028. Find the water velocity and the flow rate. ' V=(LOmXR™\(6")—n=0.013 (from Table A-13) R= Alpe = (I20/25)/(25 + 12.0425) = 1.765 m v= (1.0/0.013)(1.765)°%0.0028)'?= 5.945 m/s Q= Av = [(12.0)(2.5)](5.945) = 178 m’/s 14.2 Water flows in the symmetrical trapezoidal channel lined with asphalt shown in Fig. 14-1. The channel bottom drops 0.1 ft vertically for every 100 ft of length. What are the water velocity and flow rate? ' v= (1.486/n\(R°Y6") n= 0.015 (from Table A-13) R=Alpe A= (16.0/4.5) + 2)(4.51G)(4.))/2) = 132.882 16.04 2VES FTE = 44.46 R= 132.8/44.46 = 2.9878 1/100=0.00100 w= (.486/0.015)(2.987)°¥0.00100)"? = 6.498 ft/s Q = Av = (132.8)(6.498) = 863 ft/s - Se aon 143° Water ig to flow at a rate of 30 m°/s in the concrete channel shown in Fig. 14-2. Find the required vertical drop of the channel bottom per kilometer of length, HA = (3.62.0) + 4.0-2.0)[(1.6+3.6)/2]=12.40m? v= (1.0/n)(R2\%" pe 364204 VG0-20PF GS-16} +16=1003m — R=Alp, 2.419 = (1.0/0.013)(1.236)"%(s)""_-s = 0.000746 ‘This slope represents 2 drop of the channel bottom of 0.000746 m per meter of length, or 0.746 m per kilometer of length. Q/A =30/12.40=2.419 m/s .40/10.03 = 1.236m * 36m Mig. 142 us Ms 4a FLOW IN OPEN CHANNELS 0 357 ‘Water flows in the triangular steel channel shown in Fig. 14-3 at a velocity of 2.9 ft/s. Find the depth of flow if the channel slope is 0.0015. 1 .486/m)\(RY(5) R= Alpy = 2{{(d){d tan 27.5°)/2Y/ (2d Joos 27.5") 2.9 (1.486/0.014)(0.23004)"(0.0015)" d= 2.578 23094 4 J i Fig. 143 ‘After flood had pasted an observation station ona river, an engncer visited the site and, by locating ood tmarks, performing appropriate surveying, and doing necessary computations, determined tha the crost sectional area, wetted perimeter, and water-surface slope at the time ofthe peak flooding were 2960 m, Sst m, and 0.00076, respectively. The engineer also noted thatthe channel bottom was “earth with grass and = 0.030), Estimate the peak flood discharge. 1.0/n)(R2)(s"*) = (1.0/0.030)(2960/341)°*(0.00076)"* = 3.881 m/s = Av = (2960)(3.881) = 11 490 m/s ‘A rectangular, concrete channel 50 ft wide is to carry water at a flow rate of 800 cfs. The channel slope is 0.00025. Find the depth of flow. ' (1.486/n)(R*)(69) = Q/A = 800/50d = 16.00/d R= A/p, = 504/(50+ 2d) 16,00/d = (1.486/0.013){50d /(S0 + 24))°*(0.00025)"? "This equation is not readily solvable, but a trial-and-error solution (not shown here) reveals that d = 3.92. Prepare a computer program that will determine the depth of flow of water in a rectangular channel (as in Prob. 14.6). ' [THIS PROGRAM DETERMINES THE DEPTH OF FLOW FOR OPEN CHARGIEL PLOW TW RECTANGUTAR SECTIONS. IT CAN BE USED FOR PROBLERS 1H BOTH THE [ENCLISH SYSTER OF UNITS AND THE INTERNATIONAL SYSTEM OP UNITS. can 1 coum t TBYTER © (ZERO) OR BLANK IF ENGLISH SYSTEM Of rts 15,10 BE USED. ENTER 1 (ONE) TF cous 1-10 ENTER NUMBER INCLUDING DECIMAL, GIVING WIDTH OF CHANNEL (IN FEET O8 METERS) cous 11-20 ENTER NUWBER INCLUDING DECIMAL, GIVING [FLOW RATE (IN CUBIC PEEP PER SECOND OR CUBIC HETERS PER SECOND) . couvimis 21-30 EXTER NOMBER INCLUDING DECIMAL, GIVING cous 31-40 ENTER NOMBER INCLUDING DECIMAL GIVING MULTIPLE DATA SETS FOR SOLVING ANY NUMBER OF PROBLEHS MAY BE 8 8 DIMENSION TITLE(13) 1 READ(5,100,EN=2) ww2Ts ,71TLE 368 0 CHAPTER 14 48 m9 100 Fonva (31 1346) warTe(6, 105) TET 108 roman 12" 1386, ////) Comte 1.486, EP (UMITS.B9.1)COEFF=1.0 READS, 101) 4,995.8 101 roman (4Pi0.0) ool TRY =COEEF /¥* (W4D/(4e2.0°D) )*¥(2.0/3.0) #99R(8)-2/8/D 104-040-001 ‘TRYD=COEFF/t*(WD/(W+2-00) )** (2.0/3.0) *S0RE(S)-Q/4/0 ap (revimmy2) 102, 102,109 103 TRYIeTRY? 20 70 108 102 b+p-0.0008 1 (ONITS.E9.0) WRITE 6, 105)4,9,8,88,0 106 FORMAT 1x, ‘GivEN DATA'FOR AN OPEN CHANNEL, FLOW THA RECTANGULAR SE ACTION", //5K, "WIDTH =" P7.1," FE" //5k,"FLOW RABE =" ,P7<1," CU FT/S Ns //SK, SLOPE. =",P1007 1/5, ARMING NAVRLUE. = P6.36/// 71% THE, SEPTH OF Plow WILL GE,P2.2," PT") 1F (QUITS. BQ. 1) WRITE (6, 107)¥,9.8,8,D 307 PORMAT(1x,"GIVEN DATA’FOR AN OPEN CHANNEL FLOW IM A RECTANGULAR SE ACTION", //5%, "MOTH ="4F7.1," W*, //Sk, FLOW RATE =" 7.1," CU M/S Ts//5%, "SLOPE =" F10.7,//5k, HMONG N-VALUE #*,P6.3,////1%, "BME D ‘EPR OF PLOW WILL BE’,P9.2," MN") 701 2 sto Solve Prob. 14.6 using the computer program of Prob. 14.7, 1 Taput OSAMPLE ANALYSIS QF PEN CHANNEL FLOW IN A RECTANGULAR SECTION 50.0 800.0 0.00025 0.013 (Output SAWPLE ANALYSIS OF OPEN CHANNEL PLOW 3M A RECTANGULAR SECTION GIVEN DATA FOR AW OPEN CHAMNEL FLOW IN A RECTANGULAR SECTION wrod = 50.0 Fr FLOW RATE = 200.0 CU FT/S SLOPE * 0,0002500 savornnc W-VALUE = 0.013 {Me DEPTH OF Flow WILL BE 3.92 Pr ‘A-rectangular channel (n= 0.016) 20 m wide isto carry water at a flow rate of 30 m'/s at a slope of 0.00032. Find the depth of flow using the computer program of Prob. 14.7. 1 Input ISAMPLE ANALYSIS GP GPEN CHANNEL PLOW INA RECTANGULAR SECTION 2.0 30.0 0.00032 0,016 Mai FLOW IN OPEN CHANNELS J 359 Output SAWPLE ANALYSIS OP OPEN CHANEL FLOW IN A RECTANGULAR SECTION wipmi = 20.0 4 30.0 cv M/s store = 0.000220 sannG NVALUE = 0.016 ‘we DEPTH OF FLOW WILL BE 1.25 4 ‘A corrugated metal pipe of 500 mm diameter flows half-full ata slope of 0.0050 (see Fig. 14-4). What is the flow rate for this condition? ! v (1.0/n)(R?°)(s"%) = (1.0/0.024)|(88)/4}?(0.0050)"* = 0.7366 m/s Q= Av = {(()(88)7/41/2} (0.7366) = 0.0723 m/s (Note: The hydraulic radius for both a circular cross section and a semicircular cross section is one-fourth the diameter.) Lo Fig. 144 ‘A 2tin-diameter cast iron pipe on a a slope carries water at depth of 5.6in, as shown in Fig. 15a. What is the flow rate? I v= (1.486/n)(R)(s"2), R= Alp. The applicable area in tis problem is the shaded area (AECDA) in Fig. 14.55: AB=BC= 12in (both are radii), BE = 12 —5.6=6.4in, Therefore, AE = EC = (12) = ca 10.13 in and &ABE = 4 EBC = arccos (6.4/12) = 57.77", (Atea)sacoa = {(2)(24) /4][)S7.77)/360" 4145.19 in, (Area) anes = (Area) pcen™ (6.4)(10.15)/2 = 32.48 in", (Area) arcon = (AFE@)ancoa ~ Q)(Atea) ges = 145-19 — (2)(32.48) = 80.23 in®. The applicable wetted perimeter in this problem is the are distance ADC in Fig. 14-5: pe = ADC = ()(24)[()(57.77)/360"] = 24.20 in. R = 80.23/24.20 = 3.315 in, oF (0.2763, v = (1.486 /0.012)(0.2763)>(as)"* = 2.627 ft/s, Q'= Av = (80.23/144)(2.627) = 1.46 €/s. ie «@ co) Fig. 145 360 0 CHAPTER 14 42 143 1414 was ‘A 500-mm-diameter concrete pipe on a ay slope is to carry water at a flow rate of 0.040 m°/s. Find the depth of flow. See Fig. 14-64. HF v= (1.07my(R\(6"7) = Q/A=0.040/A 0, 040/A = (1.0/0.013)(R°™Y(a)"?——AR® = 0.01163 Since R= A/p,, AM ip? = 0.01163 o Equation () contains two unknowns, A and p,.; however, both unknowns can be expressed in terms of the ‘unknown depth of flow, d. The applicable area in this problem is the shaded area (AECDA) in Fig, 14-66: AB=BC=0.25m (both ae rai), BE~0.25 Therefore, AB = CE = VOSS) tO I}, xABE= EBC = arceos (0.25 — d)/0.25}, (Ates)sacon™ [2s =a er 1025-410 251} = o0m01 (aon 25 -¢VOBl=OB aE (Ate) nea = (Atea)gora= 225 = OVO2T = O25= (Area) acon = (Ate8)ancos ~2(Are8) anes = (0.001091) (aroos 025 0.25-d) | ef 225= ova = OB=3) 0.25 ~ 025 pe =ADC=(2¥(0. sy{ Crass (025 2/0.25)) = (.008727)(arceos ‘Therefore, substituting into Eq, (1), {0.010 (ares (0.25 — 4/025) = 025 — AVOBF=OB=DY_ 4 yyyggm? [(0.008727) (arecos [(0.25 = 4)/0.25]} > s ‘This equation is not readily solvable, but a trial-and-error solution (not shown here) reveals that d = 0.166 m or 166mm. w wy Mig. 14-6 Solve Prob. 14.11 utilizing Fig. A-18. Bs = dp = 0.0025 ft/ft. From Fig. A-15, Oras = 11.4 ft/s and gy = 3.6 0/8. These values of Ona and Una must be adjusted for an m value of 0.012 for the given cast iron pipe (Fig. A-15 is based on an n value of 0.013): (Qua)n-oora/ 11.4 = 0.013/0.012, (Qpr)n-nor2 = 12.4 fs, (Upu)na0ia/3-6 = 0.013/0.012, (Undennor2 = 3-9/5, {aya = 5.6124 = 0.23, or 23 percent. Enter the ordinate of Fig. A-18 with a value of d/dya of 23 percent, move ‘horizontally to the line marked “discharge,” and then vertically downward to the abscissa to read Q/Quy = 12 percent. In a similar manner using the “velocity” lin, read v/v, = 63 percent. Therefore, Q = (0.12)(12.4) = LS ft/s, v = (0.63)G.9) =2.5 f/s. Solve Prob. 14.12 utilizing Fig, A-18. Is = sts = 0.00200. From Fig. A-16, Ona =0.169 m/s: 0 / Qn = 0.040/0.169 = 0.24, or 24 percent, From Fig. AZ18, d/dys= 34 percent; d = (0.34)(500) = 170mm. ‘A.30-in-diameter concrete storm sewer pipe must carry a low rate of 9.0cfs at a minimum velocity of 2.5 ft/s. Find the required slope and water depth. ' A= Ql =9.0/25=3.6008 —— Agu [(x)(8)'/4] = 4.909 8 AlAga,=3.600/4.909=0.73 or 73 percent 14.16 war FLOW IN OPEN CHANNELS 0 361 From Fig. A-18, d/dy,= 69 percent and R/Rya = 116 percent. = (0.65)(30)=20.7in v= (1.486/n)(R™\(s"%) 2.5 (1.486 /0.013)((1.16)(¢8)/4])7%0) 000734 Acconcrete pipe must carry water at a slope of 0.0075, ata velocity of 0.76 m/s, and at a depth of flow equal to ‘one-tenth its diameter. What is the required pipe diameter? 1 v= (LO/m\(RYs") 0.76 = (1.0/0.013)(R**)(0.0075)"* R= 0.03853 ft From Fig. A-18 with d/dsai= 10 percent, R/Ra= 25 percent. 0.03853/Ryn=0.25 Ryu =0.1541m —0.1541=d/4 d= 0.616m or 616mm Prepare a computer program that will determine either the depth of flow or the flow rate for open channel flow in circular sections. (© THIS PROGRAM DETERNINES EITWER THE DEPTH OF FLOW OR THE FLOW RATE © -PROBLEWS IN BOTW THE ENGLISH SYETEM OF WAITS AND TE INTERNATIONAL © INPUT DATA MUST BE SET UP AS POLLOKS. c © cant conum 1 EXTER 0 (ZERO) OR BLANK IF MIGLISH SYSTEM © (OF UNITS 15 TO BE USED. ENTER 1 (OWE) IF c seo. © CoLvmS 2-79 ENTER TITLE, DATE, AND OTHER INFORMATION, c 1 DESIRED. © CARD 2 COLUMNS 1-10 ENTER NUMBER INCLUDING DECIMAL GIVING e DIAMETER OF CHANNEL (IN INCHES OR MILLI- c eres). ¢ CconwmS 11-20 ENTER MUMDER INCLUDING DECIMAL GIVING € DEPTH OP FLOW (IN INCHES OR MILLIMETERS) . c (cous 21-30 ENTER NUMBER INCLUDING DECIMME, GIVING c Ccouvmis 21-40 ENTER NUMER INCLUDING DECIMAL GIVING c NANNING NVALUE. c FLOW RATE (IN CUBIC FEET PER SECOND OR c ‘CUBIC METERS PER SECOND) . e cl . © * NOTE WELL....EITHER THE DEPTH OF FLOW (COLUWMS 11-70) oR THE # © # Flow RATE’ (COLUMIS 41-50), WIICHEVER ONE IS TO BE DETERMINED BY + ‘THIS PROGRAK, SHOULD BE LEFT BLANK ON CARD 2. : WOLTTPLE DATA SETS FOR SOLVING ANY NUMBER OF PROBLEMS HAY INCLUDED FOR PROCESSING. DIMENSION TETLE(13) 1 READ(S, 100,END~2) WITS TITLE, 100 FoRaT( 11 3K6) wnaTE (6,105) TITLE 105 FORMATS 13K6,////) PACTOR®I2.0 TF (UNITS.bp. 1)C0EFP=1.0 {IP (UNTTS BQ. 1) FACTOR=1000.0 READ (5, 101) DIAN,,5,0,0 101 FORMAT (5P10.0) ReDIAH/2.0 1#(9.67-0.0001)G0 70 102 962 0 CHAPTER 14 CALL AReaWe HReAREA/MP (Q*AREATOBFE /4*HR** (2.0/3.0) 898 (5) {Te (UNITS. BQ.0) WRITE (6,103) DIAM,0,8,4.0 103 FORT( 1X, ‘GIVEN DATA’FOR AM OPEN CHANNEL FLOW IN A CIRCULAR SECTI 2088 //5%, "DIAMETER "87.2," IN", //5K, "DEPTH OF PLOW =" 477.2," IN", #7/5R, SLOPE = (P10.7¢/79R, SMRRUING NVALUE ©" F603, ////1%, "THE FLO RATE WILL BE!,P8.3," CU FT/S") 4F (QUITS. BD. 1) WRITE (6, 104)DIAM,0, 5.0.0 104 PORIE( IX, "GIVEN DATA’FOR AN OPEN CHANNEL PLOW IN A CIRCULAR SECTI ove //5K, (DIAMETER =",P7.1," 0 ,//5%,"DEDTH OF FLOM «"4P7-1," HM" S/19K,"S1OPE =" 10.7, //5K, "HARMING NVALUE =" P63, ////1K, "EME PLO SNe RATE WILL BE*,FB.3," cU'N/S") 701 102 ANPHQ*N/COEFF /SQRT(S) =0.01 bid "TRYIoAREA"* (5.0/3.0) /MP**(2.0/3.0)-a4P 108 D=D+0.01 IF (D.GF-DIAM)GO TO 112 "TRY2“AREA*® (5.0/3.0) /MP*¥(2.0/3-0)-AMP Ep (TRY *TRY2) 106,106,107 107 mwvi=rRy2 0 0 108, 105 D+p-0.005, IP (UNITS. nO) MRITE(6, 1091104, 0,5,4,D 109 FORMAT (1X," GIVEN DATA'OR AN OPEN CHAPEL PLOW IN A CIRCULAR SECTI SON //5K, DIAMETER =" ,P7.2," IN", //5%,"FLOW BATE =" ,F8.3," CU FT/S SLR, SLOPE =" P13.7, 1/5, HRNING NVALUE =" 6.5,////1%, HE D sebau OF Flow WILL. BE',P?.2," IM"), {IF (UNITS. BQ. 1) WRITE (611009184, Q,8.8,D 110, FORMAT(1x,"GIVEN DRTA FOR AN OPEN CHANNEL FLOW TW A CIRCULAR SECT SON //SK, ;DIAMETER =" P71," WH" //5%, FLOM RATE =",F8.3," CU M/S SS (75K SLOPE =! P13. //5k," PARNING NVALUE =" 26.3, ////AK, "THE D SEPTH OF FLOM MILL BE‘,#7.1,! HM) 112 wea7E 6,116) 116 FORMAT(ix, "THIS CIRCULAR CONDUIT CANNOT CARRY THIS GREAT A FLOW AS ‘OPEN CHAIMEL PLOW.") m1 2 stor [ABCOASDIAM*#2/4."ARCOS ((R-DI) /R) [RBEA™ (R-D1) #59 (Re¥2=(HHDI) *¥2) /2.0 [AREN-ABCDA-2.0°ABEA P=DIAMPARCOS ((R-D1) /R) IF (D. GT. RYAREA“PI*O2AH#*2/4. 1F (0.6. R)WP=PE*DIAN-WP [AREA-AREA/PACTOR**2 e=ir/PACTOR AREA 14.18 Solve Prob. 14.11 utilizing the computer program of Prob. 14.17. Osa ANALYSTS ge AN OPEN CHANNEL FLAW IN A CINGULAR SECTION 24.0 5.6 0.0025 0.012 M9 ua FLOW IN OPEN CHANNELS 0 363 Ontpat SAVELE ANALYSIS OF AM OPEN CHAMUEL FLOW TH A CIRCULAR SECTION GIVI DATA FOR AN OPEN CHANEL, FLOW IN A CIRCULAR SECTION DIAMETER = 24.00 18 DEPIH OF FLOW = 5.60 18 swore = 0.025000 agate N-VALUE = 0.012 {THE PLOW RATE WILL BE 1.463 CU FT/S Solve Prob. 14.12 utilizing the computer program of Prob. 14.17. A input LSANPLE' ANALYSIS @P AN OPEN CHADWEL, FLAW IN A CIRCULAR SECTION 500. 0,002 0.013 0.040 Outpat (GIVEN DATA FOR AN OPEN CHANEL FLOW IN A CIRCULAR SECITON FLOW RATE = 0.040 CU W/S oun NVALUE = 0.013 ‘THE DEPTH OF FLOM MILL BE 165.7 HH ‘An open channel isto be designed to carry 1.0 m’/s at a slope of 0.0065. The channel material has an n value of 0.011. Find the most efficent cross section for a semicircular section, 1 v= (L.0/m\(RY(8")_— Q/A=(1.0/n)(A/p.)%5"7) A 1p = On/s"? = (1.0)(0.011)/0.0065"* = 0.1364 =n@18 pom ad/2 — (xd?/8)°U(nd/2)=0.1364 = d=0.951m or 951mm (dis the diameter ofthe semicircular section; the depth of flow would, of course, be half of d.) Find the most efficient cross section for Prob. 14.20 for a rectangular section. IA /p%=0.1364 (from Prob. 14.20). The most efficient rectangular section has a width equal to twice its depth. Letting d= depth, A= (d)(2d) = 2d, py = d + 2d + d = Ad, Q2d?)*"/(Ad)” =0.1364, d 434 mm; width = 2d = (2)(434) = 868 mm. Find the most efficient cross section for Prob. 14.20 for a triangular section. 1 A°/p2 = 0.1364 (Grom Prob. 14.20). The most eficient triangular section has a XF angle and 1:1 side slopes (see Fig. 14-7). A = (3)(@V2\(dV2) = d°, py = (2)(dV2)= 2.828d, (d?)**/(2.828d)** = 0.1364, d= 0.614, or 614 mm; sides = 4V2 = (614)(V2) = 868 mm. 364 0 CHAPTER 14 1423 m8 14.25 14.26 14.27 Mig. 147 Find the most efficient cros section for Prob. 14.20 fora trapezoidal section. 1 4°°/p® =0.1364 (from Prob, 14.20). The mos efficient trapezoidal section is half a regular hexagon (see Fig, 148), A= (L.155d)(d) + (2)[(d){d tan 30°)/2] = 1.73242, p_ = (3)(0.155d) = 3.465d, (1.7324°)"9/(3.465d)>? =0.1364, d =0.459 mi; sides and bottom: each = 1.155d = (1.155}(0.459) = 0.530m. Rer7 Fase] res For the same conditions given in Prob. 14.2, determine the status of flow (i.e., is it critical, subcritical, or supercritical?). IN, =v/Vedq v= 6.4988t/s (from Prob. 14.2) dq =A/B A =132.8f0 (from Prob. 14.2) 3)(4.5)+16.0+(3)(4.5)= 43.0 dy, = 132.8/43.0=3.088 ft Ny = 6.498/ V@ZI3. 088) = 0.652 Since Np <1.0, the flow is subcritical. ‘The triangular channel (n = 0.012) shown in Fig. 14-9 isto carry water at a flow rate of 10m’/s. Find the critical depth, critical velocity, and critical slope of the channel. HBiAt=g/0* B= 6d, A=24)GA)/2=3d2 6A /(342)'= 9.80/10 AO2m/s = (mv sM(LORP) R= Alp (C)(1.178V1@)(VI)CL.178)] =0.5588m = {(0.012)(2.402)/1(1.0)0.5588)"]}° = 0.00181 Fig. 149 ‘The semicircular channel (n = 0.010) shown in Fig. 14-10 is to carry water at a depth of 1.0. Find the velocity, slope, and discharge at the critical stage. 1 = A/B = [()(*\(2.07/4)/2.0= 0.78548 uv. = Vad = V@Z2(0.TBS4) = 5.009 ft/s <= {nvf[(0.486)(R??)]}? = {(0.010)(5.029)/[(1-486)(2.0/4)°°]}* = 0.00289 = Av, =[2)()(2.07/4)(5.029) =7.90 f8/s {A flow rate of 2.1 m’/s is to be carried in an open channel at a velocity of 1.3 m/s. Determine the dimensions of the channel cross section and required slope if the cross section is rectangular with depth equal to one-half the width. Use n = 0.020. FLOW IN OPEN CHANNELS J 365 el Fig. 1610 4H Q=Av. Let B= channel width. Then channel depth = B/2;2.1 =[(BX(B/2)K1.3), B required width s 1.797 m and depth is 1.797/2, or 0.898 m R= Alp, = (1.797(0.898)/{1.797 + (2)(0.898)] = 0.4491 m 8 = {nv/{(1.0)(R*)]}? = {(0.020)(1.3)/[(1.0)(0.4491)")}? = 0.00197, 1797 m, Hence, the M28 Repeat Prob. 14.27 ifthe depth must be equal to twice the width. Compare answers with Prob. 14.27. 4H Q=Av. Let B= channel width. Then channel depth = 2B; 2.1 = ((B)(2B)|(1-3), B= 0.899 m. Hence, the required width is 0.899 m and depth is (2)(0.899), or 1.798 m. R= Alp, = (1.798)(0.899)/[0.899 + (2)(1.798)] =0.3596 m 5 = (nv/{(1.0)(R*)]}? = {(0.020)(1.3)/{(1.0)(0.3596)""]}? = 0.00264 ‘The chan area isthe same (oeleing roundoff err) but stepe slope is requted for he narrower 1429 Repeat Prob. 14.27 if the channel cross section is semicircular. ' 21= [Aad /4]0.3) d= 2.028m r= 2.028/2= 1.014m 5 (mv/{(1.0)(R*)}}* = ((0.020)(1.3)/[(1.0)(2.028/4)""})? = 0.00167 1430 Repeat Prob. 14.27 if the channel cross section is trapezoidal, with depth equal to the width of the channel bottom and side slopes of 1:1. 1 Q= Av. Let depth and channel bottom width = B (see Fig. 14-11). Then surface width = 3B; A= (GBB) ~ (2){4)(8\B)] = 2B", 2.1 = 2BV(1.3), B= 0.899 m. R= Alp. = (2)(0.899)"/{0.899 + (2)(0.899)V2] = 0.4696 m_ = {nv /H(1-0)(R°*)]} = {(0.020)(1.39/(1.0)(0.4696)""}? = 0.00185 Feeeereeeeea 2 ——F1 fee vy 431 For each of the channel cross sections shown in Fig. 14-12, compute the area, wetted perimeter, and hydraulic radius. Bw A= 3634.07] = 6.283? p= MI(H(4.0)]=6.28m_——R= Alp, =6.283/6.283 = 1.000m ) A= (5.0)(2.5) = 12.50 m? Pw =2.5+5.0+2.5=10.00m R= 12.50/10.00 = 1.250 © A= (5.01.2)+ QNGVL2L2)] =7-M0m?—_ py =5.0+ 2){(1.2)(V9)]=8.394m R= 7.440/8.394 = 0.886 m 366 0 CHAPTER 14 1432 433 1434 14.36 A.36-in-diameter concrete pipe on a 0.0015 slope carries water at a depth of 26 + ;_—s9=— © Fe. 1412 ‘Water is to flow in a rectangular flume at arate of 1.42 m°/s and at a slope of 0.0028. Determine the dimensions of the channel cross section if width must be equal to twice the depth. Use n = 0.017. B_Q=(Ay(1.0/n)(R)(s"), Let B = channel width and B/2= channel depth; 1.42 = [8(B/2)]0.0/0.0171(8)B/2)/(B/2 + B + B/2)}*(0.0028)", B = 1.366 m. Hence, required channel width = 1.366 m and depth = 1.366/2, or 0.683 m. ‘Rework Prob. 14.32, assuming width must be equal to the depth. Note which solution gives the smaller (and therefore more efficient) cross section, HQ =(A)(.0/n)(R™Y(s"). Let B = channel width and depth; 1.42 = [(B)(B)|(1.0/0.017)((B)(B)/(B + B+ £B)P*(0.0028)", B = 0.981 m. Hence, required channel width and depth are each 0.981 m. A = (0.981)(0.981) = (0.962 m®. For Prob. 14.32, A = (1.366)(0.683) = 0.933 m?, The cross section of Prob. 14.32 has the smaller 1.0m? /s: Q = (A)(1.0/n)(R™™Y(s"?), A= (0.84)(aD7/4) = 0.6597D*, R = (1.21)(D/4) = 0.3025D, 2.76 = (0.6597D"\(1.0/0.014)(0.3025D)**(0.00018)'", D = 2.34 m. 440 A TLiin-diameter vitrified sewer pipe (1 = 0.014) is laid on a slope of 0.00025 and carries wastewater at a flow rate of 50 fs. What is the depth of flow? H From Fig. A-5, Ona = 67 0/5: (Qua)o-aoul/67 = 0.013/0.018, (Qna)enoone = 62-2 0/8; O/Qua= '$0/62.2 = 0.804, or 80.4 percent, From Fig. A-18, D/Diy = 69 percent, D = (0.69)(72) = 49.7. 968 0 CHAPTER 14 u aL ua 1443 aus 14.45 1447 ‘A 1.O-m-diameter pipe must carry a discharge of 0.40 m’/s at a velocity of 0.80 m/s. Determine the slope and the depth of water. 1 A= Q/v=0.40/0,80=0.5000m? Aga = (x)(1.0)7/4 = A/Aca = 0-5000/0.7854'=0.64 or 64 percent From Fig. A-18, D/Dua =0.63 and R/Rea=1.12. D=(0.63)(1.0)=0.630m == v= (L.0/m\(RY(6)_— Ryn = 1.0/4 = 0.2500m R= (1.12)(0.2500) =0.2800m 0.80 =(1.0/0.013)(0.2800)"%s"") = 0.000590 7854 mi? ‘The trapezoidal channel of Fig. 14-16 is to carry 500 cfs of water. The maximum allowable velocity of flow is 3.0 fps to avoid scouring. Determine the depth of flow, d, and the width of the channel bottom, B, if the hhydraulic radius of the channel is one-half the depth of flow. Also, determine the slope of the channel bottom. Use n =0.025. HT R=d2=Alp. Am BA+AQNLSAY(d)]=Bd+ 150 — p= B+2VEE CSA = B+3.6064 d/=(Bd+150)(B+3.606d) B+3.606d~=28-+3.0d B=0.606d = A=Q/v Bd + 1.5d* = 500/3.0 Substituting B = 0.606d, (0.606d)(d) + 1.5d?= 166.7, d = 8.90ft; (0,606)(8.90) = 5.39 ft. v= (1,486/n)(R™)(s"7) R=8.90/2=4.45 ft 3.0 = (1.486/0.025)(4.45)""(s'7) 5 = 0.000348 aa -—9-1—4 ve 46 ‘An open channel to be made of concrete is to be designed to carry 1.5 m/s ata slope of 0.00085. Find the most efficient cross section for a semicircular section. 1 v= (1O/m\(R*KS") QI = (1.0/n)(Alp.)™8") AP [pi = Qnis'* = (1.5)0.013)/0,00085"" = 0.6688 A= nd?/8 Pom nd/2 — (ad?/8)/(nd 29" =0.6688 d= 1.727m (dis the diameter of the semicircular section; the depth of flow would, ofcourse, behalf of d.) Find the most efficient cross section for Prob. 14.43 for a rectangular section. FA Ip = 0.6688 (from Prob. 14.43). The most efficent rectangular section has a width equal to twice its depth. Letting d = depth, A= (d)(2d) = 2d", py =d-+ 2d + d= 4d, (2d?)*"/(Ad)"® = 0.6688, d = 0.789 m; width = 2d = (2)(0.789) = 1.578, Find the most efficient cross section for Prob. 14.43 for a triangular section. BA /p2=0.6688 (rom Prob. 14.43). The most efficient triangular section has a 9? angle and 1:1 side slopes (see Fig. 14-7): A = (1)(d y3)(d/2) = a, p, = 2)(d V2) = 2.8284, (f°) / (2.8284) = 0.6688, d= 1.115 m; sides = d2 = (.115)(V2) = 1.577 m. Find the most efficient cross section for Prob. 14.43 for a trapezoidal section. I A®/p2 =0.6688 (rom Prob. 14.43). The most efficient trapezoidal section is half a regular hexagon (see 1.155d)(d) + 2{(d)(d tan 30°)/2] =1.7324", p, = (8) 1.1550) (1.7324?)*9/(3.465d)°” = 0.6688, d = 0.832 m. Sides and bottom: each = 1.155. 1.155)(0.832) = 0.961 m, For the conditions given in Prob. 14.32, determine whether the flow is critical, subcritical, or supercritical. He v= Q/A = 1.42/{(1.366)(0.683)]= 1.522 m/s Np = v/ Vgdy = 1.522/ V(@-BU7( 0.683) = 0.588 Since N, <1.0, the flow is subcritical ee re) ust FLOW IN OPEN CHANNELS 369 ‘A rectangular channel with a width of 3.0m and an n value of 0.014 is to carry water at a flow rate of 13.¢m°/s. Determine the critical depth, velocity, and channel slope. HB d.= (0/8) /e) = (03.4/3.0)/9.807} = 1.267m v= Q/A =13.4/|(1.267)(3.0)] =3.525 m/s R= Alp = (1.267)3.0)/(1.267 + 3.0 + 1.267) = 0.6868 m 5e= (nvel{ (LORY)? = {(0.014)(3.525)/[(1-0)(0.6868)""})? = 0.00402 “The semicircular channel (n = 0,013) shown in Fig. 14-17 isto carry water while flowing full (.e., at a depth of 1.58). Determine the velocity, slope, and discharge when low is critical. ' dg = ALB = 3(#)(3.0)'/4)/3.0 = 1.178 ft Vad. = VG2.2)(1.178) = 6.159 ft/s m{(2.486)(R°)]}? = {(0.013)(6.159)/{(1.486)(3.0/4)”*}) = 0.00426 Q= Av = (I{(x)G.09/4)}(6.159) =21.81°/s = Determine the dimensions of the most economical trapezoidal brick-lined (n = 0.016) channel to carry 200 m/s with a slope of 0.0004. I The most economical trapezoidal channel has a cross section as shown in Fig. 14-8 and R= d/2 and A=V3d"; Q= (A)(1.0/n)(R°*)(s!7), 200 = (V34°)(1.0/0.016)(d /2)°"(0.0004)"*, d= 6.491 m. Bottom width = (1.155)(6.491) = 7.497 m. Determine the discharge for a trapezoidal channel with a bottom width of 8 ft and side slopes 1:1. The depth is 64t, and the slope of the bottom is 0.0009. The channel has a finished concrete lining (n = 0.012). 1 A= (6+ ANEMONE) = 84.008 —— py = B+ (2)[(6(V2)] = 24.97 = (A)(1.486/n)(R??)(s"*) = (84.00)( 1.486 /0.012)(84.00/24.97)°°(0.0009)"" = 701 ft/s ‘What depth is required for 4-m’/s fow in a rectangular planed-wood (n = 0.012) channel 2m wide with a bottom slope of 0.002? ' Q=(ANLO/m RV) 4 = Ba)(1.0/0.012)[24/(2 + 24)P%(0.002)"* £72424)" = 0.3381 d= 0.888m (by trial and error) ‘A developer has been required by environmental regulatory authorities to line an open channel to prevent erosion, The channel is trapezoidal in cross section and has a slope of 0.0009. The bottom width is 10 ft and side slopes are 2:1 (horizontal to vertical). If he uses rubble (y, = 135 lb/ft) for the lining, what is the minimum Diy of the rubble that can be used? The design flow is 1000 cfs. Assume the shear that rubble can withstand is described by t= (0.040)(y, ~ y)(Dse) (lb/ft), in which y, is the unit weight of rock and Dy is the average rock diameter in feet. IA Manning n of 0.03 is appropriate far rubble. = (A)(1.486/n)(R™\(5"9) 1000 = [(4)(10 + 2)](1.486/0.08){()(10 + 2dy/{10 + (2)(V5)(4)1)*9(0.0000)"* (00+ 24)P7/10+ ANVIAP?= 672.9 d= 8.634 by tial and error) y= YR R= 8.63(10 + (2(8.63)]/[10-+ (2)(V5)(8.63)] =4.881 f Fo = (62.4)(4.841)(0.0009) = 0.2719 b/ ‘Tofind the Diy size for incipient movement r = rq and 0.2719 = (0.040)(135 ~ 62.4)(Ds), Dig = 0.0936 t. ‘A metal-lined rectangular sluiceway is to carry 1.0 m’/s ata slope of 0.010. Determine the minimum area of galvanized iron (n = 0.011) needed per meter of length. Neglect freeboard. 370 0 CHAPTER 14 I For minimum area, D = B/2 and R = B/4. = (AL.O/m(R\(5)——1.0= [(8)(B/2)]0.0/0.011)(8/4)"0.010)"" B= 0.8015 m D-=0.8015/2=0.4008m —— Aguyy/L= 0.4008 + 0.8015 + 0.4008 = 1.6 m?/m. 14.55 The sides of a trapezoidal channel are inclined 63.4° to the vertical; the channel isto carry 18 m'/s with a ota ips of 00, Determine ho tem width, depth, and velocity for the best hydraulic section ' O=(A\(L.0/m\(R\(0") and m= tan 63.4" = 2.000 Px = 4d V+ mi? — 2md = (4)(d)VT-+ 000) ~ (2)(2.000)(d) = 4.9440 = B+ 2VSd AN9d A= Bd +2d*=(0.4719d)(d) + 2d? = 2.4720" 2. 472d*)(1.0/0.026)(2.4724"/4.944d)>"(0.0009)"? d= 2.373m B= (0.4719)(2.373)=1.120m v= Q/A=18/[(2.472)(2.373)"] = 1.293 m/s 14.56 A semicircular corrugated-metal (n= 0.025) channel must transport 2.4 m’/s a distance of 1000 m with a head loss of 2m. Compute the required radius. ' = (AN(0/m)(R\(5")_— 2.4-= (7 /2)(1.0/0.025)(r/2)"%adm)"* r= 1-121 m 1457 Determine the best hydraulic trapezoidal section to convey 86 m/s with a bottom slope of 0.002. The lining is finished concrete (n= 0.012). ' Q=(A)(L.0/n)(R)(5'7)__ 86 = (A)(1.0/0.012)(A/p.)°™(0.002)'* A Ip? = 23.08 “The best trapezoidal section is half a regular hexagon (see Fig. 14-8) for which A= 1.7324" and p. =3.46Sd (om Prob. 14.23). (1.7324d°)**/(3.465d)?? = 23.08, d = 3.141 m. Sides and bottom: each = 1.155d = (1.155)@.141) = 3.628, 14.58 Calcolate the discharge in steady flow through the channel and floodway of Fig. 14-18; take s = 0.0010 and y= 2438m. ' 2 =(AYLO/n RY") (12)(5-+ 2.498) + 2)(5 + 2.438)(5 + 2.438)/2 ~ (2.438)(2.438)/2 = 141.6 m? VS DABS + G+ ZARB + 12+ VFS =29.59m 141.6)(1.0/0,025)(141.6/29.59)°%(0.0010)"* = 508.6 m"/s ‘Ag = (120)(2.438) + (2.438)(2.438)/2 = 295.5 m? (po): 120+ VERIT TARE = 123.4m Qs = (295.5)(1.0/0.040)(295.5/123.4)°%(0.0010)"* = 418.1 m/s =O, +0, = 5086+ 418.1 = 926.7 m/s Fig. 14:18 14.59 For 25000 fs through the section of Fig. 14-18, find the depth of flow in the floodway (i.e., evaluate y) in eet ‘when the slope of the energy grade line is 0.0004. HQ = (A). O/MYRYS') AL = (12)(5 +9) + AUS + VG + y)/2— (IZ = 712 + 22y +85 | (Dai = VETS # GF) + 12+ Vy = (SMV2) + 12 + (2y)(VE) Aza 120y + (y)(y)/2= 120) + 97/2 | (pa)a= 120+ VyFF y= 120+ (y)(V2)——_ = (25 0009(0.3048)? = 707.9 m/s 107.9 = (y2/2-+ ay + 85)(1.0/0.025){(y2/2 + 22y + 85)/[(S)(V2) + 12 + (2y)(V2)1}°7(0.0004)"* + (420y + y*/2)(1.0/0.040){(120y + y°/12)/[120 + (y)¢V2)}}°*0.0004)"" y=279m or 9.1Sft (by trial and error) FLOW IN OPEN CHANNELS J 371 14.60 Find the critical depth for flow at 1.6 m’/s per meter of width. ' Y= Gl)!” = (1.69.80) = 0.639 m 1461 Compute the critical depth for flow at 0.4 m°/s through the cross section of Fig. 14-19. ! QTgA a1 T= OY y tanG/2)=1155y A=(y)(y tan 60"/2) = 0.5774y?_—_(0.4)(1.155y)/[(9.807)(0.5774y?))} y= 0.628m Fig. 1419 1442 Determine the critical depth for flow at 8.49.m"/s through a trapezoidal channel with bottom 2.5 m wide and with 45 sides ' OTA =1 — TH2542y — A=By+myt=25y + (DO? 4925+ 2y)MO.BONZSy+y]=1 —-y=0.928m (by trial and error) Design a transition from a trapezoidal section, 8 ft bottom width and side slopes 1 on 1, depth 4 ft, to a rectangular section, 6ft wide and 6 ft deep, for a flow of 250 cfs. The transition is to be 20 ft long, and the loss is one-tenth the difference between velocity heads. Show the bottom profile, and do not make any sudden changes in cross-sectional area. 1 A, =8 x44 4 = 48 ft, Ay = 36 ft, loss = 0.1[(v3/2g) — (vi/2g)], and ys + (vi/2g) + 21 = ys + (3/28) + 22+ toss, Assume a linear change in area, b, and T: = 8 ~ 2(x/L) and T = 16 10(x/L). Hence: (6 + Ty/2) = 48 ~ 12(2/L) and y = 2{([4— @/L)\/[2 ~ (&/L))). sik | A | vi27g| tos | EGE | y z 0 48 | 0421 4a | 40 0 (0,008 0333 | 44 | oso 443 | 44 | -0.488 ‘0.011 0.667 | 40 | 0.607 4402 | s.oor | -1.206 0.014 10 | 36 | 0749 438 [60 | -2361 ‘The profile is shown in Fig. 14-20. 372 1464 65 14.66 0 CHAPTER 14 Wah Na Fig. 14:20 Ina transition from a rectangular channel (2.5 m wide, 2m deep) to a trapezoidal channel (bottom width 4m, side slopes 2 on 1, depth 1.3 m) the energy loss is equal to 0.4 times the difference between velocity heads. The discharge is 5.8m’/s. Determine the difference between elevations of channel bottoms. ' OF Ui/2g + y= Az + VED + ys (0.4)(Ui/2g ~ v3/28) Az= (4)(L.3) + QM{C3)2)]0.3)/2) =8.580m? y= Vs = 5.8/8.580 = 0.6760 m/s vi/2g = 1.1607/[(2)(9.807)] = 0.06860m —_v3/2g = 0.6760"/[(2)(9.807)] = 0.02330 m (0+ 0.06860 + 2= Az +0.02330 + 1.3 + (0.4)(0.06860 ~ 0.02330) Az =0.727m ‘A dam gate (Fig. 14-21) admits water to a horizontal canal. Considering the pressure distribution hydrostatic at section O, compute the depth at O and the discharge per meter of width when y = 1.0m. ' dy= Coy = (0-85)(1.0)=0.85m ——pyly + vi/2g-+2.=paly + vig + mt he 0+046=0+ vi/[2)(9.807)]+0.85+0 v,=10.05m/s (0.95)(10.05) = 9.548 m/s Q = Av = ((0.85)(1)|(9.548) = 8.12 m/s per meter of width cae 085 2888 Toraeta foot, n= 0015 7, Wig. 4-21 ‘A discharge of 4.5 m/s occurs ina rectangular channel 1.83 m wide with s = 0.002 and n = 0.012. Find the ‘normal depth for uniform flow and determine the critical depth. Is the flow subcritical or supercritical? ! Q=(A\(L.O/ny(R*\(5") 4.5 = (1.83y,)(1.0/0.012)[1.83yq/(y, + 1.83 + y,)P*(0.002)'" Y= 1.060.m (by trial and error) OP lg= AB 4.57/9.907=(1.83y.) 1.83 y= 0.851 m Since y.

También podría gustarte